Evaluating Exam Review Book


530 16 17MB

English Pages 762 Year 2018

Report DMCA / Copyright

DOWNLOAD PDF FILE

Table of contents :
001 Canadian Pharmacy Review Content ver1
Misbah Biabani, Ph.D
Abbreviations
Part 1. Biomedical Sciences 15%
Part 2. Pharmaceutical Sciences 25%
Part 3. Social/Behavioural/Administrative Sciences 10%
01 Chapter Human Anatomy
Body Movements (Fig 1.1)
 Extension: Lengthening or straightening of a flexed limb.
 Flexion: Bending of a part of the body.
Orthostatic: Standing upright and lying down supine.
Weight bearing joints
Muscles:
Tips
 Adduction ( ( )
 Which one is a part of the shoulder? ( )
02 Chapter Gastrointestinal System
This chapter review anatomy, physiology and pathophysiology of the gastrointestinal system, common disease that occurs in gastrointestinal tract.
Mouth
Esophagus
Stomach
G Cell
*Deficiency of intrinsic factors causes a type megaloblastic anemia i.e. pernicious anemia.
Gastric acid secretion mechanism. In the parietal cells CO2 and H2O are converted H+ and HCO3- catalyzed by carbonic anhydrase. The parietal cells secrete HCl into the lumen of the stomach and concurrently absorb HCO3- into the blood stream.
Gastric acid stimulations. Gastric acid production is stimulated by three mechanisms.
Gastrin: It is released in response to eating a meal (protein), thus stimulates parietal cells to secrete H+.
Pathophysiology of gastric acid secretions causes gastric ulcer, duodenal ulcers and Zollinger-Ellison syndrome.
Small intestine: Consist of duodenum, jejunum, and ileum.
Duodenum: First part of the small intestine, C-shaped 10" (inch) long and curves around the head of pancreas and the entry of common bile duct.
Large Intestines: It is also known as colon.
Diseases of the gastrointestinal system
Diseases of the Colon
Digestion and Absorption
Carbohydrates digestion: The most common site of carbohydrate absorption is small intestine. Only monosaccharides such as glucose, fructose, and galactose are absorbed.
Disorder of carbohydrate absorption. Lactose intolerance results from absence of brush border lactase. Thus, non-absorbed lactose cause osmotic diarrhea.
Milk intolerance can result from 2 reasons. 1) Lactose intolerance 2) milk protein allergies
Lipid Absorption. Bile acids emulsify lipids in the small intestine, increase surface for digestion. Pancreatic lipases, hydrolyse, lipids to fatty acids, monoglycerides, cholesterol and lysolecithin.
Lipid absorption disorders. Malabsorption of lipids thus causing fatty stools, this also referred as stethorrhea. Stethorrhea can cause by
Absorption of Proteins (small intestine): Trypsin and chymotrypsin are secreted by pancreas, which helps in digestion of proteins.
Absorption of nucleic acid:
Absorption of Vitamins and Nutrients: Fat soluble vitamins (ADEK) are absorbed in small intestine along with other lipids. Vitamin B12 is absorbed in the ileum and that requires intrinsic factor.
Absorption of calcium: Mainly occurs in small intestine, which assisted by active form of vitamin D3, 1, 25-dihydroxycholecalciferol, which is produced in kidney. Chronic renal failure or vitamin D deficiency results in inadequate intestinal Ca2+ abso...
(stomach) (small intestine)
Absorption of Iron: It is absorbed as heme iron (iron bound to hemoglobin or myoglobin) or as free Fe2+. In intestinal cells, heme iron is degraded to Fe2+ and released. The free Fe2+ binds to apoferritin and is transported into the blood.
Innervations of GI tract. Autonomic innervations.
 A patient with chronic renal failure have deficiency of vitamin? ( )
 Active Vitamin D is ( ( )
03 Chapter Nervous System
3
Nervous System
Nervous system divided into central nervous system and peripheral nervous system. The central nervous system consists of brain and spine.
Brain
Meninges
Pathology of neurological disorders
Tips
 Sciatica is ( ( )
04 Chapter Cardiovascular System
Conduction System of the Heart (fig 4.4)
Blood flow sequence: Vena cava ( right atrium ( right ventricle ( left pulmonary artery( LUNGS ( left pulmonary vein ( left atrium ( left ventricle ( aorta ( systemic circulation
Septal defect: Ventricular septal defect is a hole in the wall separating the two lower chambers of the heart.
Types of pacemakers
Decrease K+ efflux out to cell
Myocardial action potential curve: Myocardial action potential curve reflects action potential, which describes electrical activity of five phases. This occurs in atrial and ventricular myocytes and purkinje fibers.
Electrocardiograph Wave Forms: The electrical activity occurred during depolarization and repolarization transmitted through electrodes attached to the body and transformed by an electrocardiograph (ECG) in to series of waveforms.
Laplace's Law: Laplace's law describes how tension in the vessel wall increases with Trans mural pressure. According to Laplace’s law, tension is proportional to the radius of a sphere.
Depolarisation and repolarisation?
05 Chapter Endocrine System
Oxytocin (formed in hypothalamus and stored in posterior pituitary gland).
Calcitonin takes Ca from Blood ( Bones
Insulin
Pancreas
Glucagon
Epinephrine
Testosterone
Prepares for pregnancy
Diabetic Patient
What decreases insulin requirement?
Pregnancy test: Human chorionic gonadotropin (hCG) hormone levels are elevated in first 3 months of pregnancy (first trimester). Progestin's in pregnancy is produced by ovaries, corpus luteum and placenta.
06 Chapter Renal System
6
Renal System
Nephron. A nephron is the basic unit of renal function. There are millions of nephron present in each kidney. Nephron has three major functions.
Causes. Some of the most important causes of prerenal ARF are dehydration, heart failure, sepsis (severe infection), and severe blood loss.
Intracellular
Hypercalcemia
Hypocalcemia
Hyperphosphatemia
 Occurs due to hypoparathyroidism (low PTH).
Potassium regulated by.
Hyperkalemia
Hypokalemia
Hyperchloremia (Cl- excess) and hypernatremia (Na+ excess in the blood)
Hypochloremia
Hyponatremia
Metabolic alkalosis (pH >7.45): ↑ Bicarbonates (HCO3-) in blood and ↓ CO2 in blood.
Respiratory Alkalosis
 Pre-renal ARF is due to ( ( )
 Normal serum potassium levels ( ( )
 Summary of electrolytes action in kidney (True/ False)
07 Chapter Liver functions and chronic liver diseases
7
Liver and Chronic Liver Diseases
Definitions
 Necrosis cellular breakdown example: Acetaminophen
 Steatosis: Hepatocytes filled with small droplet of lipid. Example: Tetracycline’s
Drugs transportation into the bile from the liver
Spontaneous Bacterial Peritonitis
Wilsons Disease
Viral Hepatitis: There are 5 types of hepatitis viral infection, hepatitis A, B, C, D, and E. However, the common infections are hepatitis A, B and C.
Hepatitis A
Hepatitis B and C
08 Chapter Respiratory System
8
Respiratory System
Obstruction of airflow is irreversible
Flu vaccine annually and Pneumococcal vaccine Q5-10y in high risk.
Causes of COPD, smoking, Alpha1-antitripsine deficiency, air pollution, secondary smoke.
09 Chapter Urinary System doc
10 Chapter The Eye and Ear doc
Higher in number
Cataract: When the eye lens becomes cloudy, decrease acuity, and no pain, and this obstruct the vision is referred as cataract.
Dental anatomy and physiology
 Blind spot is ( optic disc
 Glaucoma occurs due to ( abnormal increase IOP in eyeball
11 Chapter Blood and Anemia
45% Cells
Vitamins
Electrolytes: The Electrolytes present in blood are Na+, K+, Ca2+, Mg2+, Cl- and CO3
Agranulocytes
Rh factor: Agglutinogens in human RBCs are known as the Rh factor blood with this factor is described as Rh +ve (90% of population). Blood without this factor is described as Rh (-) negative. In Rh-negative mother, Rh-positive antigens may transfer fr...
Qualities of Blood
Megaloblastic Anemia (macrocytic anemia) Mean cell volume is increased. (MCV >100 fL)
Filgrastim. It is glycoprotein produced through recombinant DNA technology.
Hemolytic anemia: The sickle cell anemia disease is an inherited disorder caused by a defect in the gene for hemoglobin. Sickle cell anemia and thalassemia are hemolytic anemia associated with abnormal hemoglobins. Due to poor solubility of such abno...
Coombs test: The coombs test is used to distinguish immune mechanism or glucose 6-phospho dehydrogenase (G6PD) deficiency anemia. In autoimmune hemolytic anemia, coombs test is positive. Example of drugs that cause false positive coombs test penicilli...
12 Chapter Biochemistry
Carbohydrates
Classification
Common questions in pharmacy exam is to ask!
Carbohydrate metabolism. Glycogenesis, Glycogenolysis, Glycolysis, Gluconeogenesis.
Gluconeogenesis
Amino Acids: The amino acids from proteins are precursor of compounds and energy source like converted to acetyl CoA. Amino acids degradation eliminated -NH2- group and this converts to NH3 and this may be toxic. Ammonia eliminates through conversion ...
Urea cycle
Liver
 Arginine ( Nitric oxide (NO)( vasodilator
Glycine ( Porphyrine ring ( Hemoglobin
Zwitter Ion: Amino acids are ionisable +ve ions as amines, -ve ions as acid. (no net charge)
Proteins
Fats and Lipids
13 Chapter Fluids Electrolytes and Nutrition
This chapter review the important feature of the vitamins is that they generally cannot be synthesized by mammalian cells and, therefore, must be supplied in the diet.
Canada's Food Guide, eating well with Canada Food Guide provides evidence base information on nutrient standards and the prevention of chronic diseases.
Infants nutrition
Folic Acid (vitamin B9)
Vitamins
Fat soluble
Water soluble
14 Chapter Microbiology
14
Microbiology
Bacterial Structure
Fungal infections: Candidiasis (yeast), Athletes foot
GRAM –ve and aerobic bacteria
GRAM +ve and aerobic bacteria
Gram –ve cocci
Moraxella catarrhalis
Salmonella
Clostridium
C. tetani
C. botulinum
Gram + ve cocci
S. viridans (alpha hemolytic)
S. aureus (Coagulase +ve)
Cyanobacteria diphtheriae
GRAM –VE and Anaerobic bacteria
B. fragilis
 
 
BACTERIA
 
Gram +ve Cocci
Found on the skin and in the nose
Boils, and Septicemia
Penicillin G and Penicillin V, cefalexin
Food poisoning
Staphylococcus aureus
Wound infections, skin infections impetigo, and cellulitis.
Catalase positive
Toxic shock syndrome (TSS)
Clarithromycin
Tonsillitis, Cellulites, impetigo. Scarlet fever, Septicemia may cause immune-mediated disease (e.g. rheumatic fever).
Beta-hemolytic streptococci e.g. Strep pyogenes (Group A Strep: GAS)
Erythromycin
Azithromycin
Penicillin G, cloxacillin, cefalexin
Amoxicillin
Penicillin G
Pneumonia (CAP), Otitis media, Meningitis, Sinusitis, Pharyngitis
Alpha-hemolytic
Clarithromycin
Streptococcus pneumonia
Azithromycin
Amoxicillin, Penicillin G, Clindamycin
Endocarditis,
Alpha-hemolytic
Dental caries
Streptococci viridans
Instrument contamination. Catheter infections, UTI
 
S. epidermidis
Gram +ve bacilli
Erythromycin or penicillin's (to eliminate carrier state)
Diphtheria (disease due to toxin production)
Corynebacterium diphtheria
Tetracycline
Clostridia sp.
Tetanus,
Cl. tetani
Metronidazole, or vancomycin
Gas gangrene
Cl. perfringens
Botulism
Cl. botulinum
Pseudo membranous colitis
Cl. difficile
 
 
Gram-ve bacilli
Uncomplicated UTI. Sulfa drugs (cotrimoxazole), nitrofurantoin, Trimethoprim, fluoroquinolones (cipro, norfloxacin, Ofloxacin)
Urinary tract infections (90%),
E. coli
Traveler's diarrhea
Proteus sp.
E. coli (diarrhea). Ciprofloxacin, and Levofloxacin,
Wound infection, sepsis. Normal inhabitants of the gut.
Klebsiella sp
Enteric fever (typhoid), food poisoning
Chloramphenicol (typhoid)
Most sp. are animal pathogens (e.g. eggs etc). S. typhi infects man only, causes typhoid.
S. typhi
Salmonella sp
Ciprofloxacin
Dysentery (bloody diarrhea or shigellosis).
Shigella sp.
Carbapenems (imipenem, meropenem)
Nosocomial (hospital acquired) and opportunist infections (most common S. aureus).
Aminoglycosides +/-
Pseudomonas aeruginosa
Ampicillin,
Ceftazidime (3rd)
Clarithromycin,
Azithromycin
Pneumonia, meningitis, Otitis media
Hemophilus influenza
Ampicillin, amoxicillin
Tetracycline
 
 
Gram -ve cocci
Penicillin G
Meningococcal meningitis +/- shock commensal of upper respiratory tract
Neisseria meningitides
Cefixime
Ceftriaxone im/iv
Ciprofloxacin
Gonorrhea (STIs). Always pathogenic.
Neisseria gonorrhea
Levofloxacin
Ofloxacin
Azithromycin 1g
Chlamydia
Chlamydia trachomatis
 
 
Acid-fast bacilli
Isoniazid
Tuberculosis. (Weight loss, coughing, fever, sweating, chest pain)
Rifampicin
Streptomycin
Mycobacterium tuberculosis
Ethambutol
The most common cause of infectious death world-wide.
Pyrazinamide
Dapsone
Leprosy
Mycobacterium leprae
Rifampicin
 
 
Spirochetes
Syphilis (STIs) (genital ulcers or chancres), single large ulcer, and painless.
Penicillin G im inj.
Treponema pallidum
Doxycycline
 Have thick, ergosterol containing cell walls and grow in humans as budding yeast cells and slender tubes (hyphae).
Fungi
Nystatin
Thrush, mucocutaneous infection, vulvovaginitis
Candida albicans (yeast)
Clotrimazole, miconazole
Tolnaftate (topical)
Athlete’s foot
Tinea pedis
Abscesses (puss)
Sporotrichosis or granulomas
Skin, nail and hair infections, sometimes acquired from animals.
Ringworm
Dermatophytes
Ubiquitous airborne filamentous fungus
Allergic reactions, opportunistic infections
Aspergillus sp.
Meningitis in immunocompromised
Present in soil and pigeon droppings
Cryptococcus neoformans
 
 
Protozoa
Malaria. Four sp. infect man via biting female anopheles mosquito.
Chloroquine, Mefloquine
Plasmodia sp
Primaquine, Doxycycline
Low grade gastrointestinal disease: giardiasis.
 Metronidazole
Giardia lamblia
Amoebic dysentery (are infective when swallowed, traveler’s diarrhea). Severe, may invade and spread to the liver.
Metronidazole
Entamoeba histolytica
Ciprofloxacin
and Giardia lamblia (intestinal protozoa)
Cotrimoxazole
 
 
DNA viruses
 
Conjunctivitis, Sore throat
Adenoviruses
HSV1 and HSV2 can cause oral and genital lesions. HSV1 causes cold sores and Keratoconjunctivitis.
Acyclovir
Famciclovir
Herpes viruses
Herpes simplex virus
VZV can cause (Varicella: chickenpox, zoster: Shingles), glandular fever,
Foscarnet
Herpes zoster
Ganciclovir
Cytomegalovirus (CMV)
Epstein-Bar (EB virus)
Roseola infantum (sixth disease)
Hepatitis B Transmitted via blood and body fluids and sexual contact.
Interferon alpha
Hepadnavirus. Hepatitis B
Slapped cheek disease, (fifth disease, and erythema infectious).
Can cause aplastic crises
Parvovirus: parvovirus B
HPV vaccine. Implicated in cancer of the cervix.
Papovaviruses: papillomavirus,
Warts, cervical cancer,
Vaccine Gardasil. Quadrivalent human papillomavirus (types 6, 11, 16, 18) recombinant vaccine.
Hemorrhagic cystitis.
Polyomavirus
Molluscum contagiosum, smallpox
 
Poxviruses
 
 
RNA viruses
Amantadine, Ribavirin, Rimantadine (influenza A).
Orthomyxoviruses:
Influenza (flu)
Influenza A and B
Yellow fever, chronic hepatitis
Flaviviruses. Yellow fever, Hepatitis C
 
Paramyxoviruses: parainfluenza, RSV, Measles Mumps
Respiratory infections: Croup
May be severe in infants
Measles, mumps
Meningitis, Common cold (rhinoviruses)
Picornaviruses: Enteroviruses (e.g. poliovirus), rhinoviruses, Hepatitis A
 Rhino has runny nose
Hepatitis
 
Gastroenteritis
Reoviruses: rotavirus
NRTI, NNRTI
AIDS, T-cell leukemia,
Retroviruses: HIV-1, 2
Spastic paraparesis
HTLV I, II
Zoonotic infection
Rabies
Rhabdoviruses: rabies
German measles (Rubella),
Togaviruses: Rubella,
 
Encephalitis
Alpha viruses
15 Chapter Cell and Molecular Biology
Prokaryotic
Animal: no cell wall but cell membrane
Yes chloroplast
Present in bacteria
Cell organs compose of a number of tissues, and each tissue composes of cells of the same type. The individual cell is the minimal self-reproducing unit in all-living species. It performs two types of functions, such as performs chemical reactions nec...
Centrosomes: Microtubule does cell division or formation spindle apparatus during cell division.
Chromosome, Gene and Genome
Gene Expression
Nucleic acid (DNA and RNA). Nucleotides are the building blocks of all nucleic acid molecules (such as DNA and RNA). These structural units consist of three essential components, i.e.
DNA
The DNA molecule consists of four bases Adenine (A), Cytosine (C), Guanine (G), Thymine (T)
There are 3 types of types of RNA based on their functions:
Codon
DNA ---------------------( mRNA ( tRNA ( rRNA ( ( ( Protein synthesis
Step 2. Translation ( mRNA ( tRNA.
Introns and exons: The coding region of a eukaryote’s gene is different from that of a prokaryote. For Eukaryotes, each gene contains introns and exons. Intron is a segment of gene situated between exons. It is not responsible for the coding of protei...
 DNA bases ( A, C, G and T (True/False)
16 Chapter Pharmacogenetics
17 Chapter Immunology and Immunization
THE ORGANS OF THE IMMUNE SYSTEM
LYMPH NODE. Lymph nodes are small, bean-shaped structures that are laced throughout the body along the lymphatic routes. Lymph nodes contain specialized compartments where immune cells congregate, and where they can encounter antigens.
B CELLS: B CELLS have thousands of identical antibodies in their membranes that allows them to bind chemically to a small group of chemically related antigen.
Less than 1%, Functions not well understood.
Ig E
IgE mediated type I: Anaphylactic reactions. It is treated by epinephrine. (Penicillin's, bee stings, latex, pea nut).
Structure of immunoglobulin's
Ig A is secreted during memory response, this accounts for 10% of serum immunoglobulins. It is secreted across mucosal surfaces into gastrointestinal, respiratory, lachrymal, mammary, and genitourinary secretions. Where this protects mucosa from col...
Natural Killer Cells
Interferon: The interferons are the family of cytokines proteins, important in the immune response. Interferon Proteins that are secreted by cells when they become infected with virus. Bind to nearby infected cells and prevent viral infection. This in...
Colony Stimulating factors:
Ig A
Cytotoxic/anti-body mediated hypersensitivity:
Antigen-antibody (IgG/IgM) complex
Active IMMUNITY
How is HIV transmitted?
Natural Immunity: Occurs when the person is exposed to a live pathogen, develops the disease, and becomes immune as a result of the primary immune response.
Flu vaccine: Influenza A&B vaccine (seasonal flu).
Hepatitis vaccine:
 Hepatitis A is given anyone over 1 year age
18 Chapter Biotechnology
Colony stimulating factors (CSF): Glycoprotein cytokines that promote proliferation, differentiation and activation of immune cells.
Interferon’s
Interleukins: synthesized by monocytes, macrophages, and lymphocytes. Interleukins are soluble messengers between leukocytes.
Hybridoma technology (Biologics)
Etanercept binds with both TNF alpha and beta: The greater risk of etanercept therapy is immuno suppression and subsequent serious infections.
Clotting Factors: Recombinant antihemophilic factor (rAHF). Indicated for treatment of classical hemophilia A. The dry concentrate of rAHF should be stored between 2 to 8o C and protect from freezing.
Human growth hormone (hGH)
Tips
19 Chapter Clinical Toxicology
GI Decontamination procedures
Gastric lavage or gastric gabage: This procedure can be used:
Syrup of Ipecac is administered within 60 min toxic dose ingestion (later has no benefit)
Decontamination
Diuresis: Promotes elimination acids and bases. This can be alkaline and acid diuresis.
Acid Diuresis: Used to promote elimination of weak bases. Example amphetamines, phencyclidines, quinidine derivative, and alkaloid drugs.
Methanol
Acetylcysteine
Q. Naloxone
Acetaminophen Overdose
Iron supplement overdose (Fe fumarate 33%, Fe sulfate 20%, Fe. gluconate 12%)
 Antidote is deferoxamine (mechanism: it works by chelation)
Benzodiazepine
Amoxicillin
20 Chapter Pharmacokinetics
Pharmacokinetics
A Absorption
Distribution: Volume of distribution is a hypothetical volume of body fluid that would be required to contain the entire drug administered so that the concentration will be the same as that found in the blood.
Factors affecting drug distribution
Albumin
K = elimination rate constant represents the fraction of drug eliminated per unit time.
Factors that affect renal clearance: As clearance is decreased half-life increases, changes in Vd cause proportional changes in half-life.
Plateau Principle
Clearance
t1/2 = 0.693 Vd/Clt
Summaries from the above formula
Formulas
21 Chapter Rates and Orders of Reactions
Example. Photochemical degradations
Zero order process application include administration of a drug as an intravenous infusion and controlled release dosage forms (SR, XL, MR) and Trans dermal patches.
First order Kinetics:
Slope –k/2.303
Third order reaction. Rate of reaction is proportional to concentration of each of the three reactions
Pseudo order. Rate of reaction is proportional to the concentration of only one reactant, in two-reactant reaction, if a reactant present in high concentrations.
 Example: Photochemical degradations
First order reactions
Slope –k/2.303
22 Chapter Pharmacodynamics
22
Pharmacodynamics
Competitive or non-competitive agonist and antagonist rates!
Type of drug interactions; Mathematical model
Therapeutic Window. The therapeutic window is the useful “opening” between the minimum therapeutic concentration and the minimum toxic concentration of a drug.
Enzyme inhibition. Chloramphenicol.
First order
Zero order
23 Chapter Medicinal Chemistry
Basics of Organic Chemistry: Organic chemistry is the study of substances that contain carbon, hydrogen, oxygen, nitrogen, sulphur etc. However, carbon is the essential element in organic chemicals.
Functional groups
Diastereoisomers. Two asymmetric carbon atoms. They are super imposable and are not mirror images.
24 Chapter Medicinal Chemistry and Pharmacology of Autonamic drugs ver1
Generally pharmacological receptors can be categorized into 4 types
 Glutamic acid (glutamate) excitatory: NMDA receptor. Example. Memantine
Cholinergic
Adrenergic
Ion channels: There are two types of ion channels
 Voltage gated: Na+ channel, Ca2+ channel, K+ channel
Transcription regulators: There are over 150 receptors of this family, which acts as transcriptional receptors. This is second major class of receptors, which include steroid hormones including estrogens, androgens, and the glucocorticoids such as cor...
Norepinephrine
Dopamine
Glycine
Glutamate
Reversible
Reversible
Dehydroxylation of pseudoephedrine gives methamphetamine
Tertiary amines
25 Chapter Medicinal Chemistry and Pharmacology of Histamines, Serotonins, Leukotrienes, NSAIDs
Histamines: The histamine is produced from mast cells. Histamines act on three receptors, of these H1, H2 receptors are excitatory and H3 receptors.
Pharmacology H1-antihistamines
H2 antagonist chemistry
Has C-O-C, C-N bonds
Ethanol amines:
Diphenhydramine
Piperizines
2nd Generation (AM)
Piperazine derivatives
NO anticholinergic effect
Less sedation due to less lipid soluble and do not penetrate the BBB.
Non selective
Ergotamine (DHE) are 5HT2 agonist.
Antagonist of 5HT2a atypical antipsychotic olanzapine
Agonist
Triptans
PGE analogs
PGI analogs (prostacylcin) chemistry
Thromboxanes (TxA2).
Chemistry of Salicylates derivatives
Onset: 5-30 min, duration 3-6 hours
Analgesic action
Epoprostenol
Dinoprostone
Dipyridamol inhibits platelet aggregation
Latano"prost"
Antipyretic action
Antiplatelets action
Irreversible platelet inhibition and inhibition of Cox-I & Cox-II action gives antiplatelets action. Antiplatelets action minimum dose is 60 to 80 mg.
Problems associated with NSAID and acetyl salicylic acid.
 Respiratory depression: Toxicity respiratory alkalosis and metabolic acidosis (increase CO2 and decrease pH).
Oxicams derivatives side effects
Glutathione is consisting of glutamic acid, cysteine, and glycine
Phenyl piperidine Derivatives
Morphinan Derivatives
Natural opium alkaloids
26 Chapter Medicinal Chemistry and Pharmacology of Cardiovascular Drugs
Thiazide diuretics chemistry: Benzene ring with sulfonamide in position 7, and halogen or trifluoro methyl group in position 6. The electron withdrawing group at position 6 is necessary. Saturation of 3, 4 double bonds (increases potency with hydroch...
Thiazide diuretics pharmacology
Thiazide diuretics therapeutic uses
Thiazides diuretics side effects (Hyper GLUC)
Loop diuretics pharmacology
Loop diuretics side effects: [OH DANG] Ototoxicity, Hypokalemia, Dehydration, Allergy (sulfa), except ethacrynic acid, Nephritis (intestinal), and Gout arthritis.
Carbonic anhydrase inhibitor chemistry: Acetazolamide. Aromatic or heterocyclic sulfonamides with a thiadiazole ring.
Carbonic anhydrase inhibitor therapeutics: Used in treatment of glaucoma (not chronically). Acute mountain sickness (respiratory alkalosis) also high altitude sickness (mountain sickness). Because of the alkaline diuresis it produces, acetazolamide ha...
Carbonic anhydrase side effects
Osmotic diuretics pharmacology
Osmotic diuretics therapeutic use
Potassium sparing diuretics: Spironolactone, eplerenone, amiloride, and triamterene
Potassium sparing diuretics chemistry
Potassium sparing diuretics pharmacology
Potassium sparing diuretics therapeutic use
Potassium sparing diuretics side effects
Vasodilators
ACE Inhibitors therapeutic use
ACE Inhibitors side effects
Antihyperlipidemic Drugs
Antihyperlipidemic drug chemistry. Categorized into non-absorbable agents and absorbable agents.
Cholestyramine chloride
Colestipol hydrochloride
Absorbable agents
Administered orally, it converted in body to nicotinamide, NAD+ and NADP+. The later two are coenzymes essential for biochemical oxidation, reduction reactions. Participation in tissue respiration oxidation, reduction reactions, which decreases hepat...
Statins
HMG-CoA reductase ( Mevalonic acid(cholesterol
Antianginal agents
Statins structure activity?
Pharmacology of nitrates
Nitrates therapeutic use: Relieve acute anginal attacks, as prophylaxis, for long-term management of recurrent angina pectoris.
Nitrates side effects
Calcium blockers pharmacology: These agents are used to treat hypertension and are effective in treating angina as well. All muscles, including the smooth muscle of the blood vessels, require calcium inorder to contract. If the CCB block the entrance ...
Dihydropyridine
Non-dihydropyridine side effects
Anticoagulants classification
Anticoagulants chemistry
Anticoagulant side effects
Thrombolytic chemistry
Thrombolytic pharmacology
Thrombolytic side effects
Antiplatelet drugs pharmacology
Antiplatelet drugs therapeutic use
Antiplatelet drugs side effects
 Dobutamine, isoproterenol ( ↑SBP & ↓ DBP
27 Chapter Medicinal Chemistry and Pharmacology of CNS drugs
Irreversible MAOi
Non-selective
5HT only
Dual action
SNRI
NDRI
Antidepressants
Tricyclic antidepressants (TCA) pharmacology
Tricyclic antidepressants (TCA) therapeutic use
Tricyclic antidepressants (TCA) side effects
Selective Serotonin Reuptake Inhibitors therapeutic use
Antipsychotic drugs pharmacology
CNS stimulants: Methylphenidate, amphetamines
Benzodiazepine pharmacokinetics
Methamphetamine have one C-methyl and one N-methyl
Benzodiazepine pharmacology
Benzodiazepine therapeutic use
Barbiturates chemistry
Barbiturates Therapeutic Use
Drugs to treat Parkinson’s disease
Anti-Parkinson’s drugs chemistry
Thiopental
Anti-epileptics chemistry
GABA analogs: Gabapentin, vigabatrin, pregabalin and baclofen (a muscle relaxant).
Local anesthetics pharmacology
General anesthetics pharmacology
 Venlafaxine at higher dose act on (
28 Chapter Medicinal Chemistry and Pharmacology of drugs that act on hormones
Antidiabetic drugs pharmacology
First generation: Tolbutamide, Chlorpropamide,
Third generation: Glimepiride
Antidiabetic drug Therapeutic uses
Orlistat (Xenical). Reduces fats stores and produce weight loss. It is an intestinal lipase inhibitor.
Antidiabetic drugs Side effects
Thyroid disorders
Medicinal chemistry of thyroid disorders
Thyroid hormones drug interactions
Thyroid hormones monitoring
Antithyroid drugs Pharmacology
Antithyroid drug mechanism:
Mechanism of antithyroid drugs
Therapeutic uses
Antithyroid drugs Contraindications / Precautions
Antithyroid drugs Pharmacokinetics
Propylthiouracil
Estrogen chemistry: Ovaries produce 17 estradiol and estrone. These hormones have 18 carbons for four rings. Three 6 membered rings and one 5-member ring. Estrogen exist as estradiol in body in equilibrium with oxidized form of estrone and further bi...
Estrogen Pharmacology
Estrogen therapeutic uses
Testosterone
Progesterone
Estrogen
Testosterone
Estrogen antagonist
Progesterone antagonist
Two types of progesterone
Progesterone therapeutic use
Progesterone side effects
Anti Progesterone pharmacology
Androgen pharmacology
Androgen side effects
Anti-androgens pharmacology: Inhibit the synthesis of androgen.
Anti-androgen side effects
29 Chapter Medicinal Chemistry and Pharmacology of Respiratory Drugs
Asthma
Long acting beta2 agonist (LABA) (Maintenance or daily): In asthma LABA combined with ICS.
 Therapeutic use. Maintenance therapy and EIA. Used in patients already taking corticosteroids. Formoterol can be used for acute and maintenance.
Corticosteroids
Oral corticosteroids (Po CST): Prednisone and prednisolone.
Drug used for the treatment of COPD: Anticholinergics. Ipratropium and tiotropium are muscarinic blocker and act as bronchodilator. Beta adrenergic agonists, Corticosteroids and theophylline.
30 Chapter Medicinal Chemistry and Pharmacology of Musculoskeletal drugs
Rheumatoid arthritis
Gout Arthritis
Acute gout arthritis
Tips
31 Chapter Medicinal Chemistry and Pharmacology of Antimicrobials
R = Substitution of R effects in solubility's salts are given orally, R = benzyl penicillin = Pen G. R = Phenoxymethyl = Pen V
Macrolides: Erythromycin, clarithromycin and azithromycin.
Inhibition of DNA gyrase (topoisomerase II) and topoisomerase IV makes cell inaccessible and leads to cell death. Different quinolones inhibit different extent of topoisomerase II and IV. DNA gyrase seems more important in gram –ve. Topoisomerase IV i...
Nitrofurantoin
Chloramphenicol
Lincosamides
Macrolides
Erythromycin
Tetracycline's
Macrolides Tips
Tetracyclin Tips
Clindamycin Tips
Quinolone and fluroquinolone Tips
Metronidazole Tips
Sulfadrugs Tips
32 Chapter Drug Metabolism
Oxidative metabolism
Alcohol oxidations
Hydrolysis
Deamination: Examples of deamination include amphetamine and dopamine.
Dehalogenation. (Halothane, Chloramphenicol)
Factors affecting Drug Metabolism
Enzymes
Glucuronidation (addition of glucuronic acid)
Nefazodone
Phenobarbital
Grapefruit
Carbamazepine
PHASE 2 REACTION
Genetic factors: Acetylation rate by N-acetyltransferase, which may result in fast acetylators or slow acetylators (pharmacogenetics).
Tips
33 Chapter Biopharmaceutics
Bioavailability and Bioequivalence
A drug is hydrophobic if partition coefficient is >1
Mechanism of drug absorption.
Transport process across membrane
 Powder [(dispersed + GI fluid → wet) absorbed]
Degree of ionization: Drugs will pass thru a membrane at a faster rate if they are unionized. The size of an ion increases due to dipole-to-dipole attraction especially water. Ionized portion of drug is less soluble in lipid but more in water.
Solubility
Surface area (Ficks law of diffusion)
Supersaturated Solution: Contains more dissolve solute then it normally would contain at a specific temperature if there were undissolved solute present. If upon cooling, the excess solute fails to crystallize from the lower temperature, the solution...
Types of Solvents
Polar solvents
Semi polar solvents
Factor Affecting solubility and Rate of Solution
Effect of Temperature
Effect of electrolytes on the solubility of non electrolytes
Action potential across cell membrane: Neuronal excitability depends on the influx of ions through specific channels in membranes.
Membrane depolarization: Excitation = increase Na entry (influx), decrease K exit (efflux)
Tips
 Write sequence of absorption for oral dosage, from higher to lower: solution > suspension > liquid gel caps > powder>
34 Chapter Pharmaceutics
State of the matter
Sublimation is?
Interfacial Phenomena
Wetting Phenomena: A solid is said to be wetted by a liquid if the liquid spontaneously spreads over the solid. A solid is not wetted by a liquid if the latter cannot spread over the former spontaneously. The contact angle is an important parameter ...
Methyl alcohol (methanol) and isopropyl alcohol, ethylene glycol is toxic, and butyl and amyl alcohol are irritating. Volatile ethers paralyse the central nervous system, and are irritating to mucous membrane increases. Ketones are mildly irritating a...
London forces in molecules are weak intermolecular forces in liquid hydrocarbon are not true chemical bonds.
Critical solution temperature: It is the maximum temperature above which homogenous liquid is formed regardless to any concentration of phenol.
Wetting Phenomenon?
Chemical kinetic & Drug stability
Change pH effect on degradation of drugs. The magnitude of the rate of hydrolytic reaction catalyzed by acid (H+) and base (OH-) can change with pH. Acid (H+) catalysis predominates at lower pH, whereas base (OH-) catalysis operates at higher pH.
Modes of pharmaceutical degradation:
Buffers and Buffer Calculations: A buffer is a compound or a mixture of compounds that has the ability to resist changes in pH when limited amounts of acid or base are added to the solution of the buffer or when the solution is diluted with solvent.
1) Examples of colligative properties?
 Hypotonic. The clinical significance of all this is to insure that isotonic or iso-osmotic solutions do not damage tissue or produce pain when administered.
35 Chapter Pharmaceutical Exipients
Sterile water for injection USP
Bacteriostatic water for injection USP
Packed in single or multiple use dose containers.
Sterile purified water USP
Triethnolamine
Ethyl paraben
Sodium benzoate
Benzalkonium chloride
Benzyl alcohol
Phenol
Thimerosal
Antioxidant
Ascorbic acid
Ferric oxide gives red color
Sodium lauryl sulfate
Gelatin
Humectants
Surfactant
Filler/
diluents
Binder
Disintegrant
Glidants
Lubricant
36 Chapter Rheology
Non-Newtonian flow is characterized into three types: plastic, pseudo plastic and dilatants.
Plastic Flow
Dilatants flow
Anti-thixotropy systems (RHEOPEXY): Products that exhibit opposite action of thixotropy are referred to as anti-thixotropy. Anti-thixotropy occurs when solutions to gel transformation. Example dilatants flow.
Tips
37 Chapter Pharmaceutical dosage forms
Suppository
Capsules
Lozenges
External Use
Inhalants
Miscellaneous
Emulsions
Solid Dosage
Friability
Question Alerts!
Trituration
Levigation
Spatulation
Eutectic mixtures
Suppository Bases
Physical and chemical properties of suspension
Purpose. Sustaining effect it necessitates drug dissolution prior to absorption.
Sedimentation: The Stoke’s law can express the relationship of the rate of sedimentation with various parameters.
LARGE PARTICLE SIZE
( SEDIMENTATION
HIGH VEHICLE DENSITY
( SEDIMETATION
Clays. Bentonite, veegum
Liquid Dosage Form
Water and lipid soluble base. Poly ethylene glycol (PEG)
Examples
Mechanism
Spirits
38 Chapter Drug Delivery Systems
Intravenous
IM
Intraarticular
Parenteral preparations
Vaginal products
Controlled/targeted delivery:
Controlled delivery systems
Technological methods
Liposomes for drug delivery systems
39 Chapter Pharmaceutical Analysis
Gas
Column
Planar
GSC
GLC
HPLC
Paper
Parameter that affects resolution.
GLC (Gas Liquid Chromatography)
 Stationary phase is liquid surface on solid
GSC (gas solid chromatography)
TLC Spray Reagent
Paper Chromatography
 Stationary phase. The stationary phase is cellulose paper (paper is made from cotton fibres and highly purified about 90% alpha cellulose).
Question Alerts!
Ultra Violet/Visible light spectrophotometer:
 Ultraviolet radiation 140 mm Hg, a diastolic blood pressure >90 mm Hg. Diagnosis criteria from joint national committee (JNC8) report recommendations for follow up in adults.
Calcium channel blockers (CCBs)
86 Chapter Ischemic Heart Disease
NSTEMI
STEMI
Tips
87 Chapter Stroke
Stroke
88 Chapter Congestive Heart Failure
Causes of CHF. In 65% of patient’s coronary artery disease is the cause of heart failure, other causes include nonischemic cardiomyopathy example hypertension, thyroid disease or valvular disease. These patients usually have reduced left ventricular d...
Diagnostic tests
ACE Inhibitors (captopril, enalapril, lisinopril, ramipril, and trandopril)
Beta blockers (Bisoprolol and carvedilol, labetalol, Metoprolol)
Digoxin
Tips
89 Chapter Anti Arrhythmic Drugs
Supraventricular (atrial) SVA Ventricular arrhythmias (VA)
praventricular (atrial) SVA treatment. Rate control drugs
 Cardiac glycoside (digoxin), Beta blockers (Propranalol, atenolol, metoprolol, nadolol), CCBs (verapamil and diltiazem), Antiarrhythmic class 1C: flecainide, propafenone; Antiarrhythmic Class III: Sotalol, Amiodarone, and dofetilide
Ventricular tachycardia treatment
 Class 1A (quinidine, procainamide); Class 1B (mexiletine); Class 1C (flecainide, propafenone); Class III (sotalol, Amiodarone); Beta blockers (metoprolol)
Antiarrhythmic Drugs Classification
Class III
Class I
Quinidine
Procainamide
Disopyramide
Ib: Lidocaine
Class IV
Verapamil
Diltiazem
90 Chapter Peripheral Vascular Diseases
91 Chapter Chronic Pain Conditions
91
Chronic Pain Conditions
92 Chapter Anxiety Disorders
SSRIs
Azapirones
5HT1a agonist
93 Chapter depression
 Caution: when starting other antidepressants after fluoxetine discontinuation.
Tips
94 Chapter psychosis and schizophrenia
94
Anti Psychotic Drugs
High potency
 First line treatment
Risperidone:
Negative Symptoms
Disorganizations. Disorganized speech
Disorganized behavior: Agitation, Hostility
95 Chapter Dementia ver1
Mechanism
96 Chapter Antiseizure or epileptic drugs
96
Epilepsy
Generalized seizures
97 Chapter Parkinson’s Disease ver2
98 Chapter Antimicrobial drugs
S. aureus
Bacterial Conjunctivitis
Canaliculitis
Skin and Soft Tissue Infections
Impetigo
Cellulitis
Strep. pyogenes and/or Staphylococcus aureus
Impetigo
Follculitis, boil (furuncles) and carbuncles
S. aureus
Erysipelas
Necrotising fasciitis
Neisseria meningitis
Bacterial infection of kidney substances.
Pseudomembranous colitis
Infection of Skin by fungi
Infection of the CNS
Infections of the joint and bones
LOWER RESPIRATORY TRACT INFECTIONS
Tetracyclins = Take empty stomach
SKIN INFECTIONS
CNS INFECTIONS
URINARY TRACT INFECTIONS
SEXUALLY TRANSMITTED INFECTIONS
Osteomyelitis
99 Chapter Anticancer Drugs and Chemotherapy
Chemotherapy
GI toxicity
Hepatotoxicity
Hemorrhagic cystitis
999 Chapter Pharmacognosy and Natural Products ver1
Cranberry
Dong Quai
Echinacea
Evening Primrose
Garlic
Licorice
Ginger
Ginkgo Biloba
Ginseng
9999 Abbreviations et al
Abbreviations
Dys-
Above; excess
Recommend Papers

Evaluating Exam Review Book

  • 0 0 0
  • Like this paper and download? You can publish your own PDF file online for free in a few minutes! Sign Up
File loading please wait...
Citation preview

www.pharmacyprep.com

Evaluating Exam Review Book 2018

Pharmacy Prep

Evaluating Exam Review Book

Misbah Biabani, Ph.D

Toronto Institute of Pharmaceutical Sciences (TIPS) Inc. Toronto, ON M2N 6K7

2018 Pharmacy Prep

Professional Exams Preparation Center 4789 Yonge St. Suites # 417, Toronto, ON, M2N 5M5 WWW.PHARMACYPREP.COM 416-223-PREP (7737)/647-221-0457 Toronto Institute of Pharmaceutical Sciences Inc. © 2000 to 2018 TIPS Inc. All Rights Reserved. Copyright © 2000-2018 TIPS Inc. Unauthorized reproduction of this manual is strictly prohibited and it is illegal to reproduce without permission. This manual is being used during review sessions conducted by PharmacyPrep.

1

www.pharmacyprep.com

Evaluating Exam Review Book 2018 Disclaimer

Your use and review of this information constitutes acceptance of the following terms and conditions: The information contained in the notes intended as an educational aid only. It is not intended as medical advice for individual conditions or treatment. It is not a substitute for a medical exam, nor does it replace the need for services provided by medical professionals. Talk to your doctor or pharmacist before taking any prescription or over the counter drugs (including any herbal medicines or supplements) or following any treatment or regimen. Only your doctor or pharmacist can provide you with advice on what is safe and effective for you. Pharmacy prep make no representation or warranty as to the accuracy, reliability, timeliness, usefulness or completeness of any of the information contained in the products. Additionally, Pharmacy prep does not assume any responsibility or risk for your use of the pharmacy preparation manuals or review classes. In our teaching strategies, we utilize lecture-discussion, small group discussion, demonstrations, audiovisuals, case studies, written projects, role play, gaming techniques, study guides, selected reading assignments, computer assisted instruction (CAI), and interactive video discs (IVD). Our preparation classes and books are not intended as substitute for the advise of NABPLEX®. Every effort has been made to ensure that the information provided herein is not directly or indirectly obtained from PEBC® previous exams or copyright material. These references are not intended to serve as content of exam nor should it be assumed that they are the source of previous examination questions. ©2000-2018 TIPS Inc. All rights reserved. Foreword by Misbah Biabani, Ph.D Coordinator, Pharmacy Prep Toronto Institute of Pharmaceutical Sciences (TIPS) Inc 4789 Yonge St. Unit 415-417 Toronto ON M2N 6K7, Canada

Copyright © 2000-2018 TIPS Inc. Unauthorized reproduction of this manual is strictly prohibited and it is illegal to reproduce without permission. This manual is being used during review sessions conducted by PharmacyPrep.

2

www.pharmacyprep.com

Evaluating Exam Review Book 2018

Content Abbreviations

Part 1. Biomedical Sciences 15% 1. Human Anatomy 2. Gastrointestinal System 3. Nervous System 4. Cardiovascular System 5. Endocrine System 6. Renal System 7. Liver Function and Pathophysiology 8. Respiratory System 9. Urinary System 10. The Eye and Ear 11. Blood and Anemia 12. Biochemistry 13. Nutrition 14. Microbiology 15. Cell and Molecular Biology 16. Pharmacogenetics 17. Immunology and Immunizations 18. Biotechnology 19. Toxicology Part 2. Pharmaceutical Sciences 25% 20. Pharmacokinetics 21. Rates and Orders of Reactions 22. Pharmacodynamics 23. Basics of Medicinal Chemistry 24. Medicinal Chemistry and Pharmacology of Autonomic Nervous System Drugs. 25. Medicinal Chemistry and Pharmacology of Histamines, Serotonin, Prostaglandin and Non-Steroidal Anti-inflammatory Drugs

Copyright © 2000-2018 TIPS Inc. Unauthorized reproduction of this manual is strictly prohibited and it is illegal to reproduce without permission. This manual is being used during review sessions conducted by PharmacyPrep.

3

www.pharmacyprep.com

Evaluating Exam Review Book 2018

26. Medicinal Chemistry and Pharmacology of Cardiovascular Drugs 27. Medicinal Chemistry and Pharmacology of Psychiatric & Neurological Drugs 28. Medicinal Chemistry and Pharmacology Endocrine Drugs 29. Medicinal Chemistry and Pharmacology of Respiratory Drugs 30. Medicinal Chemistry and Pharmacology of Musculoskeletal Drugs 31. Medicinal Chemistry and Pharmacology of Antimicrobial Drugs 32. Drug Metabolism 33. Biopharmaceutics 34. Physical Pharmacy 35. Pharmaceutical Excipient 36. Rheology 37. Pharmaceutical Dosage Forms 38. Drug Delivery Systems 39. Pharmaceutical Analysis Part 3. Social/Behavioural/Administrative Sciences 10%

40. Canadian Healthcare System 41. Canadian Pharmacy Law and Jurisprudence 42. Pharmacist Scope of Practice in Canada 43. Pharmacy Management 44. Pharmacoeconomics 45. The New Drug Approval Process 46. Evidence Based Medicine and Epidemiology 47. Biostatistics 48. Hospital Pharmacy

Part 4a. Pharmacy Practice (50%)

Professional Practice Skills (15%)-workflow

49. Pharmacy Calculations. Basics 50. Pharmacy Calculations. Dose Calculations 51. Pharmacy Calculations. Dilutions and Allegations 52. Brand and Generic Name Indexes Copyright © 2000-2018 TIPS Inc. Unauthorized reproduction of this manual is strictly prohibited and it is illegal to reproduce without permission. This manual is being used during review sessions conducted by PharmacyPrep.

4

www.pharmacyprep.com

Evaluating Exam Review Book 2018

53. Prescription Processing and Medication Dispensing 54. Safety of Medications in Special Populations 55. Promoting Medication Adherence 56. Professional Pharmacy Communication Skills 57. Bioethics and Professional Ethics 58. Drug Information Resources and Literature Evaluation 59. Medication Errors and Patient Safety Practices 60. Health Promotion and Disease Prevention 61. Collaboration and Teamwork 62. Sterile Preparations 63. Compounding and Storage Conditions Part 4b. Pharmacy Practice-Clinical Pharmacy (35%) 64. Pharmaceutical Care and Drug Related Problems 65. Adverse Drug Reactions and Management 66. Drug Interactions 67. Clinical Biochemistry and Therapeutic Drug Monitoring 68. Quality Assurance in Pharmacy Practice 69. OTC and Prescription Drugs for Dermatological and Foot Conditions 70. OTC and Prescription Drugs for Ophthalmic, Ear and Mouth Disorders 71. OTC Drugs Antihistamine, Decongestants, Antitussives, Expectorants 72. OTC Drugs for Nausea, Vomiting, Constipation, Diarrhea, Hemorrhoids 73. Analgesics, and Topical Pain Relievers 74. Asthma and Chronic Obstructive Pulmonary Disease (COPD) 75. Smoking Cessation 76. Insomnia 77. Eating Disorders 78. GERD, Ulcers, Inflammatory Bowel Disease (IBD) and Irritable Bowel Syndrome (IBS) 79. Diabetes Mellitus Type I and Type 2 80. Thyroid Disorders 81. Contraception 82. Gynaecologic and Genitourinary Disorders Copyright © 2000-2018 TIPS Inc. Unauthorized reproduction of this manual is strictly prohibited and it is illegal to reproduce without permission. This manual is being used during review sessions conducted by PharmacyPrep.

5

www.pharmacyprep.com

Evaluating Exam Review Book 2018

83. Rheumatoid Arthritis, Osteoarthritis and Gout arthritis 84. Osteoporosis 85. Hypertension 86. Coronary Artery Diseases 87. Stroke 88. Congestive Heart Failure 89. Cardiac Arrhythmias 90. Peripheral Vascular diseases 91. Neurological Disorders and Pain Management 92. Anxiety Disorder 93. Depression 94. Psychosis and Schizophrenia 95. Dementia 96. Seizures and Epilepsy 97. Parkinson’s Disease 98. Antimicrobial Agents 99. Anticancer Drugs and Chemotherapy 100. Pharmacognosy and Natural Products

Copyright © 2000-2018 TIPS Inc. Unauthorized reproduction of this manual is strictly prohibited and it is illegal to reproduce without permission. This manual is being used during review sessions conducted by PharmacyPrep.

6

Pharmacyprep.com

Human Anatomy

1 Human Anatomy Questions Alerts! Common questions in pharmacy exam is to ask! · Anatomy of body movements like abduction, adductions, supine and prone. · Anatomical planes such as sagittal and midsagittal plane. · Skeletal bones and joints. Patella (kneecap), hip joints or bowl and socket (ilium, ischium, pubis), skull bones, knee joints have popliteal spaces. · Muscles. Flexor and Extensor muscles, Actin and myosin muscle fibers for muscle contraction, masseter muscles are attached to mandibles. This chapter reviews essentials and definitions of systemic human anatomy terminology and provide a basic understanding of how the human body is structured with emphasis on clinical applications. This chapter also reviews cellular mechanism in human physiology. A special emphasis is on drug-induced diseases and effects of adverse drug reactions on various organs.

Body Movements (Fig 1.1) · · · · · · · · · · · · · ·

· ·

Abduction: Movement away from the midline of the body. Adduction: Movement toward the midline of the body. Extension: Lengthening or straightening of a flexed limb. Flexion: Bending of a part of the body. Dorsiflexion: Backward (upward) bending of the foot. Plantar flexion: Bending of the sole of the foot downward toward the ground. Pronation: Act of turning the hand so that the palm faces downward. Supination: Act of turning the hand so that the palm is uppermost. Eversion: Outward turning. Fascia: Fibrous membrane separating and enveloping muscles. Anterior (ventral): Front side of the body (example: Abdomen is anterior to the spinal cord). Posterior (dorsal): Back of the body (example. Spinal cord is posterior to the stomach). Lateral view = from the side of the body Medial view = from the middle of body (between two legs)

Deep: Away from the surface. Superficial: On the surface (example. Superficial veins can be viewed through skin).

Copyright © 2000-2018 TIPS Inc. Unauthorized reproduction of this manual is strictly prohibited and it is illegal to 1-1 reproduce without permission. This manual is being used during review sessions conducted by PharmacyPrep.

Pharmacyprep.com ·

Human Anatomy

Proximal: Near the point of attachment to the trunk or near the beginning of a structure (Example. The proximal end of the stomach is at the esophagus or the proximal end of the upper bone joins with shoulder bone). Fig 1.1

Distal: Far from the point of attachment to the trunk or from the beginning of a structure (Example. The distal end of the stomach is at the small intestine). Inferior. Below another structure. Caudal (pertaining to the head) means inferior in human. (Example. The urinary bladder lies inferior to the kidney)

opposite Abduction Extension Lateral Dorsal Proximal

opposite Adduction Flexion Medial Ventral Distal

Copyright © 2000-2018 TIPS Inc. Unauthorized reproduction of this manual is strictly prohibited and it is illegal to 1-2 reproduce without permission. This manual is being used during review sessions conducted by PharmacyPrep.

Pharmacyprep.com

Human Anatomy

Anatomical planes (Fig 1.1) · Sagittal plane: Plane created by an imaginary line that is parallel to the median plane. Separates the body into right and left portions. · Midsagittal plane (median plane): Plane created by an imaginary line that divides the body into right and left halves. Separates the body/body part into equal right/left portions. · Parasagittal plane: Divides the body into unequal right and left portions. · Coronal plane/frontal. Divides the body/body part into anterior and posterior portions. · Transverse plane/horizontal. Divides the body/body part into superior and inferior portions. · Oblique plane: Passes through the body/body part at an angle. Anatomical positions Postural: Positions such as standing, sitting, lying down, turning right or left. Orthostatic: Standing upright and lying down supine. Fowler’s position: Seated position with back support (Head elevated) Trendelenburg position: Sleeping position with Feet elevated and head lowered. Skeletal Joints (Fig 1.2) Weight bearing joints Shoulder joint: consists of humerus, scapula, synovial membrane, articular cartilage, articular capsule, articular liquid, and ligament. Knee joint: consists of femur (longest bone), tibia, patella, meniscus, articular cavity, serous bag, and articular capsule & cartilage. Patella (knee cap) bone is present in knee joint. Popliteal spaces or nerves are present in knee joint. Hip joint (socket and ball): consists of Ilium, ischium, and pubis. Shoulder joint Knee joint Hip joint

Ball & socket (Rotary cuff: is a group of muscles that stabilize the shoulder) Hinge joint Ball &Socket

Cranial bones “ PEST OF”

The major skull bones include Cranial bones (8), Facial bones (14), Ossicles (ear bones) (3) Cranial bones (protect brain): Frontal bones, parietal, occipital, temporal, sphenoid and ethmoid bone.

Parietal (2), Ethmoid, Sphenoid, Temporal (2), Occipital, Frontal

Tennis elbow (lateral epicondylosis): inflammation and pain of outer side of elbow involving humerus and usu. This results from excessive use of forearm or twisting. Tendons: Connect muscle to bone Ligaments: Connect bone to bone Anterior cruciate ligament is connecting between three bones of knee such as thighbone (femur), shinbone (tibia) and kneecap (patella). Muscles: · Trapeziusà neck · Triceps brachià shoulder (anterior). Triceps are in only in arms. Copyright © 2000-2018 TIPS Inc. Unauthorized reproduction of this manual is strictly prohibited and it is illegal to 1-3 reproduce without permission. This manual is being used during review sessions conducted by PharmacyPrep.

Pharmacyprep.com · · · · · · · · · · · ·

Human Anatomy

Biceps brachià upper arm (biceps) and thighs. Femorus is present in back of thigh and leg. Quadriceps --> thighs Gluteus medium is in hip. Psoas muscleà hip Largest muscleàgluteus maximus (which forms part of the buttock). Fastest muscle isà eyelid elevator. Longest single muscleà Sartorius muscle (which stretches from the pelvis to below the knee (more than 15 inches or 40 cm long). Smallest muscle is stapedius (found inside the middle ear and less than 0.04 inch (1 mm) long). Strongest muscle: Masseter muscle (which elevates during mastication. It can exert a force equivalent in 100 kg (220 lbs). Muscles account for 40% of total body weight. Muscle fibers actin and myosin helps in muscle contraction.

Three types of muscle tissues Skeletal muscle (muscle attached to skeleton tendons, bones); Voluntary Smooth muscle (organs, stomach, blood vessels muscle): involuntary Cardiac muscle (Heart muscle): involuntary Types of tissues and functions. Four basic types of tissues, epithelial (covering), connective (support), muscle (movement), and nervous (control/integration). · Epithelium: its functions include covering, secretion, absorption, and sensitivity. · Connective tissue: Support, cartilage, bone, blood, fibrous tissue of ligament (chondrocytes). · Muscle tissue, skeletal muscle tissue, cardiac muscle tissue, smooth muscle tissues. · Nervous: Control and integration. Tissue functions: Protection, absorption, filtration, excretion, secretion, and sensory reception. Epithelium tissue present at sites of rapid diffusion, such as the lining of lung alveoli. Endothelium tissue present in the lining of blood vessels (arteries, veins, capillaries). Mesothelium present at sites where very little activity is occurring, such as Bowman's capsule in the kidney and the lining of major body cavities. Epithelial tissue: Covering/lining or glandular, are 2 basic types endocrine "ductless" produce hormones. Exocrine have ducts, sweat, oil, saliva, bile enzymes, mucin (mucus).

Copyright © 2000-2018 TIPS Inc. Unauthorized reproduction of this manual is strictly prohibited and it is illegal to 1-4 reproduce without permission. This manual is being used during review sessions conducted by PharmacyPrep.

Pharmacyprep.com

·

Human Anatomy

Connective Tissue: support protection, insulation, transportation. Characteristics, large extra cellular matrix. Four basic classes of connective tissue: · Connective tissue proper. Loose, adipose, areolar storage, support organs or vessels, Dense. Regular, elastic (tendons and ligaments). · Cartilage. Cushion, structure, support, and laid down before bone. · Osseous (bone): Bring in beef bone, compact, rigid, and spongy marrow. · Blood: RBCs, WBCs, and platelets, and plasma matrix.

Copyright © 2000-2018 TIPS Inc. Unauthorized reproduction of this manual is strictly prohibited and it is illegal to 1-5 reproduce without permission. This manual is being used during review sessions conducted by PharmacyPrep.

Pharmacyprep.com

Human Anatomy

Tips 1 4 7 10 13 16 19 22 25

Supination 2. flexion Extension 5. adduction Sagittal plane 8. Midsagittal plane Biceps bronchi 11 Triceps bronchi Joints 14 Hormone glands Extensor muscles 17 Flexor muscles Ilium 20 Ischium Endocrine glands 23 Arteries Moving away from 26 Moving closer to body body 28 slicing vertically from 29 Slicing vertically from side middle line lines 31 dysuria 32 Dyspnea · Adduction à ( ) · Abduction à ( ) · Sagital planes à ( ) · Para sagital plane à( ) · Mid sagital plane à( ) · Transverse plane à ( ) · Hip joints have à ( ) · Flexor muscles are present inà ( ) · Extensor muscles are present inà ( ) · Epithelial tissue is present in à ( ) · Endothelial tissues is present in à ( ) · Skull bones are à??? · Movement away from the midline of the body ( ) · Act of turning the hand so that the palm is uppermost ( ) · bending part of the body ( ) · movement toward the midline of the body ( ) · lengthening or straightening of the flexed limb ( ) · found in arms and thighs ( ) · Found in arms only ( ) · Separates the body into unequal right and left portions ( ) · Separates the body into equal right and left portions ( ) · Separates the body into right and left portions ( ) · It protects the front of the joint ( ) · Difficulty in breathing ( ) · Difficulty in swallowing ( ) · Difficulty in urination ( ) · Found in limbs, foot, arms ( ) · Hip joints have ( ) · Which one is a part of the shoulder? ( ) · Popliteal space is present knee ( )

3. 6. 9. 12 15 18 21 24 27

Abduction Parasagittal plane Kneecap dysphagia Blood vessels Pubis transverse plane Skull bones slicing vertically

30

Slicing horizontal

Copyright © 2000-2018 TIPS Inc. Unauthorized reproduction of this manual is strictly prohibited and it is illegal to 1-6 reproduce without permission. This manual is being used during review sessions conducted by PharmacyPrep.

PharmacyPrep.com

Gastrointestinal System

2 Gastrointestinal System Questions Alerts! Common questions in pharmacy exam is to ask! · Stomach secretions (intrinsic factor, HCL, gastrin). Pepsin is digestive enzyme present in GIT break downs proteins. · Role of small intestine in absorption of nutrients, drugs and supplements · Large intestine (colon) bacteria and excessive absorption of water that cause constipation. · Disease of GI system like GERD, peptic ulcers, Crohn's disease, ulcerative colitis and irritable bowel syndrome (IBS) symptoms.

This chapter review anatomy, physiology and pathophysiology of the gastrointestinal system, common disease that occurs in gastrointestinal tract. Mouth · Tongue has bony attachments (styloid process, hyoid bone) attached to the floor of the mouth by frenulum. · Posterior exit from mouth guarded by a ring of palatine/lingual tonsils. · Ducted salivary glands open at various points into the mouth. This process involves teeth (muscles of mastication move jaws) and tongue (extrinsic and intrinsic muscles). · Mechanical breakdown, plus some chemical (ptyalin, enzyme in saliva) secretion. · Saliva amylase does hydrolysis of starch and glycogen into maltose. Esophagus · The esophagus is about 10" long. · Food moves through esophagus by peristalsis.

Copyright © 2000-2018 TIPS Inc. Unauthorized reproduction of this manual is strictly prohibited and it is illegal to reproduce without permission. This manual is being used during review sessions conducted by PharmacyPrep.

2-1

PharmacyPrep.com

Gastrointestinal System

Stomach Question Alerts! 1) Intrinsic factor secreted from parietal cells deficiency cause? 2) Pernicious anemia should be treated by parenteral (SC/IM) vitamin B12. 3) Elderly persons have deficiency of vitamin B12 4) Gastrin is secreted from pyloric gland of stomach. 5) What are stomach secretions occur in response to protein diet? Gastrin and pepsin.

Fig 2.2 · · · · ·

Cardioesophageal sphincter guarding entrance from esophagus. Pyloric sphincter guarding the outlet is much better defined. Fundus, body and pylorus recognised as distinct regions. Stomach secretes both acid and mucus (for self protection). Surface area increased by rugae, which serves as temporary store for food.

Stomach Secretions

Purpose

Mucus Lubricant, protects surface from acid. Intrinsic factor Vitamin B 12 absorption (in small intestine ilium). Acid (H+) Kills bacteria, breaks down food, converts pepsinogen. Pepsinogen Broken down to pepsin (a protease) Gastrin Stimulates acid secretion (in response protein) *Deficiency of intrinsic factors causes a type megaloblastic anemia i.e. pernicious anemia.

Source

Mucus Cell Parietal cell Parietal cell Chief Cell G Cell

Gastric acid secretion mechanism. In the parietal cells CO2 and H2O are converted H+ and HCO3- catalyzed by carbonic anhydrase. The parietal cells secrete HCl into the lumen of the stomach and concurrently absorb HCO3into the blood stream. Gastric acid stimulations. Gastric acid production is stimulated by three mechanisms. Vagal stimulation. Vagal nerve innervates parietal cells and stimulates H+ secretion directly. Histamine release. Histamine is released from mast cells in the gastric mucosa and diffuses to nearby parietal cells.

Gastrin: It is released in response to eating a meal (protein), thus stimulates parietal cells to secrete H+.

Pathophysiology of gastric acid secretions causes gastric ulcer, duodenal ulcers and Zollinger-Ellison syndrome. Question Alerts! 1) What enzymes are released into small intestines? Pancreatic and bile secretions. 2) A patient with ileostomy, what oral dosage is NOT suitable? Oral drugs especially Sustain release (SR, CR MR CD) d f

Copyright © 2000-2018 TIPS Inc. Unauthorized reproduction of this manual is strictly prohibited and it is illegal to reproduce without permission. This manual is being used during review sessions conducted by PharmacyPrep.

2-2

PharmacyPrep.com

Gastrointestinal System

Gastric emptying Time: The caudad region of stomach contract to propel food into the duodenum. The rate of gastric emptying time is fastest if gastric content is isotonic. Fat inhibits gastric emptying time (i.e. increase gastric emptying time). Stomach ---à (Pyloric sphincter)-à Duodenum à Jejunum àileum Prokinetic drugs (metoclopramide, domperidone) decrease gastric emptying time

Small intestine: Consist of duodenum, jejunum, and ileum. Duodenum: First part of the small intestine, C-shaped 10" (inch) long and curves around the head of pancreas and the entry of common bile duct. · Highest drug absorption in the body takes place here. · Pancreases is a large glandular organ attached near the stomach. · Pancreas secretes intestinal enzymes (pancreatic lipase, amylase, protease), and these helps in the digestion of carbohydrates. · Bile secretions are bile salts, bilirubin, phospholipids, and cholesterol. Jejunum: It is 8 to10 feet long: The majority of food absorption takes place in the jejunum. Secretion · Secretin stimulates pancreas to produce watery fluid, high in bicarbonates concentration. · Pancreozymin stimulates pancreas to produce a viscous fluid low in bicarbonate concentration. Ileum: It is 12 feet long. Towards the end of the small intestine, accumulations of lymphoid tissue (Peyer’s patches) are more common here. Large Intestines: It is also known as colon. · Jejunum terminates at caecum. · Highest basic or pH. · Animals digest cellulose in colon. · The large intestine reabsorbs water then eliminates drier residues as feces. · Its primary purpose is to extract (absorbed) water from feces. · Colon consists of higher flora and fauna in GI tract 90 to 99% anaerobic bacteria. Example B. fragilis and C. difficle anaerobic and aerobic E. coli. · Colon bacterial produce vitamin K 2 (menaquinone). Diseases of the gastrointestinal system Diseases of the Mouth and Jaw · Oral thrush is caused by Candida albicans, and moniliasis. · Gingivitis (gum inflammation) is caused by Fusobacterium sp. · Stomatitis is Inflammation of mouth cavity. Herpes stomatitis caused by herpes infection, and aphthous stomatitis caused by oral hygiene or damage to mucus membrane. Disease of the Salivary Glands: Sjogren syndrome (dry mouth, dry eyes) is autoimmune disease, it is associated with rheumatoid arthritis. Copyright © 2000-2018 TIPS Inc. Unauthorized reproduction of this manual is strictly prohibited and it is illegal to reproduce without permission. This manual is being used during review sessions conducted by PharmacyPrep.

2-3

PharmacyPrep.com

Gastrointestinal System

Sialorrhea is an excessive secretion of saliva in infants, children, Parkinson’s disease. Can cause by mucosal irritation. Dyspepsia: Defined as pain or discomfort in the upper abdomen. Symptoms are nausea, fullness, early satiety, bloating or regurgitation. The dyspepsia could be due to esophagitis, GERD, peptic ulcer (GU or DU) 15-25%, Reflux esophagitis, 5-15%, gastric or esophageal cancer ( Proteins and oligopeptides ………………….> oligopeptide à amino acids (stomach) (small intestine)

Absorption of nucleic acid: · Nucleaseà Nucleic acid into nucleotide. · Ribonuclease à Hydrolyses RNA · Deoxyribonuclease à Hydrolyses DNA

Absorption of water (H 2 O): It is isosmotic in the small intestine and gallbladder. Absorption of Vitamins and Nutrients: Fat soluble vitamins (ADEK) are absorbed in small intestine along with other lipids. Vitamin B12 is absorbed in the ileum and that requires intrinsic factor. Absorption of calcium: Mainly occurs in small intestine, which assisted by active form of vitamin D3, 1, 25-dihydroxycholecalciferol, which is produced in kidney. Chronic renal failure or vitamin D deficiency results in inadequate intestinal Ca2+ absorption, causing rickets in children and osteomalacia in adults. The mechanism of calcium absorption is passive absorption.

Copyright © 2000-2018 TIPS Inc. Unauthorized reproduction of this manual is strictly prohibited and it is illegal to reproduce without permission. This manual is being used during review sessions conducted by PharmacyPrep.

2-6

PharmacyPrep.com

Gastrointestinal System

Absorption of Iron: It is absorbed as heme iron (iron bound to hemoglobin or myoglobin) or as free Fe2+. In intestinal cells, heme iron is degraded to Fe2+ and released. The free Fe2+ binds to apoferritin and is transported into the blood. The iron absorbed from small intestine in the form of ferrous Fe2+ Transferrin: Free Fe2+ circulates binds transferring and transports it from small intestine to its storage sites in the liver and from the liver to the bone marrow for the synthesis of hemoglobin. Innervations of GI tract. Autonomic innervations. Cholinergic: It is usually excitatory on functions of GI tract. It is carried via the vagus and pelvic nerves. · Vagus nerve innervates the esophagus, stomach, pancreases and upper large intestine · Pelvic nerve innervates the lower large intestine and rectum, and anus. Adrenergic · It usually inhibitory on the functions of GI tract · Direct post ganglion adrenergic innervations of blood vessels and some smooth muscles.

Tips Practice answering tips from table: 1. 4. 7. 10

diarrhea cramps 2 glucoses Alpha glucosidase

2. 5. 8. 11

13 16 19

Peptidase Trypsin Parenteral vitamin B 12 rye

14 17 20

constipation Proteases Colon 95-100% anaerobic bacteria Enterokinase Vitamin D 3 Alcohol dehydrogenase

23

oats

22 · · · · · · · · · · · · ·

3. 6. 9. 12

Bloating nuclease gluten present in cereal Fructose + glucose

15 18 21

Chymotrypsin Deficiency of intrinsic factors wheat

The most basic part of the GI tract ( ) Irritable bowel disease symptoms ( ) The proteins are digested by ( ) What converts nucleic acid into nucleotides ( ) The pernicious anemia is caused by ( ) What digest peptides into amino acids ( ) pernicious anemia is treated by ( ) What converts inactive trypsinogen into trypsin ( ) What enzyme oxidizes alcohol to aldehyde and acids ( ) What are the major bacteria present in colon ( ) Breakdown sucrose & starch to glucose ( ) Gluten is present in ( ) Allergic component in milk ( )

Copyright © 2000-2018 TIPS Inc. Unauthorized reproduction of this manual is strictly prohibited and it is illegal to reproduce without permission. This manual is being used during review sessions conducted by PharmacyPrep.

2-7

PharmacyPrep.com · · · · · · · · · · ·

Gastrointestinal System

Celiac is caused by ( ) Soya milk allergies due to ( ) A patient with chronic renal failure have deficiency of vitamin? ( Pernicious anemia is caused by à ( ) Pernicious anemia is treated by à ( ) Maltase breakdowns maltose to à ( ) Sucrase breakdowns sucrose to à ( ) Alcohol dehydrogenase: ethanol à acetaldehyde à acetic acid Irritable bowel symptoms (IBS) include à ( ) Active Vitamin D is à ( ) Bacteria in colon makes --> ( )

)

Copyright © 2000-2018 TIPS Inc. Unauthorized reproduction of this manual is strictly prohibited and it is illegal to reproduce without permission. This manual is being used during review sessions conducted by PharmacyPrep.

2-8

PharmacyPrep.com

Nervous System

3 Nervous System Questions Alerts! Common questions in pharmacy exam is to ask! · What section of brain controls voluntary and involuntary movements? · Blood brain barrier definition and functions · Peripheral nerves, radial nerves, ulnar nerves. Sciatica. Cranial nerves. · Types of Neurological disorders: Multiple sclerosis, Chronic spasticity, Bell's Palsy, Neuralgia, Seizures or epilepsy, Fibromyalgia, and Parkinson's disease. · Sciatica pain site is buttocks and back of thighs. · Causes of multiple sclerosis

Fig 3.1

Fig 3.2

Question Alerts! 1) Voluntary and involuntary movements are controlled by? 2) What section of brain coordination and control balance? Copyright © 2000-2018 TIPS Inc. Unauthorized reproduction of this manual is strictly prohibited and it is illegal to reproduce without permission. This manual is being used during review sessions conducted by PharmacyPrep.

3-1

PharmacyPrep.com

Nervous System

Nervous system divided into central nervous system and peripheral nervous system. The central nervous system consists of brain and spine. Brain · Cerebrum: largest section of brain and controls voluntary (Think and decide what to say) and involuntary movements (autonomic functions). · Brain stem: Is the posterior part of the brain consist of pons and medulla oblongata and mid brain. · Cerebellum: Controls balance (GAIT) and modifies body movements (Motor). Maintain body coordination and balance. · Spinal cord. Vertebral column, epidural space, meninges, spinal cord, dorsal vertebra, and spinal nerve. · Thalamus: affects sensory levels, awareness and alertness. Corpus Collasum connects and communicates between right and left hemisphere of brain. Mesolimbic pathways are present in? Frontal lobes Medulla oblongata is present in? Temporal lobe

Frontal lobe Motor Voluntary movements Planning, Initiation Spontaneity Language Language expression Eye movement

Cognitive Memory Problem solving Judgment Abstract Abstract thinking Executing functions

Frontal lobe: Motor, cognitive, behavioral and arousal. Temporal lobe (auditory, speech, memory information retrieval) Brain stem ( breathing, digestion, heart control, blood vessel control, alertness) Occipital lobe (visual reception, interpretation)

Behavior Personality Social and sexual Impulse control Mood and affect

Arousal Attention

Parietal lobe (processing sensory input, sensory discrimination, body orientation, somatic area). Wernicke’s area: In temporal lobe language comprehension. Vestibular system: Reflex adjustment of head, eyes and postural muscles provide a stable visual image and steady posture. Vestibular ocular reflexes. Nystagmus The direction of the nystagmus is defined as the direction of the fast (rapid eye) movement. Therefore, the nystagmus occurs in the same direction as the head rotation. Normally initial Question Alerts! rotation of the head causes the eyes to 1) Blood brain barrier is present at? move slowly in the opposite direction to 2) Drugs that cross BBB. Rifampin, Cefuroxime sodium, Cefotaxime, maintain visual fixation. Carabapenam, Atropine, physostigmine, Diphenhydramine, and ethanol. Meninges · The meninges are three concentric membranes that surround and protect brain and spinal cord. · The dura mater: outer most membrane. Copyright © 2000-2018 TIPS Inc. Unauthorized reproduction of this manual is strictly prohibited and it is illegal to reproduce without permission. This manual is being used during review sessions conducted by PharmacyPrep.

3-2

PharmacyPrep.com · ·

Nervous System

The arachnoid: middle layer, transparent, flexible The pia mater: inner layer, is fine, and delicate.

Cerebro Spinal Fluid (CSF): The CSF is outside of the brain and circulates through the cavities inside the brain called ventricles. Blood brain barrier: The blood brain barrier (BBB) is the barrier between cerebral capillary blood and cerebrospinal fluid (CSF). BBB is formed by capillary endothelial cells that line cerebral micro vessels form tight junctions and lacks large intracellular spaces. Further neural tissue covers capillaries. Together constitutes forms BBB. The CSF fills the ventricles and the subarachnoid space. Three functions of BBB · Protects brain from endogenous or exogenous toxins. It prevents escape of neurotransmitters from CNS into blood circulations. · Lipids soluble drugs cross faster than water-soluble (polar) drugs. In capillary lining of BBB have, enzymes such as monoamine oxidase (MAO), cholinesterase and some other enzymes. These enzymes prevent catecholamines, serotonin, and acetylcholine, to enter into brain. Peripheral Nervous System: All nerves of the body residing outside of the brain and spinal cord comprise the peripheral nervous system. Periphery can be divided into sensory (somatic) and autonomic. · Ulnar nerve: Passes through the shoulder, elbow to wrist. · Sciatic nerve runs through buttock, thighs down to foot. It divides into tibia and common fibular nerve. Which supplies the muscles of posterior thigh and all of the leg and foot. · Intercostal nerve is that anterior divisions of the thoracic spinal nerves. · Radial nerve runs through forearm, wrist to finger tips. It supplies to muscles of forearm. · Popliteal nerve. Passes in knee joint. · Axillary nerve (circumflex): It supplies the deltoid and teres minor muscles, shoulder joint, and skin on back of arm. · Phrenic nerve: connect from neck down to lungs. Phrenic nerve injuries can result in to brachial palsy. (phrenic nerve palsy) · Vagus nerve: A parasympathetic nerve innervate four organs liver, GI, Heart, and lungs. PERIPHERAL NERVE Radial nerve damage cause

COMMENTS Wrist drop

Ulnar nerve damage cause

Claw hand (small fingers hand contract)

CRANIAL NERVES CRANIAL NERVE: origin from brain and spread to facial function. Olfactory smell Optic vision Copyright © 2000-2018 TIPS Inc. Unauthorized reproduction of this manual is strictly prohibited and it is illegal to reproduce without permission. This manual is being used during review sessions conducted by PharmacyPrep.

3-3

PharmacyPrep.com

Nervous System

Oculomotor: Trochlear: Trigeminal: Abducens Facial Acoustic Glossopharyngeal:

Eye upward, medial, downward movement Eye down and in movement Touch forehead and cheek, clench teeth Look side to side Taste for the anterior 2/3 of tongue Hearing Posterior 1/3 of the tongue

Vagus Spiral accessory:

Defecation, slowed heart rate shoulder shrug

Hypoglossal:

tongue movement

Nerve Cell: Nerve cell consists of dendrite, cell body, axon, myelin sheath, and synapse. Question Alerts! Myelin sheath damage is associated with? MS

Fig 3.3 Pathology of neurological disorders

Fig 3.2

Degenerative diseases: Alzheimer’s, Parkinsonism, Multiple sclerosis, and ALS (Amyotrophic lateral sclerosis). Alzheimer’s: deficiency of acetylcholine Parkinson’s: deficiency of dopamine Multiple sclerosis: autoimmune or degeneration of myelin sheet ALS: unknown Copyright © 2000-2018 TIPS Inc. Unauthorized reproduction of this manual is strictly prohibited and it is illegal to reproduce without permission. This manual is being used during review sessions conducted by PharmacyPrep.

3-4

PharmacyPrep.com

Nervous System

Seizures or epilepsy: Excessive excitation of neurons due to disorderly inhibition of cortical neurons. Parkinson's disease: Decrease in dopamine or imbalance of dopamine negrostratial pathway. Suppressing (antipsychotic drugs) the dopamine cause extra pyramidal symptoms (EPS) Extra pyramidal symptoms (EPS is side effect of antipsychotic drugs) "Akathisia, Dystonia, Parkinsonism, Tardive Dyskinesia". These symptoms are the side effect of antipsychotic drugs like haloperidol. Akathisia = restlessness or cannot sit still. Parkinsonism: (“TRAP”) Tremors (hand shaking), rigidity, ataxis, postural instability, Dyskinesia = involuntary movement or shaking Dystonia = involuntary muscle spasm Tardive dyskinesia = involuntary movement of lips, tongue, and chewing motion. Ataxia = lack of muscle coordination in voluntary movements. Migraine Headache: Vasodilatation of intracranial extra cerebral blood vessels. Vertigo: False sensation of moving or spinning or object moving usually accompanied by nausea and loss of balance. Meniere's disease produces sudden episode attack of vertigo along with ringing in ears (tinnitus) and progressive deafness. Episodes can last from minutes to hours. Associated with nausea and vomiting. Betahistine is used for treatment. Chronic spasticity: Spasticity is an involuntary velocity dependant increase in muscle tone resulting in injury to motor pathway in brain or spinal cord. It is common in MS, stroke, spinal cord injury and cerebral palsy. It can impair feeding, dressing, bowel function, hygiene and gait. Bell's Palsy: Paralysis of lower motor neuron of facial nerve (effects on eye). It is often due to herpes simplex virus (HSV 1 ) infection causing inflammation and edema. Multiple Sclerosis: The multiple sclerosis (MS) is characterized by destruction of myelin sheet (demylenation) and axonal degeneration & loss in CNS. The MS is chronic and can be caused by autoimmune mediated action. Treatment: Interferons beta (first line therapy), glatiramer acetate (Immunomodulators similar to interferon beta), Mitoxantrone, natalizumab, Fingolimod (spingosine-1-phosphate receptor agonist), Teriflunomide, and laqinimod. Temperature regulation: The homeostatic mechanisms regulate body temperature (37.5 °C) or 98.6 °F Sympathetic nervous system innervate heat loss by vasodilatation and sweat production. Sympathetic nervous system innervate adrenal gland than increase metabolic rate. Thalamus à pituitary gland à thyroid à Increase metabolic rate. Hyperthermia = >38.2 °C or 100 °F High fever is defined >40.5 °C Hyperpyrexia (fever) = a fever >41.5 °C are rare Hypothermia = 40.6 °C Antipyretics (NSAIDs) reduce fever by inhibiting cyclooxygenase, this inhibits prostaglandin synthesis. Therefore, analgesics decrease set-point temperature. In response that cause heat loss in the form of sweating, vasodilatation. Diagnostic techniques. Electroencephalograph (EEG): The EEG consist of alternating excitatory and inhibitory synaptic potential in the pyramidal cells of the cerebral cortex. CT scan (computed tomography) of brain. Demonstrates generalized waves of spike and wave discharge. Cerebrospinal fluid (CSF) sample is taken by Lumbar puncture. FMRI. The functional MRI is used brain scanning.

Tips 1. 4

Sciatic nerve Tardive dyskinesia

7 1 0

Nissl substance it prevents escape of neurotransmitter from CNS into blood circulations

· · · · · · · ·

· ·

2 5 8 1 1

Blood brain barrier Protects brain from endogenous &exogenous toxins Multiple sclerosis lipid soluble drugs cross faster than H 2 O soluble drugs

3 6

Adrenal medulla Bradykinesia

9

Cerebrum

What is the barrier between cerebral capillary blood and cerebrospinal fluid (CSF) the CSF fills the ventricles & the subarachnoid space ( ) A CNS disease where the myelin sheath of motor neurons is degenerating or being destroyed, which interferes with neuronal impulses ( ) The nerve that pass through buttocks, thighs down to foot ( ) What part of brain controls voluntary and involuntary movements ( ) Inappropriate posture of neck, face and limbs is referred as ( ) Functions of blood brain barrier ( ) Slow movement ( ) The dark granular inside neuronal cell bodies ( ) Sciatica is à ( ) The longest and largest nerve is --> (

)

Copyright © 2000-2018 TIPS Inc. Unauthorized reproduction of this manual is strictly prohibited and it is illegal to reproduce without permission. This manual is being used during review sessions conducted by PharmacyPrep.

3-6

PharmacyPrep.Com

Cardiovascular System

4 Cardiovascular System

Question Alerts! Carotid artery supply blood to? Brain What arteries supply blood to eyes? external carotid artery

Copyright © 2000-2018 TIPS Inc. Unauthorized reproduction of this manual is strictly prohibited and it is illegal to reproduce without permission. This manual is being used during review sessions conducted by PharmacyPrep.

4-1

PharmacyPrep.Com

Cardiovascular System

Questions Alerts! Common questions in pharmacy exam is to ask! · Definitions and disease associated with thrombus, embolus, ischemia, aneurism, atherosclerosis, plaques, and edema. · Concept of depolarization and repolarization · Electrode potential curve (P wave is atrial depolarization, QRS is ventricular depolarisation, QT wave is mechanical contractions of ventricles). · Diagnostics. ECG. Electrocardiography, and Echocardiography and biological markers.

Conduction System of the Heart (fig 4.4)

Fig 4.4

Fig 4.5

Copyright © 2000-2018 TIPS Inc. Unauthorized reproduction of this manual is strictly prohibited and it is illegal to reproduce without permission. This manual is being used during review sessions conducted by PharmacyPrep.

4-2

PharmacyPrep.Com

Cardiovascular System

Blood flow sequence: Vena cava à right atrium à right ventricle à left pulmonary arteryà LUNGS à left pulmonary vein à left atrium à left ventricle à aorta à systemic circulation Septal defect: Ventricular septal defect is a hole in the wall separating the two lower chambers of the heart. Types of pacemakers Natural (main) pacemaker of heart is SA node. Latent pacemaker of heart is AV node, bundles of His and purkinje fibres. · Pulse direction’s SA node à AV node à Bundles His à Purkinje Important concept! fibres Depolarisation and repolarisation? Depolarization (inward current): Carrying +ve charge into cell Increase Na+ influx into cell Decrease K+ efflux out to cell Repolarization (outward current or hyper polarization). Take +ve charge out of cell Increase K+ efflux out to cell Increase Cl- influx into cell Myocardial action potential curve: Myocardial action potential curve reflects action potential, which describes electrical activity of five phases. This occurs in atrial and ventricular myocytes and purkinje fibers. · Phase 0: Rapid depolarization: Na+ enters the cell · Phase 1: Early rapid repolarisation: K+ leaves the cell · Phase 2: Plateau: Ca2+ enters the cell · Phase 3: Final rapid repolarisation: K+ pumped out of the cell · Phase 4: Slow depolarization: K+ inside the cell and Na+, Ca2+ outside the cell. Phase 1 to starting phase 3 is absolute refractor period or effective refractory period. The cell cannot respond to any stimuli. (NO action potential can be initiated). During Phase 3 is relative refractory period. The cell ability to respond stimuli increases or cell can respond to strong stimuli. Electrocardiograph Wave Forms: The electrical activity occurred during depolarization and repolarization transmitted through electrodes attached to the body and transformed by an electrocardiograph (ECG) in to series of waveforms. · P wave indicates atrial depolarization. · PR interval indicates the spread of the impulse from the atria through Purkinje fibres. (Beginning of initial depolarisation of ventricle). · QRS complex indicates ventricular depolarization. · ST segment indicates phase 2 of the action potential the absolute refractory period. · T wave shows phase 3 of the action potential ventricular repolarization. Copyright © 2000-2018 TIPS Inc. Unauthorized reproduction of this manual is strictly prohibited and it is illegal to reproduce without permission. This manual is being used during review sessions conducted by PharmacyPrep.

4-3

PharmacyPrep.Com · ·

Cardiovascular System

Q-T interval. Mechanical contraction of the ventricles (Torse de pointes). U wave caused by hypokalemia.

Torsade de pointes: This is also called the Q-T interval. A problem in one of the ion channels can prolong the Q-T interval. A prolonged Q-T interval can increase risk for a type of arrhythmia called torsade de pointes. · · · ·

Thrombus is blood clot. Embolus is moving blood clot. Aneurysm is abnormal dilatation of arteries. Can cause stroke. Stenosis is constriction or narrowing of opening. Atherosclerosis is increased in LDL, progressively hardens the arteries and veins. Cause CAD (angina, MI), stroke, ischemia, and PVD.

Question Alerts! 1) Definition of Atherosclerosis. 2) Diseases that cause by plaques? Angina, MI, ischemic stroke.

Plaques are progressive accumulation of lipids and inflammatory cells. Site of injuries in arteries results formation of plaques. Sheer stress may result in plaque rupture, collagen exposure, platelet aggregation, and clot formation. Examples of diseases that comes from plaques are angina, myocardial infarction, atrial fibrillation, cerebral stroke, embolism and peripheral vascular diseases (DVT and PE). Cardiac oxygen consumption: When increased size of the heart.

Laplace's Law: Laplace's law describes how tension in the vessel wall increases with Trans mural pressure. According to Laplace’s law, tension is proportional to the radius of a sphere. Autonomic effects on heart rate and conduction velocity. Inotropic: Force of contraction (The ability of the cardiac muscle to develop force at given muscle length). Positive (+ve) inotropics: Digoxin, ACEI, DHP-CCB Negative (-ve) inotropics: BBs, verapamil, diltiazem Chronotropic: Heart rate (the number of action potential that occur per unit time). Positive (+ve) chronotropic. DHP-CCB Negative (-ve) chronotropic: Amiodarone, BBs, NDHP-CCBs, digoxin, "(ABCD)" Dromotropic: Conduction Positive (+ve) dromotropic: amitriptyline Negative (-ve) dromotropic: Na+& K+ channel blockers. Stroke volume: The volume of blood ejected from the ventricle on each beat. (Pulse).

Copyright © 2000-2018 TIPS Inc. Unauthorized reproduction of this manual is strictly prohibited and it is illegal to reproduce without permission. This manual is being used during review sessions conducted by PharmacyPrep.

4-4

PharmacyPrep.Com

Cardiovascular System

Ejection fraction: The fraction of end-diastolic volume ejected in each stroke volume. Ejection fraction in congestive heart failure is 140/90 mm Hg, in presence of DM, renal, atherosclerosis, and cerebrovascular. If the average SBP/DBP is 140-159/90-99 mmHg, treatment is recommended in the presence of risk factors smoking, FH, truncal obesity, sedentary lifestyle, male >55 yo, female >60yo. Coronary artery disease: ECG and biological marker (Troponin and Creatine kinase CK-MB) Electrocardiogram (ECG), and measures cardiac rhythms. ECG - used for excluding atrial fibrillation. Echocardiogram: Shows the presence of regional valve motion abnormalities, size of heart chambers. Echocardiogram allows for identification of valvular abnormalities and other MI problems. MRI, MR angiography (MRA), or CT angiography used for confirmation of degree of arterial occlusion or neurologic conditions like cerebral ischemia.

Tips Find answers from the table. 1. Absolute refractory period 4. Phase 0 7. Phase 3 10 Digoxin 13 Beta blockers 16 cerebral embolism

· · · · · · · ·

2. 5. 8. 11 14

Repolarization Phase 1 to starting phase 3 + ve inotropic ACEI stroke

3 6 9 12 15

arrhythmia Relative refractory period –ve inotropic Dihydropyridine CCBs brain attack

Absence of rhythm ( ) Drugs that cause +ve inotropic effect ( ) Rapid depolarization ( ) Increase in force of contraction ( ) The cell cannot respond to any stimuli ( ) The cell ability to respond stimuli increases or cell can respond to strong stimuli ( Decrease in force of contraction ( ) Excessive negative charge in cell occurs ( )

)

Copyright © 2000-2018 TIPS Inc. Unauthorized reproduction of this manual is strictly prohibited and it is illegal to reproduce without permission. This manual is being used during review sessions conducted by PharmacyPrep.

4-5

PharmacyPrep.Com

Cardiovascular System

Select True/False Statements · · ·

A brain attack that occurs when a wandering clot (embolus) or some other particle forms in a blood vessel usually in the heart and flow into in the brain cerebral vessel is cardiogenic cerebral embolism. True/False Drugs that cause –ve chronotropic effect (digoxin, beta blockers) True/False Stroke or brain attack happens when brain cells die because of inadequate blood flow to the brain (True/False)

Copyright © 2000-2018 TIPS Inc. Unauthorized reproduction of this manual is strictly prohibited and it is illegal to reproduce without permission. This manual is being used during review sessions conducted by PharmacyPrep.

4-6

PharmacyPrep.Com

Endocrine System

5 Endocrine System Questions Alerts! Common questions in pharmacy exam is to ask! · Hormone of anterior and posterior pituitary gland, thyroid hormone, Insulin, corticosteroid hormones. · Hypothyroid and hyperthyroidism symptoms. Lab investigations of serum TSH · Hypoglycemia and hyperglycemia symptoms. · Insulin function, Pathophysiology of diabetes and diabetic ketoacidosis · Hypo corticosteroids (Addison diseases) and hyper corticosteroids (Cushing's disease). Definitions · Amniocentesis: surgical puncture of the amniotic sac · Cystoscopy: process of viewing the urinary bladder · Dysmenorrhea: Painful periods · Embryology: study of the growth and development of the human organism · Gynecologist: specialist in the diseases of the female reproductive system · Hydrocele: accumulation of water in the scrotum; · Menorrhagia: Excessive bleeding during menstruation · Nephritis: Inflammation of the kidney · Primigravida: first pregnancy · Spermatogenesis: creation of new sperm · Urology: study of urinary tract Endocrine system Consists of a group of organs that have NO DUCTS and therefore are also known as DUCTLESS GLANDS that secrete hormones directly into the blood stream.. Major endocrine glands: Pituitary Gland (present under hypothalamus), the master endocrine gland. Testes, Ovaries, Thyroid Gland (neck), Adrenal Gland (on kidney), Pancreas Gland (endocrine and exocrine) Other glands · Parathyroid Gland (neck) · Thymus Gland (chest) · Pineal Gland (brain)

Copyright © 2000-2018 TIPS Inc. Unauthorized reproduction of this manual is strictly prohibited and it is illegal to reproduce without permission. This manual is being used during review sessions conducted by PharmacyPrep.

5-1

PharmacyPrep.Com

Endocrine System

Pituitary Gland · Located at the base of the brain. · Consists of two parts: anterior lobe and posterior lobe. · It is sometimes known as the master gland. · It controls the functions of other endocrine glands and is in turn controlled by the hypothalamus. Endocrine Gland Hypothalamus Anterior Pituitary gland

Types of hormone Houses releasing and inhibiting hormones Thyroid-stimulating hormone (TSH) Adrenocorticotropic (ACTH) Growth hormone (GH) Follicle-stimulating hormone (FSH)

Luteinizing hormone (LH)

Prolactin Posterior Pituitary gland

Pineal

Thyroid

Vasopressin (antidiuretic hormone) Oxytocin (formed in hypothalamus and stored in posterior pituitary gland). Melatonin

Parathyroid

Thyroid hormone (Triiodo T 3 and levothyroxine T 4 ), and calcitonin. Parathyroid hormone (PTH)

Thymus

Thymosin

Target tissue Anterior pituitary Thyroid Adrenal cortex Bones; soft tissues Females; ovary Males: Testes Females: Ovary Males. Testes Females: breast

Physiologic actions Controls release of anterior pituitary hormone Production of thyroid hormone (T 4 and T 3 , and calcitonin). Secretion of cortisol Stimulates growth of bones and soft tissues Promotes growth of ovarian follicle; Stimulates estrogen secretion Stimulates sperm production Stimulates ovulation Stimulates progesterone secretion Stimulates testosterone secretion Promotes breast development; stimulates milk secretion

Kidney

Causes water retention

Uterus Breasts

Causes contraction Causes ejection of milk

Brain; anterior pituitary; reproductive organs; possibly other sites. Most cells

Sets the body’s “time clock”. Causes sleep in response to darkness

Bone; kidney; intestine

T lymphocytes

Increases the metabolic rate; necessary for normal growth and development. Calcitonin takes Ca from Blood à Bones Increase amount of calcium in the bloodstream. ¯ amount of phosphate in the bloodstream - PTH = - Ca (by bone resorption) - PTH = ¯ phosphate Enhances the production of T lymphocytes

Copyright © 2000-2018 TIPS Inc. Unauthorized reproduction of this manual is strictly prohibited and it is illegal to reproduce without permission. This manual is being used during review sessions conducted by PharmacyPrep.

5-2

PharmacyPrep.Com Pancreas

Adrenal Medulla Adrenal cortex

Endocrine System

Insulin Secreted from beta cells Glucagon Secreted from alpha cells Somatostatin and gastrin Secreted from delta cells F cells? pancreatic polypeptides Epinephrine

Most cells

Kidney

Promotes use and storage of nutrients particularly glucose, after eating Maintains glucose levels in the bloodstream during periods of no food Inhibits digestion and absorption of nutrients. Inhibit secretion of insulin, glucagon and gastrin. Somatostatin are growth hormone inhibiting hormone (GHIH). Increases Na+ retention and K+ excretion

Zona glomerulosa (out) Z. fasciculata

Aldosterone

Kidney

Increases Na+ retention and K+ secretion

Cortisol

Most cells

Increases glucose in the bloodstream

Z. reticularis

Androgens

Females: bone and brain Male sex organs; body as a whole. Female sex organs; body as a whole Uterus

Puberty growth spurt and sex drive in females.

Testes (male)

Testosterone

Ovaries (female)

Estrogen Progesterone

Most cells Digestive system

Stimulates production of sperm; responsible for development of sex characteristics. Promotes sex drive. Stimulate uterine and breast growth; responsible for sex characteristics. Prepares for pregnancy

Question Alerts! 1) Insulin & glucagons released from? Beta cell & alpha cells 2) Epinephrine released? 3) Aldosterone hormones released from? 4) Aldosterone antagonist spironolactone act on collecting duct and prevent K+ secretion causes hyperkalemia. Physiological effects of some pituitary hormones · Somatostatin: opposes the effects of Growth Hormone-Releasing Hormone (GHRH) · Prolactin: It is synthesized and secreted by lactotrope cells in the anterior pituitary gland, breast and the deciduas. Effects · Stimulates the mammary glands to produce milk (lactation). · Provides the body with sexual gratification after sexual acts · immune tolerance of the fetus by the maternal organism during pregnancy. · Stimulate proliferation of oligodendrocyte precursor cells which differentiate into oligodendrocytes, the cells responsible for the formation of myelin coatings on axons in the central nervous system. Thyroid Gland (Fig 5.3): Secretes thyroid hormones (LEVOTHYROXINE, TRIIDOTHYRONIN AND CALCITONIN), which in turn control the body’s metabolic rate. Copyright © 2000-2018 TIPS Inc. Unauthorized reproduction of this manual is strictly prohibited and it is illegal to reproduce without permission. This manual is being used during review sessions conducted by PharmacyPrep.

5-3

PharmacyPrep.Com

Endocrine System

Thyroxin or Levothyroxine (T 4 ): Naturally occurs in levo (L) isomer form produced in the thyroid gland. T 4 Converts in the liver and other organs to T 3 by deiodination (deiodinase). · Controls the rate of metabolism in the body. Triiodothyronine (T 3 ): Metabolically active form. Calcitonin (a peptide): Hypocalcemic hormone. · Secreted by parafollicular cells (C-cells). · Reduces blood calcium ion concentration by moving Ca from blood to bones. · Used in treatment of osteoporosis associated vertebral fracture. · Hypercalcemia stimulates calcitonin production. Functions of thyroid hormones: Question Alerts! · Growth and development 1) Conversion of T4 to T3 by deiodination. · Proper function of all body system 2) Calcitonin production is stimulated by? · Maintenance of all body tissues. Carbohydrate, fat, protein, and vitamin metabolism (Basal Metabolic Rate). · Affects the secretion of other hormones (insulin, NE, Epi, cortisol, estrogen and testosterones. Mechanism of action At the target cell, proteases split protein carrier off from the thyroid hormone and most of T 4 is · deiodinated to T 3. · T 3 (and probably some T 4 ) enter the cell through membrane transport proteins and bind to a specific nuclear receptor. Hypothyroidism Thyroid gland is under active and produces insufficient thyroid hormone. Symptoms

Fatigue Sensitivity to cold Dry flaky skin and Coarse hair Slowed speech (deep voice) Puffy face, hands, feet Hearing loss Decreased libido Weight gain Constipation Impaired memory Hypertension, bradycardia Slow return of deep tendon reflexes

Hyperthyroidism Overactive thyroid gland causing an abundance of thyroid hormone. Thyrotoxicosis is the general term for over activity of the thyroid gland. Heat intolerance Profuse sweating Diffusely enlarged nontender goiter. Nervousness, irritability, anxiety and insomnia Weigh loss in spite of increased appetite Tremor and muscle weakness Tachycardia Diarrhea

Copyright © 2000-2018 TIPS Inc. Unauthorized reproduction of this manual is strictly prohibited and it is illegal to reproduce without permission. This manual is being used during review sessions conducted by PharmacyPrep.

5-4

PharmacyPrep.Com Diseases

Endocrine System

Hashimoto (autoimmune, the most common type of hypothyroidism. Common on in elderly). Myxedema (If untreated Myxedema and coma may develop. Dwarfism Mental retardation

Graves disease (diffuse toxic goiter) the most common form of hyperthyroidism, autoimmune disorder. Antibodies (longacting thyroid stimulators) bind to and activate TSH receptors.

¯ serum TSH

(Free T 4 )

The most sensitive test for detecting the hypothyroid state. - serum TSH Commonly used in patient receiving replacement therapy (levothyroxine) to control treatment. This is not separate test but estimation of free T 4 level mathematical interpretation of relationship of RT 3 U and serum T 4 levels. Serum free thyroxine ¯ FT4

(TT 3 )

Serum total triiodothyronine ¯ TT3

Pregnancy

Levothyroxine is used to treat. Adequate dose thyroxin, necessary for development of the fetal brain. >6 mU/L

Disproportionate rise indicated hyperthyroidism. Useful in early detection and rule out of hyperthyroidism Propylthiouracil the treatment of choice.

Serum TSH assay Sensitive TSH assay Free thyroxin index (FTI)

Serum TSH

Plummer’s disease (toxic nodular goiter)

Sensitive TSH assay

Elevated T 4 indicates hyperthyroidism

( ) Albuminuria is indicator of --> ( ) The most common extra cellular cation is--> ( ) The most common extra cellular anion is--> ( ) What happens in metabolic acidosis? ( ) What happens in metabolic alkalosis? ( ) Write the examples of drugs that cause metabolic acidosis? ( ) Write the examples of drugs that cause metabolic alkalosis? ( ) What is the abundant metal in body? ( ) Pre-renal ARF is due to à ( )

Creatinine clearance Azotemia Bladder or prostate Tonicity Hypercalcemia

)

Select True or False statements · · · · · ·

Normal serum potassium levels à ( ) If it is defect in renal filtration, CrCl à ( Normal CrCl is à ( ) In renal disease CrCl is à ( ) Azotemia or uremia is à ( ) Potassium sparing diuretics gives à (

)

)

Copyright © 2000-2018 TIPS Inc. Unauthorized reproduction of this manual is strictly prohibited and it is illegal to 6-7 reproduce without permission. This manual is being used during review sessions conducted by PharmacyPrep.

PharmacyPrep.Com

Renal System

· ·

Pyuria and dysuria is symptoms of à ( Lactic acidosis is SE of à ( )

)

· · · · · ·

Summary of electrolytes action in kidney (True/ False) Proximal convoluted tubule = Reabsorbs Na+, Cl-, Ca2+(True/ False) Distal convoluted tubule = Reabsorbs Na+, Cl-, Ca2+(True/ False) (True/ False) Thin descending loop of Henle = Reabsorbs H 2 O Thick ascending loop of Henle = Reabsorbs: Na+, K+, Cl-, Mg2+, Ca2+ (True/ False) Collecting tubule = Reabsorbs Na+ in exchange of K+ or H+ (regulated by aldosterone). Reabsorption of H 2 O is regulated by ADH (vasopressin). (True/ False)

Copyright © 2000-2018 TIPS Inc. Unauthorized reproduction of this manual is strictly prohibited and it is illegal to 6-8 reproduce without permission. This manual is being used during review sessions conducted by PharmacyPrep.

www.pharmacyprep.com

Liver function and Pathophysiology

7 Liver and Chronic Liver Diseases Questions Alerts! Common questions in pharmacy exam is to ask! · Causes of chronic liver diseases like ascites (peritonitis). · Hepatitis A, B, C infections causes of infections · Hepatitis A and B vaccines and treatment. Definitions · Necrosis cellular breakdown example: Acetaminophen · Steatosis: Hepatocytes filled with small droplet of lipid. Example: Tetracycline’s Drugs transportation into the bile from the liver · There are transporters for anions, bile salts, cations, and neutral organic compounds. · Release small intestine.

Question Alerts! 1) Enterohepatic recirculation is recirculation bile from small intestine to liver. 2) Drugs that involve in enterohepatic recirculation? Increase action of oral drugs with phase II metabolism. 3) Erythromycin estolate cause cholestatic jaundice. 4) Sulfa drugs in last trimester of pregnancy can cause KERNECTERUS. 5) Cholestyramine binds with bile and prevents reabsorption of bile into liver. 6) Decrease in blood flow to liver alters extent of drug metabolism. Oral drugs passage to liver: Mesenteric veins à portal veins à liver à hepatic vein à heart à systemic circulationà Renal or hepatic elimination. Enterohepatic recirculation ·

This term refers to drugs emptied via bile into the small intestine and then reabsorbed from the intestinal lumen into PORTAL VEIN to the systemic circulation.

Copyright © 2000-2018 TIPS Inc. Unauthorized reproduction of this manual is strictly prohibited and it is illegal to 7-1 reproduce without permission. This manual is being used during review sessions conducted by PharmacyPrep.

www.pharmacyprep.com · ·

Liver function and Pathophysiology

It can allow the body to conserve endogenous substances such as bile acids, vitamins D and B 12 , estrogen etc. It may be responsible for some of the long half-lives of drugs. Antibiotic therapy interferes with the process of enterohepatic recirculation by drugs, which have been conjugated can be hydrolyzed by gut enzymes such as glucuronidase and then reabsorbed as the active drug or as a metabolite. A decrease in bacterial flora as a consequence of antibiotic therapy can decrease the amount of sulfatase and glucuronidase containing bacteria. This could then lead to an increased rate of elimination of the drug.

Chronic liver disease: Ascites, hepatic encephalopathy, cholestatic disease, Wilson’s disease, alcoholic liver disease and viral hepatitis. CHRONIC LIVER DISEASES Spontaneous Bacterial Peritonitis

Chronic cholestasis (Jaundice or obstructive)

Ascites

Hepatic encephalopathy

history of fever, abdominal pain.

Yellow skin, eye, itchy skin. Abdominal pain, dark urine.

Aldosterone retain water. Excessive fluid retention in peritoneal cavities.

Confusion, psychosis.

hyperbilirubinemia

Abdominal cancers, abdominal infections, TB and GI problems, Drug of choice is spironolactone.

Ammonia released in blood.

Treatment: antibiotics

Treatment. Antihistamine or cholestyramine

Lactulose can trap ammonia gas.

Peritoneal dialysis associated infection: Associated risk with S. aureus in peritoneal catheter in dialysis. Chronic Cholestasis (cholestasis jaundice, obstructive jaundice, or jaundice). · Cholestasis is a condition in which obstruction of bile from liver to intestine, it is also referred to as obstructive jaundice. Symptoms are pruritus (itching) and are due to hyperbilirubinemia associated with liver diseases. Cholestyramine removes excess of bilirubin from the body. Antihistamines can be used for non-specific pruritus. · Jaundice: The jaundice is a yellowish discoloration of skin and whites of eye caused by high levels of pigment bilirubin in the blood stream. The urine is dark because excessive bilirubin in blood excreted through kidney. The other characteristic symptoms are pale stools and generalized itchiness. Ascites or Hydroperitoneum · The accumulation of fluid in peritoneal cavity and cause abdominal distention is referred to as ascites, this is due to high plasma aldosterone levels. Symptoms include abdominal distention. Ascites is caused due to infections such as tuberculosis, heart failure, cirrhosis, and portal hypertension, and various cancers. · Drug of choice is spironolactone is inhibitor of aldosterone, because aldosterone hormone increases Na/H 2 O retention. Alternate furosemide can be added to enhance diuresis.

Copyright © 2000-2018 TIPS Inc. Unauthorized reproduction of this manual is strictly prohibited and it is illegal to 7-2 reproduce without permission. This manual is being used during review sessions conducted by PharmacyPrep.

www.pharmacyprep.com

Liver function and Pathophysiology

Hepatic Encephalopathy: (Porto Systemic Encephalopathy). · Condition in which brain function is impaired by presence of toxic substances, absorbed from colon, which is normally detoxify and removed by liver. This condition occurs in severe liver damage such as liver cirrhosis. · Symptoms include: Drowsiness, confusion, difficulty in performing task (e.g. writing) and coma. · Drug of choice is lactulose to achieve 2 to 3 bowel movement a day. If no improvement in 1 to 2 days add metronidazole.

Fig 7.2

Wilsons Disease · Excessive copper can cause Wilsons disease. The drug of choice is the penicillamine and treatment is lifelong. Pyridoxine (vitamin B 6 ) 25 mg daily should be given with penicillamine to counteract its antipyridoxine effect. · Avoid food that has high copper content such as peanuts, chocolate, shellfish, mushrooms, and liver. The liver cirrhosis is the destruction of normal liver tissue. Cirrhosis results from permanent damage (IRREVERSIBLE) or scarring of the liver. It is end stage of chronic liver diseases, hepatitis B & C, ascites, chronic hepatitis and hepatic encephalopathies. The most common cause is due to alcohol abuse or continued excessive intake of alcohol over long period of time. Cholestatitis: Retention of bile acids because of the obstruction of bile ducts. Cholestatitis can lead to hyperbilirubinemia. Viral Hepatitis: There are 5 types of hepatitis viral infection, hepatitis A, B, C, D, and E. However, the common infections are hepatitis A, B and C. Most common symptoms of hepatitis are flu like symptoms, loss of appetite, fatigue, mild fever, muscle or joint aches, nausea and vomiting. Less common symptoms are light colored stools, jaundice (yellowing of skin, and whites of the eye), generalized itching, internal bleeding, and altered mental state. Hepatitis A · Hepatitis A is acute infection · Transmits through food contaminations such as water, food or orofecal · Vaccine available · Hepatitis A vaccine is recommended to travelers. Hepatitis B and C · Hepatitis B and C are chronic. Hepatitis B is 90% chronic in children and 5% in adults. · The most common is hepatitis B. · Hepatitis B is DNA type of virus, where as other hepatitis are RNA type. · Transmission through body fluids, such as blood transfusion, sexual contact and sharing needles (drug abuse and spa). · Hepatitis C is often chronic in adults (acute hepatitis C, is 80% becomes chronic likely within first year of infection). · Hepatitis C has NO vaccine. · Hepatitis B vaccine also protects hepatitis D infections. Copyright © 2000-2018 TIPS Inc. Unauthorized reproduction of this manual is strictly prohibited and it is illegal to 7-3 reproduce without permission. This manual is being used during review sessions conducted by PharmacyPrep.

www.pharmacyprep.com

Liver function and Pathophysiology

Interferon alfa (IFN alfa) is the drug of choice to treat acute viral hepatitis and chronic hepatitis.

·

Hep A Hep B&C Liver cirrhosis Jaundice (cholestasis)

ALT (liver specific) ↑ ↑ ↑ ↑

ALP

↑ ↑(specific)

AST

Bilirubin

↑ ↑ ↑

↑ ↑ ↑Serum unconjugated bilirubin

Alpha 1- antitrypsin deficiency occurs in liver cirrhosis. (deposition of excessive abnormal (A1AT) occurs in cirrhosis). Drugs associated with liver cirrhosis are amiodarone, methotrexate and methyldopa. ALP (alkaline phosphatase); have 5 minor sources (liver, bile duct, kidney, bone, placenta) ALT>AST by 1000 x in acute viral hepatitis AST can also come from muscle DRUGS THAT CAN CAUSE HEPATOTOXICITY Drugs Acetaminophen Tetracycline Methotrexate Vitamin A Salicylates Iron Cyclophosphamide 6-Mercaptopurines

Maximum dose/TIPS > 4g daily >2g daily >25mg/wk chronic use over 40,000 U daily chronic use >2g daily single dose >1g

Tips

· · · · · · ·

1. 4.

Penicillamine Cholestatitis

2. 5.

7. 10 13 16

Hepatitis C Cancer Sexual contact Interferon alfa

8. 11 14 17

Ascites hepatic encephalopathy Infections GI surgeries water Portal hypertension

What type of hepatitis is chronic? ( ) What is the treatment of hepatitis? ( ) Hepatitis A transmits by? ( ) Hepatitis B and C transmits by? ( Ascites is caused by? ( ) What is a DNA type of virus ( ) Accumulation of fluid in peritoneal cavity (

3. 6.

Wilsons disease Hepatitis B

9. 12 15 18

Tuberculosis Chronic liver disease Orofecal Spironolactone

) )

Copyright © 2000-2018 TIPS Inc. Unauthorized reproduction of this manual is strictly prohibited and it is illegal to 7-4 reproduce without permission. This manual is being used during review sessions conducted by PharmacyPrep.

www.pharmacyprep.com · · · · · · · · · · · ·

Liver function and Pathophysiology

It is caused by excessive copper ( ) Drug of choice to treat Wilsons disease ( ) Retention of bile acids because of obstruction of bile ducts ( A type of hepatitis that has no vaccine ( ) The drug of choice to treat ascites ( ) Lactulose is used in what type of chronic liver disorder? ( ) What type of hepatitis is chronic à ( ) What is treatment of hepatitis à ( ) Hepatitis A transmits by à ( ) Hepatitis B and C transmits by à ( ) Ascitis is caused by à( ) If takes Hep B vaccine, this also protects à( )

)

Select True or False statements: · · · · ·

Cholestatitis: Retention of bile acids because of the obstruction of bile ducts. Example: Penicillins (isoxazole type) (True/False) Cholestatitis can lead to hyperbilirubinemia example: Rifampin (True/False) Bacterial peritonitis: Chronic liver disease, history of fever, abdominal pain. (True/False) Causes of ascites include à infections, TB, cancer, GI surgeries, chronic liver disease (True/False) Lactulose is used in what type of chronic liver disorder à hepatic encephalopathy (decrease NH 3 ) (True/False)

Copyright © 2000-2018 TIPS Inc. Unauthorized reproduction of this manual is strictly prohibited and it is illegal to 7-5 reproduce without permission. This manual is being used during review sessions conducted by PharmacyPrep.

www.pharmacyprep.com

Liver function and Pathophysiology

Copyright © 2000-2018 TIPS Inc. Unauthorized reproduction of this manual is strictly prohibited and it is illegal to 7-6 reproduce without permission. This manual is being used during review sessions conducted by PharmacyPrep.

www.pharmacyprep.com

Respiratory System

8 Respiratory System Questions Alerts! Common questions in pharmacy exam is to ask! · Tidal volume. The volume inspired or expired with each normal breathing. · Asthma symptoms and triggers. · Definition of emphysema is COPD. · Differences between COPD (terminal bronchioles or alveoli) and asthma (bronchus).

Fig 8.1 Question Alerts! Which of the following does NOT open into respiratory tract? A) Nasopharynx B) Laryngeal pharynx C) Frontal sinus D) Bronchus E) Alveoli

Copyright © 2000-2018 TIPS Inc. Unauthorized reproduction of this manual is strictly prohibited and it is illegal to reproduce without permission. This manual is being used during review sessions conducted by PharmacyPrep.

8-1

www.pharmacyprep.com

Respiratory System

Lung volume Tidal volume: Can be measured by Spirometer (FEV 1 ). (In asthma, expiratory flow rate decreased). The volume inspired or expired with each normal breath. Residual volume: The volume that remains in the lung after a maximal expiration. The residual volume cannot be measured by spirometer. Lung capacities: Total lung capacity: The sum of all four lung volumes (tidal volume, inspiratory lung volume, expiratory lung volume, residual volume). The volume in the lungs after maximal inspiration. Forced expiratory volume (FEV 1 ): The volume of air that can be expired in 1 second after maximal inspiration. This is measured by spirometer. Spirometer: Measure volume that has been exhaled at the end of the first second (FEV 1 ). The spirometer is used for diagnosis of obstructive lung diseases such as asthma and COPD. Peak flow meter: is test to determine asthma severity for patient at home. This test measures the highest forced expiratory flow. LOWER RESPIRATORY TRACT Physiology Factors that decrease the respiration

Ventilation. Air moves from Atmosphereà URTàLRTàalveoli Respiration. Gaseous exchange occurs at alveoli capillary membrane. Increased in resistance to air flow Decrease ventilation Decrease diffusion Mucosal edema. Increased bronchial secretion Bronchospasm Tracheobronchial tubes have smooth muscle fibers arranged on a spiral around the tube. Asthma, COPD is Emphysema and Bronchitis

· · · · · · ·

Arrangement of Bronchiole smooth muscles Disease UPPER RESPIRATORY TRACT Anatomy Sinus (prenasal cavity in & around the nasal cavity) Nasal cavity Pharynx Larynx Trachea (past outside thoracic cavity) Disorders Acute rhinitis Acute pharyngitis Acute tonsillitis Acute laryngitis Common cold (most prevalent of URI, and NOT life threatening but causes severe discomfort). Treatment Antihistamineà Runny nose Decongestants (Sympathomimetics)ànasal congestion

Copyright © 2000-2018 TIPS Inc. Unauthorized reproduction of this manual is strictly prohibited and it is illegal to reproduce without permission. This manual is being used during review sessions conducted by PharmacyPrep.

8-2

www.pharmacyprep.com

Respiratory System

AntitussiveàCough Antibioticsà Infections ExpectorantsàBring up mucus RESIRATORY PATHOLOGY ASTHMA Obstructive Site Bronchus or bronchial tubes Cause

Inflammation in bronchus

Symptoms

Wheezing, cough, sputum, and SOB.

Treatment

Bronchodilators Steroids (ICS, PO, IV)

Eosinophilic Obstruction of airflow is reversible Immunization Flu vaccine annually Pneumococcal vaccine Prevention Avoid triggers (allergens, exercise, emotional stress, cold air, (NO trigger is warm air)

COPD (Emphysema + Chronic bronchitis) Obstructive Alveoli (enlarged, over-stretched, loss of long elasticity). Air trapped in lung. Permanent (irreversible) enlargement alveoli (emphysema) and /or Chronic bronchitis. Shortness of breath (SOB) (dyspnea), fatigue, cough, sputum Bronchodilators (SABD, LABD, LAMA) acute COPD bronchodilators, anticholinergic, for exacerbations oral steroids (not used? inhaled corticosteroids) Antibiotics (pneumonia) Neutrophil due to bacterial infections. Obstruction of airflow is irreversible Flu vaccine annually and Pneumococcal vaccine Q5-10y in high risk. Causes of COPD, smoking, Alpha1-antitripsine deficiency, air pollution, secondary smoke.

Cystic fibrosis: Autosomal recessive disorder that is caused by a mutation on chromosome 7. This mutation results in a defective membrane CT channel (CFTR). Signs: The mutation leads to secretion of thick mucus, which lodges in lungs, liver and pancreas.

Copyright © 2000-2018 TIPS Inc. Unauthorized reproduction of this manual is strictly prohibited and it is illegal to reproduce without permission. This manual is being used during review sessions conducted by PharmacyPrep.

8-3

www.pharmacyprep.com

Respiratory System

Tips 1 4 7 · · · · · · · · · ·

asthma COPD Wheezing

2 5 8

emphysema Emboli formation in lungs Vital capacity

3 6

dyspnea Autosomal recessive disorder

Difficulty in breathing (dyspnea) Permanent enlargement of the alveoli (emphysema) Emphysema is…(COPD) What respiratory condition occurs in bronchus or bronchial tube (Asthma) What respiratory condition can occur in bronchioles & alveoli (COPD) Pulmonary embolism is floating clot in blood circulation in the lung tissues. Cystic fibrosis is a --> multi organ disease but mainly effect on lungs due to the long thick lung secretions blocks small airway and thus inflammation (a restrictive respiratory disease) Cough in asthma sounds like--> Wheezing Alpha1-antitripsine deficiency? COPD When maximum amount of air a person can expel from the lung after a maximum inhalation, is referred as. Vital capacity. This depends on age, gender, height, mass and ethnicity. ?

Copyright © 2000-2018 TIPS Inc. Unauthorized reproduction of this manual is strictly prohibited and it is illegal to reproduce without permission. This manual is being used during review sessions conducted by PharmacyPrep.

8-4

www.pharmacyprep.com

Urinary System

9 Urinary System Questions Alerts! Common questions in pharmacy exam is to ask! Urinary tract infection symptoms Bladder (cystitis), Urethra (urethritis), Ureter (ureteritis), kidney (pyelonephritis). Complicated and uncomplicated UTI. Symptoms of benign prostatic hyperplasia (BPH). Types and symptoms of urinary incontinence and over reactive bladder. Prostate cancer screening is prostate specific antigen (PSA), and Digital rectal exams. The urinary system includes two kidneys, two ureters, the bladder, two sphincter muscles, and the urethra. Urinalysis is a test that studies the content of urine for abnormal substances such as protein or signs of infection. Urodynamic tests evaluate the storage of urine in the bladder and the flow of urine from the bladder through the urethra.

Urinary Tract Infection: The name of the UTI depends on its location in the urinary tract. An infection in the bladder is called cystitis. If the infection is in one or both of the kidneys, the infection is called pyelonephritis. This type of UTI can cause serious damage to the kidneys if it is not adequately treated.

Copyright © 2000-2018 TIPS Inc. Unauthorized reproduction of this manual is strictly prohibited and it is illegal to reproduce without permission. This manual is being used during review sessions conducted by PharmacyPrep.

9-1

www.pharmacyprep.com

Urinary System

Pathological defense mechanism of UTI:

Organ Urethra Bladder Ureter Kidney

Infection Urethritis Cystitis. The most common UTI Ureteritis Pyelonephritis (Symptoms: flank pain, dysuria, pain at costovertebral angle, and same symptoms as cytitis)

Treatment Cotrimoxazole 3d, nitrofurantoin 5d, trimethoprim. Ciprofloxacin, amoxi+clav, 3rd gen cephalosporin’s

Copyright © 2000-2018 TIPS Inc. Unauthorized reproduction of this manual is strictly prohibited and it is illegal to reproduce without permission. This manual is being used during review sessions conducted by PharmacyPrep.

9-2

www.pharmacyprep.com

Urinary System

Urine pH 6.5 to 7 (slightly acidic in morning, slightly base by evening) Physiological Factors that increase risk of urinary tract infections. Decrease resistance of mucus membrane. (e.g. menopause) Increase in vaginal pH Colonize colon spread to UTI Colonization of urethra and peri-urethral tissue. Causative agents of UTIs. E. coli is the most common in community acquired (80%), and hospital (50%). Uncomplicated UTI: Occurs in females with normal genitourinary tract. Cystitis, the most common infecting organism is E. coli (80-90%). Usual presenting symptoms. Internal dysuria, frequency, suprapubic discomfort and urgency. Complicated UTI: occurs in individuals with functional or structural abnormalities of genitourinary tract. Virtually all episodes of UTI in men are complicated. E. coli (50%). Patient with cystitis. Uncomplicated UTI Complicated UTI Internal dysuria, frequency, suprapubic discomfort, urgency, and fever. Burning upon urination. Nausea & vomiting Cotrimoxazole, nitrofurantoin,

High fever, blood in urine (turbid urine or cloudy, hematuria), vomiting and sepsis. Burning upon urination. Nausea & vomiting Ciprofloxacin+amoxi

Benign prostatic hyperplasia (BPH) is a condition in men that affects the prostate gland, which is part of the male reproductive system. The prostate is Question Alerts! located at the bottom of the bladder and surrounds the urethra. BPH is an What is NOT a symptom of enlargement of the prostate gland that can interfere with urinary function in BPH? older men. It causes blockage by squeezing the urethra, which can make it Jet urination (stream) difficult to urinate. Men with BPH frequently have other bladder symptoms including an increase in frequency of bladder emptying both during the day and at night. Symptoms: Urine obstructions symptoms includes frequent urine, drop by drop, incomplete voiding, nocturea and also include irritation symptoms. Treatment. The drug of choice to decrease prostate size Finasteride 5 mg. Alpha blocker tamsulosin, alfuzosin, terazosin is used to treat symptoms. Bladder neck/sphincter contraction is due to alpha agonist activity. Painful bladder syndrome/Interstitial cystitis (PBS/IC): is a chronic bladder disorder also known as frequencyurgency-dysuria syndrome. In this disorder, the bladder wall can become inflamed and irritated. The inflammation can lead to scarring and stiffening of the bladder, decreased bladder capacity, pinpoint bleeding, and, in rare cases, ulcers in the bladder lining. The cause of IC is unknown at this time. Copyright © 2000-2018 TIPS Inc. Unauthorized reproduction of this manual is strictly prohibited and it is illegal to reproduce without permission. This manual is being used during review sessions conducted by PharmacyPrep.

9-3

www.pharmacyprep.com

Urinary System

Kidney stones is the term commonly used to refer to stones, or calculi, in the urinary tract system. Stones form in the kidneys and may be found anywhere in the urinary system. They vary in size. Some stones cause great pain while others cause very little. The aim of treatment is to remove the stones, prevent infection, and prevent recurrence. Both nonsurgical and surgical treatments are used. Kidney stones affect men more often than women. Stones (calculi) can form anywhere in urinary tract and pain radiates in flank area, sharp, sudden severe pain, may be intermittent depends on stone movement, Nausea, vomiting, bleeding (hematuria), obstruction of flow of urine, or an infection. Depending on the site of stone it is referred as kidney stone, ureteral stone, or bladder stone. The process of stone formation is urolithiasis, renal lethiasis, or nephrolithiasis. Risk factors: Infections, urinary stasis, immobility, hypercalcemia, hyperuricemia, high urinary oxalate levels. Increased incident in men over age 40. List of drugs associated with urolithiasis/ nephrolithiasis

Causes/recommendation

Sulfa drugs (antibiotics/sulfonylureas) Fibrates Topiramate

Drink lots of fluids/water Avoid use in renal stones. Drink lots of fluids/water

Vit. D overdose Bisphosphonates Calcium supplements Allopurinol Prostatitis is inflammation of the prostate gland that results in urinary frequency and urgency, burning or painful urination, a condition called dysuria, and pain in the lower back and genital area, among other symptoms. In some cases, prostatitis is caused by bacterial infection and can be treated with antibiotics. But the more common forms of prostatitis are not associated with any known infecting organism. Antibiotics are often ineffective in treating the nonbacterial forms of prostatitis. Proteinuria is the presence of abnormal amounts of protein in the urine. Healthy kidneys take wastes out of the blood but leave in protein. Protein in the urine does not cause a problem by itself. But it may be a sign that your kidneys are not working properly. Nephropathy associated with high albumin urine. Urinary incontinence: Uncontrollable loss of urine or involuntary leakage of urine. There are many causes and types of incontinence, and many treatment options. Treatment range from simple exercises to surgery. Women are affected by urinary incontinence more often than men. To treat incontinence, anticholinergic drugs oxybutynin is the drug of choice. Detrusor muscle contraction due to parasympathetic activity (voiding). Detrusor muscle relaxation and tightening of sphincter is due to anticholinergic activity (storage). Bladder relaxation is due to beta agonist activity. Bladder neck/sphincter contraction is due to alpha agonist activity.

Copyright © 2000-2018 TIPS Inc. Unauthorized reproduction of this manual is strictly prohibited and it is illegal to reproduce without permission. This manual is being used during review sessions conducted by PharmacyPrep.

9-4

www.pharmacyprep.com

Overflow incontinence Urethral blockage. Bladder overfull Diuretics

Urinary System

Stress incontinence Relaxed pelvic or leakage with - abdominal pressure, cough/sneezing. Men à surgery Women à hypomobility of spincter Estrogen vaginal cream, suppositories, ovule.

Urge incontinance Neurological problems, Parkinson’s, sensitive to infection. Feeling urgency Anticholinergic drug: Oxybutinin, tolterodine

Tips · · · · · · · · · ·

Urinary incontinence symptoms àNO control on bladder or urination, urine leakage. Drugs that are used for treatment of urinary incontinenceà Oxybutynin or anticholinergic drugs. Drugs that are avoided in-patient with overflow urinary incontinence à anticholinergics Enuresis (bed wetting) drug of choice à ADH (vasopressin) or imipramine. Benign prostatic hyperplasia (BPH) is à enlargement of prostate Benign prostatic hyperplasia symptoms are --> dysuria, frequent urination, urine by drop by drop, nocturia, and irritation (NOT a symptoms, stream (Jet urination). Drug of choice to treat benign prostatic hyperplasia à Finasteride (Proscar 5 mg), and also Propecia 1 mg to treat baldness. Saw palmetto is herbal product is used for --> benign prostatic hyperplasia. Pyuria and dysuria is symptoms of à complicated UTI Hypertrophy: Increased size of an organ or tissue by increase in size of its cell. Hyperplasia: increase in size of an organ or tissue by increase in number of its cells. Can cause tumor (cancer).

Copyright © 2000-2018 TIPS Inc. Unauthorized reproduction of this manual is strictly prohibited and it is illegal to reproduce without permission. This manual is being used during review sessions conducted by PharmacyPrep.

9-5

www.pharmacyprep.com

Urinary System

Copyright © 2000-2018 TIPS Inc. Unauthorized reproduction of this manual is strictly prohibited and it is illegal to reproduce without permission. This manual is being used during review sessions conducted by PharmacyPrep.

9-6

www.pharmacyprep.com

The Eye and Ear

10 The Eye and Ear Questions Alerts! Common questions in pharmacy exam is to ask! 1) Photoreceptors rods are sensitive for dim light and cones cells sensitivity to daylight and colors. 2) Cornea is upper layer of eye is rate determine step in ophthalmic drops. 4) Eye disorders like conjunctivitis (red or pink eye), blepharitis, and sty (hordeolum), Age related macular degeneration, Cataract, and glaucoma. 5. External and middle ear problems.

Copyright © 2000-2018 TIPS Inc. Unauthorized reproduction of this manual is strictly prohibited and it is illegal to reproduce without permission. This manual is being used during review sessions conducted by PharmacyPrep.

10-1

www.pharmacyprep.com

The Eye and Ear

Question Alerts! 1) More sensitive photoreceptors for dim light in eye? Rods 2) Color sensitive photoreceptors are? cones 3) Rhodopsin is red photosensitive pigment in the retinal "rods" important vision in dim light. 4) Iodopsin is pigment present in the retinal “cones” important in daylight. 5) What arteries supply blood to eye? External carotid arteries.

Optic nerve from both eye merge at optic chiasm and become optic tract. This optic tract connect to thalamus then goes to right and left brain. Primary visual area in occipital lobes of cerebral cortex. · · · · · · · · · · ·

Cornea: In the front of the eyeball is a transparent opening known as the cornea. RATE LIMITING STEP FOR OPHTHALMIC DROPS. Pupil: After light passes through the cornea, a portion of it passes through an opening known as the pupil. Iris: Pupil opening can be adjusted by the dilation of the iris. Ciliary muscles: The lens is attached to the ciliary muscles. Ciliary gland: secrete aqueous humor. Retina: The inner surface of the eye is known as the retina. Macula: Small central area of retina. Optic nerve: The network of nerve cells is bundled together to form the optic nerve on the very back of the eyeball. Optic disk: The nerve cells is bundled at very back of eyeball, is also known as blind spot. Rods and cones are NOT present on the optic disk, therefore blind spot. Myopia: If the incoming light from a far away object focuses before it gets to the back of the eye, that eye’s refractive error is called “myopia” (nearsightedness). Hyperopia: If incoming light from something far away has not focused by the time it reaches the back of the eye, that eye’s refractive error is “hyperopia” (farsightedness).

The retina contains two types of photoreceptors, rods and cones. These rods are responsible for night vision, our most sensitive motion detection, and our peripheral vision. The rods are more numerous, some 120 million, and are more sensitive than the cones. However, they are not sensitive to color. · ·

The 6 to 7 million cones provide the eye's color sensitivity and they are much more concentrated in the central yellow spot known as the macula. The image forms in eye at retina. Ophthalmic drug rate limiting step is cornea.

Eye anatomy Cornea Retina (photoreceptors) Rhodopsin Idopsin Optic disk Optic nerve Celiary gland

Functions

Copyright © 2000-2018 TIPS Inc. Unauthorized reproduction of this manual is strictly prohibited and it is illegal to reproduce without permission. This manual is being used during review sessions conducted by PharmacyPrep.

10-2

www.pharmacyprep.com Myopia Hyperopia Mydriatic pupils Miotic pupils Vitreous humor Aqueous humor

The Eye and Ear

Gel like fluid filled between retina and lens. Fluid Filled between cornea and lens.

Beta carotene --> Retinol (vitamin A) --> 11-Cis retinal-> opsin RODS CONE Rhod”opsin” (retinal) Iod”opsin” Rhodopsin is rod cells pigmentation. ResponIodopsin is retinal cone cells responsible for day light sible for dim light vision. vision color vision More sensitive in dark Less sensitive in dark Higher in number Less in number Rods: Light on the retina converts 11-cis retinal to all Trans retinal. The retinal is a vitamin A is essential for the regeneration of 11-cis retinal. Deficiency of vitamin A causes night blindness.

Carotenoids Aqueous humor is present in anterior eye chamber Vitreous humor is present in posterior eye chamber Glaucoma: Due to angle closure increasing intra ocular pressure (IOP) cause glaucoma. This is due to; · Increase aqueous humor production cause increase IOP. · Decrease aqueous humor secretion from shlemn canal. Aqueous humor is secreted from ciliary tissue (ciliary gland). Treatment of glaucoma Beta blockers (Timolol). They decrease IOP by inhibiting formation of aqueous humor. Prostaglandin analogues ("prost"). Lower IOP by increase outflow of aqueous humor through uveoscleral pathway. Topical CA inhibitors (Acetazolamide, Dorzolamide): is diuretics. Decrease IOP by inhibiting enzyme that involved in formation of aqueous humor. Cholinergic agonist (Pilocarpine, carbachol): Directly stimulate muscarinic receptors to contract ciliary muscle and increase trabecular outflow.

Copyright © 2000-2018 TIPS Inc. Unauthorized reproduction of this manual is strictly prohibited and it is illegal to reproduce without permission. This manual is being used during review sessions conducted by PharmacyPrep.

10-3

www.pharmacyprep.com

The Eye and Ear

Age related macular disorder (AMD): It is due to gradual deterioration of macular in central vision. It is two types; 1) Dry. This is characterized by drusen (white to yellow spots in the central retina). May or may not cause vision loss. 2) Wet. Caused by presence of choroidal neovascular membrane (CNM). This is common cause of severe vision loss. The Amsler grid is useful for self-monitoring by patients. Multivitamins without carteronoids are used prophylaxis of AMD. Lutein & Zexanthin are two types of carotenoids, which are yellow to red pigments found widely in vegetable. These may contain in multivitamin that are used for AMD prophylaxis. Cataract: When the eye lens becomes cloudy, decrease acuity, and no pain, and this obstruct the vision is referred as cataract. Cataract surgery postoperative care: Antibiotics, Dilators and anti-inflammatory drugs. Antibiotics: Fluoroquinolones 7-10 d (besifloxacin, ciprofloxacin, gatifloxacin, moxifloxacin and ofloxacin). Aminoglycosides 7-10 d. Gentamicin, neomycin, tobramycin Dilators and cycloplegic: used to keep iris away from implant during early healing period and improve comfort by decrease ciliary muscle spasm. Cyclopentolate, phenylephrine, tropicamide. Anti-inflammatory 3-4 wks: Dexamethasone, prednisolone, diclofenac, and ketorolac. Combination eye drops: Ciprodex (ciprofloxacin+ dexamethasone). EYE CONDITIONS SUMMARY Glaucoma Cataract AMD Eye drops Technique: tilt head backward, place drop in conjunctiva sac; put pressure on inner cantus. Terms: OD = OS = OU =

Ear (outer ear, middle ear and inner ear) Otitis externa (outer ear) and Acute bacterial externa Eczematous otitis externa: Drainage resulting from mild otitis externa may be self treated. (Ear pain associated with ear drainage, the patient should be referred to a physician). Otitis media with perforation of the tympanic membrane or drainage from the middle ear, the patient should be referred to a physician. OTITIS EXTERNA Self-treated by non-prescription

OTITIS MEDIA Require referral to physician

When to refer? Wax buildup, foreign objects in ear canal.

Fluid is indicator of infection of ear bones or tympanic membrane. Child presents with fever and ear pain? The drug of choice for otitis media? Amoxicillin or azithromycin/clarithromycin

1) Ceremonious gland produce? earwax 2) Earwax is removed by carbamide peroxide and mineral oil.

Inner ear: Noise, drug exposure, such as ASA, ototoxic drug, vestibular toxicities and Meniere’s disease. Copyright © 2000-2018 TIPS Inc. Unauthorized reproduction of this manual is strictly prohibited and it is illegal to reproduce without permission. This manual is being used during review sessions conducted by PharmacyPrep.

10-4

www.pharmacyprep.com

The Eye and Ear

EAR CONDITIONS SUMMARY Otitis externa What symptoms to refer? Otitis media Meniere’s disease Ear drops admin Technique: For infants and toddlers pull ear down and back For adults and older children pull ear up and back Lie on unaffected side Stay on side for 5-10 min after drops instilled. Can put cotton moistened with medication in ear to keep drops in place. Terms: AD = AS = AU =

Dental anatomy and physiology · · · · · · · · · · · · · · · · ·

There are 20 primary teeth and 28 to 32 permanent teeth. The last 4 is being wisdom teeth. In adult, there are 16 teeth in maxilla and 16 in the jaw. Wisdom teeth may or may not grow in. Incisors 8, canine 4, premolar 8, molar 8, wisdom (third molar) 4=32. Baby teeth start to grow during intrauterine first trimester (8 weeks). Teeth are made of enamel, dentin, and cement. Dentin composes most of the root. Crown is covered by the enamel. The root embedded inside the maxilla and jaw bones has a bulb canal. The bulb canal contains blood supply and nerve terminals. The root teeth may be single in number or multiple. Gingival or gums consist of mucosal tissue lies over the alveolar bone. Oral hygiene is practice of keeping mouth and teeth clean, to prevent bad breath, and dental problems. Plaque is yellow sticky films that form on the teeth and gums. Bacteria in plaque release acid that harm the enamel. Brushing and flossing daily the teeth will prevent tartar forming. Fluorides are a primary protector against dental cavities.

Copyright © 2000-2018 TIPS Inc. Unauthorized reproduction of this manual is strictly prohibited and it is illegal to reproduce without permission. This manual is being used during review sessions conducted by PharmacyPrep.

10-5

www.pharmacyprep.com · · · · · · ·

The Eye and Ear

Fluorides make teeth surface more resistant to acids. Drinking water contains enough fluorides. Sugar free gum increases salivation and help to clean teeth surface. Good food items for teeth; green tea, milk and yogurt, cheese, apples, onion. Smoking and chewing tobacco causes multiple dental diseases. Bulimia nervosa and repeated vomiting causes significant damage to enamel. Bad dental hygiene shows direct link to systemic diseases; · Cardiovascular disease. · Bacterial pneumonia. · Low birth weight. · Complication of diabetes. · Osteoporosis.

Tips · · · · · · · · ·

·

Blind spot is à optic disc Age related macular degeneration cause due toà retinal detachment Glaucoma occurs due to à abnormal increase IOP in eyeball Drugs that are used to treat glaucoma are à sympathetic blockers (B-blockers), Prostaglandin analogs (Latanoprost), CA diuretics (acetazolamide). Photoreceptor sensitive to daylight? cones Photoreceptor sensitive to dark or dim light? rods What are the most sensitive photoreceptors? rods Age related macular degeneration cause due toà retinal detachment Glaucoma occurs due to à Increase IOP

What are the mechanism of Drugs that are used to treat glaucoma are à inhibitor formation of aqueous humor or increase outflow of aqueous humor. · Meniere's disease is characterized as? Vertigo, hearing loss and tinnitus. Vitreous humor is? The clear gel fills the space between the lens and the retina of eyeball of human.

Select True/False Statements · · · · ·

The retina contains two types of photoreceptors, rods and cones. (True/False) These rods are responsible for night vision, our most sensitive motion detection, and our peripheral vision. (True/False) The rods are more numerous, some 120 million, and are more sensitive than the cones. However, they are NOT sensitive to color. (True/False) The 6 to 7 million cones provide the eye's color sensitivity and they are much more concentrated in the central yellow spot known as the macula. (True/False) Cones are sensitive to color vision(True/False)

Copyright © 2000-2018 TIPS Inc. Unauthorized reproduction of this manual is strictly prohibited and it is illegal to reproduce without permission. This manual is being used during review sessions conducted by PharmacyPrep.

10-6

Pharmacyprep.com

Blood and Anemia

11 Blood and Anemia This chapter reviews blood cells and functions. Types of anemia and their morphological changes. Terminology Questions Alerts! Common questions in pharmacy exam is to ask! · Blood cells such as platelets, red blood cells, and white blood cells · Hemostasis · Morphological changes of anemia. Iron deficient microcytic anemia and megaloblastic anemia. · Elemental iron supplements (ferrous fumerate > dried ferrous sulfate > ferrous sulfate>ferrous gluconate). · Iron supplement drug interactions, thyroid hormone, tetracycline and quinolones, cholestyramine. · Vitamin B12 and Folic acid supplements

· · · · · · · · ·

Question Alerts! 1) The highest number of blood cells? 2) The second highest number of cell? 3) Reticulocytes are? 4) Life of thrombocytes (platelets) ? 5) Life of erythrocytes (RBC)? Myelosuppresion is?

Agranulocytosis: Decrease in number of neutrophil Neutropenia. Decrease in neutrophil Neutrophilia. Increase in neutrophil Esinophilia. Increase in esinophil Thrombocytopenia. Reduced platelets to less than 150 x 109/L Parasthesias: Abnormal tingling sensation as described as tingling & needles. Hemocromotasis: Excessive iron Polycythemia: excessive hemoglobin or RBC concentration. Function of blood include transportation of gases, nutrients, hormones, metabolic wastes regulates body temperature, pH, electrolyte balance, fluid volume protects, prevents blood loss (clotting), and prevents infection (WBC and antibodies).

Copyright © 2000-2018 TIPS Inc. Unauthorized reproduction of this manual is strictly prohibited and it is illegal to 11-1 reproduce without permission. This manual is being used during review sessions conducted by PharmacyPrep.

Pharmacyprep.com

Blood and Anemia

Blood

55% Fluid

Water containing salts Proteins Antibodies Hormones Electrolytes Fats and lipids Sugar (carbohydrates) Mineral Vitamins

45% Cells

Erythrocytes (RBC)= 5-6 mil/mL (44%) Leukocytes + Platelets = (1%)

Erythrocytes

Leukocytes

Platelets

Copyright © 2000-2018 TIPS Inc. Unauthorized reproduction of this manual is strictly prohibited and it is illegal to 11-2 reproduce without permission. This manual is being used during review sessions conducted by PharmacyPrep.

Pharmacyprep.com

Blood and Anemia

Blood constitute 55% fluid and 45% cells. Blood proteins · ·

Blood proteins (albumin, glycoprotein, and fibrinogen). Question Alerts! Albumin is the most common blood proteins (55%) in blood. The major protein in blood? Which is carrier of, buffer, osmotic pressure, transport lipids, fatty acids and acidic drugs. Globulins carry ions, hormones, and steroids and bind base drugs. Fibrinogen changes to fibrin, helps in blood clotting.

Electrolytes: The Electrolytes present in blood are Na+, K+, Ca2+, Mg2+, Cl- and CO3 Extracellular ion Na+, Cl- and Ca2+. The most common extra cellular cation is sodium (Na). The most common extra cellular anion is chloride (Cl). Intracellular ions K+, Mg2+ and phosphate. Antibodies (Immunoglobulin) IgG, IgM, IgE, IgA, and IgD. Erythrocytes (Red blood cells): RBC is composed of globin (protein), iron (metal), phospholipid (lipids), potassium phosphate (salt), hemoglobin (contain porphyrin ring system) bind loosely to oxygen and carbon dioxide. In the overdose of carbon monoxide, the hemoglobin binds with carbon monoxide. RBC synthesis (Erythropoiesis): Pre-erythrocyte is the first step in the synthesis of RBC series. · The normal value of the RBC in adult men would be 4.7 million. · Erythrocytes are originated in sequence of cells Hemocytoblastà Megaloblastà Erythroblast (normoblast) --> Reticulocytesà Erythrocytes · Erythropoietin, a glycoprotein is a key hormone for the production of RBC. In the absence of erythropoietin. Hypoxia is unable to stimulate the production of RBC. The kidney is a principal organ for the synthesis of erythropoietin, therefore, kidney failure would result in severe anemia. RBC destruction: Life span of RBC is 120 days (4 months). RBCs are destroyed in the spleen (spleen also referred to as "grave yard" of RBC). Hemoglobin (Hgb): Blood protein-containing iron metal, carries oxygen in blood. · Hemoglobin content 14.5 g/100 ml. Hemoglobin consists of porphyrin ring. Oxygen binds with porphyrin ring. Porphyrin ring present in haemoglobin is tetramer. · The metabolic degradation of hemoglobin takes place principally in the reticuloendothelial system (endoplasmic reticulum). The non-protein portion of hemoglobin is a "ferrous" complex of porphyrin. The O 2 binds the ferrous form (Fe2+) of iron present in hemoglobin to form oxyhemoglobin. During this process one electron is transferred. (Ferrous = Fe2+; Ferric = Fe3+). Methemoglobinemia: Abnormal state of hemoglobin contains oxidized iron as Ferric Fe3+ · · ·

The protein portion of hemoglobin is globin. Glycine is an aminoacid is precursor of hemoglobin. Myoglobin. Protein present in tissues, which is essential for oxygen transport in tissues. Carries oxygen in tissue. The porphyrin ring present in myoglobin is monomer.

Copyright © 2000-2018 TIPS Inc. Unauthorized reproduction of this manual is strictly prohibited and it is illegal to 11-3 reproduce without permission. This manual is being used during review sessions conducted by PharmacyPrep.

Pharmacyprep.com ·

Blood and Anemia

Methemoglobin: Nitrite ions react with hemoglobin and produce methemoglobin, which has a low affinity for oxygen and a high affinity for cyanide ions. This forms iron CN complex, it is referred as cyanomethemoglobin.

Hemoglobin Myoglobin Cytochrome oxidase Transport oxygen in blood Transport oxygen in tissue Catalyzes phase I metabolism Porphyrin tetramer Porphyrin monomer Porphyrin Has affinity and binds with Has affinity and binds with Has affinity and binds with O 2 , O 2 , CO, CO 2 and CN. O 2 , CO, CO 2 and CN. CO, CO 2 and CN 2+ 2+ Ferrous ion (Fe ) Ferrous ion (Fe ) Ferrous ion (Fe2+) Death due to cyanide poisoning results from cyanide binding to hemoglobin, myoglobin and inhibiting cytochrome oxidase. Glycine is precursor of -CH 2 - in porphyrin ring. Carboxyhemoglobin: Carbon monoxide bound hemoglobin. Platelets (Thrombocytes): Platelets are referred to as thrombocytes. Platelets help in the process of blood clotting. Platelets lack nuclei and platelets are produced in bone marrow. Platelet life span is 7 to 10 days. Range in blood 150,000 to 300,000 mm3. Deficiency of platelets is referred as thrombocytopenia. Leukocytes (WBC) · Leukocytes produced in bone marrow like RBC’s. Leukocytes consist of clearly defined nuclei. About 30% are lymphocytes and about 60% are neutrophils and 8% are monocytes. Normal range of WBC’s (white count) in blood is 4000 to 11000/mm3 (4000 to 11000/ cmm). · Neutrophils: About 60% of white blood cells are neutrophils. Responsible for immune defence phagocytosis. · Monocytes: About 8% of white blood cells are monocytes. · B-lymphocytes and T lymphocytes are primary cell of specific immune response. · Basophils: Responsible for inflammatory response. · Eosinophils: Defence against parasites. · Monocytes: Immune defence (precursor of tissue macrophage) also called big eaters. · B-lymphocytes. Antibody production (precursor of plasma cells). · T-lymphocytes. Cellular immune response. Granulocytes are Neutrophils, eosinophils, and basophils, cells that stain. · Neutrophils gives stain with acidic or basic dyes · Eosinophils gives stain with acidic dye · Basophils gives stain with basic dye Agranulocytes Cells that do not stain are agranulocytes. There are 2 types of cells lymphocytes and monocytes.

Copyright © 2000-2018 TIPS Inc. Unauthorized reproduction of this manual is strictly prohibited and it is illegal to 11-4 reproduce without permission. This manual is being used during review sessions conducted by PharmacyPrep.

Pharmacyprep.com

Blood and Anemia

Rh factor: Agglutinogens in human RBCs are known as the Rh factor blood with this factor is described as Rh +ve (90% of population). Blood without this factor is described as Rh (-) negative. In Rh-negative mother, Rh-positive antigens may transfer from Rh-positive fetuses to the mother via placenta. This may lead to production of Rhpositive antibodies in the mother’s blood. These same antibodies may transfer back from the mother’s blood into fetus via the placenta, and produce antigen antibody reactions. This leads to lysis of red blood cells in the fetus, and miscarriage. Rho gram prevents the formation of anti-Rh antibodies in a mother who bears Rh positive fetus. Rh- women who have + babies get a Rhogam shot. Blood Groups Question Alerts! Surface of RBC's have antigen What blood group can be given to anyone? A RBC has A antigen, plasma has B antibodies B has B antigen, A antibodies AB has A and B antigens, no antibodies. O (universal donor) has no antigens, has antibodies against A and B, transfusion reaction can lead to kidney failure. DONOR Qualities of Blood o o O A B AB · Temperature. 38 C (100.4 F), warfarin AB (anticoagulant) added in blood storage a� a� a� a� Viscosity: 5xH 2 O sticky, cohesive, and B a� a� resistant to flow pH 7.35 to 7.45 A a� a� volume: 4 to 6 litres (7% body weight in O a� kg) (2.2 lb/kg). · Transport O 2 and CO 2 2% of cells in whole blood males: 40-54 (androgens stimulate production) females: 37-47 (estrogens inhibit production). · Hemoglobin (Hb): binds transports O 2 and CO 2 (280 million/RBC) Hemoglobin - Fe+2 males: 14 to 18 g/100ml; females: 12 to 16 g/dl, fetal hemoglobin has a higher affinity for O 2 .

Anemia Anemia has categorized based on morphological changes of RBC. Mean cell volume (MCV): The MCV detects changes in cell size. Decrease in (↓) MCV indicates a microcytic cell anemia, which is due to iron deficiency. Increase in (↑) MCV indicates macrocytic anemia (megaloblastic anemia), which is due to deficiency of vitamin B 12 and folic acid. Mean cell hemoglobin concentration (MCHC): Weight of hemoglobin in average red blood cell. In microcytic anemia decrease in (↓) MCHC. This is not significant in determining megaloblastic anemia. Serum ferritin: Iron stores are measured by serum ferritin. The concentration of ferritin is proportional to iron stores. Total iron binding capacity (TIBC). Increased in microcytic anemia.

Copyright © 2000-2018 TIPS Inc. Unauthorized reproduction of this manual is strictly prohibited and it is illegal to 11-5 reproduce without permission. This manual is being used during review sessions conducted by PharmacyPrep.

Pharmacyprep.com

Blood and Anemia

Anemia is characterized as deficiency of red blood cells and this can occur in three forms. Morphological changes in anemia MCV MCHC Serum ferritin TIBC Transferrin saturation Cyanocobalamine

Microcytic (Hypochromic) (Deficiency of Fe)

↓ ↓ ↓ ↑ ↓

Macrocytic (Megaloblastic) (Deficiency vit. B 12 or folic acid) ↑ No change

↓ Fe Chronic diseases Lead poisoning Thalassemia Bleeding.

↓ ↓B 12 or Folate Chronic alcohol

Normocytic (Normochromic) (Sickle cell anemia) No change No change

Aplasia Renal disease Inflammation

Transferrin: Free Fe2+ circulates binds transferrin and transports it from small intestine to its storage sites in the liver and from the liver to the bone marrow for the synthesis of hemoglobin. Hematocrit: (Hct) this is ratio of the volume of RBC to volume of whole blood.

Question Alerts!

1) What is the common cause iron deficient anemia is? 2) What type of anemia is the most common in pregnancy? 3) What is common cause anemia in pregnancy? 4) In microcytic anemia? MCV and MCHC and serum ferritin decrease. 5) What test is used to diagnose iron deficient anemia?

Serum ferritin: The major iron storage in protein in blood. · Iron storage is measured by serum ferritin (The concentration of ferritin is proportional to iron stores). · Response to oral and parenteral iron occurs at the same rate in normal circumstances. · Good reticulocyte response indicates active red cell production.

Microcytic (hypochromic) anaemia: Symptoms fatigue or dyspnea on exertion may occur during pregnancy and in infants. Mainly occurs due to iron deficiency. This is could be due to. · Low mean cell volume (MCV) · Impaired heme (protein) synthesis · Deficiency of serum iron · Increased total iron binding capacity (TIBC) · Decreased serum ferritin Iron supplements Iron salt Ferrous fumarate Ferrous sulfate Ferrous gluconate Polysaccharide-iron complex Iron dextran inj. Iron base suspension

Elemental iron 33%; 100 mg Fe/300 mg 20% 60 mg Fe/300 mg 12% 35 mg Fe/300 mg (150 mg Fe/150 mg)

TIPS Salt with Highest elemental iron Highest GI absorption Iron polysaccharide complex.

Copyright © 2000-2018 TIPS Inc. Unauthorized reproduction of this manual is strictly prohibited and it is illegal to 11-6 reproduce without permission. This manual is being used during review sessions conducted by PharmacyPrep.

Pharmacyprep.com

Blood and Anemia

Iron supplements are regulated as schedule 2 drugs (behind the counter). Iron containing 100 fL)

· · · ·

The main source of vitamin B 12 is meat and dairy product. Deficiency of vitamin B 12 or folate due to decreased of DNA synthesis. Megaloblastic anemia (large abnormal form or precursors to RBC). Impaired DNA synthesis usually due to folate or vitamin B 12 deficiency. Major causes of megaloblastic anemia include folate or vitamin B 12 deficiency this is due to, chemotherapy and alcoholism and seniors. Drugs that cause megaloblastic anemia are acyclovir, alcohol, antiepileptics (carbamazepine, valproic acid), methotrexate (dose dependent), nitrofurantoin, oral contraceptives, proguanil, sulfasalazine, trimethoprim (usually due to worsening of pre existing folate deficiency).

Vitamin B 12 deficiency: Pernicious anemia is a type of autoimmune disorder. This occurs due to deficiency of intrinsic factor. The intrinsic factor helps in absorption of vitamin B 12 in stomach/ileum. · Treatment. Parenteral vitamin B 12 supplement. Oral vitamin B 12 supplements are NOT effective in pernicious anemia due to deficiency of intrinsic factor. · Schilling Test: To detect pernicious anemia that caused by deficiency of vitamin B 12 . Urinary excretion test is used to diagnose deficiency of vitamin B 12 caused by decreased vitamin B 12 absorption (lack of intrinsic factor cause pernicious anemia). Vitamin B 12 available as oral, im/sc 100 mcg daily for 1 wk or 200 mcg weekly sc/im until Hb normalized. The daily requirement of cyanocobalamine or hydroxycobalamine 6-9 mcg. Vitamin B 12 only nutrient need gastric secretion (intrinsic factor) to be absorbed from GIT. Vitamin B 12 deficiency cause enlargement of bone marrow, RBCs, WBCs and platelets. It effects all proliferating cells. Folate Deficiency: The most often occurs in chronic alcoholics, pregnancy and drug induced folic acid deficiency by drugs such as carbamazepine, phenytoin, sulfonamides, methotrexate, dapsone, and oral contraceptives. Intestinal malabsorption caused by Giardia lamblia infection.

Copyright © 2000-2018 TIPS Inc. Unauthorized reproduction of this manual is strictly prohibited and it is illegal to 11-7 reproduce without permission. This manual is being used during review sessions conducted by PharmacyPrep.

Pharmacyprep.com

Blood and Anemia

Folic acid (folate) supplements: The daily requirement is 200 mcg. For therapy 1 mg, 5 mg is used. Folinic acid (Lucovarin) is synthetic form of folate. The folate available in natural product. In pregnancy prophylaxis folic acids 1 mg supplements should begin 1 month before conception. Folic acid supplements in pregnancy 5 mg daily po x 10-12 wks with history of neural tubule (NTD) defect reduce the risk of neural tubule defect. Certain medical conditions like type 1 DM, seizure in pregnancy therapy with valproic acid, carbamazepine, BMI >35 kg/m2, and malabsorption disorder. Folic acid: Pteridine nucleus is bonded to nitrogen of p-amino benzoic acid, and p-amino benzoic acid is bonded to glutamic acid through an amide linkage. H2N

N N

N N

O N

Question Alerts! Pteridyl ring bind with enzyme

COOH N H

Pteridyl group

Glutamyl moity

Pteridyl group analog Methotrexate

COOH p-amino benzyl group (PABA)

.

PABA analog

Trimethoprim/sulfonamides Sulphamethoxazole

Folic acid

Erothropoetins · Erythropoetins are glycoprotein produced through recombinant DNA technology. Epoetin is same with natural erythropoietin. · Therapeutic use of erythropoietins such as Epoetin and Darbepoetin are anemia. associated with cancer chemotherapy and renal disease. Oprelvekin therapeutic use is thrombocytopenia. Filgrastim. It is glycoprotein produced through recombinant DNA technology. Therapeutic. Neutropenia associated with cancer chemotherapy. Oprelvekin (IL-11). A recombinant DNA produced non-glycosylated polypeptide growth factor. Drug that cause folic acid deficiency Methotrexate

Folic acid dose 5 mg once week

Carbamazepine in pregnancy Sulfonamide antibiotics Phenytoin Dapsone

1 to 5 mg daily As needed As needed

Normocytic (Normochromic) · In normocytic anemia MCV is normal. · Due to acute hemorrhagic or RBC hemolysis due to immune or non-immune mediated. · Examples of normocytic anemia includes hemolytic anemia, sickle cell anemia, and thalassemia. Question Alerts! 1) The most common cause folic acid deficiency is diet, drugs and alcoholism. 2) Oral folic acid supplements recommended at childbearing age to decrease neural tubule defects. 3) The common source of vitamin B12 is meat and dairy products. (can be deficient in vegans in pregnancy). 4) Elderly are commonly deficient in Vitamin B12 and can cause? Megaloblastic .

Copyright © 2000-2018 TIPS Inc. Unauthorized reproduction of this manual is strictly prohibited and it is illegal to 11-8 reproduce without permission. This manual is being used during review sessions conducted by PharmacyPrep.

Pharmacyprep.com

Blood and Anemia

Hemolytic anemia: The sickle cell anemia disease is an inherited disorder caused by a defect in the gene for hemoglobin. Sickle cell anemia and thalassemia are hemolytic anemia associated with abnormal hemoglobins. Due to poor solubility of such abnormal hemoglobin in a reduced state, semicrystalline bodies are formed inside of RBC. These crystalline bodies are pointed and elongated inside of the cell, and rupture the red blood cells. Glucose-6-phosphate dehydrogenase (G6PD) deficiency hemolytic anemia: The enzyme G6PD necessary to maintain the reduce glutathione level (GSH) in red blood cells. This enzyme is necessary to prevent the hemolysis. The deficiency of this enzyme may cause severe hemolytic anemia in patients with the use of certain oxidant drugs such as primaquine, sulfonamide, nitrofurantoin, nalidixic acid, probenecid, chloroquine, and dimercaprol. Glutathione is an antioxidant, which prevents the oxidation of hemoglobin to methemoglobin. Coombs test: The coombs test is used to distinguish immune mechanism or glucose 6-phospho dehydrogenase (G6PD) deficiency anemia. In autoimmune hemolytic anemia, coombs test is positive. Example of drugs that cause false positive coombs test penicillin's, cephalosporin's and methyldopa. Aplastic anemia: (Total or partial destruction of the bone marrow). Reduced red blood cell and white blood cell and platelets (thrombocytopenia) counts is decreased. Aplastic anemia is due to inadequate production or release of myeloid stem cells. Drugs that cause cell aplasia: Azathioprine, phenytoin, isoniazid, penicillamine, chlorpropamide, chloramphenicol, erythropoietin, cephalosporin's, penicillin's, tetracycline's, insulin. Methotrexate, isoniazid, quinidine, quinine, rifampicin, sulphonylureas, methyldopa, mefenamic acid, drugs with oxidant effect on cell membrane (particularly in G6PD deficiency). Agranulocytosis Agranulocytosis is profound reduction of blood cells such as neutrophil, this can cause symptoms like fever, mouth, throat ulcers and this can lead to prostration and death. Recovery usually 2 to 3 weeks after the drug is withdrawn. Repeat exposure to causative drug is not recommended due to sensitization. Drugs that can cause agranulocytosis: Antibiotics (penicillin's, cephalosporin's, cotrimoxazole, chloramphenicol, sulphonamides), antidepressants (imipramine, clomipramine, desipramine, mianserin), antiepileptics (carbamazepine, phenytoin), anti-inflammatory (gold, penicillamine, leflunomide, sulfasalazine, NSAIDs), antipsychotics (chlorpromazine, thioridazine, clozapine), antithyroid drugs (methimazole, propylthiouracil), captopril, procainamide, and ticlopidine. Thrombocytopenia (reduced platelets to less than 150 x 109/L). May present as easy bleeding, bruising or purpura. Prolong bleeding time but INR remains normal. Usually occurs 7 to 10 days after drug administered. Stimulation of erythropoiesis: The most nutritional deficient anemia (iron, vit.B 12 and FA), have elevated levels of endogenous erythropoietins. Pharmacologic stimulation of RBC production using erythropoiesis stimulating agents such as Epoeitin alpha (Eprex), and Darbepoetin alpha (Aranesp). Used in chronic renal failure, cancer chemotherapy, HIV, hepatitis C patient receiving ribavarin.

Copyright © 2000-2018 TIPS Inc. Unauthorized reproduction of this manual is strictly prohibited and it is illegal to 11-9 reproduce without permission. This manual is being used during review sessions conducted by PharmacyPrep.

Pharmacyprep.com

Blood and Anemia

Tips 1.

Vit C& E

2.

4. 7. 10 13 16

Vit B12 Neurotubule defect megaloblastic anemia Methotrexate Phenytoin

19 22 25 28

Microcytic anemia Empty stomach diarrhea bronze disease, excessive absorption and storage of iron Primaquin

31

· · · · · · · · · · · · · · · · · · · · · · · · · · · · · · ·

3.

5. 8. 11 14 17

Anemia associated chronic renal disease Intrinsic factor thrombocytopenia iron Sulfa drugs Ferrous gluconate

20 23 26 29

Sickle cell anemia constipation melena Deferoxamine

21 24 27 30

32

Use straw

6. 9. 12 15 18

Parietal cells in stomach Skin cell cancer pernicious anemia proximal convoluted tubule (PCT) OCP Breathlessness when lying down Folic acid ferrous gluconate vomiting Penicillin's

Anemia due to deficiency of iron ( ) Vitamin supplements recommended in elderly ( ) Intrinsic factors secreted from.. ( ) Deficiency of intrinsic factors cause.. ( ) Megaloblastic anemia is due to ( ) Oprelvekin (interleukin 11) is approved for ( ) Epoietin alpha are used to ( ) Deficiency of folic acid supplements in pregnancy can cause ( ) What is the meaning of melanoma? ( ) Drug that gives folic acid deficiency ( ) Vitamin that decrease oxidative degradation ( ) Anemia in pregnancy is due to ( ) The most abundant metal in the body ( ) Site of calcium reabsorption ( ) ) Characteristic of both vitamin B 12 and folic acid deficiency ( ) Which component is required for vitamin B 12 absorption ( What is orthopnea? ( ) What iron salts have higher GI absorption? ( ) A patient G6PD deficiency, sulfa drug can cause… ( ) Moon shaped RBC are seen in.. ( ) The highest elemental iron present in…( ) A patient taking methotrexate for cancer treatment, to treat bucal ulcers give…( How do you take iron supplement? ( ) Common side effect of iron supplement ( ) Overdose symptoms of iron ( ) Iron supplement antidote ( ) Liquid iron is taken by using.. ( ) What type of anemia can cause G6PD deficiency? ( ) Drugs that induce hemolytic anemia? ( ) What is hemochromatosis? ( ) Excessive bleeding like menorrhea cause the type of anemia --> ( )

)

Copyright © 2000-2018 TIPS Inc. Unauthorized reproduction of this manual is strictly prohibited and it is illegal to 11-10 reproduce without permission. This manual is being used during review sessions conducted by PharmacyPrep.

www.PharmacyPrep.Com

Biochemistry

12 Biochemistry This chapter reviews basic and essentials of biochemistry topics such as, intermediary metabolism, carbohydrates, lipids, proteins, enzyme kinetics and porphyrins. Question Alerts! Common questions in pharmacy exam is to ask! · Primary metabolism of glucose (glycolysis, glycogenesis, gluconeogenesis, glycogenolysis). · Proteins, and aminoacids. Examples of non-essential and essential amino acids (PVT TIM HALL) · Example of essential fatty acids (Omega 3, 6 and 9). · Fatty acid oxidation, formation of ketone bodies. Catabolism: This pathway convert pyruvate (glycolysis), acetyl Co-A (fatty acid degradation), and amino acid to carbon dioxide and water with release of energy. This cycle is strictly oxygen dependent (aerobic). Catabolism examples includes glycogenolysis and glycolysis. Anabolism: This pathway forms amino acid such as aspartate and glutamate from cycle intermediates also the porphyrin ring of the heme (hemoglobin, myoglobin and cytochrome) is formed from intermediates cycle. Anabolism examples includes Glycogenesis and gluconeogenesis Fermentation: The formation of ethanol and lactate from glucose are examples of fermentation. Carbohydrates Classification · Monosaccharide's (C 6 H 12 O 6 ) Examples glucose, Fructose. · Disaccharides (C 12 H 22 O 11 ) Examples. Sucrose, lactose, and maltose · Polysaccharides. More than two monosaccharides. Examples. starch, cellulose · Oligosaccharides. 2 to10 monomers. Carbohydrate digestion and absorption: Dietary carbohydrate is digested in the mouth and intestine and absorbed from the small intestine. Disaccharides (e.g. sucrose, lactose), oligosaccharides (e.g. dextrins), and polysaccharides (e.g. starch) are cleaved into monosaccharide's (e.g. glucose, fructose).

Copyright © 2000-2018 TIPS Inc. Unauthorized reproduction of this manual is strictly prohibited and it is illegal to 12-1 reproduce without permission. This manual is being used during review sessions conducted by PharmacyPrep.

www.PharmacyPrep.Com

Biochemistry

Question Alerts! 1) What is an end product of anaerobic glycolysis? 2) What is an end product of aerobic glycolysis? 3) Definition of Gluconeogenesis is? glucose formation from non carbohydrate sources. 4) Glucose is stored in liver cells and tissue as? Carbohydrate metabolism. Glycogenesis, Glycogenolysis, Glycolysis, Gluconeogenesis. Glycogenesis (glycogen synthesis): Glycogen (glycogenesis is synthesis of glycogen from glucose. This glycogen is stored in liver and muscle.) Glycogenolysis (glycogen breakdown): Glucose (glycogenolysis is break down of glycogen to glucose). Glycolysis: Glucose à CO 2 + H 2 O Glycolysis is breakdown of glucose to water and carbon dioxide. Glycolysis occurs in the cytosol and mitochondria in most organs of the body. Cystosolic process Aerobic Glucose

Mitochondria Pyruvate --------------------à CO 2 + H 2 O

anaerobic Lactate Mitochondrial Process Pyruvate Oxidative phosphorylation

CO 2 + H 2 O

Under anaerobic conditions (absence of O 2 ): Glycolysis involves the conversion of glucose to lactate (lactic acid). This can occur in cells without mitochondria. Under aerobic conditions (presence of O 2 ). Glycolysis involves the conversion of glucose to pyruvate (pyruvic acid), this occurs in mitochondria. Citric acid cycle (Krebs cycle): This citric acid cycle pathway is also known as the Krebs cycle, which serves both breakdown and synthetic purposes, and occurs in mitochondrial compartment. Under aerobic conditions pyruvate enters mitochondria, citric acid (Krebs cycle), where it is completely oxidized to CO 2 +H 2 O, if the supply of O 2 is insufficient as in actively contracting muscles, pyruvate is converted to lactate. Mature RBC lack mitochondria, hence there is no Krebs cycle activity. In bacteria use glyoxylate cycle in place of Krebs cycle. The glyoxylate cycle, centers on the conversion of acetyl-CoA to succinate for the synthesis of carbohydrates. In microorganisms, the glyoxylate cycle allows cells to utilize simple carbon compounds as a carbon source when complex sources such as glucose are not available. The glyoxylate cycle is absent in animals.

Copyright © 2000-2018 TIPS Inc. Unauthorized reproduction of this manual is strictly prohibited and it is illegal to 12-2 reproduce without permission. This manual is being used during review sessions conducted by PharmacyPrep.

www.PharmacyPrep.Com

Biochemistry

The pentose phosphate pathway (also called the phosphogluconate pathway and the hexose monophosphate shunt) For most organisms, it takes place in the cytosol. It is a process that generates NADPH and pentoses (5carbon surgars). There are two distinct phases in the pathway. The first is the oxidative phase, in which NADPH is generated, and the second is the non-oxidative synthesis of 5-carbon sugars. This pathway is an alternative to glycolysis. While it does involve oxidation of glucose, its primary role is anabolic rather than catabolic. Gluconeogenesis Gluconeogenesis is the synthesis of glucose from non carbohydrate sources. This process, which occurs primarily in the liver and kidney is the synthesis of glucose from small noncarbohydrate precursors such as lactate and alanine.

Pyruvate (Pyruvic Acid)

aerobic

CO2 + H2O anaerobic

Lactic (Citric Acid Cycle) GLYCOLYSIS

Glycogenolysis GLYCOGEN

GLUCOSE Glycogenesis Gluconeogenesis

FAT & PROTEIN Protein metabolism: Amino acid à proteins àpeptides Amino Acids: The amino acids from proteins are precursor of compounds and energy source like converted to acetyl CoA. Amino acids degradation eliminated -NH2- group and this converts to NH3 and this may be toxic. Ammonia eliminates through conversion of urea in animals. The NH2 group is removed by transamination and oxidative deamination to urea. Urea cycle Urea is formed from NH 3 and amino (NH 2 ) group of Asp bicarbonate (HCO 3 ) in urea cycle in liver. Five enzymes involved in urea cycle, two enzyme are in mitochondria and three enzyme involved in cytosol, thereby the urea cycle partially occurs in mitochondria and partially in cytosol. Ammonia (NH 3 ) is produced in all tissue, but the urea cycle is only carried out in liver. Thus, NH 3 must be transported to liver with non-toxic form. NH 3 is converted to glutamine (Gln) which is not toxic. 20 amino acids are converted to 7 common intermediates. Those are: Alanine, Cysteine, Glycine, Pyruvate Serine, and Threonine are

glucogenic and ketogenic intermediate

Copyright © 2000-2018 TIPS Inc. Unauthorized reproduction of this manual is strictly prohibited and it is illegal to 12-3 reproduce without permission. This manual is being used during review sessions conducted by PharmacyPrep.

www.PharmacyPrep.Com

Biochemistry

degraded to Pyruvate

α-ketoglutarate SuccinylCoA. Fumarate. Oxaloacetate. Acetyl-CoA. Acetoacetate

Glucogenic intermediate (Form glucose) Ketogenic intermediates (Form ketone bodies)

Ketogenesis or fatty acids oxidation: It occurs when there is a high rate of fatty acid oxidation in the liver. Three types of substances betahydroxy butyric acid (80%), acetoacetic acid (20%) and acetone (trace amounts). These three substances are collectively known as the ketone bodies (also called acetone bodies or acetone). Enzymes responsible for ketone bodies formation are associated with mitochondria. Adipose tissue Lipolysis Triglycerides → Free Fatty Acids Ketone bodies FFA= Free fatty acids

Blood →

FFA

Beta oxidiation → Acyl CoA

Liver →

AcetylCoA→

Amino acid biosynthesis products: Amino acids are not only makes proteins and also precursor of severs products such as neurotransmitters (dopamine, nor epinephrine and epinephrine), hormones, and porphyrines. Phenylalanine à Tyrosine à levodopa à Dopamineà NorepinephrineàEpinephrine. · Tyrosine à Thyroxine (thyroid hormone). · Tyrosine à Catalization by tyrosinase give phenyl 3-4 quinone than polymerization gives Melanin (black skin pigment). · · · · · · ·

Tryptophan à 5-hydroxy tryptophan à 5-hydroxy tryptamine (5HT or serotonin). Tryptophan à Niacin Histidine àhistidine decarboxylase produce histamine (allergic response) Arginine à Nitric oxide (NO)à vasodilator Arginine à Urea Arginine à Creatinine Glutamate à catalization by glutamate decarboxylase produce Gamma amino butyric acid (GABA neurotransmitter).

Hemoglobin: The hemoglobin is protein consist of globulin + Heme (Porphyrin + Fe2+). Porphyrines: Amino acids are precursor of prophyrines, and these if are component of heme biosynthesis in mitochondria and cytosol. Porphyrins are derived from succinyl-CoA and glycine. Glycine à Porphyrine ring à Hemoglobin Glycine is only amino acid without chiral centre. There are several genetic defects in heme biosynthesis. · Uroporphyrinogen III cosynthase deficiency, congenital erythropoietic porphyria. · Red urine, reddish teeth, photosensitive skin, and increased hair growth. · Ferrochelatase deficiency = erythropoietic porphyria. Copyright © 2000-2018 TIPS Inc. Unauthorized reproduction of this manual is strictly prohibited and it is illegal to 12-4 reproduce without permission. This manual is being used during review sessions conducted by PharmacyPrep.

www.PharmacyPrep.Com

Biochemistry

Essential amino acids "PVT TIM HALL" Arginine, Histidine, Isoleucine, Leucine, Lysine, Methionine, Phenylalanine, Threonine, Tryptophan, and Valine Arginine although it is produced but most degrade to urea.

Non-essential amino acids Alanine, asparagine, Cysteine, Glutamate, Glutamine, Glycine, Proline, Serine and Tyrosine

Essential amino acids (EAAs) are the components of proteins that make them essential in the diet, of the 20 amino acids in proteins, 10 are essential i.e. required in the diet because they cannot be synthesized in the body. All humans require eight EAAs. Infants require histidine. Only essential amino acids taken through diet because they are not synthesized in body in sufficient amounts. Essential amino acids are “PVT TIMHALL“ phenylalanine, valine, tryptophan, threonine, isoleucine, methionine, histidine (in infants), arginine (in infants), leucine and lysine. ACID Base properties of amino acids. At all physiological pH all amino acids have both negative and positive charge. When pH = pKa, there is 50% ionized and 50% unionized. Amino acids can act either as an acid or base and are defined as amphoteric or ampholytes. Zwitter Ion: Amino acids are ionisable +ve ions as amines, -ve ions as acid. (no net charge) pKa values indicate the pH at which the group (acid or amine) is 50% dissociated All amino acids have two titration curves. H 3 N(+)-CH 2 -COO(-) Isoelectric Point. (pl) The pH at which there is no net charge on the structure. · At a pH> pl the structure has net negative charge · At a pH < pl the structure has net positive charge. · Every structure has one isoelectric point but can be many pKa values. Proteins Proteins composed of amino acids. proteins are formed by condensation of amino acid. Structural role within the cell and also within the connective tissue and skeleton of the whole organism. Primary structure is linear sequence of amino acids.

Each position occupied by one of 20 amino acids and linked by peptide bonds. Secondary structure occurs with hydrogen bonds Copyright © 2000-2018 TIPS Inc. Unauthorized reproduction of this manual is strictly prohibited and it is illegal to 12-5 reproduce without permission. This manual is being used during review sessions conducted by PharmacyPrep.

www.PharmacyPrep.Com

Biochemistry

forming between the carboxyl portion of amino acids and the amino group of another. Alpha helix = coiling into a helix Beta pleated sheet = a folded sheet as polypeptide folds back on itself. Weak hydrogen bonds between amino and carboxyl groups and different amino acids form at regular intervals, creating a regular structure. (Not from interactions between variable R-groups). Not all of a polypeptide forms secondary structure in most proteins. Tertiary structure: Three-dimensional structure formed by disulfide (S-S) bonds. · Interactions between variable R-groups forming. · Hydrophobic interactions between nonpolar amino acids. · Hydrogen bonds between polar amino acids · Ionic bonds between ionic amino acids · covalent bonds between sulfur containing amino acids. Producing the three-dimensional folded structure of most proteins. Quaternary structure: A protein consisting of more than 1 amino acid chain. Separate polypeptide chains that assemble together. Aggregations of polypeptides form interactions between more than one polypeptide. Types of proteins: There are 20 amino acids commonly found in proteins, they are linked together by peptide bonds. Protein is generally classified into three different categories. Simple protein, conjugated protein and derived protein. Simple protein is naturally occurring proteins, which upon hydrolysis yield only alpha-amino acids such as albumins, globulins, prolamines, glutelins, and albuminoids. Plasma proteins: albumin, globulins. Conjugated protein: Conjugated proteins are further classified on the nature of their prosthetic groups polypeptides Polypeptides Polypeptides Polypeptides Polypeptides Polypeptides Polypeptides

+ non-proteinaceous Metals (chromoprotein) Metalloprotein Vitamins Nucleic acids Carbohydrates Lipids Phosphoric acid

Example Iron in hemoglobin, colored group Enzyme cofactors Ribosomes Glycoproteins, mucins Lipoproteins Casein, ovovitellin

Derived proteins: They are formed from primary or conjugated proteins by the actions of the acid, alkali, heat, water, enzyme or alcohol. They generally differ in physical and chemical properties from the protein they are derived from. They are subdivided into primary derived protein (denatured protein) or secondary derived protein. Denaturation of proteins: A protein denaturation results in the unfolding and disorganization of the protein structure, which does not occurs by hydrolysis. Denaturising agents include: heat, organic solvents, mechanical mixing, strong acids or base, detergents and ions of heavy metals such as lead or mercury. Denatured proteins are insoluble and precipitate. Denaturation process is often irreversible.

Copyright © 2000-2018 TIPS Inc. Unauthorized reproduction of this manual is strictly prohibited and it is illegal to 12-6 reproduce without permission. This manual is being used during review sessions conducted by PharmacyPrep.

www.PharmacyPrep.Com

Biochemistry

Globular and Fibrous proteins: Globular hemeproteins. A hemeproteins are groups of specialized proteins that contain heme as tightly bound prosthetic groups. The most common heme proteins in human are haemoglobin and myglobin. These proteins bind oxygen reversible. These proteins have high affinity to carbon monoxide. Hemoglobin complex of porphyrin ring and ferrous ion (Fe2+). Transport oxygen in blood only. Carbon dioxides and carbon monoxide also binds with haemoglobin reversible. Myoglobin. complex of porphyrin ring and ferrous ion (Fe2+). Transport oxygen in tissues. Myoglobin present in heart and skeletal muscles. Hemoglobinopathies. Example of hemoglobinopathies. Sickle cells anemia, haemoglobin C disease (HbC) and the thalassemia syndrome. Cell Membrane · Composed of lipid bylayer with proteins embedded within the membrane · Lipid bilayer- Phospholipids contain hydrophilic head (outer portions of membrane) and Hydrophobic chains (inner portions of membrane). · Membrane proteins functions are either transport mechanisms or as receptors. · The extrinsic proteins bound to outer and inner portions of membrane, easily removed. · Intrinsic protein strongly bound in the membrane, cannot easily removed. May extend completely through the membrane.

Question Alerts!

1) Amino acid are linked with -CONH- peptide bond to form proteins. 2) Caseine? Phosphoprotein 3) Keratin is an albuminoid present in? 4) Sphingosine are? Enzymes: Protein catalysts substance that alter the rate of a chemical reaction without itself being permanently changed into another compound. Fats and Lipids Fatty Acid Synthesis: Palmitate is an end product. Associated with hexose monophosphate (HMP) Shunt. Lipids can be divided into five classes according to their chemical structure Example Glycolipids Cerebrosides Spingosine + carbohydrate Gangilosides sphingolipids + sialic acid or choline phosphate Glycolipids: Also known as cerebrosides. They are isolated from the brain. Upon hydrolysis, they yield fatty acid, galactose and sphingosine. They are also known as galactolipids due to the presence of galactose, such as phrenosin, and kerasin. Glycosolipid metabolism takes place in cytoplasm. Cerebrosides = sphingosine + lipids + sugars Gangliosides = sphingolipids + sialic acid or choline phosphate

Copyright © 2000-2018 TIPS Inc. Unauthorized reproduction of this manual is strictly prohibited and it is illegal to 12-7 reproduce without permission. This manual is being used during review sessions conducted by PharmacyPrep.

www.PharmacyPrep.Com

Biochemistry

Cerebroside are? They occur in myelin sheath of nerves and white matter of the brain tissues and cellular membrane. Important for nerve conduction. Cerebroside are glycolipids contain b-galactose (carbohydrate) + sphingosine + fatty acid but no phosphoric acid. Sphingolipids contains sphingosine formed from palmitoyl CoA and serine. Sphingosine forms ceramide backbone when joined to fatty acids. The addition of sugars, sialic acid or choline phosphate forms compounds such as cerebrosides, gangliosides or sphingomyelin found in nerve tissues and membrane. Phospholipids: Known as phosphatides. They are esters that consist of fatty acid, phosphoric acids and nitrogenous compounds, such as lecithin. These are important part of membrane. Sterols (steroids): The sterols are alcohols structurally related to steroids. They are obtained from plants and animals such as cholesterol and ergosterol. Steroid structures have 3 cyclohexane rings and 1 cyclopentane ring. Steroids are converted to compounds such as bile acids, vitamin D and steroidal hormones. They are not broken down completely. Waxes: Waxes are defined as high molecular weight esters. They consist of monohydric alcohol and high molecular weight of fatty acids. Fixed oils and fats: They are esters of glycerol and fatty acids, such as olive oil. Fixed oils, such as hard, which are solid at room temperature, are known as fat. Lipid metabolism: Catabolism triglycerides stored in fat cells are hydrolyzed by hormone, sensitive liposes into 3 fatty acids and glycerol. · Fatty acids- are broken down by B-oxidation to Acetyl CoA (2Carbon Units), which enter the Kreb cycle to complete oxidation to CO 2 + H 2 O with release of considerable energy, too rapid breakdown of fatty acids leads to ketone bodies (Ketogenesis) as in diabetes Mellitus. · Glycerol enters glycolysis and is oxidizes to pyruvate and via the Krebs cycle to CO 2 + H 2 O. · Steroids may be converted to other compounds such as bile acids, vitamin D or steroidal hormones, they are not broken down completely. Anabolism. Biosynthesis forms fatty acids, steroids and other terpene-related metabolites. Fatty acids are formed in the cytoplasm and unsaturation occurs in the mitochondria or endoplasmic reticulum. Human cannot make Linoleic acid. Terpende compounds- are derived from Acetyl CoA via mevalonate and include: Cholesterol and other steroids, Fat-soluble vitamins (A,D,E and K) and Bile acids. Sphigolipids contains sphinegenine formed from palmitoyl CoA and serine. Sphingenine forms ceramide backbone when joined to fatty acids. The addition of sugars, sialic acid or choline phosphate forms compounds such as cerebrosides, gangliosides, or sphigomyelin found in nerve tissues and membranes. Phosphatidyl compounds- such as phosphatidyl choline (Lecithin), phosphatidyl serine or ethandamine are also important parts of membranes. Biosynthesis of lipids, regulation by insulin, glucagon, and atherosclerosis: Cholesterol production is regulated by intracellular cholesterol concentration and by the hormones glucagon and insulin. The rate-limiting step in the pathway to cholesterol is the conversion of (3-hydroxy-(3)-methylglutaryl-CoA (HMG-CoA) into mevalonate, and the enzyme that catalyzes this reaction. HMG-CoA reductase is a complex regulatory enzyme whose activity is modulated over a 100-fold range. It is allosterically inhibited by as yet unidentified derivatives of Cholesterol Copyright © 2000-2018 TIPS Inc. Unauthorized reproduction of this manual is strictly prohibited and it is illegal to 12-8 reproduce without permission. This manual is being used during review sessions conducted by PharmacyPrep.

www.PharmacyPrep.Com

Biochemistry

OMEGA Omega 3

Double bond 3

Linolenic acid (alpha linolenic acid) ALA EPA DHA Omega 6 Linoleic acid (LA) 2 Gamma linoleic acid (GLA) Arachidonic acid (ARA) Omega 9 Oleic acid (OA) 1 and of the key intermediate mevalonate. HMG-CoA reductase is also hormonally regulated. The enzyme exists in phosphorylated (inactive) and dephosphorylated (active) forms. Glucagon stimulates phosphorylation (inactivation), and insulin promotes dephosphorylation, activating the enzyme and favoring cholesterol synthesis. HMG-CoA reductase AcetylCoA --> B-hydroxy, B-methyl, glutaryl CoA --> --> --> Mevalonate --> Cholesterol When the sum of the cholesterol synthesized and obtained in the diet exceeds the amount required for the synthesis of membranes, bile salts, and steroids, pathological accumulations of cholesterol in blood vessels (atherosclerotic plaques) resulting in obstruction of blood vessels (atherosclerosis). Essential fatty acids: Essential fatty acids (EFAs) are required in amounts equalling 6 to 10% of fat intake (equivalent to 5 to 10 g/day). 15

12

9

1 COOH

Alpha-Linolenic Acid (omega-3)

They include -6 (n-6) fatty acids are linoleic acid (cis-9, 12-octadecadienoic acid) and arachidonic acid (cis-5, 8,11,14-eicosatetraenoic acid) and -3 (n-3) fatty acids are linolenic acid (cis-9, 12, 15-octadecatrienoic acid), cis-5, 8,11,14,17-eicosapentaenoic acid (EPA), and cis-4,7,10,13,16,19-docosahexaenoic acid (DHA). EFAs must be provided by the diet. Vegetable oils provide linoleic acid and linolenic acid, and marine fish oils provide eicosapentaenoic acid and docosahexaenoic acid. However, some EFAs can be made from others. For example, the body can make arachidonic acid from linoleic acid, and eicosapentaenoic acid (EPA) and docosahexaenoic (DHA) acid can be partially synthesized from linolenic acid, although fish oil is a more efficient source. Fats (worst to best): Trans fats > Saturated > Cholesterol > Monounsaturated > polyunsaturated (Omega). Essential trace elements Trace element Complication associated with deficiencies Copper Wilson’s disease (excess), leucopenia, neutropenia (deficiency), Minks’ (defect of Cu+). Penicillamine is used for treatment of Wilson’s disease. Copper present in: Cytochrome oxidase Iron The most abundant metal in body. Deficiency of iron microcytic anemia, and hypochromic anemia Hemochromatosis (excess of iron). Enzyme or proteins contain iron: Hemoglobin, myoglobin, Cytochrome oxidase, myeloperoxidase. Copyright © 2000-2018 TIPS Inc. Unauthorized reproduction of this manual is strictly prohibited and it is illegal to 12-9 reproduce without permission. This manual is being used during review sessions conducted by PharmacyPrep.

www.PharmacyPrep.Com Zinc Selenium Chromium Molybdenum Iodine

Biochemistry

Children: poor growth, impaired sexual development (deficiency) Adults: dermatitis (alcoholics). Selenium deficiency can cause cardiomyopathy. Impaired glucose tolerance Present in xanthine oxidase enzyme which catalyzes conversion of purine to uric acid Deficiency of iodine may cause goiter disease

Tips 1. 4. 7. 10 13

· · · · · · · · · · · · · · · · · · · · · · · · ·

linolenic (Omega 3) 2. linoleic (omega 6) 3. Arachidonic Arginine 5. Oxidized hemoglobin 6 HMG Co- Reductase Methylated hemoglobin 8. Prostaglandin 9. Ferrous Hemoglobin 11 Myoglobin 12 Cytochrome oxidase Excessive phenylalanine in 14 Tryptophan the urine Glycolysis; Glucose→(gives H 2 O and CO 2 ) Glycogenesis; Glucose→( to glycogen formation) Glycogenolysis; Glycogen→(breakdown to glucose) Gluconeogenesis: fats & proteins→(formation to glucose) Nitric oxide (NO) is a derivative of what amino acid? (Arginine) The serotonin is produced by? (Tryptophan) Phenylketonuria (PKU) is? (Excessive phenylalanine) Cholesterol synthesis rate limiting step is catalyzed by… (HMG Co Reductase). The most basic amino acid (Arginine). Zwitter ionà amino acid with negative and positive charge. All amino acids have two titration curve. Isoelectric point (pl)à the pH of solution, where an amino acid exhibit zwitter ion At pH>pl the structure has net negative charge At pH75,000 IU associated with osteoporosis bone resorption. Newborn (infant) on breast-feeding should get vitamin D drops. Diet Cholecalciferol

7-Dehydrocholesterol Skin (ultraviolet)

↓ [ Ca2+] Liver - PTH 25-OH-cholecalciferol (vit. D 2 ) Storage from of vitamin D ↓ [ phosphate] (+) Kidney 1, 25-(OH) 2 -cholecalciferol (Active vit. D 3 )

24, 25-(OH) 2 -cholecalciferol (Inactive)

1) Vitamin D 3 is steroidal hormone 2) What vitamin is deficiency in chronic renal failure? 3) What is dose of vitamin D for > 50y ? 800 U 4) Infants on breast-feeding have deficiency of? Vitamin D drops 5) Active vitamin D is? D 3 (1,25 dihydroxy cholecalciferol) 6) Deficiency of Vitamin D? Osteomalacia, osteoporosis, rickets. a-Tocopherol (vitamin E) Act as a natural antioxidant by scavenging free radicals and molecular oxygen. Storage site of vitamin E is in adipose tissue (fatty tissues). Alpha tocopherol is the strongest antioxidant among all tocopherols. Occurs only with fat malabsorption. Dietary requirements: Increased intake of vitamin E is recommended in premature infants fed formulas that are low in vitamin as well as in persons consuming a diet high in poly saturated fatty acids. Symptoms Increase in red blood cell fragility (increase cell integrity) Deficiency: peripheral neuropathy, retinopathy, skeletal muscle atrophy. Vitamin K Vitamin K 1 (Phylloquinone) is derived from green vegetables Vitamin K 2 (Menaquinone) is produced by intestinal bacteria. Vitamin K 3 is a synthetic menadione. When vitamin K 3 is administered. It will be alkylated to one of the vitamin K 2 forms of menaquinone. Dietary requirements: It maintains normal levels of blood clotting protein factors 2, 7, 9, 10, and protein C, and protein S. Copyright © 2000-2018 TIPS Inc. Unauthorized reproduction of this manual is strictly prohibited and it is illegal to 13-7 reproduce without permission. This manual is being used during review sessions conducted by PharmacyPrep.

www.PharmacyPrep.Com

Nutrition

Deficiency Hemorrhagic syndrome (for infant’s due to less bacterial in colon), bleeding, elevated prothrombin time. Drug (¯)Antidote of warfarin Fat diet

Bacteria (colon) Inactive vitamin K (oxidized)

Liver epoxide reductase

warfarin

Active vitamin K (reduced) 1) What vitamins are synthesized by bacteria? Vitamin K 2 (Menaquinone) 2) What are vit. K clotting factors! 2,7, 9, 10 (01972) 3) Vitamin K is essential for prothrombin in liver. 4) Vir. K sources? Dark green vegs and synthesis by intestinal bacteria. 5) Vitamin K? Facilitates normal blood coagulations. 6) Warfarin antidote is? Vit. K 7) Protein C and cofactor protein S? Localize blood clot 8) Some new born babies get the vitamin K injections.

Tips

· · · · · · · · · · · · · ·

·

1. 4. 7.

Vitamin D Vitamin D3 neurotubule defect

10

25 hydroxycholecalciferol Vitamin D2 in liver

2 5 8

Vit B 12 ileum Vitamin A

3 6 9

Trans 1, 3 retinoic acid (Vit A) folic acid 1,25 dihydrocholecalciferol Vit D 3

What vitamin is found only in animal products? ( ) The most common vitamin deficiency in United States and Canada ( ) What is precursor acid ? ( ) Active form of vitamin D is? ( ) Storage form vitamin D is? ( ) Supplement of folic acid in early pregnancy reduces? ( ) Sun exposed skin forms the type of vitamin D is? ( ) Retin A is topical used for wrinkles and acne is isomer of? ( ) Vitamin A, D, E, K absorption takes place in what part of gastrointestinal tract? ( ) All B-complex vitamin washouts from body except? ( ) what vitamin is essential for the synthesis of nitrogenous bases in DNA and RNA? ( People who do not eat from animal sources have deficiency of? ( ) what vitamin overdose causes toxicity? ( ) Chronic alcoholics have deficiency of? ( ) Chronic renal disease patient should receive vitamin à ( )

)

Copyright © 2000-2018 TIPS Inc. Unauthorized reproduction of this manual is strictly prohibited and it is illegal to 13-8 reproduce without permission. This manual is being used during review sessions conducted by PharmacyPrep.

www.PharmacyPrep.Com

Nutrition

Select True/False Statement · · · · · · · · · · · · · · · ·

Deficiencies in newborns treated for hyperbilirubinemia by photo therapy; Riboflavin Niacin is not a true vitamin. (True/False) Niacin; derived from the amino acid tryptophan. (True/False) Pellagra is due to deficiency of vitamin B 3 (niacin) (True/False) Pernicious anemia is due to vitamin B 12 deficiency. (True/False) Pteridine ring structure is present in folic acid (True/False) Scurvy is due to deficiency in vitamin C. (True/False) b-carotenoids is precursor of vitamin A. (True/False) Vitamin D deficiency in children is rickets and in adults is osteomalacia.. (True/False) Vitamin D supplements are recommended in newborn that are on breast-feeding. (True/False) Folic acid supplements are now recommended for pregnant women to prevent neural tube defects (spina bifida) in their children. (True/False) Omega 6 is Lenoleic acid. (True/False) Omega 3 is Lenolenic acid à act like aspirin à Antiplatelets. (True/False) Lenolenic acid mainly present in fish and walnut. (True/False) Vitamin E toxicity à More than 1100 units (average capsule is 400 units) à Prevent the synthesis vitamin K coagulant factors (act as anticoagulant). (True/False) Severe Vitamin B 1 thiamine deficiency; Beriberi and Wernicke-Korsaskoff syndrome. (True/False)

Copyright © 2000-2018 TIPS Inc. Unauthorized reproduction of this manual is strictly prohibited and it is illegal to 13-9 reproduce without permission. This manual is being used during review sessions conducted by PharmacyPrep.

www.PharmacyPrep.Com

Nutrition

Copyright © 2000-2018 TIPS Inc. Unauthorized reproduction of this manual is strictly prohibited and it is illegal to 13-10 reproduce without permission. This manual is being used during review sessions conducted by PharmacyPrep.

www.PharmacyPrep.com

Microbiology

14 Microbiology Questions Alerts! Common questions in pharmacy exam is to ask! · Endotoxin (pyrogen) of gram negative and exotoxins gives positive organism · Infective organism of common infections like pneumonia, traveler's diarrhea, toxic shock syndrome, endocarditis, cellulites, meningitis, syphilis, athlete’s foot and warts. · Herpes virus: HSV1, HSV2, VZV, Epstein bar, and CMV · Hepatitis: Hepatitis A, B, C · Influenza A and B · HIV transmission Fungal infections: Candidiasis (yeast), Athletes foot Bacterial Structure · Bacteria. Contain cell membrane and cell organs · Bacterial nucleus: Not surrounded by cell membrane · Bacterial ribosome are 30S, 50S, and 70S. · Cell membrane consist of cytochrome and lipids and enzymes. · Mesosomes convoluted invagination of mitochondria. · Plasmid (bacterial resistant) is closed circular extra chromosomal DNA. · Endospore = Metabolically inactive cell. Contain calcium dipicolinate (resistant to sever environmental conditions). · External layer = Capsule (resistant to phagocytosis) · Cell wall = Portion external to cell membrane, osmotic protection. · Peptidoglycan = Present in cell membrane of Gram –ve & +ve · Mucopeptide = (protein + carbohydrate) is a peptidoglycan · Techoic acid = Water-soluble polymer. Present in gram +ve only · Periplasmic space = Found in gram +ve cell, between cell membrane and outer cell membrane contains proteins. · Outer membrane = Present in Gram –ve, phospholipid layer, embedded proteins/porins. · Lipopolysaccharide present in Gram –ve, consist of lipid A, also known as endotoxin. · Glycocalyx present in external layer, Slime layer, and adhesive. · Appendages: Flagella, Pili/Fimbriae, ordinary pili or sex pili. · Bacterial growth curve lag increase in individual size (many nutrient) · Exponential or log is increase in population. · Stationary division. Death (accumulate toxin, decrease in nutrient). Copyright © 2000-2018 TIPS Inc. Unauthorized reproduction of this manual is strictly prohibited and it is illegal to 14-1 reproduce without permission. This manual is being used during review sessions conducted by PharmacyPrep.

www.PharmacyPrep.com · · · · ·

Microbiology

Obligate aerobe or facultative bacteria: Generate H 2 O 2 , act as bactericidal. Detoxification mechanism by Superoxide dismutase. Enzyme to neutralize peroxide like hydrogen peroxide (H 2 O 2 ). Obligate anaerobe has no superoxide dismutase. (No detoxification mechanism so bacterial death). Facultative anaerobe: Most pathogenic bacteria, can shift from fermentative to respiratory metabolism. Aerotolerant anaerobes: Similar to facultative, remains fermentative. Capnophilic anaerobes: Require CO 2 example bacteroid fragilis (gram –ve anaerobe)

Oxygen requirement Obligate aerobe. Generate H 2 O 2 , it is bactericidal. Super oxide dismutase enzyme neutralizes H 2 O 2 . · · Obligate anaerobe. No super oxide dismutase, killed by O 2 . · Facultative anaerobe: Most pathogenic bacteria can shift from fermentative to respiratory metabolism. · Aerotolerant anaerobe: Similar to facultative to remains fermentative. Bacterial shapes: Round (coccus), rod like (bacillus), and spiral (spirochete).

· · · ·

·

Virus § No cell membrane and consist of DNA or RNA and proteins. § Some viruses have single strand of DNA. Fungi § Cell membrane contain ergosterol layer. Protozoa § Protozoa are unicellular or single cell organisms and classified based on flagellates. Atypical bacteria § Mycoplasma: Have no cell wall § Rickettsia: Can be transmitted by ticks, mites etc. § Chlamydia: Lack ATP synthesis. Mycobacterium § Cell membrane contain mycolic acid layer. § Acid-fast test detects mycobacteria.

Gram +ve Stain blue or purple Techoic acid Peptidoglycan layer is thick Exotoxin is a metabolic product Exotoxin is thermolabile (heat sensitive) destroyed at a temperature over 60 oC

Gram –ve Stain red or rose pink Lipopolysaccharide (LPS) in outer membrane Peptidoglycan layer is thin Endotoxin is a metabolic product Endotoxin is more toxic than exotoxin, destroyed at higher temperatures.

Copyright © 2000-2018 TIPS Inc. Unauthorized reproduction of this manual is strictly prohibited and it is illegal to 14-2 reproduce without permission. This manual is being used during review sessions conducted by PharmacyPrep.

www.PharmacyPrep.com

Microbiology

GRAM +ve and aerobic bacteria Gram + ve cocci Streptococcus (in short Strep.) Catalase (-ve) S. pyogenes (Group A) S. agalactiae (Group B) S. bovis (Group D) S. pneumonia (alpha hemolytic) S. viridans (alpha hemolytic) Staphylococcus (in short Staph.) S. aureus (Coagulase +ve) S. saprophyticus S. epidermidis Enterococcus Gram +ve bacilli Cyanobacteria diphtheriae Listeria monocytogenes Bacillus cereus

GRAM –ve and aerobic bacteria Gram –ve cocci NISSERIA N. gonorrhea N. meningitis Moraxella catarrhalis Gram –ve bacilli (rods) Escherichia coli (E. coli) Klebsiella pneumonia Enterobacter spp. Shigella Proteus mirabilis Salmonella S. typhi S. enteritidis

GRAM +VE and Anaerobic bacteria Clostridium C. perfringens C. tetani C. difficile C. botulinum

Vibrio cholerae Pseudomonas aeruginosa Pasteurella multocida H. influenza H. duceryl Legionella pneumophila Yersinia Y. enterococolitica Y. pestis GRAM –VE and Anaerobic bacteria Fusobacterium Bacteroides B. fragilis

(Coagulase +ve) Staph.

(Coagulase –ve) Strep. Strep categorized as Non-hemolytic: S. viridians Hemolytic: Group A (Strep. pyogenes) (GAS), Group B (Strep. pneumonia), and Group C.

Copyright © 2000-2018 TIPS Inc. Unauthorized reproduction of this manual is strictly prohibited and it is illegal to 14-3 reproduce without permission. This manual is being used during review sessions conducted by PharmacyPrep.

www.PharmacyPrep.com

Microbiology

Infections causative organisms

GRAM +VE BACTERIA Gram +ve Cocci Staphylococcus aureus Catalase positive

Beta-hemolytic streptococci e.g. Strep pyogenes (Group A Strep: GAS)

Found on the skin and in the nose Boils, and Septicemia Food poisoning Wound infections, skin infections impetigo, and cellulitis. Toxic shock syndrome (TSS) Tonsillitis, Cellulites, impetigo. Scarlet fever, Septicemia may cause immune-mediated disease (e.g. rheumatic fever).

Alpha-hemolytic Streptococcus pneumonia

Pneumonia (CAP), Otitis media, Meningitis, Sinusitis, Pharyngitis

Alpha-hemolytic Streptococci viridans

Endocarditis, Dental caries Instrument contamination. Catheter infections, UTI

S. epidermidis Gram +ve bacilli Corynebacterium diphtheria Clostridia sp. Cl. tetani Cl. perfringens Cl. botulinum Cl. difficile

Penicillin G and Penicillin V, cefalexin Clarithromycin Erythromycin Azithromycin Penicillin G, cloxacillin, cefalexin Amoxicillin Penicillin G Clarithromycin Azithromycin Amoxicillin, Penicillin G, Clindamycin

Diphtheria (disease due to toxin production)

Erythromycin or penicillin's (to eliminate carrier state) Tetracycline

Tetanus, Gas gangrene Botulism Pseudo membranous colitis

Metronidazole, or vancomycin (NOT used aminoglycosides)

Copyright © 2000-2018 TIPS Inc. Unauthorized reproduction of this manual is strictly prohibited and it is illegal to 14-4 reproduce without permission. This manual is being used during review sessions conducted by PharmacyPrep.

www.PharmacyPrep.com

Microbiology

GRAM -VE Gram-ve bacilli

E. coli Proteus sp. Klebsiella sp

S. typhi Salmonella sp Shigella sp.

Pseudomonas aeruginosa

Hemophilus influenza

Urinary tract infections (90%), Traveler's diarrhea Wound infection, sepsis. Normal inhabitants of the gut. Enteric fever (typhoid), food poisoning Most sp. are animal pathogens (e.g. eggs etc). S. typhi infects man only, causes typhoid. Dysentery (bloody diarrhea or shigellosis). Nosocomial (hospital acquired) and opportunist infections (most common S. aureus).

Pneumonia, meningitis, Otitis media

Uncomplicated UTI. Sulfa drugs (cotrimoxazole), nitrofurantoin, Trimethoprim, fluoroquinolones (cipro, norfloxacin, Ofloxacin) E. coli (diarrhea). Ciprofloxacin, and Levofloxacin,

Chloramphenicol (typhoid) Ciprofloxacin Ciprofloxacin Carbapenems (imipenem, meropenem) Aminoglycosides +/Ampicillin, Ceftazidime (3rd) Clarithromycin, Azithromycin Ampicillin, amoxicillin Tetracycline

GRAM -VE COCCI Gram -ve cocci

Neisseria meningitides

Meningococcal meningitis +/- shock commensal of upper respiratory tract

Neisseria gonorrhea

Gonorrhea (STIs). Always pathogenic.

Chlamydia trachomatis

Chlamydia

Penicillin G Ceftriaxone, cefuroxime Na Rifampin Cefixime Ceftriaxone im/iv Ciprofloxacin Levofloxacin Ofloxacin Azithromycin 1g

Copyright © 2000-2018 TIPS Inc. Unauthorized reproduction of this manual is strictly prohibited and it is illegal to 14-5 reproduce without permission. This manual is being used during review sessions conducted by PharmacyPrep.

www.PharmacyPrep.com

Microbiology

ACID FAST BACILLI Acid-fast bacilli

Mycobacterium tuberculosis

Tuberculosis. (Weight loss, coughing, fever, sweating, chest pain) The most common cause of infectious death world-wide.

Mycobacterium leprae

Leprosy

Isoniazid Rifampicin Streptomycin Ethambutol Pyrazinamide Dapsone Rifampicin

SPIROCHETE Spirochetes

Syphilis (STIs) (genital ulcers or chancres), single large ulcer, and painless. Penetrate through broken skin or mucus Treponema pallidum membrane usually through sexual contact. Genital herpes ulcers are often multiple, small and painful.

Penicillin G im inj. Doxycycline

FUNGI Fungi

Candida albicans (yeast)

Tinea pedis

Have thick, ergosterol containing cell walls and grow in humans as budding yeast cells and slender tubes (hyphae). Thrush, mucocutaneous infection, vulvovaginitis Athlete’s foot

Sporotrichosis or granulomas

Abscesses (puss)

Dermatophytes

Ringworm

Aspergillus sp. Cryptococcus neoformans

Allergic reactions, opportunistic infections Meningitis in immunocompromised

Nystatin Clotrimazole Miconazole Fluconazole Clotrimazole, miconazole Tolnaftate (topical)

Skin, nail and hair infections, sometimes acquired from animals. Ubiquitous airborne filamentous fungus Present in soil and pigeon droppings

Copyright © 2000-2018 TIPS Inc. Unauthorized reproduction of this manual is strictly prohibited and it is illegal to 14-6 reproduce without permission. This manual is being used during review sessions conducted by PharmacyPrep.

www.PharmacyPrep.com

PROTOZOA Protozoa Plasmodia sp Giardia lamblia Entamoeba histolytica and Giardia lamblia (intestinal protozoa)

VIRUS DNA viruses Adenoviruses

Herpes viruses Herpes simplex virus Herpes zoster Cytomegalovirus (CMV) Epstein-Bar (EB virus)

Hepadnavirus. Hepatitis B Parvovirus: parvovirus B

Microbiology

Malaria. Four sp. infect man via biting female anopheles mosquito. Low grade gastrointestinal disease: giardiasis. Amoebic dysentery (are infective when swallowed, traveler’s diarrhea). Severe, may invade and spread to the liver.

Conjunctivitis, Sore throat HSV1 and HSV2 can cause oral and genital lesions. HSV1 causes cold sores and Keratoconjunctivitis. VZV can cause (Varicella: chickenpox, zoster: Shingles), glandular fever, Roseola infantum (sixth disease) Hepatitis B Transmitted via blood and body fluids and sexual contact. Slapped cheek disease, (fifth disease, and erythema infectious).

Papovaviruses: papillomavirus, Polyomavirus

Warts, cervical cancer, Hemorrhagic cystitis.

Poxviruses

Molluscum contagiosum, smallpox

Chloroquine, Mefloquine Primaquine, Doxycycline Metronidazole Metronidazole Ciprofloxacin Cotrimoxazole

Acyclovir Famciclovir Valacyclovir Foscarnet Ganciclovir

Interferon alpha Can cause aplastic crises HPV vaccine. Implicated in cancer of the cervix. Vaccine Gardasil. Quadrivalent human papillomavirus (types 6, 11, 16, 18) recombinant vaccine.

Copyright © 2000-2018 TIPS Inc. Unauthorized reproduction of this manual is strictly prohibited and it is illegal to 14-7 reproduce without permission. This manual is being used during review sessions conducted by PharmacyPrep.

www.PharmacyPrep.com

Microbiology

RNA VIRUS RNA viruses Orthomyxoviruses: Influenza A and B

Flaviviruses. Yellow fever, Hepatitis C Paramyxoviruses: parainfluenza, RSV, Measles Mumps Picornaviruses: Enteroviruses (e.g. poliovirus), rhinoviruses, Hepatitis A Reoviruses: rotavirus Retroviruses: HIV-1, 2 HTLV I, II Rhabdoviruses: rabies Togaviruses: Rubella, Alpha viruses

Amantadine, Ribavirin, Rimantadine (influenza A). Neuraminidase inhibitors. Oseltamivir (A and B) Zanamivir (A and B).

Influenza (flu) Yellow fever, chronic hepatitis Respiratory infections: Croup Measles, mumps Meningitis, Common cold (rhinoviruses) Hepatitis Gastroenteritis AIDS, T-cell leukemia, Spastic paraparesis Rabies German measles (Rubella), Encephalitis

May be severe in infants Rhino has runny nose NRTI, NNRTI Protease inhibitors Zoonotic infection

Tips Find answers for the tips from the following table: 1. S. pneumonia 2. Chlamydia trachomatis 4. S. aureus 5. Treponema pallidum 7. E.coli 8. H. pylori 10. Haemophilus 11 M. catarhalis influenza 13. Herpes simplex 14. Cytomegalovirus virus 16. Herpes zoster 17 Influenza A& B 19. Eptein barr 20 Shigella · · · · ·

3.

Pseudomonas aeruginosa

6. Corynebacterium 9. Borrelia burgdorferi 12. Group B Strep 15. Rubella 18. N. meningitis 21. Compylobacter jejuni

E. coli is classified as? à Beta hemolytic bacteria example is? à Toxic shock syndrome is caused by? à Lyme disease is caused? à Techoic acid is present in? à

Copyright © 2000-2018 TIPS Inc. Unauthorized reproduction of this manual is strictly prohibited and it is illegal to 14-8 reproduce without permission. This manual is being used during review sessions conducted by PharmacyPrep.

www.PharmacyPrep.com · · · · · · · · · · · · · · · · · · · · · · · · · · · ·

Encephalitis is? à Chlamydia neonatrum (C. trachomatis) is? à Non-gonococcal infections that cause UTI are? Diphtheria is caused by? à Syphilis is caused by? à Antrax is caused by? à What bacteria catalase degrades H 2 O 2 ? à Examples of live attenuated vaccines? Example of killed vaccine --> Viral diarrhea is caused by à The most common cause of community acquired pneumonia ( The most common cause of subacute endocarditis ( ) The causative organism of syphilis ( ) The causative organism of lyme disease ( ) Tick born infection is ( ) The causative organism of bacterial diarrhea ( ) The causative organism of otitis externa ( ) The most common pathogen isolated from middle ear ( Infection when aspiration of ear is performed. ( ) The most common cause of bacterial meningitis ( ) The most common cause of encephalitis ( ) Example of gram +ve bacilli ( ) The causative organism of sinusitis ( ) The causative organism of nosocomial (hospital) infections ( A types of herpes virus include,( ) Causative organism of shingles ( ) Causative organism of seasonal flu ( ) Inoculum effect?

Microbiology

)

)

)

Copyright © 2000-2018 TIPS Inc. Unauthorized reproduction of this manual is strictly prohibited and it is illegal to 14-9 reproduce without permission. This manual is being used during review sessions conducted by PharmacyPrep.

www.PharmacyPrep.com

Microbiology

Copyright © 2000-2018 TIPS Inc. Unauthorized reproduction of this manual is strictly prohibited and it is illegal to 14-10 reproduce without permission. This manual is being used during review sessions conducted by PharmacyPrep.

www.PharmacyPrep.Com

Cell and Molecular Biology

15 Cell and Molecular Biology Questions Alerts! Common questions in pharmacy exam is to ask! · Cell and cell organs · Nucleic acids DNA and RNA bases, nucleotide (phosphate, sugar and base). · Complimentary base pair (A-T and C-G). · Transcription (DNA to mRNA) and translation (mRNA to rRNA) in protein synthesis. · Sequence of protein synthesis DNA --> mRNA --> tRNA --> rRNA --> protein synthesis. · DNA recombination methods and role of plasmid. What is cDNA? · What is gene therapy? Antisense technology · Cloning? This chapter reviews basics of chromosomes, gene, nucleic acids, DNA structure and functions, replication, Mutations and recombination. RNA structure and functions, transcription from DNA and translation to synthesize proteins. Also review topics such as recent development of gene cloning and genetic engineering.

Fig 16.1

Copyright © 2000-2018 TIPS Inc. Unauthorized reproduction of this manual is strictly prohibited and it is illegal to 15-1 reproduce without permission. This manual is being used during review sessions conducted by PharmacyPrep.

www.PharmacyPrep.Com Prokaryotic Primitive nucleus (no nuclear membrane) Large single DNA molecule Contain cell wall (rigid): covalently bonded short chains of amino acids. Cell wall contain murein (polysaccharide chain) NoàChloroplast Noà mitochondria (cell membrane) Present in bacteria Bacteria are reproduced by type of cell division called binary fission.

Cell and Molecular Biology Eukaryotic Have nucleus Animal: no cell wall but cell membrane No chloroplast Yesà mitochondria Plantà yes cell wall Yes chloroplast Present in: Animal, plant, fungi, parasites, algae, and protozoa

Cell organs compose of a number of tissues, and each tissue composes of cells of the same type. The individual cell is the minimal self-reproducing unit in all-living species. It performs two types of functions, such as performs chemical reactions necessary to maintain our life. The second is passes the information for maintaining life to the next generation. Since the cell is the vehicle for transmission of the genetic information in all living species, it needs to store the genetic information in the form of double-stranded DNA. The cell replicates its information by separating the paired DNA strands and using each as a template for polymerization to make a new DNA strand with a complementary sequence of nucleotides. The same strategy is used to transcribe portions of the information from DNA into molecules of the closely related polymer, RNA. The RNA is the intermediate between DNA and protein and it guides the synthesis of protein molecules by the complex machinery of translation, i.e. the ribosome. The resultant proteins are the main catalysts for almost all the chemical reactions in the cell. In addition to catalyst, proteins are performing also building block, transportation, signalling etc. Cell Organs Endoplasmic reticulum (ER): It is a series of membranes extending throughout the cytoplasm of eukaryotic cells. Cytochrome P450 present in endoplasmic reticulum. · Rough endoplasmic reticulum helps in protein synthesis. · Smooth endoplasmic reticulum helps in lipid synthesis, this does not contain ribosome. Golgi body (Golgi apparatus): Series of flattened sacs. Synthesize the cell’s proteins and lipids. Lysosomes: Drop like sac of enzymes in the cytoplasm. Help digestion within cells. Mitochondria: (power house of the cell). It releases energy in the form of ATP. Chloroplast (chlorophyll): It is normally present in green plants. Its principal function is to absorb energy from the sun. Vacuoles: They are big, fluid like structures, and may occupy more than 75% of the plant cells. · Store nutrients as well as toxic wastes. · Flagella. It exists as single and it helps bacteria to move. · Cilia. It exists as numerous and it helps bacteria to move.

Copyright © 2000-2018 TIPS Inc. Unauthorized reproduction of this manual is strictly prohibited and it is illegal to 15-2 reproduce without permission. This manual is being used during review sessions conducted by PharmacyPrep.

www.PharmacyPrep.Com

Cell and Molecular Biology

Centrosomes: Microtubule does cell division or formation spindle apparatus during cell division. Peroxisomes: Contains enzymes that break down fatty acids and carbohydrates and decomposes hydrogen peroxides. Chromosome, Gene and Genome Genome: The genome of an organism is its complete set of DNA. All the genetic information in an organism is referred collectively as a “genome”. .

Fig 16.2 Chromosome: The 3 billion bases of the human genome are not all in one continuous strand of DNA. Rather the human genome is divided into 23 separate pairs of DNA, called chromosomes. Chromosomes are structures within the cell nucleus that carries genes. A chromosome contains a continuous molecule of DNA which is wrapped around histones. Human has 22 pairs of autosomes and 1 pair of sex chromosome, hence make up to 23 pairs of chromosomes. Autosomes are nonsex determining chromosomes, while sex chromosomes are X and Y chromosome. Male will have XY sex chromosomes, whereas female will carry XX sex chromosomes. The collection of chromosomes in an individual is called karyotype. For example, the typical male karyotype has 22 pairs of autosomes, one X and one Y chromosome.

Copyright © 2000-2018 TIPS Inc. Unauthorized reproduction of this manual is strictly prohibited and it is illegal to 15-3 reproduce without permission. This manual is being used during review sessions conducted by PharmacyPrep.

www.PharmacyPrep.Com

Cell and Molecular Biology

Gene Expression

A gene is a DNA sequence that encodes a protein or an RNA molecule. Each chromosome contains many genes, i.e. the basic physical and functional units of heredity. Each gene exists in the particular position of particular chromosome. In human genome, it is expected that there are 30,000 to 35,000 genes. In prokaryotic genome, one gene is corresponding to one protein. Whereas, in eukaryotic genome, one gene can correspond to more than one protein because of the process called as “alternative splicing”. What is functional unit of inheritance? Gene Human genome: Mapping the entire sequence of human gene. The human genome contains approximately three billion nucleotides bases, which code for approximately 20,000 to 25, 000 protein-coding genes. Most nucleotides base pairs are identical from person to person, with only 0.1% contributing to individual differences. A gene is a series of codons that specifies a particular protein.

Nucleotide structure: Consist of 3 units that is base, sugar and phosphate Nucleic acid (DNA and RNA). Nucleotides are the building blocks of all nucleic acid molecules (such as DNA and RNA). These structural units consist of three essential components, i.e. · A pentose sugar deoxyribose (in DNA) and ribose (in RNA). · Phosphate (bound to the 5’ carbon) · Base (bound to the 1’ carbon), nitrogenous base. Forms of Nucleotides · Nucleotides can have 1, 2, or 3 phosphate groups. Monophosphate nucleotides · Have only 1 phosphate, which are the building blocks of DNA. Diphosphate nucleotides have 2 phosphate groups and triphosphate nucleotides have 3 phosphate groups, which are used to transport energy in the cell. Nucleoside consist of sugar and base There are two chemically different nucleic acids deoxyribosenucleic acid (DNA) and Ribonucleic acid (RNA).

Copyright © 2000-2018 TIPS Inc. Unauthorized reproduction of this manual is strictly prohibited and it is illegal to 15-4 reproduce without permission. This manual is being used during review sessions conducted by PharmacyPrep.

www.PharmacyPrep.Com

Cell and Molecular Biology

Nitrogen Bases "Pure As Gold"

Purine Bases Adenine (A) Guanine (G)

DNA

Adenine (A) Cytosine (C) Guanine (G) Thymine (T) Double stranded 2-Deoxyribose

Pyrimidine Cytosine (C) Uracil (U) Thymine (T)

CUT Py

RNA Adenine (A) Cytosine (C) Guanine (G) Uracil (U) Single stranded Ribose

Pairs of purine with pyrimidines formed by hydrogen bonds. When strands base pair, they are said to be complementary. · A pairs with T (2 hydrogen bonds) · G pairs with C (3 hydrogen bonds) · ·

Purine can form 2 hydrogen bonds Pyrimidine can form 1 hydrogen bond.

Mutations: If complimentary pair incorrectly compliments other than A to T and G to C can result into mutations. There are 3 possible mutations like purine to purine, pyrimidine to pyrimidine and purine to pyrimidine. Deoxyribonucleic acid (DNA): The molecule that carries the genetic information for most living systems. DNA is present in chromosomes of eukaryotic organisms, mitochondria, chloroplast of plants. Prokaryotes are singlecelled organisms with no nuclei (e.g. bacteria). They have no distinct nuclear compartment to house their DNA and therefore the DNA swims within the cells. Eukaryotes, on the other hand, are organisms whose cells contain a nucleus surrounded by cytoplasm which is contained within a plasma membrane. The DNA locates within the nucleus. Eukaryotes are organisms with single or multiple cells, for example, plant and animal. Prokaryotic organism, which does not have nucleus but contain single chromosome. The DNA is present in single chromosome of prokaryotic organism. Structure of DNA: Double helix a term often used to describe the configuration of the DNA molecule. The helix consists of two spiralling strands of nucleotides (a sugar, phosphate and base) joined crosswise by specific paring of the bases and 3,5-phosphodiester bonds. Some viruses contain single stranded DNA. There are proteins associated with DNA present in eukaryotic nucleus, these proteins referred as nucleoproteins. In prokaryotic this protein present in DNA complex present in neucleoid

Copyright © 2000-2018 TIPS Inc. Unauthorized reproduction of this manual is strictly prohibited and it is illegal to 15-5 reproduce without permission. This manual is being used during review sessions conducted by PharmacyPrep.

www.PharmacyPrep.Com

Cell and Molecular Biology

The DNA molecule consists of four bases Adenine (A), Cytosine (C), Guanine (G), Thymine (T) A sugar-phosphate backbone, arranged in two connected strands to form a double helix. Ribonucleic acid (RNA): Nucleotide structure for RNA. Similar to the nucleotide of DNA, the nucleotide for RNA also has Phosphate and Base. The only difference is that the nucleotide here has Ribose Sugar, instead of deoxyribose in the DNA nucleotide. The ribose has an extra OH group at 2’, which is different from the H group at the same place of deoxyribose. That’s why we call these two different things “ribonucleic Acid” and “deoxyribonucleic acid” one is with the OH group, which contains the “O” molecule, yet the other one without. RNA Polymerases: Theses enzyme helps in synthesis of rRNA, tRNA and mRNA. There are 3 types of types of RNA based on their functions: · Ribosomal RNA (rRNA): normally synthesize ribosome 80% of total RNA. The rRNA is present in ribosome in cell. · Transfer RNA (tRNA): 15% of total RNA. Each tRNA amino acid carries the specific amino acid to the site of protein synthesis. Each tRNA molecule contain anticodon that generally recognizes all the codons on mRNA. · Messenger RNA (mRNA): 5% of total RNA. mRNA carries the genetic information from DNA to cytosol for protein synthesis. Codon · The codon are present in the messenger RNA (mRNA), they are Adenine (A), Guanine (G), Cytosine (C), Uracil (U). These four nucleotide bases produce three base codons. There are 64 different combinations of these bases. Sixty one of 64 codons normally produce 20 common amino acids. However, there are 3 codons UAG, UGA and UAA, do not produce amino acids. · The following codon do not code for amino acids, they are known as stop, nonsense or termination codons. When one of the codons appears the synthesis of peptide chain is stopped.

Gene transcription and translation process

Transcription

Translation Occurs in the cytoplasm at the ribosome and involves mRNA and tRNA

Post-translational Modification (Including glycosylation, phosphorylation, and sulfatation.

Protein

DNA ---------------------à mRNA

Protein

à tRNA à rRNA à à à Protein synthesis

Step 1. Transcription à DNA à mRNA Step 2. Translation à mRNA à tRNA.

Copyright © 2000-2018 TIPS Inc. Unauthorized reproduction of this manual is strictly prohibited and it is illegal to 15-6 reproduce without permission. This manual is being used during review sessions conducted by PharmacyPrep.

www.PharmacyPrep.Com

Cell and Molecular Biology

Important concept! 1) What step comes first in protein synthesis? Transcription 2) Anticodon is present on? tRNA 3) Making mRNA from DNA is called? Transcription 4) Making DNA from mRNA is? Reverse Transcription (retrovirus) 5) Antisense therapeutic agents target mRNA Transcription: This is first step in cell protein synthesis, during this process information from DNA copied to mRNA. DNA --à mRNA Translation: This is second step in cell protein synthesis. The protein synthesis occurs in ribosomes. During translation information from mRNA is brought to ribosomes by tRNA. This will determine the sequence of amino acids and protein synthesis. Reverse transcription: This begins when the viral particles (retrovirus) enters the cytoplasm of target cell. Reverse transcriptase is an enzyme used to generate complementary DNA (cDNA) from an RNA template. mRNA à DNA Complementary DNA (cDNA): DNA synthesized from a messenger RNA rather than from a DNA template. This type of DNA is used for cloning or as a DNA probes for locating specific genes in DNA hybridization studies. Introns and exons: The coding region of a eukaryote’s gene is different from that of a prokaryote. For Eukaryotes, each gene contains introns and exons. Intron is a segment of gene situated between exons. It is not responsible for the coding of protein. So the introns will be ultimately spliced out of the mRNA. And exon is a nucleotide sequence in DNA that carries the code for the final mRNA molecule and thus defines the amino acid sequence during protein synthesis. The process of removing the introns for the mRNA sequence is called RNA splicing. This process is done with the help of spliceosomes. Though the Introns seem “useless”, it is quite Amazing that in eukaryotes, each gene can have many introns, and each intron may have thousands of bases. Introns in eukaryotic genes normally satisfies the GT-AG rule that is intron begins with GT and ends with AG. Introns can be very long. Plasmid is extra chromosomal substance of DNA. Plasmids are often used for DNA recombination and cloning. Restriction Endonuclease Enzymes: Restriction enzymes or restriction endonuclease is a class of bacterial enzymes. They are DNA cutting enzymes, which recognize certain point, called restriction site, in the doublestranded DNA with a specific pattern and break the phosphodiester bonds between the nucleotides. Such process is called digestion. Naturally, restriction enzymes are found and isolated from various bacterial species, which are used to break foreign DNA to avoid infection or disable the function of the foreign DNA. Restrictive endonuclease or restriction enzyme. Enzyme that breaks DNA in a specific site in the interior of molecule. There are two types of restrictive enzymes lyase and lygase. The lyase split the DNA on specific site. Lygase is the joining of DNA on specific site.

Copyright © 2000-2018 TIPS Inc. Unauthorized reproduction of this manual is strictly prohibited and it is illegal to 15-7 reproduce without permission. This manual is being used during review sessions conducted by PharmacyPrep.

www.PharmacyPrep.Com

MOLECULE A

Cell and Molecular Biology

MOLECULE B

C-T-A-G…5’

5’ … G-A-T-C C-T-A-G-G-A-T-C

(Sealed with DNA ligase)

Cloning: Given a piece of DNA X, the process of duplicating it into many pieces is called Cloning. The basic steps involve: 1) Insert X into a plasmid vector with antibiotic-resistance gene and a recombinant DNA molecule is formed. Plasmids and DNA fragments must have compatible RE ends for ligation by T4 DNA ligase. A linear product of DNA and the linearized plasmid is firstly formed, followed by the joining of the opposite ends to form a circular product. rDNA. or recombinant DNA molecules formed by lab methods of genetic recombination (molecular cloning).

Copyright © 2000-2018 TIPS Inc. Unauthorized reproduction of this manual is strictly prohibited and it is illegal to 15-8 reproduce without permission. This manual is being used during review sessions conducted by PharmacyPrep.

www.PharmacyPrep.Com

Cell and Molecular Biology

2) Insert the recombinant into the host cell (usually, E. coli). This makes use of a chemical based transformed method, where the bacterial cells are made “competent” to take up foreign DNA by treating with calcium ions. After the recombinant DNA molecules are mixed with the bacteria cells, a brief heat shock is applied to facilitate uptake of DNA. 3) Grow the host cells in the presence of antibiotic. Note that only cells with antibiotic resistance gene can grow. Note that when we duplicate the host cell, X is also duplicated. 4) Select those cells contain both the antibiotic-resistance genes and the foreign DNA X. Some cells only contains plasmid vector but without the foreign DNA due to unsuccessful ligation in step 1. The cells with foreign DNA X can be correctly selected by the complementation of beta-galactosidase, in which the correct colony will show blue color. 5) Kill them and extract X. Genetic Diseases: Hemophilia, sickle cell anemia, cystic fibrosis. Genetic diseases: Hemophilia · Hemophilia. This is a genetic disease often associated with X chromosome only. Thus, men can hemophilic or no hemophilic, however there is no carrier in men. · Hemophilia causes slow blood clot formation. · There are two types of hemophilia, that is type A and B. · Hemophilia type A is due to deficiency of clotting factor 8 (antihemophilic factor). · Hemophilia type B is due to deficiency of clotting factor 9 (Christmas factor). Gene therapy Gene therapy cures genetic diseases such as cystic fibrosis, sickle cell anemia, and hemophilia, in which defected genes are identified and altered or by altering gene expression to prevent or cure genetic diseases and cancers, Copyright © 2000-2018 TIPS Inc. Unauthorized reproduction of this manual is strictly prohibited and it is illegal to 15-9 reproduce without permission. This manual is being used during review sessions conducted by PharmacyPrep.

www.PharmacyPrep.Com

Cell and Molecular Biology

ADA deficiency (adenosine diaminase deficiency), this cause autosomal recessive metabolic disorder that cause immunodeficiency. Knockout mice: A knockout mouse or knock-out mouse is a genetically modified mouse (Mus musculus) in which researchers have inactivated, or "knocked out", an existing gene by replacing it or disrupting it with an artificial piece of DNA. Apoptosis: In normal component of cell regulation, the suppressor gene (tumor suppressor gene), arrest replication of a cell with damaged DNA until the DNA is repaired. Failure to repair DNA and resume normal function will result in programmed cell death, is referred as apoptosis. In Gene therapy tumor suppressor genes are inserted into tumor lines to stimulate apoptosis killing the tumor cells. Necrosis: cell death due to diseases like ischemia etc. Cirrhosis: liver cell death due to chronic liver disease.

Tips Select answers from the following table 1. plasmid 2. lygase 3. lyase 4. gene 5. transcription 6. nucleotides 7. Uracil 8. adenine 9. guanine 10 hemophilia 11 genome 12 Intron 13 A DNA sequence of specific organism. · How many type of nucleic acid? · Building blocks of all nucleic acid molecules are the ( ) · Purine bases ( ) · Pyrimidine bases ( ) · Base found only in RNA ( ) · All the genetic information in an organism is referred collectively as…( ) · DNA sequence that encodes a protein or an RNA molecule is a ( ) · This is the first step in cell protein synthesis ( ) · A segment of gene situated between exons is ( ) · Split the DNA on specific site ( ) · Join the DNA on specific site ( ) · Small circular, extra chromosomal DNA molecule called….( ) · This is a genetic disease often associated with X chromosome only. ( ) · Genome (DNA library), complete genetic information of one species. · What is plasmid, except? · What is genome? à

Copyright © 2000-2018 TIPS Inc. Unauthorized reproduction of this manual is strictly prohibited and it is illegal to 15-10 reproduce without permission. This manual is being used during review sessions conducted by PharmacyPrep.

www.PharmacyPrep.Com

Cell and Molecular Biology

Select True/False Statements · · · · · · · · · · · · · · · · · · · · · · · · ·

Prokaryotes à Have cell membrane. (True/False) Eukaryotic à animal cell have no cell wall (True/False) Nucleotide à base + sugar + phosphate (True/False) DNA bases à A, C, G and T (True/False) RNA bases à A, C, G and U (True/False) Complimentary bases à A-T, G-C or A-U (True/False) Transcription is à DNA to mRNA (True/False) Translation is à mRNA to tRNA (True/False) cDNA is à complementary DNA that produced from mRNA (True/False) Anticodons are present on à tRNA (True/False) The Largest type of RNA? r-RNA (80%) (True/False) The Smallest type of RNA? m-RNA (5%) (True/False) Single strand DNA is present in? Some virus (True/False) RNA polymerase I make? m-RNA (True/False) Haploid à single chromosome (True/False) Daploid à double chromosome (True/False) DNA transferase catalyzes the transfer of various groups such as phosphate and amino groups. (True/False) DNA hydrolase’s = hydrolyses various substances. (True/False) DNA lyase = catalyzes the removal of various functional groups other than the process of hydrolysis. (True/False) DNA isomerase’s = catalyzes various isomerisation's (True/False) Reverse transcriptase found in some viruses, they are referred as retrovirus, is an RNA dependent DNA polymerase. This enzyme requires an RNA template to direct the synthesis of new DNA. Example: NRTI – Nucleoside reverse transcriptase inhibitors. NNRTI-Non nucleoside reverse transcriptase inhibitors. DNA synthesis by reverse transcriptase can be inhibited by AZT (zidovudine). (Anti HIV) Retrovirus : Virus that contain reverse transcriptase enzyme. Example. HIV utilizes this enzyme to replicate their RNA genome. (True/False) Hapten is a low molecular weight compounds that act as immunogens after chemically complexing to a larger molecule or cell surface. (True/False)

Copyright © 2000-2018 TIPS Inc. Unauthorized reproduction of this manual is strictly prohibited and it is illegal to 15-11 reproduce without permission. This manual is being used during review sessions conducted by PharmacyPrep.

www.PharmacyPrep.Com

Cell and Molecular Biology

Copyright © 2000-2018 TIPS Inc. Unauthorized reproduction of this manual is strictly prohibited and it is illegal to 15-12 reproduce without permission. This manual is being used during review sessions conducted by PharmacyPrep.

www.PharmacyPrep.Com

Pharmacogenetics

16 Pharmacogenetics Pharmacogenetics is study of inherited genetic differences in drug metabolic pathways which can affect individual responses to drugs. Pharmacogenetics search for genetic variations that lead to individual differences in drug response. E.g. examining the influence of carvedilol, a beta blocker gene on blood pressure. Pharmacogenomics refers to entire spectrum of genes like e.g. study examines the interactions between CYP 450 and beta 1 , beta 2 , and alpha 1 receptors effects on gene that beta blocker effects. Fast metabolizers: CYP2C19. The presence of a single “17 variant causes a slight increase in CYP2C19 enzyme activity. CYP2D6 presence of “2 variant gene result in increase in CYP2D6 activity. CYP2D6 Codeine ----------------------à morphine Slow metabolizers: No enzyme activity or metabolizes certain medications at a significantly lower rate than normal. The pharmacogenomics is integration of pharmacology and genetics. The study of pharmacogenomics allows to design and develop drugs that are customized to each person’s genetic mark up. The pharmacogenomics also utilized to study cytochrome enzymes that are responsible for drug interactions. Genetic variations: A genetic variation occur as either rare defects or polymorphism. Polymorphism are defined as variations that occur at a frequency of at least 1% in the human population. Example the gene encoding cytochrome CYP450 enzymes CYP2A6, 2C9, 2C19, 2D6 and 3A4 are polymorphic. Single nucleotide polymorphism (SNP) occurs when one base pair of nucleotide replaces another. A single base differences that exist between individual. This is the most common genetic variation in DNA. To perform pharmacogenetics, the first step is detailed analysis of patient list of single nucleotide polymorphism. Defective splicing: In which an internal polypeptide segment is abnormally removed, and the ends of the remaining polypeptide are joined. Individual variability in drug therapy: The required daily dose of warfarin for inhibition of thrombosis and embolism in many disease conditions varies up to 20-30 fold from patient to patient. Therefore frequent blood

Copyright © 2000-2018 TIPS Inc. Unauthorized reproduction of this manual is strictly prohibited and it is illegal to reproduce without permission. This manual is being used during review sessions conducted by PharmacyPrep.

16-1

www.PharmacyPrep.Com

Pharmacogenetics

coagulation testing in patient receiving warfarin therapy is mandated to achieve safe and effective anticoagulation's. The clinical use of statins to treat high cholesterol is large and dose dependant variations in drug efficacy and drug safety. Study suggest the genetic polymorphism of HMG coreductase and drug transporter which regulates hepatic uptake or efflux of statins and statin metabolites contributes to the variability efficacy and the side effects of cholesterol lowering drugs.

Tips · · ·

What is pharmacogenetics à Pharmacogenetics à The study which allows to design and develop drugs that are customized to each person’s genetic mark up ( )

References. Made especially for you: pharmacogenomics and pharmacy practice, CPJ. Jan 2008 vol. 141, No.1

Copyright © 2000-2018 TIPS Inc. Unauthorized reproduction of this manual is strictly prohibited and it is illegal to reproduce without permission. This manual is being used during review sessions conducted by PharmacyPrep.

16-2

www.pharmacyprep.com

17 Immunology and Immunizations Questions Alerts!

Common questions in pharmacy exam is to ask! • Immune cells response (B cell (Humoral immune response) and T cells (Cell mediated immune response). • Immunoglobulin's (IgE asthma and anaphylactic reaction) • Mechanism of inflammation (bacterial infections neutrophil and viral infections lymphocytes). • Mechanism and Types of hypersensitive reactions (poison ivy, Hashimoto, Montaux test, anaphylactic reactions).

THE ORGANS OF THE IMMUNE SYSTEM The organs of the immune system are stationed throughout the body. They are known as lymphoid organs because they are concerned with the growth, development, and deployment of lymphocytes, white blood cells (WBC) that are key operatives of the immune system. LYMPH NODE. Lymph nodes are small, bean-shaped structures that are laced throughout the body along the lymphatic routes. Lymph nodes contain specialized compartments where immune cells congregate, and where they can encounter antigens.

Copyright © 2000-2018 TIPS Inc. Unauthorized reproduction of this manual is strictly prohibited and it is illegal to 17-1 reproduce without permission. This manual is being used during review sessions conducted by PharmacyPrep.

www.pharmacyprep.com

B lymphocytes and T lymphocytes are primary cell of specific immune response. Innate mechanism: Innate immunity Immediate Skin, pH, mucus, saliva, temperature interferon’s, cellular; neutrophils, eosinophil, basophil First line of defense No memory cells Cells are destined to die

Adaptive immunity Delayed (gradual) Lymphocytes (Bcells and Tcells) Lifelong immunity. Specific to antigen (bacteria, virus etc) Memory cells

Humoral immune response (B cells immunity): Acquired immunity (immunity that is NOT inherited) is humoral immunity associated with antibody production. Cells involved in the immune system: White blood cells (WBC) or leukocytes are two types, polymorphonuclear leukocytes (granulocytes) and mononuclear leukocytes without granules in their cytoplasm. The polymorphonuclear leukocytes (granulocytes) Neutrophils, Eosinophils and Basophils. • •

Normal range of white blood cells are 4000 to 11000/cmm Lymphocytes. About 30% of white blood cells are lymphocytes.

Copyright © 2000-2018 TIPS Inc. Unauthorized reproduction of this manual is strictly prohibited and it is illegal to 17-2 reproduce without permission. This manual is being used during review sessions conducted by PharmacyPrep.

www.pharmacyprep.com • •

Neutrophils: About 60% of white blood cells are neutrophil. Monocytes: About 8% of white blood cells are monocytes

Neutrophil • Most abundant WBC. • Not only phagocytes but also granulocytes • Uses its prepackaged chemicals to degrade the microbes it ingest. • ↑ in number of neutrophil indicates bacterial infection. Eosinophils • Play a role in defending against parasitic worms. They secrete their granule contents onto worms, which helps kill them. • ↑ in eosinophils indicates parasite infection and allergies. Basophils • Smallest circulating granulocytes. • Discharge the contents of their granules, releasing a variety of mediators such as histamine, serotonin, prostaglandins, and leukotriene, which leads to inflammation and other symptoms associated with and infections.

Question Alert! 1) Mast cells and basophiles produce histamines! 2) Chromoglycate Na and Nidocromyl are mast cell stabilizers 3) What cells are not distinguishable?

LYMPHOCYTES: The LYMPHOCYTES are a type of white blood cells found in the blood and many other parts of the body. TYPES OF LYMPHOCYTES INCLUDE B cells, T cells, Natural killer cells. B CELLS: B CELLS have thousands of identical antibodies in their membranes that allows them to bind chemically to a small group of chemically related antigen. VIRGIN B CELLS: never respond to an antigen since they release into the circulation from bone marrow. Their membrane antibodies are of the immunoglobulin M and D (Ig M and Ig D).

Copyright © 2000-2018 TIPS Inc. Unauthorized reproduction of this manual is strictly prohibited and it is illegal to 17-3 reproduce without permission. This manual is being used during review sessions conducted by PharmacyPrep.

www.pharmacyprep.com MEMORY B CELLS: are derived from cell division form another B cell that has responded to an antigen. Their membrane antibodies are Ig A, Ig E, Ig G B CELLS (B lymphocytes) mature into plasma cell that secrete antibodies (immunoglobulins), the proteins that recognize and attach to foreign substances known as antigens. Each type of B cell makes one specific antibody, which recognizes one specific antigen. B lymphocytes Bcells Acquired immunity or Humoral immunity

T Cell Helper T cells (glycoproteins CD4) Cytotoxic T cell Natural killer T cell Cell immunity

Leukocytes (granulocytes) Neutrophil Eosinophil Basophil

Classes of ANTIBODIES (Immunoglobulins, Ig): Antibodies respond to antigens by latching on to, or binding with, the antigens. Specific antibodies match specific antigens, fitting together much the way a key fits a lock. Ig A Ig D Ig E

Ig G IgM

10%, Ig A1 and Ig A2, present in saliva, tears, urine, and external body fluids. Less than 1%, Functions not well understood. 1% located on the cell surface of blood basophils and on connective tissue mast cells to trigger the secretion of inflammatory mediators from these cells in the presence of specific antigen. IgE mediates allergic reactions (asthma). Eosinophils and basophils have IgE antibodies receptor. Inflammation is related to eosinophils. Serum half is 2 to 3 days. When this bound mast cells, the serum half-life could be several months. Ig E levels increased in allergic reactions. 70%, most common (abundant) of all Ig’s found in all body fluids. This is secreted at the end of primary immune response and during memory responses. IgG1 to IgG4, and can cross placenta. 20% IgM is the most potent activator of all immunoglobulins. IgM1 to IgM2. First immunoglobulin produced in body.

Structure of immunoglobulin's

Copyright © 2000-2018 TIPS Inc. Unauthorized reproduction of this manual is strictly prohibited and it is illegal to 17-4 reproduce without permission. This manual is being used during review sessions conducted by PharmacyPrep.

www.pharmacyprep.com

Ig A Ig A is secreted during memory response, this accounts for 10% of serum immunoglobulins. It is secreted across mucosal surfaces into gastrointestinal, respiratory, lachrymal, mammary, and genitourinary secretions. Where this protects mucosa from colonization of pathogen (Bacteria) and other microorganisms. SERUM HALF LIFE IS ~ 5 DAYS. Subclass: IgA1 and IgA2 Ig G Ig G is the predominant immunoglobulin, (~ 70%). This is secreted at the end of primary immune response and during memory responses. It diffuses from blood into other extra cellular fluids, particularly in inflamed vasculatures, and it crosses the placenta to enter the fetal circulation. SERUM HALF LIFE IS 25-35 DAYS. Subclass: IgG1 – IgG4

Ig E This accounts for ~ 1%. This binds to Ig E receptors located on the cell surface of blood basophils and on connective tissue mast cells to trigger the secretion of inflammatory mediators from these cells in the presence of specific antigen. IgE mediates allergic reactions (asthma). SERUM HALF IS 2 to 3 DAYS. When this bound mast cells, the serum half life could be several months. Ig M It does not leave the blood in significant amount because of it PENTAMERIC STRUCTURE (molecular size 900,000 daltons). This accounts for ~ 20%. Serum halflife 9 to 11 days. IgM is the most potent activator of all immunoglobulins. Subclass: IgM1 and IgM2 Accounts for less than 1%. Has no known function.

Ig D

The mononuclear leukocytes without granules in their cytoplasm are Monocyte and Lymphocyte (T-lymphocyte: Helper T-cell, Cytotoxic T-cells and Suppressor T-cells), B-lymphocytes and natural killer cells. T CELLS T cells contribute to the immune defenses in two major ways. Some help regulate the complex workings of the immune system, while others are cytotoxic and directly contact infected cells and destroy them. Chief among the regulatory t cells are "helper/inducer" t cells. They are needed to activate many immune cells, including b cells and other t cells. Another subset of regulatory t cells acts to turn off or suppress immune cells. Cytotoxic t cells help rid the body of cells that have been infected by viruses as well as cells that have been transformed by cancer. They are also responsible for the rejection of tissue and organ grafts. THYMUS GLAND: T cell do not enter the circulation directly from bone marrow. But first enter the thymus gland to mature. Most developing T cells die in thymus gland. VIRGIN T CELLS: Release from thymus to circulation are virgin T cells. Copyright © 2000-2018 TIPS Inc. Unauthorized reproduction of this manual is strictly prohibited and it is illegal to 17-5 reproduce without permission. This manual is being used during review sessions conducted by PharmacyPrep.

www.pharmacyprep.com MEMORY T CELLS: Originate through cell division and responses of other T cells. T Cell receptors (T antigen receptors) T Cell have two membrane proteins (alpha and beta or gamma or delta). These proteins define specificity of each T Cell and several other membrane proteins know as CD3 complex occurs in cell mediate immune response. TCell mediated immunity: T cells: Helper T Cells, Cytotoxic T Cells, and Suppressor T Cells The helper T cells. Glycoproteins. The most T cells can be classified by the presence of membrane glycoproteins. The helper T cells (TH cells) CD4 and Cytotoxic T cells (CTL) or TC Cells CD8. Normal CD4 count is from 500 to 1,500 cell per cubic millimeter of blood.

Question Alerts! CD4 counts indicates?

The helper T cells (TH cells). These can be divided two type TH1 and TH2. These cells produce lymphokines (cytokines) are small proteins that act on other cells in autocrine, paracrine, and endocrine manner. TH1 activate other cells, inhibit antibody production by inhibiting the formation of TH2. TH2 Activate B cells to divide and produce antibody. Natural Killer Cells At least two types of lymphocytes are killer cells; cytotoxic T cells and natural killer cells. Can increase the number of red blood cells and reduce the need for red blood cell transfusions in patients receiving chemotherapy; and Oprelvekin can reduce the need for platelet transfusions in patients receiving chemotherapy. MONOCYTES are white blood cells that can swallow and digest microscopic organisms and particles in a process known as phagocytosis. Monocytes can also travel into tissue and become MACROPHAGES, or "big eaters." BCell Humoral immunity or acquired. NOT inherited. Antibodies: Monoclonal antibodies

Tcell Adaptive immunity Activate antigen-specific cytotoxic T cells that able induce apoptosis in body. Cytokines (lymphokines): Interferons, interleukins, and colony stimulating factors

CELLS IN THE IMMUNE SYSTEM SECRETE TWO TYPES OF PROTEINS. ANTIBODIES and CYTOKINES CYTOKINES (Lymphokines): Soluble protein molecules released by participating and interacting cells in the adaptive immune system. Copyright © 2000-2018 TIPS Inc. Unauthorized reproduction of this manual is strictly prohibited and it is illegal to 17-6 reproduce without permission. This manual is being used during review sessions conducted by PharmacyPrep.

www.pharmacyprep.com Substances produced by some immune system cells to communicate with other cells. Therapeutic cytokines: Types of cytokines (Lymphokines) includes. • Interferons • Interleukins • Colony stimulating factors (CSF). Interferon: The interferons are the family of cytokines proteins, important in the immune response. Interferon Proteins that are secreted by cells when they become infected with virus. Bind to nearby infected cells and prevent viral infection. This increased resistance of cells to viral infection and slows the spread of disease. There are three major types of interferon: Interferon’s inhibit viral infections and may have anticancer properties. Interferons are indicated in hepatitis infections. • Alpha (leukocyte) interferon is used for treatment of chronic hepatitis B. (peginterferon alpha 2a). • Contraindications of interferon. Autoimmune disease (SLE, RA), severe depression, or psychosis, neutropenia, thrombocytopenia and cardiac arrhythmias. • Interferon Beta (fibroblast) is used for treatment of multiple sclerosis. • Gamma (immune) Interleukin (Class I cytokine receptors IL-2, IL-3, IL-7, IL-11, IL-13, IL-15): A type of lymphokine that regulates the growth and development of white blood cells. The interleukins also called lymphokines. Twelve interleukins (IL-1 through IL-12) have been identified to date. Interleukin-3 (hematopoietic growth factor), Oprelvekin (interleukin-11) is a polypeptide growth factor obtained by recombinant DNA technology. Increases platelet production via stimulation of hematopoietic stem cells. Therapeutic use: Chemotherapy related thrombocytopenia. Side effects: Fluid retention, peripheral edema and dyspnea.

IL-3

Secreted by Tcells and stimulate bone marrow and causes cell proliferation.

Therapeutic use Thrombocytopenia (hemopoietic growth factor)

IL-11

Tcell. Induce megalocyte proliferation

Anemia, (hemopoietic growth factor)

Colony Stimulating factors: Colony-stimulating factors (CSFs) also called hematopoietic growth factors. Usually do not directly affect tumor cells rather, they encourage bone marrow stem cells to divide and develop into white blood cells, platelets, and red blood cells. Bone marrow is critical to the body's immune system because it is the source of all blood cells. The CSFs stimulation of the immune system may benefit patients undergoing cancer treatment. Some examples of CSFs and their use in cancer therapy erythropoietin, epoetin alpha, and epoetin beta. Darbepoetin (chemotherapy induced anemia occurs within weeks to months). Therapeutic use: Treatment of anemia resulting from chronic renal failure. Side effects: Increase BP, thus monitor blood pressure. Granulocyte Colony Stimulating Factors. Produced by recombinant DNA technology. Filgrastim (Pegfilgrastim) and Sargramostim. These are glycoproteins produced via recombinant DNA technology.

Copyright © 2000-2018 TIPS Inc. Unauthorized reproduction of this manual is strictly prohibited and it is illegal to 17-7 reproduce without permission. This manual is being used during review sessions conducted by PharmacyPrep.

www.pharmacyprep.com Filgrastim sc 5 mcg/kg QD for 7-10 days given after 24 h of chemotherapy. (Chemotherapy induced neutropenia can occur within days to weeks). Therapeutic use: Treatment of chronic and chemotherapy induced neutropenia. Sargramostim is approved for myeloid reconstitution (in bone marrow transplantation). Side effects: Skin allergies, respiratory allergies, and cardiovascular allergies. Filgrastim and pegfilgrastim are contraindicated for patient with Allergic to E. coli derived proteins. Cell surface proteins Tcell Bcell NK Cells Macrophages

types MHC I, CD-2, CD-4, CD-8 MHC I, MHC II, B7, CD-19, CD-20 MHC I, CD-16, CD-56 MHC II, CD-14, CD-40

MAJOR HISTOCOMPATIBILITY COMPLEX (MHC) proteins: Recognize peptide epitopes (fragment of antigens), combined with chemical and MHC proteins to produce two major classes of MHC proteins. Class 1 proteins: Present on all surfaces of body cells. Class II proteins: Present on specific antigen presenting cell (APC’s).

Type IV

Type III

Type II

Type I

Hypersensitivity Reactions: Excess, inappropriate and prolonged immune responses that cause damage to normal tissue. HYPERSENSITIVE REACTIONS IgE mediated type I hypersensitivity reaction Respiratory allergies grass, animal fur, carpet mites. GI allergies. Dairy products, shellfish, and peanut. Skin allergies, topical drugs (procaine). Intravenous allergiesinsect venoms. IgE mediated type I: Anaphylactic reactions. It is treated by epinephrine. (Penicillin's, bee stings, latex, pea nut).

Symptoms/EXAMPLES Inflammation of upper and lower respiratory tract (asthma), GI and skin. Atopic dermatitis, pruritis, rhinitis, asthma, and food allergies. Urticaria, eczema. Approximately 50% of patient with asthma secret IgE.

Cytotoxic/anti-body mediated hypersensitivity: Transfusion mismatches, hemolytic anemia, Rh disease, specific autoimmune diseases Hashimoto thyroiditis, and myasthenia gravis. Antigen-antibody (IgG/IgM) complex Non-specific autoimmune disorders such as systemic lupus erythematous, rheumatoid arthritis. Hepatitis infections, local respiratory form of fungal reactions. Penicillin and sulfonamides. Delayed type reactions mediated by cell (T cells). Prolong action of protozoa. Mycobacterium Tuberculin test and poison ivy.

Hemolytic anemia and thrombocytopenia are more common. Hyper acute graft rejection Lymphadenopathy, fever, and rash- first symptoms. More serious: Glomerulonephritis, vasculitis and lupus, arthralgia and arthritis. Symptoms of type IV: Contact dermatitis, microvesicle formation, and spongiosis. Tuberculin (Monteux test) reaction gives erythema.

Mnemonics. Copyright © 2000-2018 TIPS Inc. Unauthorized reproduction of this manual is strictly prohibited and it is illegal to 17-8 reproduce without permission. This manual is being used during review sessions conducted by PharmacyPrep.

www.pharmacyprep.com Hypersensitive reaction is "ACID" Type 1 - Allergic ( Levodopa --> dopamine --> norepinephrine --> epinephrine). · Structure activity relationship of direct acting acetylcholine agonist. · Structure activity relationship of atropine and muscarinic blockers. · Pharmacology and structure activity relationship of sympathomimetics like pseudoephedrine, ephedrine, and crystal meth. · Pharmacological actions of sympathetic receptors alpha1&2 and beta 1&2. Parasympathetic receptors like muscarinic and nicotinic.

Nervous System

Central Nervous System (CNS) Spinal cord and brain

Peripheral Nervous System (PNS)

Autonomic Nervous System (ANS)

Cholinergic

Sensory somatic nervous System 12 pairs of cranial nerves (optic & vagus) 31 pairs of spinal nerves

Adrenergic

Drug receptors Generally pharmacological receptors can be categorized into 4 types · Seven trans membrane proteins · Ion channels · Transcriptional regulators · 1-Transmembrane proteins Copyright © 2000-2018 TIPS Inc. Unauthorized reproduction of this manual is strictly prohibited and it is illegal to 24-1 reproduce without permission. This manual is being used during review sessions conducted by PharmacyPrep.

www.Pharmacyprep.com

Medicinal Chemistry

Seven Trans membrane proteins The G protein coupled receptors are the largest types of pharmacological receptors, which almost 200 human receptors are known to date. The G proteins coupled receptors are the major therapeutic significance with well-established therapeutics studies. Examples of G proteins coupled receptors as therapeutic targets are Acetylcholine, muscarinic, norepinephrine, beta receptors, angiotensin AT 1 , dopamine, Serotonin, histamines, and opioids. · · · · · ·

Glutamic acid (glutamate) excitatory: NMDA receptor. Example. Memantine GABA inhibitory: Example Benzodiazepine, barbiturates, and antiseizure drugs. Dopamine inhibitory: G protein linked cAMP. Antiparkinson drugs and antipsychotics. Norepinephrine excitatory: antidepressants, and antianxiety. Serotonin excitatory and inhibitory. Antidepressant and antianxiety. Opioids peptide inhibitory neurotransmitters Enkephalins, endorphins and dinorphine

Ion channels: There are two types of ion channels · ·

Voltage gated: Na+ channel, Ca2+ channel, K+ channel Transmitter gated: neurotransmitter interact with specific receptors.

Transcription regulators: There are over 150 receptors of this family, which acts as transcriptional receptors. This is second major class of receptors, which include steroid hormones including estrogens, androgens, and the glucocorticoids such as corticosteroids, vitamin D, retinoic acid, and thyroxin. One transmembrane proteins: These receptors include several growth factors such as tumor necrosis factor, serine/threonin kinase, neurotropins, and cytokines. Most neurotransmitters interact primarily with postsynaptic receptors, but some receptors are located on presynaptic neurons, providing fine control of neurotransmitter release. Post synaptic Alpha1 Serotonin

presynaptic Alpha2 Beta 1 Beta 2 Serotonin (REUPTAKE)

Receptors in autonomic nervous system:

Cholinergic receptors are classified as nicotinic N 1 (in the adrenal medulla and autonomic ganglia) or N 2 (in skeletal muscle) or muscarinic M 1 (in the autonomic nervous system, striatum, cortex, and hippocampus) or M 2 (in the autonomic nervous system, heart, intestinal smooth muscle, hindbrain, and cerebellum).

Question Alerts! Identify structure of Ach, Epi, NE, 5HT Copyright © 2000-2018 TIPS Inc. Unauthorized reproduction of this manual is strictly prohibited and it is illegal to 24-2 reproduce without permission. This manual is being used during review sessions conducted by PharmacyPrep.

www.Pharmacyprep.com Neurotransmitters Acetylcholine (ACh)

Medicinal Chemistry

Chemical structures

Characteristics

CH3 O

N

H3C

CH3

CH3

O

Acetylcholine (ACh)

Epinephrine

OH H N

CH3

HO Epinephrine

Is a neurotransmitter of peripheral nervous system neuromuscular junction, parasympathetic system, visceral motor nuclei in the brain stem, and basal nucleus of Meynert. Uses a1 , a 2 or b 1 , b 2 , b 3 adrenergic receptors, which are G-protein linked receptors. Plays insignificant role in CNS and is found in the adrenal medulla.

OH

OH

Norepinephrine

NH2

HO Norepinephrine OH

Dopamine

NH2

HO

Dopamine

Uses a1 , a 2 or b 1 , b 2 , b 3 adrenergic receptors, which are G-protein linked receptors is the transmitter of post ganglianic sympathetic neurons and CNS (locus ceruleus), plays role in anxiety states, panic, attacks, depression Uses D 1 and D 2 dopamine receptor, which are Gprotein-linked reception is depleted in Parkinson disease and is increased in schizophrenia.

OH

Serotonin (5hydroxytryptamine. 5HT)

NH2 HO N H 5-HT

g-Aminobutyric acid (GABA)

Glycine

Uses 5HT receptor, which is a transmitter-gated ion channel that is permeable to Na+ and K+ ions is neurotransmitter of the raphe nuclei of the brainstem whose neurons project to widespread areas of the CNS.

Uses the GABA receptor, which is a transmittergated ion channel that permeable to Cl- ions. Uses Gama-Aminobutyric Acid (GABA) the GABA receptor, which is G-protein-linked receptor is a major inhibitory neurotransmitter in the CNS. COOH Uses the glycine receptor, which is transmittergated ion channel that is permeable to Cl- ion is H2N H the major inhibitory neurotransmitter in the spinal H cord. Glycine has NO chiral carbon. Glycine H2N

Glutamate

COOH

COOH H2N

COOH H

Uses the N-methyl-D-apartate (NMDA), kainite, or quisqualate. A receptors, all of which are transmitter gated ion channels that are permeable to Na+, K+ and Ca2+ ions.

Glutamate

Copyright © 2000-2018 TIPS Inc. Unauthorized reproduction of this manual is strictly prohibited and it is illegal to 24-3 reproduce without permission. This manual is being used during review sessions conducted by PharmacyPrep.

www.Pharmacyprep.com

Medicinal Chemistry

Adrenergic receptors: are classified as 1 (postsynaptic in the sympathetic system), 2 (presynaptic in the sympathetic system and postsynaptic in the brain), 1 (in the heart), or 2 (in other sympathetically innervated structures). Dopaminergic receptors are classified as D 1 , D 2 , D 3 , D 4 , and D 5 . The D 3 and D 4 play a role in thought control (limit negative symptoms of schizophrenic processes), whereas D 2 receptor activation controls the extra pyramidal system. GABA receptors are classified as GABA A (activating chloride channels) and GABA B (potentiating cAMP formation). The GABA A receptor consists of several distinct polypeptides and is the site of action for several neuroactive drugs, including benzodiazepines, newer anticonvulsants (e.g. lamotrigine), barbiturates, picrotoxin, and muscimol. Serotonergic (5HT) receptors (with at least 15 subtypes) are classified as 5HT 1 (with four subtypes), 5HT 2 , and 5HT 3 . 5HT 1A receptors, which occur presynaptically in the raphe nucleus (inhibiting presynaptic uptake of 5HT) and postsynaptically in the hippocampus, modulate adenylate cyclase. 5HT 2 receptors, located in the fourth layer of the cortex, are involved in phosphoinositide hydrolysis. 5HT 3 receptors occur presynaptically in the nucleus tractus solitarius. Glutamate receptors are classified as inotropic NMDA (N-methyl-D-aspartate) receptors, which bind NMDA, glycine, zinc, Mg2+, and phencyclidine (PCP, also known as angel dust) and affect the influx of Na+, K+, Ca2+ and non-NMDA receptors, which bind quisqualate and kainate. Non-NMDA channels are permeable to Na+ and K+ but not to Ca2+. These excitatory receptors mediate important toxic effects by increasing calcium, free radicals, and proteinase. In neurons, synthesis of nitric oxide (NO) involving NO synthase increases in response to glutamate. Opioid receptors and neurotransmitters are peptide type. Endorphin-enkephalin (opioid) receptors are classified as µ 1 and µ 2 (affecting sensor motor integration and analgesia), 1 and 2 (affecting motor integration, cognitive function, and analgesia), and 1 , 2 , and 3 (affecting water balance regulation, analgesia, and food intake). µ = analgesic effect, respiratory depression + GI = development of tolerance = analgesic effect, involved in sedation + GI Sigma receptors, currently classified as non-opioid and mostly localized in the hippocampus, bind drugs. Autonomic Innervations and Primary Effects: Receptor type shown in parentheses (a alpha, β beta, M=cholinergic-muscarinic). Note that preganglionic synapses for both systems are cholinergic-nicotinic. indicate absence of direct innervations. However, receptors are present and may be stimulated by agonists.

Copyright © 2000-2018 TIPS Inc. Unauthorized reproduction of this manual is strictly prohibited and it is illegal to 24-4 reproduce without permission. This manual is being used during review sessions conducted by PharmacyPrep.

www.Pharmacyprep.com

Medicinal Chemistry

Sympathetic Drugs

Sympathetic agonist

Alpha agonist

Beta agonist

Sympathetic antagonist

Mixed agonist

Beta antagonist

Alpha antagonist

a1 antagonist

a2, antagonist

Non-selective Beta antagonist

Cardioselective Beta antagonist

a1,b2, mixed antagonist

Partial alpha & Beta antagonist

Partial agonist & antagonist

Classification of adrenergic agonist (Sympathomimetics) SYMPATHETIC AGONIST NON-CATECHOLAMINES Amphetamine Methamphetamine Methylphenidate Ephedrine Pseudoephedrine Salbutamol (Albuterol) Terbutaline Salmeterol Isoproterenol Metaproterenol

CATECHOLAMINES Dopamine Epinephrine Norepinephrine

Question Alerts! Catechol amine type of neurotransmitters. Enzymes that catalyze formation of dopamine to norepinephrine and epinephrine NH2

Tyrosine hydroxylase

HO

NH2

HO

COOH

L-dopa dopa decarboxylase

NH2

Dopamine

HO OH

Note. Phenylethanolamine Sympathetic agonist Indirect acting sympathomimetic amine. Example: Hydroxy amphetamine, ephedrine or pseudoephedrine, methyl amphetamine, and tyramine. Indirect acting sympathomimetics amines may have one,

Tyrosine

COOH

HO

dopamine b-hydroxylase

OH NH2 Norepinephrine

HO OH

phenylehtanolamine-N-methyltransferase (S-Adenosyl Methionine - SAM)

OH H N CH3 Epinephrine

HO OH

Copyright © 2000-2018 TIPS Inc. Unauthorized reproduction of this manual is strictly prohibited and it is illegal to 24-5 reproduce without permission. This manual is being used during review sessions conducted by PharmacyPrep.

www.Pharmacyprep.com

Medicinal Chemistry

two, or no hydroxyl groups. The less the hydroxyl group the higher the lipophilic and the greater the absorption and greater duration of activity after oral administration. Alkyl substitution at the alpha carbon next to amino group reduces the destruction of phenol and phenyl compounds and increases lipophylic characters meth crystal structure. OH CH3

NH2

N

NH2

CH3

H

OH CH3

N

CH3

Amphetamine

Dextroamphetamine

Ephedrine

A

B

C

H

H CH3

CH3

N

CH3

CH3 Methamphetamine

Pseudoephedrine

D

E Alpha 2 agonist H N HN

R

Imidazoline ring

Clonidine

HO

OHCH3

OH

NH2

HO

CH3 NH2

COOH Decarboxylation Methyl Norepinephrine

Methyl dopa - a prodrug

Sympathetic agonist and antagonist share chemical features of natural ligand of epinephrine. Sympathetic antagonists Alpha Blockers

Beta Blockers O

CH3 CH2CHCH2NHCH CH3

CH3O CH3O

N

N

O NC

O

N NH2

Propanolol

Prazosin

All alpha blockers contain 4-amino-6-7-dimethoxyquinazoline ring system attached to piperazine ring. Reduction of furan ring of prazosin to the tetrahydrofuran ring of terazosin increases the duration of action by altering rate of metabolism. Terazosin, doxazosin, Tamsulosin have long half life and duration of and that allows once daily dosing. Tamsulosin & Terazosin, alfazosin used for BPH.

Copyright © 2000-2018 TIPS Inc. Unauthorized reproduction of this manual is strictly prohibited and it is illegal to 24-6 reproduce without permission. This manual is being used during review sessions conducted by PharmacyPrep.

www.Pharmacyprep.com

Medicinal Chemistry

Beta blockers Lipophilicity: The lipophilic BBs primarily cleared by liver (Propranolol, pindolol, and metoprolol) hepatic elimination). Hydrophilic (atenolol and Nadolol) are renally cleared. Propranolol can penetrate into CNS thus can be used for anxiety and CNS SEs insomnia, depression, and vivid dreams present only in propranolol. Cardioselective beta selective blockers are 4-substituted aryloxypropranolamine. Labetalol and Carvedilol: Alpha 1 , beta 1 , and beta 2 Timolol, betaxolol, metoprolol block B 2 receptors in ciliary muscles. The ciliary muscles (gland) produce aqueous humor. SAR OF SYMPATHETIC DRUGS AGONIST LEVODOPA, DOPAMINE, NE, EPINEPHRINE

ANTAGONIST PROPRONOLOL

Cholinergic

Muscarinic Antagonist

Cholinergic Agonist Direct cholinergic agonist

Indirect cholinergic agonist

Acetylcholine agonist Choline ester Bethanachol Carbachol Methanacol

anticholinesterase

Tertiary amines .Atropine Scopolamine Benztropine Trihexyphenidyl

Quaternary amine Ipratropium Tiotropium Glycopyrrolate

Pilocarpine Reversible Quaternary alcohols

Donepezil

Reversible Carbamate .Physostigmine Neostigmine Rivastigmine Pyridostigmine

Organophosphate (Irreversible cholinesterase inh) Echothiophate, Malathion Parathion Sarin

Cholinesterase

Acetylcholine -------------> Acetyl + Choline

Copyright © 2000-2018 TIPS Inc. Unauthorized reproduction of this manual is strictly prohibited and it is illegal to 24-7 reproduce without permission. This manual is being used during review sessions conducted by PharmacyPrep.

www.Pharmacyprep.com

Medicinal Chemistry

Structure activity of direct acting cholinergic drugs CH3 O

H3C-N CH3

Acetylcholine Methanocholine

CH3

ACETYLCHOLINE Carbacholdà 1 carbamate group Methanachol à 1 methyl group Bethanachol à 1methyl &1 carbamate group

O

CH3 H3C-N

O

CH3

CH3 CH3 O Methyl group CH3

O

H3C-N Carbacholine

O

CH3

CH3 H3C-N

NH2 Carbamate group

O

CH3 CH3 O

Bethanacholine

NH2 Carbamate group

Methyl group

Drugs for Alzheimer’s disease

Reversible Acetylcholinesterase inhibitor non specific selective

Galanthamine

Specific

Donepezil

Rivastigmine (relatively selective)

Structure activity of anti-cholinergic drugs Muscarinic antagonist Tertiary amines Atropine Scopolamine Benztropine Trihexyphenidyl

Quaternary amine Ipratropium Tiotropium Glycopyrrolate

Question Alert! Methamphetamine (street name crystal meth, glass, or ice). Dehydroxylation of pseudoephedrine gives methamphetamine Pseudoephedrine has one hydroxyl group. Amphetamine or dextroamphetamine gives methamphetamine by N-methylation.

Copyright © 2000-2018 TIPS Inc. Unauthorized reproduction of this manual is strictly prohibited and it is illegal to 24-8 reproduce without permission. This manual is being used during review sessions conducted by PharmacyPrep.

www.Pharmacyprep.com

Medicinal Chemistry

Quasi ring is present in muscarinic antagonist: It is combination of piperidine + pyrrolidine. Longer the chain on nitrogen of quasi ring, lower antimuscarinic activity.

Tips 1. 4. 7. 10

Methanol One methyl group Beta 1 phenylethanolamine N-methyl transferase

2. 5. 8. 11

Carbachol Alpha 1 Beta 2 Quasi ring

3. 6. 9. 12

Bethanechol Alpha 2 Scopolamine Piperidine & pyrrolidine ring

13 Competitive muscarinic blockers, it act on vestibular system and the CNS. · Epinephrine act on? ( ) · Norepinephrine act on? ( ) · Enzyme that catalyzes norepinephrine to epinephrine ( ) · Muscarinic drugs essential group for anticholinergic action is? ( ) · Muscarinic blockers structure contain? ( ) · Acetylcholine, methanchol, carbachol and bethanechol differs in? ( · competitive muscarinic blocker, that act on vestibular system and the CNS ( · Choose 3 examples of direct acting acetylcholine agonist? ( ) · Scopolamine mechanism is à ( ) · Methanachol is structurally similar to acetylcholine, however differs in à · Write three examples direct acting acetylcholine agonist? ( ) · Organophosphate antidote is? ( ) · Atropine overdose is treated by ( ) can get into brain · Myasthenia gravis is treated by ( )

)

)

Copyright © 2000-2018 TIPS Inc. Unauthorized reproduction of this manual is strictly prohibited and it is illegal to 24-9 reproduce without permission. This manual is being used during review sessions conducted by PharmacyPrep.

www.Pharmacyprep.com

Medicinal Chemistry

Copyright © 2000-2018 TIPS Inc. Unauthorized reproduction of this manual is strictly prohibited and it is illegal to 24-10 reproduce without permission. This manual is being used during review sessions conducted by PharmacyPrep.

www.Pharmacyprep.com

Medicinal Chemistry

25 Medicinal Chemistry and Pharmacology of Histamines, Serotonin, Prostaglandin and NonSteroidal Anti-inflammatory Drugs Questions Alerts! Common questions in pharmacy exam is to ask! · Chemical structure of diphenhydramine (lipid soluble) as sedative. · Serotonin synthesis from tryptophan --> 5-hydroxy tryptophan -->Serotonin · Serotonin pharmacological actions of agonist (triptans) and antagonist of 5HT3 (ondansetron). · Structure activity of prostaglandin analogues PGE1 (misoprostol) and PGF2 (latanoprost). · Pharmacology of leukotriene receptor antagonist montelukast and zafirlukast. · Acetaminophen structure and metabolism that explains hepatotoxicity. · Glutathione chemical structure. · Acetylsalicylic acid pharmacological actions Antiplatelets, Antipyretic, Analgesic and Anti-inflammatory actions. Autocoids (local hormones): Chemical mediators that the body releases as a response to pathogens or noxious substances. Produced in the body and has profound pharmacological effects. Autocoids or chemical mediators Amine types Histamines Serotonin

Endogenous type Ecosonides Prostaglandins Prostacyclin Thromboxane Leukotrienes Bradykinin

Amines: No real clinical application in the treatment of diseases however antihistamines are of great importance. Amine types of autocoids include histamines and serotonin. Endogenous Peptides ·

Site of production for endogenous peptides are GIT, kidneys, lungs, pancreas and uterus.

Copyright © 2000-2018 TIPS Inc. Unauthorized reproduction of this manual is strictly prohibited and it is illegal to 25-1 reproduce without permission. This manual is being used during review sessions conducted by PharmacyPrep.

www.Pharmacyprep.com · · · · ·

Medicinal Chemistry

Physiological actions of endogenous peptides are. Prostaglandins. Pain sensation, development of inflammation Thromboxane. Aggregation of platelets Prostacyclin. Inhibition of platelet aggregation Leukotrienes. Inflammation, chemotactic properties (pull substances to them)

Histamines: The histamine is produced from mast cells. Histamines act on three receptors, of these H1, H2 receptors are excitatory and H3 receptors. Physiological functions of H 1 receptor typical · Allergic and anaphylactic response to histamines. · Gives bronchoconstriction, and vasodilatation. · Increase capillary permeability. · Spasmodic contractions of gastrointestinal smooth muscle.

H N

COOH

H N

NH2 N Histadine

NH2 N

-CO2

Histamine

-CH 2 - in histadine Chemistry of H 1 antihistamine. Usually lipid soluble, and similar in terms of absorption and distribution. · Good absorption after oral administration distribution with peak plasma concentration of 1 to 2 hours. · Allows some to go to the blood-brain-barrier especially structural resemblance to histamine. H 1 antihistamine chemical classification · · · · · · ·

Ethylene diamines: Pyrilamine, and triplennamine Alkylamines: Brompheniramine, chlorpheniramine, and acrivastine. Ethanolamine: Diphenhydramine, dimenhydrinate, and doxylamine. Piperazines: Meclizine, cyclizine, hydroxyzine, and cetrizine. Phenothiazine. Promethazine Dibenzocycloheptenes. Cycloheptadienes. Pthalazinone. Azelastine.

Piperidine: Terfenadine, astemizole, levocabastine, loratadine, and fexofenadine. Tertiary amine

Identify structures! O CH2CH2 N

CH3

CH N

CH3

Diphenhydramine

N CH3

Cyclizine

Diphenhydramine chemical structure! Tertiary amine is present Lipid soluble Has C-O-C, C-N bonds Has aromatic rings (hydrophobic)

Mechanism: Competitive antagonist of histamine

Copyright © 2000-2018 TIPS Inc. Unauthorized reproduction of this manual is strictly prohibited and it is illegal to 25-2 reproduce without permission. This manual is being used during review sessions conducted by PharmacyPrep.

www.Pharmacyprep.com

Medicinal Chemistry

receptors. H 1 receptors are located in the brain, heart, bronchi, GI tract and vascular smooth muscle. Mast cells and basophiles are principle histamine containing cells.

H1 Antihistamine 2nd Generation (AM) Piperazine derivatives Cetirizine (Reactine) Piperidine Derivatives Fexofenadine (Allegra) Loratadine (Claritin)

1st Generation (PM) Ethanol amines:

Diphenhydramine Dimenhydrinate Doxylamine

Alkyl amines Chlorpheniramine Piperidine Derivatives Azatadine, Cyproheptadine Piperizines Meclizine, Cyclizine Hydroxyzine

3rd Generation Piperadine Derivatives Desloratadine (Aerius)

Once daily dose

Side effects: Sedation, dizziness, nausea, constipation, diarrhea, loss of appetite and anticholinergic. Metabolism of antihistamines. Some examples of metabolites of antihistamine that are used as drugs. · Loratadine metabolizes to desloratadine. · Hydroxyzine metabolizes to Cetirizine · Terfinadine metabolizes to Fexofenadine. O H3C H3C OH CH3 CH3 CH3 HO N N HO OH

OH

Terfinadine

Fexofenadine

N

N N COOCH2CH3

Cl

NH Cl

Loratadine

Desloratadine

Copyright © 2000-2018 TIPS Inc. Unauthorized reproduction of this manual is strictly prohibited and it is illegal to 25-3 reproduce without permission. This manual is being used during review sessions conducted by PharmacyPrep.

www.Pharmacyprep.com

Medicinal Chemistry

Histamine H1-receptor blockers

Particularly Diphenhydramine, promethazine

Anti Cholinergic (muscarinic)

Particularly promethazine

Alpha Adrenergic receptor

Dopamine receptor

Particularly cyproheptadine

Serotonin receptor

All H1- antihistamines

Histamine H1 receptor

Histamine H2 receptor

Pharmacology H1-antihistamines H 1 antihistamine pharmacological actions (drugs action) takes place by blocking blocks H 1 -histamine receptor effect. This results in beneficial effect on, Allergic symptoms, seasonal rhinitis, conjunctivitis, rhino viral infections (common cold) and urticaria. H 1 -antihistamine therapeutic uses. Antihistamine may bring relief to cold symptoms, runny nose, red and itchy eyes. Allergic rhinitis symptoms such as nasal allergies. Antihistamine sedative properties are utilized to treat temporary relief of insomnia. Antihistamines are effective to treat nausea and vomiting. Used for · Allergiesà 2nd gen · Runny noseà Diphenhydramine · Insomnia à Diphenhydramine · Nausea and (motion sickness): Dimenhydrinate, and doxylamine. · Motion sickness à Dimenhydrinate, Meclizine & scopolamine, Promethazine. GI TRACT (-5HT3 )

à

VOMITING CENTRE ß VESTIBULAR NUCLEI (MEDULLA) (H1 & M) CHEMORECEPTOR TRIGGERS (D) (OPIOIDS, CHEMOTHERAPY, HORMONE (PREGNANCY)

Side effects: Anticholinergic. Dry mouth, constipation, tachycardia, and difficulty voiding urine. CNS. Dizziness, drowsiness, performance impairment (memory, work performance, visual motor coordination). GI. Constipation Contraindications with mainly first generation anti histamines. Narrow angle glaucoma, bladder neck obstruction, hyperthyroidism, cardiovascular disease, and benign prostate hyperplasia. Copyright © 2000-2018 TIPS Inc. Unauthorized reproduction of this manual is strictly prohibited and it is illegal to 25-4 reproduce without permission. This manual is being used during review sessions conducted by PharmacyPrep.

www.Pharmacyprep.com

Medicinal Chemistry

First generation H 1 antagonist

Second generation H 1 antagonist

Higher sedation and anticholinergic Tertiary amine (cause sedation) Lipids soluble, and can cross blood brain barrier More central anticholinergic side effect Anti-serotonin Anti-bradykinin Usually QID

Less sedation due to less lipid soluble and do not penetrate the BBB. NO anticholinergic effect Drug interaction Terfenadine/astemizole. Cardiac arrhythmias characterized by prolong QT interval (torse de pointes). Grapefruit juice (apple, orange) reduces oral bioavailability of fexofenadine. Cetirizine have high fatigue and somnolence (10%). Loratadine have low fatigue and somnolence.

Dimenhydrinate, Meclizine, cyclizine and promethazine are useful for the prophylaxis of motion sickness and vertigo. Promethazine is the most potent antihistamine, and limits its use due to sedation.

Pregnancy: Bromopheniramine can cause teratogenicity. All 1st gen and 2nd generation antihistamine are used in pregnancy except bromopheniramine due to its teratogenicity. 1st generation commonly used due to its wide experience. In children (less sedative) antihistamines such as loratidine or use sodium chromoglycate. First generation impairs academic and learning abilities in children. Most antihistamine and nasal cromolyn considered safe in children over 2 years of age. There is limited information about fexofenadine in under 12-year-old Question Alerts! age. Topical antihistamines like levocabastine eye drops are used allergic conjunctivitis and Topical antihistamines. Olopatadine (Pantanol), available as nasal spray. Levocabastine (Livostin), Emedastine (Emadine), Azelastine (Astelin), and Kitotifen (Zaditor). Topical applications of H 1 receptor antagonist to the eye relieves itching, congestion of conjunctiva and erythema. The density of mast cells are high conjunctiva and tear film. H 2 receptor antagonist (H 2 RA) present on parietal cell, and help to produce HCl secretions: H 2 receptors mediated functions. H 2 receptors responses to histamine such as increased secretion of gastric acid increase pepsin and intrinsic factor (Castle’s factor). Mechanism: Competitive inhibitors of H2-receptors on parietal cells, thus inhibit gastric acid secretion. Therapeutic use: GERD, upset stomach (use prophylactically before meals (longer lasting episodes, night time and predictable heartburn after trigger food.) Ach

Histamine

Gastrin

Atropine

Ranitidine

Muscarinic receptor IP 3 C a 2+

H2 receptor cAMP

?

H+ secretion PPIs Copyright © 2000-2018 TIPS Inc. Unauthorized reproduction of this manual is strictly prohibited and it is illegal to 25-5 reproduce without permission. This manual is being used during review sessions conducted by PharmacyPrep.

www.Pharmacyprep.com

Medicinal Chemistry

Classification of H 2 antagonist (H 2 RA: Ranitidine, Cimetidine, Famotidine and Nizatidine (drug names end with "tidine"). Pattern of heartburn Intermittent, short term heartburn needing fast relief. Longer lasting episodes (night time) needing a longer duration of effect. Predictable heartburn after trigger food. Frequent heartburn or episodes needing treatment for more than few days

recommendation Antacids, alginates H2RA H2RA taken before unavoidable exposure. PPI

H2 antagonist chemistry

Cimetidine, an H2-receptor antagonist CH3 HN

N

N CN CH2SCH2CH2 NH C NHCH3

H 2 antagonist pharmacology. H 2 receptor antagonist competitively blocks H 2 receptors thus blocking the effect of histamines on gastric secretions. H 2 antagonist therapeutic use. To treat heartburn, dyspepsia GERD, GI ulcers, Stress-induced gastritis. H 2 antagonist side effects: Cimetidine can cause gynecomastia. Mild diarrhea, or constipation, headache, myalgia, confusion, hallucination, and excitement. Administration: all H 2 RA are available oral and cimetidine, famotidine and ranitidine are also parenteral. Oral have rapid absorption.

Serotonin (5-Hydroxy tryptamine 5HT) Question Alerts! 1) Serotonergic system modulates mood, emotion, sleep, and appetite. 2) Serotonin contain indole ring. 3) Tryptophan to serotonin is catalysed by? Hydroxylase & decarboxylase Serotonin neurotransmitters involved in vasoconstriction, vasodilation, regulation of body temperature, sleep, depression and hormonal regulations. Tryptophan (amino acid) à 5-hydroxy tryptophan à Serotonin (neurotransmitter). Tryptophan is precursor of serotonin and tryptophan taken up in neuron and converted to serotonin. (Increase synthesis of serotonin) Conversion of tryptophan to serotonin takes place in two reactions, first hydroxylation and decarboxylation catalyzed by tryptophan hydroxylase and L-amino acid decarboxylase respectively. Serotonin contain indole ring.

Copyright © 2000-2018 TIPS Inc. Unauthorized reproduction of this manual is strictly prohibited and it is illegal to 25-6 reproduce without permission. This manual is being used during review sessions conducted by PharmacyPrep.

www.Pharmacyprep.com

Medicinal Chemistry

NH2

NH2

HO COOH N H

CH2CH2NH2

COOH N H

Hydroxylase

Tryptophan

Decarboxylase

Hydroxy tryptophan

HO

Indole ring

HO

CH2CH2NH2 N H

N H Serotonin (5HT)

Niacin Tryptophan

Serotonin Melatonin

Serotonin (5HT)

MAO (oxidative de-amination) Serotonin-----------------------------------> 5-hydroxy indole acetic acid Serotonin targets · Serotonin receptors: 5HT 1 , 5HT 2 , and 5HT 3 (migraine, N&V) · Serotonin reuptake in CNS (antidepressants) · Break down of serotonin by MAO. (antidepressants) Physiological functions of serotonin receptors. Serotonin group has several subtypes of receptors: 5HT 1 , 5HT 2 , and 5HT 3 àDeficiency of 5HT 1 , 5HT 2 and 5HT 3 gives anxiety, depression, aggression, impulsive and appetite. · 5HT 1D ; Auto receptors inhibit presynaptic activity in both serotoninergic and adrenergic neurons in the CNS. · 5HT 2 . Vasoconstriction, and platelet aggregation · 5HT 3 . Excessive of 5HT 3 gives nausea, vomiting · 5HT 4 . Release of acetylcholine in the enteric region Subtype 5HT 1a

Clinical significance 5HT IN CNS ACTS AS NEUROTRANSMITTER.

Drug Buspirone

5HT 1b/1d 5HT 1c

CNS, vasoconstriction CNS Platelets, smooth muscles. (5HT CONSTRICTS SMOOTH MUSCLES OF BRONCHI & GI) CNS (-ve schizophrenic symptom). CNS, gastrointestinal (IN GI 5HT ACT AS LOCAL HORMONE & TO REGULATE PERISTALISIS MOVEMENT) CNS, gastrointestinal

Triptan

5HT 2

5HT 3 5HT 4

Clinical Use 5HT 1a agonist Antianxiety, antidepressant, antiaggressive, antiemetic. Migraine attacks

Ergotamine Cyproheptadine

Antimigraine (acute)

Ondansetron

Antiemetic in chemotherapy

Copyright © 2000-2018 TIPS Inc. Unauthorized reproduction of this manual is strictly prohibited and it is illegal to 25-7 reproduce without permission. This manual is being used during review sessions conducted by PharmacyPrep.

www.Pharmacyprep.com

Medicinal Chemistry

Serotonin 5HT1a Agonist Buspirone

5HT2

5HT1b/1d

Non selective Ergotamine (DHE) are 5HT2 agonist. Cyproheptadine affects both 5HT2 and 5HT1 (to -appetite) Trazadone is 5HT2A ANTAgonist Antagonist of 5HT2a atypical antipsychotic olanzapine Clozapine Risperidone Quetiapine

Agonist Triptans

Sumatriptan Rizatriptan Zolmitriptan Naratriptan

5HT3

5HT4

Antagonist Ondansetron Alosetron Fabesetron Ramosetron

Agonist Cisapride

Trazodone is a triazolopyridine derivative, chemically unrelated to other available antidepressants. Trazodone's most potent pharmacologic activity involves antagonism at the 5HT2a serotonin receptor. Less potently, trazodone also inhibits the serotonin transporter, which reduces serotonin reuptake by the presynaptic neuronal membrane. Because both of these functions contribute to its antidepressant effect, trazodone is classified as a serotonin antagonist-reuptake inhibitor, or SARI. Trazodone also exhibits antagonism at alpha-1 adrenergic receptors, which accounts for its propensity to cause orthostatic hypotension, and histamine-1 receptors, which contributes to its sedative properties. 5HT1 receptor class of drugs

5HT 1B/1D receptor agonist Mechanism: Contraction of arterial smooth muscles, especially in carotid and cranial circulations. Triptans (all are indole derivatives): Sumatriptan, Rizatriptan, Naratriptan, Zolmitriptan, Almotriptan, and Frovatriptan. CH CH TRIPTANS 5HT 1D/1B receptor agonist NSO CH CH CH N 3

3

2

2

2

H

2

CH3 N H

Sumatriptan

Indole ring

5HT 1D/1B receptor agonist therapeutic uses. Triptans are used to treat migraine headache attacks. 5HT 1D/1B receptors pharmacological actions cause constriction. 5HT 1D receptor agonist side effects. Feeling of warmth, dizziness, tightness or heaviness in the chest, rarely patient may experience chest pain.

Copyright © 2000-2018 TIPS Inc. Unauthorized reproduction of this manual is strictly prohibited and it is illegal to 25-8 reproduce without permission. This manual is being used during review sessions conducted by PharmacyPrep.

www.Pharmacyprep.com

Medicinal Chemistry

5HT 4 agonist Cisapride (a benzamide), and tagaseride (indole derivative). Ergotamine serotonin partial agonist. Ergot alkaloids have agonist and antagonist properties. Ergot alkaloids and derivatives with antagonist/partial agonist activity include ergonovine, dihydroergotamine (DHE), methysergide and bromocriptine. 5HT 4 agonist

Cl H2N

O C NH

N

CH2CH2CH2O

F

OCH3 CH3O

Cisapride (a benzamide), and tagaseride (indole derivative).

Cisapride

5HT 3 receptor antagonist: Ondansetron (indole derivatives) and Granisetron (benzimidazole derivative). 5HT 3 antagonist SETRONS

O CH2 N N CH3 Indole ring

N

ONDANSETRON GRANISETRON

Ondansetron

5HT 3 receptor antagonist pharmacology act on 5HT 3 mixed receptors and can effect on nausea and vomiting. 5HT 3 receptor therapeutic use ondansetron and granisetron are used to treat chemotherapy induced or vagal stimulation and surgery nausea and vomiting. 5HT 3 receptor antagonist side effects. Ondansetron side effects. Constipation, headache, dizziness and granisetron diarrhea. Prostaglandins, thromboxane’s, prostacyclins and leukotrienes are synthesized from arachidonic acid. These four substances are naturally occurring 20-carbon cyclopentanofatty acids derivative. Ecosonides Ecosonides are metabolites of arachidonic acid (a fatty acid). Examples of ecosonides are prostaglandin analogs, thromboxane’s, and prostacyclins. Arachidonic acid is derived from linoleic acid or taken from diet and esterified to phospholipids (phosphatidylethanolamine). Site of production of endogenous peptides are GIT, kidney, pancreases and uterus. Arachidonic acid is derived from dietary linoleic acid (2 double bonds) or is ingested from the diet and esterified to phospholipids.

Prostaglandin Prostaglandin has been classified based presence and absence of keto or hydroxyl groups at 9 and 11. Subscripts relate to the number of double bond present in aliphatic chains.

Copyright © 2000-2018 TIPS Inc. Unauthorized reproduction of this manual is strictly prohibited and it is illegal to 25-9 reproduce without permission. This manual is being used during review sessions conducted by PharmacyPrep.

www.Pharmacyprep.com

Medicinal Chemistry

O

Leukotrienes

Corticosteroids

9

COOH

Lipoxygenase Arachidonic acid

Membrane phospholoipids

HO

Cyclooxygenase NSAID, ASA, COX-II Phospholipase A2 Prostaglandins G Hydroperoxidase Prostacyclin (PGI2)

PGH2

Platelet aggregation Vascular tone Bronchial tone

Paltelet aggregation Vascular tone PGE2 & PGF2

Uterine tone

Thromboxane A2

Dipyridamole

Uterine tone

11

Question Alerts! SAR of prostaglandins? PGE1 and PGF2a 1) PGE analogs have 9 keto and 11 hydroxyl group 2) PGF analogs have 9 hydroxyl and 11 hydroxyl group 3) Misoprostol is? Ecosonides 4. Misoprostol has one double bond and two hydroxyl groups (11C and 16C)

Vascular tone Bronchial tone

PHYSIOLOGICAL FUNCTIONS: Platelet aggregation, relaxes bronchial and GI smooth muscles, relax smooth muscles, and inhibit gastric acid secretion, pain, edema, and inflammation. Prostaglandin analogs

PGE1

PGE2

Protects gastric mucosa

Pyrogen elevate PGE2 contraction of uterus

Misoprostol Alprostadil Bronchial & smooth muscle dilatation

Dinoprostone Bronchial & smooth muscle dilatation

PGF2a Bronchoconstriction contraction of uterus Latano"prost" Bronchial & smooth muscle constriction

PGI2 Decrease platelets aggregation Epoprostenol Blood vessels dilatation Inhibit aggregation of platelets

TxA2 Increase Platelets aggregation

Thromboxane A2 Dipyridamol inhibits platelet aggregation

PGE analogs PGE 1 analogs classification: Misoprostol, and alprostadil. PGE 1 analogs medicinal chemistry. Misoprostol is chemically belonging to Economides.

Copyright © 2000-2018 TIPS Inc. Unauthorized reproduction of this manual is strictly prohibited and it is illegal to 25-10 reproduce without permission. This manual is being used during review sessions conducted by PharmacyPrep.

www.Pharmacyprep.com PGE1 ANALOG MISOPROSTOL Consist of 2-OH and one double bond.

Medicinal Chemistry MISOPROSTOL ¯ HCL SECRETION - MUCUS SECRETION & BICARBONATE USED FOR CYTOPROTECTIVE EFFECT. DOES NOT PREVENT GI SYMPTOMS

PGE 1 analogs pharmacology. PGE 1 and PGH 1 can be used to produce relatively local vasodilatation. PGE 1 analogs therapeutic uses: · · · · ·

Misoprostol is used for prevention of NSAID induced GI ulcers. Combination products. Naproxen + misoprostol, and diclophenac + misoprostol. Misoprostol vaginal use is for cervical priming before endometrial procedures (dilation and curettage). Alprostadil. In adults it useful for the treatment of impotence due to erectile dysfunction. Alprostadil is used for temporary maintenance of a patent ductus arteriosus when awaiting corrective surgery for congenital heart defects.

PGE 1 analogs side effects ·

Misoprostol. Abortificient side effect. The common side effects is diarrhea. Abdominal pain, uterine contraction (can cause miscarriage in pregnancy).

PGE 2 analogs classification ·

Dinoprostone derivatives. Dinoprostone

PGE 2 analogs therapeutic uses. Dinoprostone are used for their abortificient effects and to induce cervical ripening in pregnancy. PGF 2 a analogs classification: LATANOPROST, TRAVOPROS, BIMATOPROST, UNOPROSTONE PGF 2a ANALOG

Question Alerts! - outflow of aqueous humor.

PGF 2a analogs pharmacology. Lowers IOP by increasing uvescleral (aqueous humor) outflow. PGF 2a analogs therapeutic uses. · Latanoprost (Xalatan) 0.005% ophthalmic solution. Used topically to lower intra ocular pressure in OAG, combination products latanoprost + timolol indicated for open and close angle glaucoma. Copyright © 2000-2018 TIPS Inc. Unauthorized reproduction of this manual is strictly prohibited and it is illegal to 25-11 reproduce without permission. This manual is being used during review sessions conducted by PharmacyPrep.

www.Pharmacyprep.com · · · ·

Medicinal Chemistry

Travoprost (Travatan). Topical ophthalmic drug Bimatoprost (Lumigan). Topical ophthalmic drug Unoprostone (Rescula). Topical ophthalmic drug Carboprost (Hemabate). Abortificient (withdrawn).

PGF 2a analogs side effects: Eye pigmentation, lengthening and thickening eyelashes. PGI analogs. Epoprostenol (Prostcyclin) PGI analogs (prostacylcin) chemistry PGI analogs (prostacylcin) pharmacology PGI analogs (prostacylcin) therapeutic use. Epoprostenol. Used in the treatment of emergency pulmonary hypertension. It produces antiplatelets action. Thromboxanes (TxA2). · Increase platelet aggregation and potent vasoconstrictors. · Dipyridamol blocks thromboxane thus produce antiplatelet action. Leukotrienes Leukotrienes are produced from arachidonic acid; this reaction is catalyzed by 5-lipoxygenase enzyme, which oxidize polyunsaturated fatty acids possessing two cis double bonds separated by a methylene Questions Alerts! group to produce lipid hydroperoxide.

1) Leukotrienes are produced from arachidonic acid by the catalyzation of? Lipoxygenase 2) LTRAs act on? LTC4 and LTD4 3) Leukotriene inhibitors used in children > 2yrs? Montelukast

Leukocytes express lipoxygenase and release leukotrienes in lungs. Plays important role in numerous physiological functions. · Slow reacting substance of anaphylaxis. · Heart. Negative inotropic, and smooth muscles chemotaxis. · GI tract. Neutrophil chemotaxis. · Pulmonary (major): Bronchoconstriction, increase permeability, and increase mucus secretion. · Blood: Chemotactic agent for neutrophil, eosinophils, and modify lymphocyte proliferation and differentiation.

Leukotriene antagonists’ chemistry. Zafirlukast and montelukast have peptidomimetic structure and inhibit LTC 4 and LTD 4 receptors. MONTELUKAST

inhibit LTC 4 and LTD 4 receptors PEPTIDOMIMETIC STRUCTURE

Copyright © 2000-2018 TIPS Inc. Unauthorized reproduction of this manual is strictly prohibited and it is illegal to 25-12 reproduce without permission. This manual is being used during review sessions conducted by PharmacyPrep.

www.Pharmacyprep.com

Medicinal Chemistry

ZAFIRLUKAST

Leukotrienes antagonist’s therapeutic uses. Zafirlukast: For the prophylaxis and chronic treatment of asthma in adults and children 12 years of age and older. Take empty stomach to enhance it absorption. Montelukast. Can be used in children over 2-year age, Montelukast may be taken without regard of food. Available as chewable tablet (once daily in the evening) and granules. Administer granules directly into mouth or mix with teaspoon of cold or room temperature applesauce, carrot, rice or ice cream. Do not take Aspirin or NSAIDs while on this medication. Leukotriene inhibitors are drug of choice for the treatment of Aspirin induced asthma and maintenance. Side effects. GI upset, abdominal pain, diarrhea, liver dysfunction, and headache. Drug interactions. Terfenadine significantly reduces the plasma concentration of zafirlukast.

Non Steroidal Anti-inflammatory Drugs (NSAIDS) Salicylates derivatives

NSAIDs: Cox 1&II reversible competitive inhibitors

Acetyl salicylic acid (ASA)

Antiplatelet, antipyretic, analgesic and anti-inflammatory [AAAA] ASA is irreversible Cox I & II inhibitor.

Salicylic acid Methyl salicylates Salsalate, sodium thiosalicylate Choline salicylate 5-Aminosalicylic acid (Mesalamine) Olsalazine Sulphasalazine Diflunisal Pyrazolone derivatives

Wintergreen oil-topical agent, counter irritant Injectable Oral liquid Crohn's disease and ulcerative colitis Active product is 5ASA. Use for ulcerative colitis and Crohn's disease Diflurophenyl derivative of salicylic acid

Sulfinpyrazone Phenylbutazone Propionic acid derivatives Ibuprofen Ketoprofen Naproxen Carprofen Fenoprofen

The order of gastric ulcerogenic activity. Sulindac>naproxen >ASA, indomethacin, ketoprofen, ibuprofen Primarily used for pain associated with inflammation. Mild stomach upset Primarily Long lasting pain and migraines. Can be hard on stomach compare to ibuprofen.

Copyright © 2000-2018 TIPS Inc. Unauthorized reproduction of this manual is strictly prohibited and it is illegal to 25-13 reproduce without permission. This manual is being used during review sessions conducted by PharmacyPrep.

www.Pharmacyprep.com

Medicinal Chemistry

Acetic acid derivatives Indomethacin Diclofenac (po, topical, spray) Ketorolac Sulindac Etodolac Anthranilic acid derivatives Mefenamic acid Meclofenate Oxicams derivatives

The most potent. But high renal side effects, have high anti-inflammatory Voltaren Toradol. NSAID used for acute pain.

Piroxicam Meloxicam Pyrazole derivatives.

Feldene

Used for menstrual pain or dysmenorrhea.

COX II inhibitors: induced in pathological states inducible; Cox II produce prostaglandins at inflammation sites, macrophages, synovocytes, and cause inflammation and pain. COX II Enzyme are responsible for arthritis pain, thus COX2 inh. Are long term used for arthritis. Increase risk of blood clot, heart attack, or stroke and CHF. Only has no sulfa allergy, where all other Cox II inhibitors have sulfa

Celecoxib (Celebrex) Lumerocoxib

allergy. Chemistry of Salicylates derivatives F

COOH O

F

OH

OH

COOH COOCH3

CH3

O

Acetyl Salicylate

Diflunisal (Dolobid)

Methyl salicylate (Wintergreen oil)

Acetyl salicylic acid (ASA) · Pharmacological actions: ASA exhibits analgesics, antipyretics, anti-inflammatory and antiplatelet actions. ASA irreversibly inactivates both COX 1 & COX 2 , where as all other NSAIDs inactive reversible COX 1 & COX 2 . Onset: 5-30 min, duration 3-6 hours Analgesic action · Prostaglandin PGE 2 thought to sensitize the nerve endings to the action of bradykinin, histamines and other chemical mediators, thus inflammatory process may cause analgesic action. Decrease of PGE 2 synthesis represses the sensation of pain. ASA analgesic dose 325 mg every 4 to 6 h. Antipyretic action · Prostaglandin PGE 2 stimulation occurs when pyrogen an endogenous fever producing agents such as cytokines is released from the white blood cells that are activated by infection, hypersensitivity, malignancy or inflammation. Salicylates lower temperature by decreasing PGE 2 synthesis. ASA antipyretics dose 325 to 650 mg q 4 to 6 h PRN. Copyright © 2000-2018 TIPS Inc. Unauthorized reproduction of this manual is strictly prohibited and it is illegal to 25-14 reproduce without permission. This manual is being used during review sessions conducted by PharmacyPrep.

www.Pharmacyprep.com ·

Medicinal Chemistry

Reye syndrome: Children with flu and viral infection should avoid using ASA, because it may cause Reye’s syndrome.

Antiplatelets action Irreversible platelet inhibition and inhibition of Cox-I & Cox-II action gives antiplatelets action. Antiplatelets action minimum dose is 60 to 80 mg. Anti-inflammatory action: Cox-I, Cox-II and prostaglandin inhibition. The anti-inflammatory dose of ASA 650 mg. Mechanism of action of ASA. ASA

COX -1 FUNCTIONS

COOH O OCH3 O Cyclooxygenenase Active O

COX I & II

O C CH3 Cyclooxygenase (inactive)

COX-2 FUNCTIONS

Daily synthesis of prostaglandin Expressed only in response of that contribute to normal inflammation of injury. homeostatic (arrest of bleeding). Protection of the gastric mucosa through prostaglandin Hemostasis through the synthesis of thromboxane.

COOH OH

GI mucosa

Cox I

PGE2 - Gastric protection, - bicarbonates, - mucosal blood flow

Kidney

Cox I & II

PGE2 & PGI2; efferent arterial vasodilation (-GFR) and - Na&H2O excretion

Cardiovascu lar

Cox I & II

PGI2&TxA2 vasoconstriction, platelet aggregation COX2: vasodilation and inhibit platelet aggregation.

Arachidon ic acid

NSAIDs - GI bleeding, peptic ulcer ¯GFR & Na&H2O retention - stroke and MI

Problems associated with NSAID and acetyl salicylic acid. · Respiratory depression: Toxicity respiratory alkalosis and metabolic acidosis (increase CO2 and decrease pH). · GI effects: due to inhibition of prostaglandin PGEs. Prostacyclin PGI 2 inhibit gastric secretion. The most common side effects of salicylate are GI disturbances like nausea, vomiting, epigastric discomfort, GI bleeding, peptic ulcers (dyspepsia, heart burn). · PGE 2 and PGF 2a stimulates synthesis of protective mucosa in stomach and small intestine. · Indomethacin has the highest incidence of (35 to 50%) of GI ulcers. · Renal problems: PGE 2 and PGI 2 are responsible for renal blood flow. NSAID ¯ RENAL BLOOD FLOW. It can cause acute renal failure. NSAIDS INDUCE ASTHMA BY AIRWAY RESISTANCE. Reye's syndrome is an acute syndrome that may follow influenza and chicken pox infections in children. It is characterized by symptoms of sudden vomiting, violent headache, and unusual behavior. The Reye's syndrome is associated with only ASA.

Copyright © 2000-2018 TIPS Inc. Unauthorized reproduction of this manual is strictly prohibited and it is illegal to 25-15 reproduce without permission. This manual is being used during review sessions conducted by PharmacyPrep.

www.Pharmacyprep.com

Medicinal Chemistry

Sulfasalazine chemistry · Contains azo bond. Sulfasalazine contains a sulfonamide group, may cause allergy patients with sulfa allergy. · In metabolism undergoes azo reduction (phase I). Metabolism produces 5-amino salicylic acid. Sulfasalazine, olsalazine, balsalazine are metabolized in the colon by gut flora to yield 5ASA. Avoid in ASA allergic patients. · Sulfasalazine therapeutic use drug of choice for ulcerative colitis. · Sulfasalazine side effects: can cause megaloblastic anemia, and infertility (in men). NH2

Azo bond

COOH

O N S N OH

N N

HO

CO2H

OH 5-aminosalicylic acid (Mesalamine)

Azoreductase

+ O N S N OH

at Gut NH2

Sulfasalazine

Sulfapyridine

Azoreduction in colon

Diflunisal · Diflurophenyl derivative of salicylic acid. · No salicylate toxicities (does not produce salicylic acid) · No antipyretic action

Propionic acid derivatives: Ibuprofen, Ketoprofen, Naproxen, Carprofen and Fenoprofen Propionic acid IBUPROFEN LOW GI SIDE EFFECT H3C CH COOH Ibuprofen

ACETIC ACID DERIVATIVE Acetic acid derivatives: Indomethacin, Diclofenac, Ketorolac, Sulindac and Etodolac Acetic acid INDOMETHACIN Sulindac has long half-life used daily or BID. H3CO

COOH N

CH3 O Indomethacin

Cl

Acidic acid derivative therapeutic uses · · · ·

Indomethacin is used for acute gout attacks treatment, more CNS effect, aplastic anemia and contraindicated in elderly. Diclofenac (Voltaren)à ophthalmic drops Ketorolac (Toradol)à short-term use only Sulindac (Clinoril)à safest in renal failure

Copyright © 2000-2018 TIPS Inc. Unauthorized reproduction of this manual is strictly prohibited and it is illegal to 25-16 reproduce without permission. This manual is being used during review sessions conducted by PharmacyPrep.

www.Pharmacyprep.com

Medicinal Chemistry

ANTHRANILIC ACID (FENAMATES) DERIVATIVES Anthralnic acid Mefenamic and Meclofenate acid

H3C

Mefenamic acid is associated with severe diarrhea and associated with inflammation of bowel. Preferred in dysmenorrhea

CH3

NH COOH Mefenamic acid

Anthranilic acid (fenamates) derivatives therapeutic use: PREFERRED IN DYSMENORRHEA Oxicams derivatives: Piroxicam (feldene), Meloxicam (Mobic) and Tenoxicam. OH MELOXICAM CONH S O2

N

N CH3

Piroxicam

Oxicams derivatives side effects ·

Piroxicam (Feldene)à Its mean half life is 50 hours; thus, this can be used once daily dose, more bleeding, contraindicated in elderly.

p-Aminophenol derivatives: Acetaminophen chemistry. ACETAMINOPHEN

H3COCHN Dealkylation OCH2CH3

CH3CONH

Acetaminophen is an active metabolite of phenacetin or acetanilide.

OH Phenacetin

Acetaminophen

Phenacetin or acetanilideà Acetaminophen Acetanalide--> Hydroxylation--> acetaminophen Acetaminophen pharmacology: Acetaminophen has antipyretic and analgesic action. · · · ·

Inhibits central prostaglandin synthesis in CNS. Less effective in blocking peripheral prostaglandin synthesis, acetaminophen has no anti-inflammatory activity and does not effect on platelet function. Antipyretic action dose is 325 to 650 mg q 4 to 6 h PRN. Acetaminophen does NOT have Rye syndrome and may safely be able to be used in patient (child) with varicella or an influenza type viral infection. Can be used in ASA or NSAID allergic patients. Can be used in children because no Rye syndrome. Advantages are stable in liquid thus available in various solutions.

Copyright © 2000-2018 TIPS Inc. Unauthorized reproduction of this manual is strictly prohibited and it is illegal to 25-17 reproduce without permission. This manual is being used during review sessions conducted by PharmacyPrep.

www.Pharmacyprep.com

Medicinal Chemistry

Acetaminophen side effects. Skin rash, hemolytic anemia (long term phenacetin use), high doses renal dysfunction and tubular necrosis hepatotoxicity if it used more than 4 g/day. With excessive alcohol max dose 2 g/day. Acetaminophen metabolism: Acetaminophen undergoes phase I metabolism and glucuronidation, sulphate conjugation, and glutathione conjugation. Toxic intermediate of phase I metabolism is N-benzoquinoneimine, which is catalyzed by glutathione conjugation leading to cysteine and mercapturic acid.

Acetaminophen Metabolism

Question Alerts!

O CH3 C NH

S

sf e tran ulfo

r as

e

OH

Acetaminophen

UDP-GT Glucoronidation

Sulfate conjugation Cytochrome CYP 450 N-benzoquinoneimine Glutathione conjugation

1) Acetaminophen structure? 2) Phenacetin dealkylation gives acetaminophen. 3) Acetanilid hydroxylation gives acetaminophen 4) Common Side effects of acetaminophen? Skin rash 5) Acetaminophen can cause? Hepatotoxicity 6) Acetaminophen metabolism occurs in? Liver 7) Toxic metabolite intermediate produced by acetaminophen in liver is? N-Benzoquinoneimine 8) Toxic metabolite N-benzoquinoneimine is catalyzed by? Glutathione conjugation 9) Glutathione conjugation end product is ? Mercapturic acid

Mercaptopurine

Glutathione saturation causes hepatotoxicity. Antidote N-acetylcysteine binds with glutathione and activate metabolism of N-benzoquinoneimine. HS Glutathione is consisting of glutamic acid, cysteine, and O H glycine N COOH HOOC Antidote of acetaminophen is N-acetylcysteine N (CH3CONH-CH-(COOH)-CH2SH) H NAPB. N-acetyl p-benzoquinoneimine. NH2 O UDP-GT. Uridinyl diphosphate glucuronidase Glutatione transferase.

COX-2 INHIBITORS Celecoxib (Celebrex), PYRAZOLE DERIVATIVES

Celecoxib (cerebrex) SO2NH2 F3C N

N

CH3

CELECOXIB IS USED FOR ARTHRITIC PAIN Question Alerts! Drugs that have sulfa allergy? Celecoxib, hydrochlorothiazide, sulfonamide antibiotics, acetazolamide, Furosemide, sulfasalazine, rosuvastatin.

Copyright © 2000-2018 TIPS Inc. Unauthorized reproduction of this manual is strictly prohibited and it is illegal to 25-18 reproduce without permission. This manual is being used during review sessions conducted by PharmacyPrep.

www.Pharmacyprep.com · ·

Medicinal Chemistry

COX 2 inhibitors Side effects: Arrhythmias, GI upset, diarrhea, back pain, respiratory problems, and nephrotoxicity. Celebrex contain sulfa group; therefore, celecoxib has sulfa allergy. Celecoxib and valdecoxib have sulfonamide functional group.

OPIOIDS STRUCTURE ACTIVITY Benzomorphan Pentazocine. HCl derivatives Pentazocine lactate

Weak mu antagonist and potent kappa agonist action Pentazocine-kappa agonist, given with naloxone to prevent abuse.

Phenyl piperidine Derivatives

Morphinan Derivatives

Natural opium alkaloids

Phenylethylamine Derivatives

Fentanyl citrate Meperidine. HCl (pethidine) Sufentanil citrate Loperamide

Structure activity of opium alkaloids. 5) Meperidine metabolizes to normeperidine and this can cause seizures. 6) Fentanyl short half-life and lipid solubility helps to transdermal action.

Butorphanol tartrate Nalbuphine. HCl

Butorphanol. A kappa agonist, mu antagonist more potent but less efficacy than morphine and not orally active.

Codeine phosphate Hydromorphone.HCl Morphine.HCL Morphine sulfate Oxycodone.HCl Oxymorphone.HCl Buprenorphine

1) p-phenyl N-methyl piperidine is essential. 2) Morphine diacetylation gives heroin. 3) Codeine demethylation gives morphine by CYP 2D6. Buprenorphine a mixed analgesic, more potent than morphine but less efficacy.

Methadone

Methadone and propoxyphene structurally related to phenylethylamine

Propoxyphene.HCl Propoxyphene napsylate NALOXONE

Naloxone is antidote has “N-allyl group”.

Copyright © 2000-2018 TIPS Inc. Unauthorized reproduction of this manual is strictly prohibited and it is illegal to 25-19 reproduce without permission. This manual is being used during review sessions conducted by PharmacyPrep.

www.Pharmacyprep.com

9 1

HO 3

Medicinal Chemistry CH3 N

CH3 N

O

6 OH

O

CH3O

CH3 N

CH3 N

O

O

CH3O

O

Oxycodone

Hydromorphone

CH2CH CH2 N

Allyl group

O

O

O Propoxyphene

Methadone

CH3 CH2CH C CH3 N

CH2 N HO

CH3 O

Naloxone

O O C CH3

CH3 H3C CH2 O N CH2CH2 C O C CH2CH3 CH3

N H3C

HO HO

O

Heroin

H3C

HO HO

O H3C C O

OH

Codeine

Morphine

CH3 N

Acetyl groups

CH3 HO Pentazocine

HO Butorphanol

CYP2D6 (DEMETHYLATION) CODEINE ----------------------------------------> MORPHINE HYDROCODONE ---------------------------------à HYDROMORPHONE OXYCODONE ------------------------------------à OXYMORPHONE Morphine structure activity · Reactive metabolite of morphine is morphine 6-glucuronide · Phenolic hydroxyl is important for activity. Analgesic activity is depending on p-phenyl-N-alkylpiperidine moiety (4-phenylpiperidine). Piperidine ring is chair-form conformation and perpendicular aromatic ring. It has tertiary amine group (methyl). · Codeine, heroin, morphine, meperidine, hydromorphone is structurally same class and have cross allergy. Hydrocodone is derivatives of codeine, hydromorphone is derivative of morphine. · Meperidine is piperidine analgesics, meperidine metabolizes to toxic metabolite normeperidine this can cause seizures in renal disease patients. · Partial agonist/antagonist characteristics replacement of methyl moiety on the nitrogen atom with larger substituent's. · Allyl substitution is present in naloxone. This allyl group is associated with opioid antagonist action. · Substitutions at the C 3 and C 6 morphine hydroxyl group’s pharmacokinetic properties altered. · Methyl substitution at C 3 reduces first-pass hepatic metabolism by glucuronide conjugation as a consequence codeine and oxycodone have a higher oral, and parenteral potency. · Acetylation of both morphine hydroxyls gives heroin (more rapid access across the blood-brain barrier compared morphine) in the brain heroin is rapidly hydrolysed to monoacetylmorphine and morphine.

Copyright © 2000-2018 TIPS Inc. Unauthorized reproduction of this manual is strictly prohibited and it is illegal to 25-20 reproduce without permission. This manual is being used during review sessions conducted by PharmacyPrep.

www.Pharmacyprep.com

Medicinal Chemistry

Opioids as antidiarrheal drugs · Loperamide (Imodium). Has low abuse potential. Used as antidiarrheal. Maximum dose 16 mg/day. · Diphenoxylate + Atropine: Lomotil Opioids as antitussives drugs: · Codeine is used extensively as cough suppressants. · Hydro codeine bitartrate is 3 times more effective as antitussive. · Dextromethorphan HBr is the (+) isomer of the 3-methoxy form of the synthetic opioids levorphanol.

Tips 1. 4. 7. 10. 13. 16. 19. 22. 25. 28. · · · · · · · · · · · · · · · · · · · · ·

PGF2a analog histadine Mercapturic acid Misoprostol Crosses BBB 5HT3 antagonist Diacetyl morphine glutamic acid constipation analgesic

2. 5. 8. 11. 14. 17. 20. 23. 26. 29.

Ecosonide PGE1 analog LTC 4 and LTD 4 proton pump inhibitor Tertiary amine GI bleeding 5HT1B/1D agonist cysteine pinpoint pupil Type 1 PG analog antidiarrheal

3. 6. 9. 12. 15. 18. 21. 24. 27. 30.

Tryptophan N-acetyl-p-benzoquinoneimine Pyroxicam Lipid soluble Renal diseases peptide opioid neurotransmitter glycine respiratory depression steroid sparing agents antitussive

Histamine precursor is? ( ) Serotonin precursor of? ( ) Misoprostol is? ( ) 1st generation antihistamines often cause sedation, this is due to? ( ) Ondansetron is classified as? ( ) Triptans is classified as? ( ) Latanoprost is classified as? ( ) Montelukast and zafirlukast act on? ( ) Endorphins are? ( ) Acetaminophen hepatotoxicity is due to? ( ) Glutathione conjugation produce? ( ) The most common side effects associated with NSAIDs ( ) What NSAID has highest GI bleeding? ( ) Drug of choice to treat NSAID induced GI ulcer ( ) Drug of choice to prevent NSAID induced GI ulcer? ( ) Glutathione consist of? ( ) Ibuprofen, indomethacin, mefenamic acid are? ( ) Leukotriene antagonist also referred as? ( ) Opioids can be used as? ( ) Heroin structure ( ) Morphine overdose symptoms; ( ) Glutathione ( ) is a tripeptide that contain unusual tripeptide linkage between amino acids of LGlutamaic acid, L-Cysteine and Glycine

Copyright © 2000-2018 TIPS Inc. Unauthorized reproduction of this manual is strictly prohibited and it is illegal to 25-21 reproduce without permission. This manual is being used during review sessions conducted by PharmacyPrep.

www.Pharmacyprep.com

Medicinal Chemistry

Copyright © 2000-2018 TIPS Inc. Unauthorized reproduction of this manual is strictly prohibited and it is illegal to 25-22 reproduce without permission. This manual is being used during review sessions conducted by PharmacyPrep.

www.Pharmacyprep.com

Medicinal Chemistry

26 Medicinal Chemistry and Pharmacology of Cardiovascular Drugs Questions Alerts! Common questions in pharmacy exam is to ask! · Mechanism of diuretics and site of actions. Hydrochlorothiazide chemical structure. · Structure activity relation ACEi. and captopril structure · Structure activity relation ARBs. · Statins or HMG Co-A reductase inh. and structure activity relation · Calcium channel blocker (CCBs) structure activity relationship · Vasodilators: Hydralazine, dizoxide and CCB,s

Thiazide Chlorthalidone Hydrochlorothiazide Indapamide Metolazone

Loop diuretics Ethacrynic acid Furosemide

K+ Sparing Amiloride Spironolactone Triamterene Eplerenone

CA inhibitors Acetazolamide Dorzolamide

Osmotic Mannitol Urea

Distal convoluted tubule Hyper GLUC

Ascending loop

Collect duct

Proximal tubule

OH DANG Loops loose Ca Metabolic alkalosis

HyperKalemia

Proximal tubule

Metabolic alkalosis

Metabolic acidosis and intracellular alkalosis

Metabolic acidosis

Copyright © 2000-2018 TIPS Inc. Unauthorized reproduction of this manual is strictly prohibited and it is illegal to 26-1 reproduce without permission. This manual is being used during review sessions conducted by PharmacyPrep.

www.Pharmacyprep.com

Medicinal Chemistry

Diuretics In general, the opposite findings of serum electrolytes are seen in urine. Type of diuretic Thiazide

Loop Ksparing CAI Osmotic

Site of action Distal convoluted tubule Loop of henle Early Collecting duct Proximal tubule Glomerular

Ca

Mg

Na

K

-

¯

¯

¯

Uric acid -

Blood sugar -

Lipids

Metabolic Disturbances

¯

¯

¯

¯

-

-

_

Hypokalemic metabolic alkalosis

_

-

¯

-

-

_

_

_

_

_

¯

-

-

_

Hyperchloremic metabolic acidosis (↑CO 2 ) Intracellular alkalosis Hyperchloremic metabolic acidosis

-

Hypokalemic metabolic alkalosis "HYPERGLUC"

¯

Thiazide diuretics: Chlorthalidone, hydrochlorothiazide, indapamide hemihydrate, and metolazone. Thiazide diuretics chemistry: Benzene ring with sulfonamide in position 7, and halogen or trifluoro methyl group in position 6. The electron withdrawing group at position 6 is necessary. Saturation of 3, 4 double bonds (increases potency with hydrochlorothiazide). Methyl group at position 2, or lipophylic substituents at position 3, enhance the potency and prolong the activity. Replacement of sulfonyl in position 1 by a carbonyl group prolongs the activity. Cl Essential

NH2SO2

H N S O2

No Doble bond on position 3 - 4

N

Cl

NH

S O2

NH2SO2

NH

Chlorothiazide

Hydrochlorothiazide

Cl O

N

NH2SO2 N

Indapamide (Lozol)

Hydrochlorothiazide Tablets The duration of action up to 18 hrs

CH3

Cl

CH3

NH S O2

Diazoxide (Hyperstat)

Chlorothiazide Liquid The duration of action is 612 hrs.

Copyright © 2000-2018 TIPS Inc. Unauthorized reproduction of this manual is strictly prohibited and it is illegal to 26-2 reproduce without permission. This manual is being used during review sessions conducted by PharmacyPrep.

www.Pharmacyprep.com

Medicinal Chemistry

Thiazide diuretics pharmacology · Thiazide act on distal tubule of nephron and ↑ H 2 O, Na, Cl, K, excretions (↓ levels in body), hyperuricemia and retain calcium (Ca). long term use of thiazides can cause hypokalemia and hypomagnesemia thus K and Mg supplement may require. · May cause alkaline urinary pH by effecting on carbonic anhydrase. · Metabolic alkalosis (Hypochloremic alkalosis). Thiazide diuretics therapeutic uses · Thiazides are the drug of choice for uncomplicated hypertension. · Especially useful in elderly and African populations and with chronic renal diseases. · Not effective in patient renal clearance less than 50 ml/min. · Not preferable in diabetic and hyperlipidemic patient. Thiazides diuretics side effects (Hyper GLUC) · Hypokalemia, hypomagnesia, hyponatremia, hypercalcemia, hyperuricemia, hyperglycemia, metabolic alkalosis, photosensitivity rashes, acute pancreatitis, libido, difficulty with erection and ejaculation (long term). Loop Diuretics: Furosemide, ethacrynate sodium, and ethacrynic acid. Also known as high ceiling diuretics. i.e. produce a peak diuresis much greater than other diuretics. Loop Diuretics chemistry: Anthranilic acid derivatives with sulphonamide substituent, or aryloxyacetic acids without sulphonamide substituent. LOOP DIURETICS FUROSEMIDE

ETHACRYNIC ACID

2-Aminobenzoic acid derivative Sulfa allergy Reversible ototoxicity Phenoxy acetic acid derivative NO Sulfa allergy Irreversible ototoxicity

Loop diuretics pharmacology · Increase H 2 O, Na, Cl, K, Ca excretion (decrease levels in body) and Increase Ca2+ ion excretion (hypocalcemia) (Loops Lose Calcium). · No change in urinary pH. Metabolic alkalosis (hypochloremic alkalosis) · Act in the thick ascending loop of henle and inhibit the sodium/potassium dichloride co-transport system. Potent agent can excrete up to 25% of filtered Na+. No ions come into the cell therefore the sodium pump and Na/Cl symport do not work. Loop diuretics therapeutic use: Furosemide is the drug of choice in renal disease (CrCl is less than 50 ml/min), acute pulmonary edema, and hypercalcemia. Loop diuretics side effects: [OH DANG] Ototoxicity, Hypokalemia, Dehydration, Allergy (sulfa), except ethacrynic acid, Nephritis (intestinal), and Gout arthritis. Sequence of ototoxicity (ethacrynic acid > furosemide > bumetanide) Copyright © 2000-2018 TIPS Inc. Unauthorized reproduction of this manual is strictly prohibited and it is illegal to 26-3 reproduce without permission. This manual is being used during review sessions conducted by PharmacyPrep.

www.Pharmacyprep.com

Medicinal Chemistry

Ototoxicity: Ethacrinic acid ototoxicity is irreversible (permanent hearing loss). Renal disease increase risk of ototoxicity. Furosemide ototoxicity is reversible sensory neural hearing loss, i.e. discontinuing drug can be normal. Loop diuretic onset in 30 min and last 6 hrs. Carbonic anhydrase inhibitors: Acetazolamide, dorzolamide, brinzolamide. Carbonic anhydrase inhibitor chemistry: Acetazolamide. Aromatic or heterocyclic sulfonamides with a thiadiazole ring. CARBONIC ANHYDRASE INHIBITORS ACETAZOLAMIDE

All CA inhibitors the sulfonamide l group is essential for CA inhibitory activity.

DORZOLAMIDE

Carbonic anhydrase inhibitor pharmacology. Increase water, sodium, and potassium and bicarbonate excretion. Cause alkaline urinary pH, and gives metabolic acidosis. Carbonic anhydrase inhibitor therapeutics: Used in treatment of glaucoma (not chronically). Acute mountain sickness (respiratory alkalosis) also high altitude sickness (mountain sickness). Because of the alkaline diuresis it produces, acetazolamide has been used for the treatment of overdoses of acidic drugs. Carbonic anhydrase side effects · Because of the alkaline Electrolytes à Hyperchloremic metabolic acidosis (loss of HCO 3 -), hypokalemia. · Formation of renal stones (phosphaturia & hypercalciuria). · CNS à depression, drowsiness, sedation, fatigue, and disorientation. · GI à Nausea, Vomiting, and Constipation · Blood related à Bone marrow depression, thrombocytopenia (reduction of number of platelets in blood), hemolytic anemia, leucopenia (reduction in number of WBC), agranulocytosis (acute deficiency of neutrophils). Sulfa drugs antibiotics type allergies. Osmotic diuretics. Mannitol and Urea OSMOTIC DIURETICS MANNITOL

Mannitol is a carbohydrate, monosaccharide

UREA

Copyright © 2000-2018 TIPS Inc. Unauthorized reproduction of this manual is strictly prohibited and it is illegal to 26-4 reproduce without permission. This manual is being used during review sessions conducted by PharmacyPrep.

www.Pharmacyprep.com

Medicinal Chemistry

Osmotic diuretics chemistry · Water-soluble with low renal threshold, and highly polar. · They limit tubular reabsorption of water. · Promote diuresis. · Increase in urinary pH Osmotic diuretics pharmacology · This drugs limit tubular reabsorption of water and promote diuresis. · Increase excretion of H 2 O, Na, Cl and HCO 3 (decrease levels in body), increase alkaline urinary pHàIncrease excretion of HCO 3 Osmotic diuretics therapeutic use · This drugs limit tubular reabsorption of water and promote diuresis. · Mannitol used in treatment of shock, to treat drug overdose and to decrease intracranial or intraocular pressure. · Prophylaxis of acute renal failure. Osmotic diuretics side effects. Pulmonary edema, dehydration and contraindicated in CHF, and anuria (renal failure). Potassium sparing diuretics: Spironolactone, eplerenone, amiloride, and triamterene Potassium sparing diuretics chemistry Competitive inhibitor of aldosterone

Blocks Na uptake at the luminal membrane and blocks excretion of K+ Triamterene and Amiloride

Spironolactone, Eplerenone Aldosterone hormones increase K+ excretion in collecting duct and reabsorb Na+ and Cl· Spiranolactone binds at the receptors at the late distal tubule and collecting tubules thus prevents the reabsorption of Na+ and Cl- ions. Inhibits potassium secretion thus increase K+ levels. · Pteridine or pyrazine derivatives or steroid analogue antagonist of aldosterone. · Triamterene, amiloride contain pteridine or pyrazine derivatives. · Spironolactone: Contain sterol structure + lactones. H3C H3C O

CH2 CO CH2 O

NH2 N

SCCH3

H2N

N

N N

NH2

O spironolactone(Aldactone)

triamterene(Dyrenium)

Spironolactone metabolized to main active metabolite canrenone by first pass metabolism in liver. This metabolite has hepatocarcinogenic effect. Canrenone is responsible for 80% of its activity. Metabolism in liver. Spironolactone ------> Canrenone-------> canrenoic acid anion

Copyright © 2000-2018 TIPS Inc. Unauthorized reproduction of this manual is strictly prohibited and it is illegal to 26-5 reproduce without permission. This manual is being used during review sessions conducted by PharmacyPrep.

www.Pharmacyprep.com

Medicinal Chemistry

Eplerenone is a new aldosterone antagonist. Metabolized by CYP450. Eplerenone has no hormonal side effects like gynecomastia. Eplerenone is derived from spironolactone by substitution of the 17-alpha thioacetyl group of spironolactone with a carboxymethoxy group. Potassium sparing diuretics pharmacology · Act in the early collecting duct to inhibit the electrogenic reabsorption of Na+ by blocking the Na channels and hence the exchange of sodium for potassium. · After administration: ↑ Na+, Cl- elimination, ↓K+, Ca2+ (amiloride). · Increase Na, H 2 O, HCO 3 excretion (decrease levels in body), decrease K+, H+ excretion. · Alkaline urinary pH, and increase excretion of HCO 3 . · Potassium sparing diuretics gives the intracellular alkalosis. Potassium sparing diuretics therapeutic use · Amiloride, spironolactone and triamterene act as competitive antagonists of aldosterone in the kidney. Because they are weak diuretics when given alone, they are often used in combination with hydrochlorothiazide. · Spironolactone is the drug of choice for the treatment of ascites. · Spironolactone is antiandrogenic and has been used to treat hirsutisms in doses of 200 mg/day. · Amiloride used in nephrogenic diabetic insipidus · Spironolactone is the drug of choice for the treatment of ascites. Potassium sparing diuretics side effects · A side effect may be hyperkalemia, an increase in potassium levels, so that potassium supplements are usually not taken with these drugs. If they are needed, the dose of potassium is frequently administered three times a week instead of daily. · Side effects of spironolactone include endocrine à Gynecomastia, menstrual irregularities, Electrolytes à Hyperkalemia, irregularities, CNS à mental confusion Vasodilators Vasodilators cause the smooth muscle in blood vessels to relax. Relaxed or dilated blood vessels allow more blood to flow through, causing a reduction in blood pressure by decreasing peripheral resistance. As vasodilators work, the blood vessels become dilated causing a drop in pressure because there is less blood volume to fill the vessel. The body can compensate for this by retaining enough fluid to fill the blood vessel sufficiently to raise the blood pressure again. Hydralazine and minoxidil are direct vasodilators not usually used a sole therapy for high blood pressure, as their effect is usually short-lived when administered alone. Minoxidil is available in tablets and also as a topical lotion (Rogaine) for treatment of male-pattern baldness). NH2 HN NH NH

hydrazine group (Basic)

Hydralazine

Copyright © 2000-2018 TIPS Inc. Unauthorized reproduction of this manual is strictly prohibited and it is illegal to 26-6 reproduce without permission. This manual is being used during review sessions conducted by PharmacyPrep.

www.Pharmacyprep.com

Medicinal Chemistry

ACE Inhibitors Captopril, benazepril, cilazapril, enalapril, fosinopril, lisinopril, perindopril, quinapril, ramipril, and trandolapril. Question Alerts! 1) Sulfhydryl in captopril is active group, where as other ACEi, carboxylic acid is active group. 2) All ACEi are primarily eliminated by renal, except. Fosinopril (50:50). (dose adjustment not require renal impairment).

ACE Inhibitors chemistry Sulfhydril containing inhibitors. Captopril. Sulfhydryl group leads to shorter duration of action. O HS - H2C

N

CH3

CO2H

Captopril Dicarboxylate containing inhibitors. all ACEi Phosphonate containing inhibitors. Fosinopril Captopril and Lisinopril are acidic drugs while all other are amphoteric. The carboxylic attached to N-ring is common structural feature in all ACEi. The sulfhydryl group of captopril is proved to responsible for excellent activity of captopril, and this also responsible of its two most common side effects skin rashes and taste disturbances (example metallic taste and loss of taste). The sulfhydryl group also present in another drug penicillamine, which attributes to its metallic taste. Free acid group or sulfhydryl or phosphonate groups essential for the activity. O

O-CH2-CH3 O Esterase NH N CH3 HO2C

O OH O HN

N CH3

Bioactivation of Enalapril Captopril and LISinopril are acidic drugs while all other ACE inhibitors are amphoteric. ACE Inhibitors that are NOT prodrug captopril and LISinopril. All ACEi are once daily used, except captopril (2-3 times) daily. Enalapril is prodrug of Enalaprilate. Lisinopril is a lysine analogue of Enalaprilate. Fosinopril is phosphate containing ACEi. What enzyme polymorphism effect ACEi? Angiotensin converting enzyme (ACE) ACEi lowers nephropathy mechanism

Copyright © 2000-2018 TIPS Inc. Unauthorized reproduction of this manual is strictly prohibited and it is illegal to 26-7 reproduce without permission. This manual is being used during review sessions conducted by PharmacyPrep.

www.Pharmacyprep.com Aldosterone Angiotensin II -------------------àKidney

ACEi & ARBs

Medicinal Chemistry Proteinuria Na&H2O retention ¯GFR

¯proteinuria ¯ Na & H 2 O retention - GFR (renal perfusion well maintained) No rebound hypertension

ACE Inhibitors pharmacology · ↓ sympathetic output · Increase vasodilatation of smooth muscles · ↑ levels of bradykinin cause dry cough · By reducing circulating angiotensin II levels ACEi ↓ the secretion of aldosterone, resulting ↓ retention of Na and H 2 O retention leads to decrease in cardiac output (CO). · ↓ in preload and after load · Dilation of venous blood vessels leads to decrease in cardiac preload by ↑ venous capacitance. · Arterial dilator reduces systemic arteriolar resistance and ↓ after load. · Lower blood pressure by reducing peripheral vascular resistance with reflex increasing cardiac output rate. ACE Inhibitors therapeutic use · ACE inhibitors are used to treat uncomplicated hypertension and pre-hypertensive patient. However, if Angiotensin is not the contributing factor for the hypertension, the chances of ACE inhibitors working are diminished. · These drugs represent a major advance in hypertensive treatment and have most displaced digoxin as the drug of choice in congestive heart failure. 1st line treatment for: Heart failure. · ACE inhibitors also tend to protect the kidneys of diabetics from developing renal failure when used in the early stages of diabetic nephropathy. Diabetes nephropathy, post MI. · LVH (Left ventricular heart failure). · Prior CVA/TIA (Cardiovascular Attacks/ Transient Ischemic attacks) & in renal disease. ACE Inhibitors side effects · Profound low pressure (hypotension), taste abnormalities, dry cough (5 to 15%), blood cell abnormalities, and kidney problems such as proteinuria (presence of protein in urine). They are contraindicated in pregnancy. A persistent dry cough may necessitate discontinuing the drug. ACE inhibitors increase the risk of hyperkalemia. Allergic reactions angioedema (rare). Reversible neutropenia and fatigue. Dry cough Due to - bradykinins Hypotension Hyperkalemia Angioedema Proteinuria in renal stenosis - serum creatinine Reversible neutropenia Pregnancy contraindicated Copyright © 2000-2018 TIPS Inc. Unauthorized reproduction of this manual is strictly prohibited and it is illegal to 26-8 reproduce without permission. This manual is being used during review sessions conducted by PharmacyPrep.

www.Pharmacyprep.com

Medicinal Chemistry

Angiotensin Receptor Blockers (ARBs) Losartan, telmisartan, valsartan, irbesartan, and candesartan. ARBs are analogs of imidazole ring attached with 5–CH 2 -OH group (imidazole-5-acetic acid). N

Question Alerts! 1) Structure activity of Angiotensin receptor blockers imidazole is essential. 2) Telmisartan and Candesartan cilexitil have benzimidazole. This enhances hydrophobic binding and increase potency (peak/trough ratio). 3) Candesartan is prodrug. 4) all ARBs have tetrazole ring 5) All ARBs have renal and fecal elimination (avoid telmisartan (fecal 97%) in hepatic dysfunction)

Imidazole ring (essential)

Cl

N OH

Tetrazole ring N N N N H Losartan

Angiotensin receptor blockers, block the effects of angiotensin II, a naturally occurring substance that causes blood vessels to narrow (constrict). When these drugs are administered, blood vessels dilate, thereby lowering blood pressure and decreasing the workload of the heart. These drugs appear to have the same benefits as ACEi, without or less producing the common side effect of a dry cough. Pharmacological actions · Angiotensin binds to its own receptors are found on vascular muscle and in the adrenals. · Stimulation leads to vasoconstriction and release of aldosterone in the adrenal gland. · It blocks aldosterone secretion. Side effects Less dry cough (cough associated with ACEi does appear with these drugs), Bradykinin causes vasodilation of arterioles and venules results ↓ TPR, less dry cough. Dizziness, hypotension/syncope, renal dysfunction (reversible renal failure), hyperkalemia and angioedema. Contraindications. Pregnancy (renal fetal toxicity), and bilateral renal artery stenosis (stenosis; abnormal narrowing of passage or opening, such blood vessels or heart valve. Pharmacokinetics. Losartan may increase the effects of potassium supplements, potassium-sparing diuretics, and cyclosporine, leading to raise of potassium in the blood.

Antihyperlipidemic Drugs ANTIHYPERLIPIDEMIC DRUGS STATINS 3- Atorvastatin calcium (LIPITOR) Hydroxy-3Fluvastatin sodium (LESCOL Methylgluteryl Lovastatin (MEVACOR) (HMG)-CoA Pravastatin sodium (PRAVACHOL) Reductase Inhibitors Rosuvastatin ( CRESTOR)

LDL ¯¯¯¯

TG ¯¯

HDL --

Copyright © 2000-2018 TIPS Inc. Unauthorized reproduction of this manual is strictly prohibited and it is illegal to 26-9 reproduce without permission. This manual is being used during review sessions conducted by PharmacyPrep.

www.Pharmacyprep.com

Medicinal Chemistry

Simvastatin (ZOCOR) FIBRATES

Bezafibrate (BEZALIP) Fenofibrate (LIPIDIL Gemfibrozil (LOPID) Clofibrate

¯

¯¯¯¯

---

NIACIN RESINS (Bile Acid Sequestrants)

Niacin Cholestyramine resin (QUESTRAN) Colestipol HCl (COLES)

¯¯ ¯

¯¯¯ MINIMAL EFFECT

---MINIMAL EFFECT

Antihyperlipidemic drug chemistry. Categorized into non-absorbable agents and absorbable agents.

Nonabsorbable agents: Resins (Polystyrene resins). Anion-exchange resins bind with the enzymes in intestine and prevent the synthesis of cholesterol. It decreases LDL 15 to 30%, increase HDL 3 to 10%, however no change or increase in TG (disadvantage). Pharmacological actions GIT

Blood

Adipose tissue

Cholestyramine chloride · ·

A basic anion-exchange resin made of trimethylbenzylammonium groups in a large copolymer of styrene and divinylbenzene. Act on small intestine. · Available as powder only · Mixed with juice, water or non-carbonated beverages

Colestipol hydrochloride · A copolymer of diethylpentamine and epichlorodrin · Available as granules or tablets (1 g) · Do not crush, chew or cut and should be taken with plenty of water. · Hygroscopics · Water insoluble · Do not absorb orally · Do not metabolized and excreted in feces All resins do not change triglycerides or may increase TG. Avoid in cholelithiasis or complete biliary obstruction due to impaired secretion of bile acids caused by these conditions. All oral medication should be administered 1 hour before and 4 to 6 hour after resins. Absorbable agents · Niacin.Nicotinic acid (Niacin or vitamin B 3 ) · Fibrates. Clofibrate and Fenofibrate (aryloxyisobutyric acid derivatives) · Probucol (sulfur containing bis-phenol) · Statins. Lovastatin (3-hydroxy-3methylglutaryl-coenzyme A) · Fatty fish oil. Eicosapentaenoic acid (EPA) and Docosahexaenoic acid (DHA)

Copyright © 2000-2018 TIPS Inc. Unauthorized reproduction of this manual is strictly prohibited and it is illegal to 26-10 reproduce without permission. This manual is being used during review sessions conducted by PharmacyPrep.

www.Pharmacyprep.com

Medicinal Chemistry

Nicotinic acid Administered orally, it converted in body to nicotinamide, NAD+ and NADP+. The later two are coenzymes essential for biochemical oxidation, reduction reactions. Participation in tissue respiration oxidation, reduction reactions, which decreases hepatic LDL and VLDL production. At high doses lowers LDL, VLDL and raises HDL. Strongly inhibits lipolysis in adipose tissues. Inhibits tubular secretion of uric acid, causes gout or hyperuricemia.

COOH N Niacin

Adipose tissue Triglycerides

Question Alerts! 1) NAD+, NADP are active products of niacin but Nicotinamide has NO activity. 2) Niacin causes flushing is due to?

In Liver Fatty acids

Triacylglycerol

VLDL

LDL

The most common side effect and often dose limiting is cutaneous vasodilatation that gives flushing and pruritus. Taking ASA 325 mg and take with food can manage the flushing. Hepatic dysfunction (elevate LDH, AST, and ALT) is another serious side effect of niacin at higher doses and avoids alcohol. GI side effects such as nausea, vomiting, and diarrhea can be minimized be taking with meals. Usual dose of niacin if tolerated 500 mg to 4 g/day divided TID po after meal. Nicotinic acid produced from plant. Nicotinamide (reduced) produced from animal sources.

Statins Question Alerts! Statins structure activity? 1) essential group for statin action is 3,5 dihydroxycarboxylate. 2) lactone ring in lovastatin and simvastatin have bioactivates to 3,5 dihydroxycarboxylate. 2) Where does cholesterol is synthesised? Liver 3) All statins have similar onset, duration, dosing intervals and plasma protein binding. Differ in structure, kinetics, and potency. 4) peak plasma conc. in 1-4 hrs however take 2 weeks to lower plasma cholesterol. 5) Primary route of elimination is fecal. Have high hepatic metabolism and increase liver enzyme (liver trans aminases) therefore active liver disease contraindicated. 6) Atorvastatin does NOT require dose adjustment in renal disease patients therefore used in renal diseases (precaution diltiazem > nifedipine. Verapamilà avoid using in CHF (cause –ve inotropic effect) and constipation. Dihydropyridine Dihydropyridine are similar to Nitrate in effect. Act on peripherally by decrease afterload (good for vasospastic angina) or decrease total peripheral resistance. Dihydropyridine can cause reflex tachycardia in response to hypotension caused by decreased in total peripheral resistance. Dihydropyridines relax and dilating arteries. The sequence of effect on vascular smooth muscles is high to low nifedipine > diltiazem > verapamil. Calcium channel blockers therapeutic uses Calcium channel blockers are used to treat hypertension and are effective in treating angina as well. All muscles, including the smooth muscle of the blood vessels, require calcium in order to contract. If the calcium-channel blocking agents block the entrance of calcium into the muscle, the muscle will not contract. This will allow the muscle to relax and subsequently reduce the blood pressure. Other therapeutic uses: angina, migraine, and antiarrhythmic. Non-dihydropyridine side effects Flushing, profound low blood pressure, swelling of legs and feet, constipation and stomach upset. If edema (swelling) of the legs and feet occur, a diuretic may be added to the regimen. Dihydropyridine side effects. Blood pressure will fall too low and sometimes causes reflex tachycardia. The other side effects include, flushing, headache, dizziness, orthostatic hypotension, and edema. Anticoagulants Anticoagulants classification Heparin (long chain pentasaccharide)

Catalyzes the inhibition of thrombin by clotting factors

Heparin pentasaccharides are natural products, long chain pentasaccharides are responsible for heparin induced thrombocytopenia. Dalteparin sodium (Fragmin) Enoxaparin sodium (Lovenox) Nadroparin calcium (Fraxiparine) Tinzaparin sodium (innohep)

Low molecular weight Heparin (LMWH) (fractionated heparin)

Selective to factor Xa and IIa

Warfarin sodium

Vitamin K Antagonists

oral

Dabigatran

Direct thrombin (factor II) inhibitor Direct factor Xa inhibitor Indirect factor Xa inhibitor

oral

Rivaroxaban, Apixaban Fondaparinux Anticoagulants chemistry Heparin

oral SC Synthetic polysaccharides has no thrombocytopenia.

Large, highly acidic mucopolysaccharide made up of sulfated D-glucosamine

Copyright © 2000-2018 TIPS Inc. Unauthorized reproduction of this manual is strictly prohibited and it is illegal to 26-15 reproduce without permission. This manual is being used during review sessions conducted by PharmacyPrep.

www.Pharmacyprep.com

LMWH

Heparinoids: danaparoid, lipirudin and ancrod. Danaparoid

Lepirudin Coumarin derivatives

Medicinal Chemistry and D-glucuronic acid molecules Low-molecular weight heparin fragments (1-10 kDA), enoxaparin, dalteparin, tinzaparin, and ardeparin, are being produced through controlled depolymerization of Heparin A low-molecular weight heparinoid that are glycosaminoglycans extracted from porcine mucosa. Danaparoid sodium is a mixture of non-heparin low molecular weight sulfated glycosaminoglycuronans derived from porcine intestinal mucosa. Danaparoid is also indicated for the treatment of patients with an acute episode of Heparin-Induced Thrombocytopenia (HIT), and for prophylaxis in patients with a history of HIT. Mechanism. has a much higher anti-factor Xa/anti-IIa ratio (more than 20:1) A recombinant DNA-derives 65 amino acid polypeptides (nearly identical to hirudin). Warfarin, and dicumarol chemically related to vitamin K, are water-insoluble, weakly acidic 4-hydroxycoumarin lactones. Phenindione are indanedione derivatives.

Warfarin is racemic structure: The S-isomer is 3-5 times more potent than the R-isomer. The S-isomer is a substrate of CYP 2C9 and inducers or inhibitors of this enzyme have the most effect on the INR. Warfarin has a narrow range for efficacy and toxicity and inducers or inhibitors of either isoform can be significant. CYP450 drug interactions with warfarin are the most common cause of an INR increase or decrease. Warfarin is substrate of O CYP2C9........... S isomer OH CYP1A2............R isomer CYP3A4............R isomer O O CYP2C19..........R isomer Coumarin ring

Structural formula of coumadin derivative oral anticoagulants (Warfarin)

CYP2C9 rat poisons. Warfarin ………………………………> metabolite

Coumarins is

Onset of action: Anticoagulation effects 2-3 days. The delay is due to factor V turnover if 5 hours and factor 2 (thrombin) is 2-3 days. Anticoagulant pharmacology · ·

·

LMWH. These agents are not interchangeable with heparin in their actions and use. Because these highly acidic. These administered parenteral as sodium salt. Because poorly absorbed from GI tract. Heparin: Heparin is mucopolysaccharide. Enhances the serum protease antithrombin III results in inactivation of factors 2a, 9a, 10a, 11a, 12a, and 13a. Maximum effect occurs within minutes. Heparin stops the expansion of thrombi by preventing fibrin formation. Antidote of heparin is Protamine sulfate: Works by acid base neutralization. Heparin act In vitro and In vivo. It has faster onset of action Warfarin: Inhibits the hepatic synthesis of vitamin K dependent factors 2, 7, 9 and 10. Warfarin antidote is Vitamin K. This agent act only in vivo (in liver). Onset of action is slower (8 to10h). Warfarin is contraindicated in pregnancy.

Copyright © 2000-2018 TIPS Inc. Unauthorized reproduction of this manual is strictly prohibited and it is illegal to 26-16 reproduce without permission. This manual is being used during review sessions conducted by PharmacyPrep.

www.Pharmacyprep.com

Medicinal Chemistry

Anticoagulant therapeutic use · LMWH: Used in prevention of deep vein thrombosis (DVT) or pulmonary embolism (PE). · Heparin: Major antithrombotic drug for DVT and pulmonary embolism. To prevent postoperative venous thrombosis in-patient undergoing surgery. · Warfarin: used to prevent blood clots, mainly in areas where blood flow is slowest, particularly in the leg and pelvic veins. Anticoagulant side effects · LMWH: Hypersensitivity reaction (chills, fever, urticaria etc), Bleeding, Heparin induce Thrombocytopenia (HIT), Osteoporosis. · Heparin: Hypersensitivity reaction (chills, fever, urticaria etc), Bleeding and heparin induce Thrombocytopenia (HIT). · Warfarin: Bleeding; hair loss; Skin necrosis (rare), blue fingers and toes (uncommon) this also referred as purple toe syndrome Add table of comparison of anticoagulant Cardiac glycosides Digoxin is the most widely used cardiac glycoside increase myocardial contractility ad efficiency, improve systemic circulations, improve renal perfusion and reduce edema Cardiac glycosides Chemistry · Consist of one or more sugars (glycine portion) and a steroidal nucleus (aglycone or genin portion) bonded through an ether (glycosilic) linkage. Have an unsaturated lactose substituent (cyclic ester) on the genin portion. · Digoxin has an additional hydroxyl group at position 12. Duration of action is inversely proportional to the number of hydroxyl groups, which increase polarity. DIGOXIN

OH CH3 H3C OH O O O O O O OH OH OH

O O OH CH3 CH3H H

OH

Question Alerts! Position 12 hydroxyl group is present digoxin

H O O CH3

OH CH3 H3C O O O OH O O O CH3 OH OH

CH3H H

OH

H

DIGITOXIN

Cardiac glycosides Pharmacology ·

Cardiac glycoside at therapeutic doses produce. Positive inotropic effect (+ve inotropic). negative chronotropic effect (-ve chronotropic), and increased vagal tone of the sinoatrial (SA) node (vegomimetic).

Copyright © 2000-2018 TIPS Inc. Unauthorized reproduction of this manual is strictly prohibited and it is illegal to 26-17 reproduce without permission. This manual is being used during review sessions conducted by PharmacyPrep.

www.Pharmacyprep.com

Medicinal Chemistry

Digoxin mechanisms of action Electrolyte ‘ rearrangement ’ (Mechanism)

Increased Vagal tone (Mechanism)

- Ca i - Inotropic Tachyarrhythmias Brady arrhythmias ¯ Heart Rate (Desired) (Side effect) (Side effect) (Desired) · Inhibit the membrane bound Na+/K+ activated ATPase increase intracellular sodium concentration, reduce calcium transport form cell, and facilitate calcium entry via voltage gated membrane channel. Cardiac glycosides Therapeutic use · ·

Cardiac glycoside at therapeutic doses produce: Myocardial contractility and efficiency, improve systemic circulation, improve renal perfusion and reduce edema thereby it indicated in treatment of: Congestive heart failure, atrial fibrillation, atrial flutter, paroxysmal atrial tachycardia

Contraindications: Digoxin is contraindicated in ventricular arrhythmias

Thrombolytic Drugs

Streptokinase (Strepto protein from Group C-B-hemplytic steptococci bacteria

Anistreptase (Aminase) Prodrug of streptokinase

Urokinase (Abbokinase)

t-pa

(tissue type plasminogen activator) Alteplase (activase) Directly degrade Reteplase (Retevase) fibrin and fibrinogen very high affinity to plasminogen, bound to thrombus

Thrombolytic chemistry Alteplase, Recombinant DNA-derived plasminogen activators (t-PA) with 527 and 355 amino reteplase, and acids. Tenecteplase Streptokinase Nonenzymatic 47-kDa. Derived from b-hemolytic streptococci cultures. Anistreplase APSAC is a complex of human lys-plasminogen and streptokinase with an anisoyl group blocking the catalytic site. Urokinase A two chain serine protease obtained form human kidney cell culture. Thrombolytic pharmacology ·

Facilitate conversion of plasminogen to plasmin that subsequently hydrolyzes fibrin to dissolve clots.

Thrombolytic therapeutic use · Acute MI (STEMI), stroke and acute massive pulmonary embolism (PE), and deep vein thrombosis (DVT). Copyright © 2000-2018 TIPS Inc. Unauthorized reproduction of this manual is strictly prohibited and it is illegal to 26-18 reproduce without permission. This manual is being used during review sessions conducted by PharmacyPrep.

www.Pharmacyprep.com

Medicinal Chemistry

Thrombolytic side effects · Hypersensitivity reactions, Internal GI bleeding, retroperitoneal bleeding, superficial bleeding at catheter injection site. nausea and vomiting.

Antiarrhythmic agents Antiarrhythmic drugs are classified as follows: · Class IA: Procainamide, quinidine, disopyramide · Class IB: Phenytoin, lidocaine, mexiletin, tocainide · Class IC: Propafenone, Flecainide, Moricizine · Class II: All the beta-blockers · Class III: Sotalol, Bretylium, Amiodarone, dronedarone · Class IV: All the calcium channel blockers Antiarrhythmic drugs Chemistry Cinchona alkaloids, natural products (Example: quinidine, it is an optical isomer of quinine) Amides type of local anesthetics (Example: procainamide, flecainide, disopyramide) Xylyl derivatives (Lidocaine, xylocaine), Mexilitine. Quaternary ammonium salts (bretylium) Diiodobenzyl oxyethylamines (amiodarone) b-blockers (nadolol, propranolol, esmolol) Non-dihydropyridine type calcium antagonist (diltiazem, verapamil) Hydantoins (phenytoin)

Amiodarone chemical structure Amiodarone Two iodine in structure No sulfonamide group (no sulfa allergy)

Dronedarone No iodine in their structure Sulfonamide group is present (sulfa allergy) NO 4 P side effects.

Antiarrhythmic drugs pharmacology Class Ia: Quinidine (Biquin durules), and Procainamide (Procaine SR), Disopyramide (Norpace CR): Slow the phase 0 (slow entry of Sodium ion), Prolonged re-polarization, Prolonged effective refractory period Class Ib: Lidocaine, Tocainamide, and Mexiletine: Minimal effect on Phase 0 Slow phase –III repolarization (decrease K pump out). Copyright © 2000-2018 TIPS Inc. Unauthorized reproduction of this manual is strictly prohibited and it is illegal to 26-19 reproduce without permission. This manual is being used during review sessions conducted by PharmacyPrep.

www.Pharmacyprep.com

Medicinal Chemistry

Class Ic: Encainide (Enkaid), Propafenone (Rhythmol), and Flecainide (Tambocor); Very effective on slowing phase 0 depolarization, Little effect on repolarization. Class II: Beta blockers: Propranolol, Atenolol, and Timolol: Competitively block catecholamine induced stimulation of Beta receptor thereby suppressing phase IV depolarization Class III K+ channel blockers: Amiodarone, Bretylium, Sotalol: Prolong Phase III repolarization (Prolong QT interval)àTorose de pointes Class IV Ca2+ channel blockers: Verapamil, Diltiazem, And Nifedipine: Shortens action potential Digoxin: affects vagotonic response (vegomimetic) thereby increasing AV nodal refractoriness. It is contraindicated in ventricular fibrillation. Ventricular tachycardia may result from digitalis toxicities Antiarrhythmic drugs therapeutic use Class Ia: Quinidine (Biquin durules), Procainamide (Procaine SR), Disopyramide (Norpace CR): indicated to treat SVT, VT. Class Ib: Lidocaine, Tocainamide, and Mexiletine: indicated in the treatment of VT, VA Class Ic: Encainide (Enkaid), Propafenone (Rhythmol), and Flecainide (Tambocor); Indicated in the treatment of VA Class II: Beta blockers: Propranolol, Atenolol, and Timolol: Indicated to treat AT, SVT, VT, VA Class III K+ channel blockers: Amiodarone, Bretylium, Sotalol: Prolong Phase III repolarization (Prolonged QT interval can cause dangerous ventricular tachycardia )àTorose de pointes. Class IV Ca2+ channel blockers: Verapamil, Diltiazem, and Nifedipine: Indicated in the treatment of SVA, VA Digoxin: Effects vagotonic response (vegomimetic) thereby increases AV nodal refractoriness. It is contraindicated in ventricular fibrillation. Ventricular tachycardia may result from digitalis toxicities. Antiarrhythmic drugs side effects Class Ia. Quinidine, procainamide, disopyramide; Torsades de pointes. Amiodarone: blue skin, photosensitivity, photophobia, Respiratory. Pulmonary toxicity which including interstitial pneumonitis; Respiratory muscle impairment, pigmentation. GI: nausea, anorexia, constipation, hepatitis and cirrhosis, hypothyroidism, hyperthyroidism. Blue skin color, corneal deposits, hepatic toxicity, optic neuritis, and erectile dysfunction. Structure of amiodarone, dronedarone and levothyroxine. Amiodarone inhibits CYP3A4 and 2C9.

Antiplatelet drugs Antiplatelet Drugs

ASA

Dipyridamole

Ticlopidine & Clopidogrel

Glycoprotein inhibitors

Eptifibatide Tirrofiban

Prostaglandin analogs

Epoprostenol

Copyright © 2000-2018 TIPS Inc. Unauthorized reproduction of this manual is strictly prohibited and it is illegal to 26-20 reproduce without permission. This manual is being used during review sessions conducted by PharmacyPrep.

www.Pharmacyprep.com

Medicinal Chemistry

Change chart ASA, clopidogrel (Plavix), ticlopidine (Ticlid), dipyridamole dipiperdino-dinitro pyrimidine and theonopyridine. ADP inhibitors. Ticlopidine and clopidogrel, prasugrel Fab fragments of human monoclonal antibody to the glycoproteine (GPIIb/IIIa) receptors (Abciximab, tirofiban, eptifibatide). Antiplatelet drugs pharmacology ASA Inhibits platelet cyclooxygenase production of thromboxane A 2 , thus preventing platelet aggregation or clumping. ASA act as antiplatelet action at dose 60 to 80 mg Ticlopidine & Interfere ADP-induced membrane-mediated platelet-fibrinogen Clopidogrel binding. Fab fragments Monoclonal antibodies act against the glycoprotein IIb/IIIa-receptor thus prevent platelet-platelet aggregation. GPIIb/IIIa-receptor Reversible antagonist of fibrinogen, von Willebrand’s factor antagonist Antiplatelet drugs therapeutic use · Platelets of the elements of blood cells, tend to clump together. The antiplatelet drugs interfere with the coagulation by inhibiting platelet aggregation. Heart attacks and strokes occur when a blood clot that forms in a narrowed portion of an artery blood flow and cuts off the supply of oxygen and nutrients to the tissue that lies beyond the site of the clot. Antiplatelet drugs side effects · ASA: Bleeding (epistaxis to major GI bleeds), Serious GI bleeding less common with lower doses (80325mg/day) · Clopidogrel. Bleeding, diarrhea, rash, and thrombocytopenia. · Ticlopidine: Neutropenia, bleeding, diarrhea, rash, thrombocytopenia (2.5%) that is generally reversible with drug discontinuation. Monitor neutrophils every 2 wk for 1st 3 months

Tips 1. 4.

Find answers from the table: Nitroglycerin Fluvastatin and Lovastatin

2. 5.

3. 6.

APR CYP3A4

8. 11

ALS Contain sulfhydryl group decrease LDL intracellular alkalosis

7. 10

increase HDL act on distal collecting tubules

9. 12

decrease TG Sulfonamide and halogen at position 6 & 7

13 16

angina Nitrates

14

MI

15

CHF

Copyright © 2000-2018 TIPS Inc. Unauthorized reproduction of this manual is strictly prohibited and it is illegal to 26-21 reproduce without permission. This manual is being used during review sessions conducted by PharmacyPrep.

www.Pharmacyprep.com

· · · · · · · · · · · · · · · · · · ·

Medicinal Chemistry

Spironolactone (act on collecting tubules, intracellular alkalosis) Hydrochlorothiazide (thiazide), essential functional group (sulphonamide and halogen at position 6 & 7). Dihydropyridine are used for the treatment of? (angina, MI, CHF) Essential functional group that are present in thiazide diuretics? (sulphonamide at position 7) What diuretics cause intracellular alkalosis? (Potassium sparing) Nitric oxide (NO) is a neurotransmitter that acts as? (vasodilator) Drugs that cause venous pooling? (Nitroglycerin) Nitroglycerin is classified as? (Nitrates) Fibrates? (increase HDL, decrease LDL, decrease TG) Statins that should be avoided with grapefruit juice (ALS) Statins that should be taken with meals (FL) Statins that can be taken anytime of the day (APR) Dihydropyridine oxidized to pyridine by enzyme? (CYP3A4) Drug that contain sulfhydryl group (Captopril) Treatment of systolic+/- diastolic BP is used? ALL antihypertensive. Treatment isolated systolic BP (180/80) is used? CCB Which antiarrhythmic drug cause pulmonary toxicity? (Amiodarone). Amiodarone (4Ps. phototoxicity, pigmentation, pneumonitis, ↓peripheral conversion of T4 to T 3 ) Which antimalarial drug causes retinopathy side effect? (hydrochloroquine)

Write the examples of drug that affects systolic and diastolic blood pressure. · Norepinephrine à ↑ SBP & ↑ DBP · Epinephrine à ↑ SBP & ↓ DBP · Dopamine à ↑ SBP & ↑ DBP · Dobutamine, isoproterenol à ↑SBP & ↓ DBP · Reserpine à ↓SBP & ↑ DBP · Phenylephrineà↑ SBP & slightly ↑ DBP

Copyright © 2000-2018 TIPS Inc. Unauthorized reproduction of this manual is strictly prohibited and it is illegal to 26-22 reproduce without permission. This manual is being used during review sessions conducted by PharmacyPrep.

www.pharmacyprep.com

Medicinal Chemistry of CNS Drugs

27 Medicinal Chemistry and Pharmacology of Psychiatric and Neurological Disorder Agent Questions Alerts! Common questions in pharmacy exam are to ask! · Phenothiazine structures activity relation. Benzazepine (clozapine, olanzapine, quetiapine) structure activity. · Pharmacological actions of SSRIs and dual acting antidepressants and serotonin syndrome, Structure activity of TCAs. · Structure activity antipsychotic drugs. · Structure of activity antiepileptics carbamazepine, phenytoin, gabapentin · Structure activity and classification of benzodiazepine and half-lives. · Conversion to levodopa to dopamine. Selegiline and rasegiline, tolcapone · Structure activity of CNS stimulants (pseudoephedrine formation to meth amphetamine.

Classification of Antidepressants Monoamine Oxidase (MAO) Inhibitors Irreversible MAOI Phenelzine Tranylcypromine Reversible MAOi (RIMA) Moclobemide 5HT, NE Tertiary amine type (5HT > NE) Amitriptyline Imipramine Doxepin Secondary amine type (5HT = NE) Desipramine Nortriptyline

Amine reuptake inhibitors drugs

Non-selective 5HT, NE, D

a2 receptor blockers Mirtazapine 5HT only Selective serotonin Reuptake inhibitors (SSRI’s) Fluoxetine, Fluvoxamine Paroxetine, Sertraline Citalopram, Escitalopram

Dual action SNRI Venlafaxine Duloxetine Desvenlafaxine NDRI Bupropion

Copyright © 2000-2018 TIPS Inc. Unauthorized reproduction of this manual is strictly prohibited and it is illegal to 27-1 reproduce without permission. This manual is being used during review sessions conducted by PharmacyPrep.

www.pharmacyprep.com

Medicinal Chemistry of CNS Drugs

Abbreviations. 5HT = 5hydroxy tryptamine (Serotonin), NE = Norepinephrine; SNRI = Serotonin Norepinephrine Reuptake Inhibitors, NDRI = Norepinephrine Dopamine Reuptake Inhibitor; RIMAs = reversible inhibitors of monoamine oxidase. TCA = Tricyclic Antidepressants Classes of antidepressant drugs include: tricyclic compounds (TCAs), monoamine oxidase inhibitors (MAOIs), selective serotonin re-uptake inhibitors (SSRIs) and reversible inhibitors of monoamine oxidase (RIMAs). Monoamine oxidase inhibitors (MAOis) chemistry Irreversible MAO inhibitors: Phenelzine, Tranylcypromine Selegiline and Rasagiline (MAO-B selective) used for the treatment of Parkinson's disease. Reversible MAO inhibitor type A (RIMA): Moclobemide Monoamine oxidase inhibitors (MAOis) pharmacology These drugs inhibit the destruction of these brain chemicals, which leads to increased concentrations of these neurotransmitters, which may account for their antidepressant activity. Monoamine oxidase inhibitors (MAOIs) therapeutics The monoamine oxidase inhibitors (MAOis) were the first class of compounds used to treat depression. They are also used as anti-anxiety and narcolepsy. Selegiline is used for the treatment of Parkinson’s disease. Monoamine oxidase inhibitors (MAOIs) side effects ·

· ·

Patients, taking these drugs, must monitor their diet for tyramine, an amino acid present in many foods that are fermented, aged or smoked e.g. cheese. There are a number of foods containing tyramine and such` drugs must be avoided. Monoamine oxidase is an enzyme, which inactivates the neurotransmitters, epinephrine, nor epinephrine and dopamine. CVS affects à Orthostatic Hypotension, Tachycardia, Arrhythmias, Stroke. CNS effects à Sleep disturbances, CNS stimulations. Weight Gain, Sexual Dysfunction Anticholinergic àless than TCAs.

Tricyclic antidepressants (TCA) chemistry Secondary amine type: Dibenzocycloheptadine (Example Nortriptyline) and dibenzapines (Example Desipramine). Tertiary amine type. Dibenzocycloheptadine (Example Amitriptyline and Dibenzapines (Example Imipramine). Tertiary amine Amitriptyline Imipramine Doxepin High anticholinergic effect NE/5HT

Secondary Nortriptyline Desipramine Less anticholinergic More NE

Copyright © 2000-2018 TIPS Inc. Unauthorized reproduction of this manual is strictly prohibited and it is illegal to 27-2 reproduce without permission. This manual is being used during review sessions conducted by PharmacyPrep.

www.pharmacyprep.com

Medicinal Chemistry of CNS Drugs Dibenzocyclohepatadiene Derivatives

Secondary Amine type of TCA

Tertiary Amine type

CHCH2CH2NHCH3

CHCH2CH2N(CH3)2

Nortriptyline

Amitriptyline

Dibenzapine Derivatives

N

N

CH2CH2CH2N(CH3)2

CH2CH2CH2NHCH3

Imipramine (tofranil)

Desipramine (Norpramin)

Question Alerts? 1) If patient is allergic to amitriptyline can cause cross allergy with? CBZ 2) TCA structures are similar to phenothiazines (antipsychotics) 3) TCA half lives are > 15 hrs. Taken once daily 4) Tertiary amine TCA have more Anticholinergic SEs. Tricyclic antidepressants (TCA) pharmacology These drugs inhibit reuptake of NE and 5HT at receptor site, which leads to increased concentrations of these neurotransmitters, which may account for their antidepressant activity. Tricyclic antidepressants (TCA) therapeutic use These drugs are used for the treatment of major depression. Clomipramine is the drug of choice for anxiety disorder (OCD). Amitriptyline also used in migraine prophylaxis and treatment of neuropathic pain, fibromyalgia, and post herpetic neuralgia, insomnia. Imipramine was used to treat enuresis in children (Bedwetting). Tricyclic antidepressants (TCA) side effects · CVD effects: Orthostatic Hypotension, Tachycardia (AV blockade); CNS: Confusion/Delirium. Weight Gain, Sexual Dysfunction, seizure, and Bone marrow depression. · Anticholinergic side effects (Constipation, Blurred vision, Urinary retention, Dry mouth, MYDRIOSIS). Selective Serotonin Reuptake Inhibitors Chemistry (Aryloxypropyl amine is essential for serotonin selectivity). The amine group shows maximum potency when in secondary form i.e tertiary amine, reduce potency for 5HT transporter.

Copyright © 2000-2018 TIPS Inc. Unauthorized reproduction of this manual is strictly prohibited and it is illegal to 27-3 reproduce without permission. This manual is being used during review sessions conducted by PharmacyPrep.

www.pharmacyprep.com

Medicinal Chemistry of CNS Drugs

F3C

O

NHCH3

O

Cl

N N

Fluoxetine (Prozac)

N

CH2CH2CH2 N

N

N

Trazodone (Desyrel)

Fluoxetine S-isomer is 100 times more potent than R-isomer Question Alerts! Fluoxetine metabolised to norfluoxetine, t1/2 200h. It has 5 weeks wash out period and may not require dose tapering. Fluoxetine à N-demethyl metabolite (norfloxetine), has weaker effect but is responsible for long half-life.

Selective Serotonin Reuptake Inhibitors pharmacology The SSRIs are selective in blocking the reabsorption of serotonin by nerve cells by acting at the 5-HT receptors, and therefore, increasing the amount of serotonin available in the brain. Selective Serotonin Reuptake Inhibitors therapeutic use This class of antidepressants has fewer side effects than MAOI’s or TCA’s. They are also used for bulimia, obsessive-compulsive disorder and panic attacks. Selective Serotonin Reuptake Inhibitors side effects GI. Nausea (most common), headache, insomnia, nervousness, and fatigue. Sexual dysfunction (orgasmic delay). SSRI SIDE EFFECTS TCA SIDE EFFECTS NAUSEA, DIARRHEA, DRY MOUTH, HYPOTENSION, ANTICHOLINERGIC, CONSTIPATION, SUICIDAL IDEATION, SEXUAL DYSFUNCTION, SEDATION. INSOMNIA, SEXUAL DYSFUNCTION. Cardiovascular (AV node blockade). Serotonin syndrome? When combined with MAOi plus SSRI or MAOi plus TCA.

Copyright © 2000-2018 TIPS Inc. Unauthorized reproduction of this manual is strictly prohibited and it is illegal to 27-4 reproduce without permission. This manual is being used during review sessions conducted by PharmacyPrep.

www.pharmacyprep.com

Medicinal Chemistry of CNS Drugs

Antipsychotic drugs Antipsychotics are used to treat schizophrenic individual is out of touch with reality, hallucinates, hears voices and exhibits bizarre behaviour. These symptoms are only one aspect of schizophrenia. Other symptoms are social withdrawal, and an inability to communicate or to concentrate. Typical or 1st generation Phenothiazine Thioxanthenes Butyrophenones Diphenylbutyrylpiperidine Dibenzoxazepine Dihydroindolone Chlorpromazine Chlorprothixene Haloperidol Pimozide Loxapine Molindone Fluphenazine Thiothixene Droperidol Trifluoperazine Thioridazine Phenothiazines: Chlorpromazine, thioridazine, and prochlorperazine, Must contain nitrogen containing side chain substituents on nitrogen for antipsychotic activity drug. The ring and nitrogen must be separated by three-carbon chain such as chlorpromazine (Thorazine). Phenothiazines. Such as thioridazine in which the ring and side chain nitrogen are separated by a two-carbon chain has only antihistaminic or sedative activity. Difference of one CH 2 - in side chain referred as homolog. Side chain containing piperazine derivatives have the greater potency and highest pharmacological activity. Substituents on X increase anticholinergic activity. Thioridazine piperidine ring in side chain low EPS risk but central muscarinic and QT prolongation.

S N

X

Side chain

R R = CH2CH2CH2N(CH3)2

Phenothiazine skeleton

Butyrophenones: Haloperidol Butyrophenones are chemically NOT related to phenothiazines but have similar activity. Haloperidol produces a high incidence of extra pyramidal side effects and less sedative, anticholinergic effect than chlorpromazine and produce less autonomic effect like mild hypotension at high doses. Thioxanthene’s: Thiothixene, chlorprothixene: Lack of ring nitrogen and side chain is attached through double bond.

Copyright © 2000-2018 TIPS Inc. Unauthorized reproduction of this manual is strictly prohibited and it is illegal to 27-5 reproduce without permission. This manual is being used during review sessions conducted by PharmacyPrep.

www.pharmacyprep.com

Medicinal Chemistry of CNS Drugs

Atypical or 2nd generation: Clozapine, olanzapine, risperidone, paliperidone, quetiapine, ziprasidone, laurasidone and aripiprazole. Benzazepine derivatives 2nd generation antipsychotics such as benzazepine derivative act on less on D 2 , and high on 5HT 2A. Where as in typical antipsychotic haloperidol binds equally high affinity to D 2 , serotonin and putative alpha 2 receptors. Dibenzazepine: Clozapine and quetiapine Clozapine acts less potent D 2 , more potent 5HT 2A, (D 2 /5HT 2 ratio is very low) H 1 , a1&2 , low on M. Olanzapine acts potent D 2 and 5HT 2A . Quetiapine acts less potent D 2 , Most effectively binds 5HT 2A , H 1 , a 1&2 , low on M (D 2 /5HT 2 ratio is low). Risperidone blocks 5HT 2 greater than D 2 receptors. , (D 2 /5HT 2 ratio is very low) Paliperidone: Active metabolites of risperidone. This is not metabolized in liver thus have low drug interactions. Available as PO daily dose (taken in morning due to insomnia), and prolong released injectable suspension. Ziprasidone: Agonist of 5HT 1A and moderate inhibition of synaptic reuptake of 5HT and NE. (D2/5HT1a ratio is low). Thus ziprasidone has potential efficacy in –ve symptoms and depression. Take with high calorie food. Aripiprazole: partial agonist at 5HT 1a and D 2 receptors. (D2/5HT1a ratio is medium) Thus has potential efficacy in –ve symptoms and depression. Available oral single daily dose. Taken with or without food. · ·

D 2 inhibitory effect effective produce more pharmacological benefit to treat positive symptoms. 5HT 2A inhibitory effect effective in producing more pharmacological benefit to treat negative symptoms.

Clozapine Olanzapine Risperidone Quetiapine

D2 5HT 2 least high

least high

Paliperidone Ziprasidone + Aripiprazole +

M1 +/(less)

H1 high

+/(less)

high

Alpha 1 +

Alpha 2 +

5HT 1A

QT

Weight glucose +++ +++ +++ + +

yes yes

+ yes +/+/-

+++ + ++ + +/+/-

Antipsychotic drugs pharmacology · ·

1st generation antipsychotic mainly binds to D 2 thereby block dopamine receptors in cortical and limbic areas of brain. Blockade in dopamine at basal ganglia leads to side effects such as Parkinson’s (Extra pyramidal side effects). 2nd generation antipsychotics bind and inhibit serotonin receptors and dopamine receptors. Some examples clozapine 5HT 2 , D 1 and D 2 , D 4 , muscarinic, and alpha-adrenergic receptors. Risperidone blocks 5HT 2 greater than D 2 receptors. Quetiapine, Least effect on dopamine.

Antipsychotic drugs therapeutic use The antipsychotic agents are used to treat schizophrenia and other psychoses (major mental disorders).

Copyright © 2000-2018 TIPS Inc. Unauthorized reproduction of this manual is strictly prohibited and it is illegal to 27-6 reproduce without permission. This manual is being used during review sessions conducted by PharmacyPrep.

www.pharmacyprep.com

Medicinal Chemistry of CNS Drugs

1st generation antipsychotics are preferred to treat positive schizophrenia symptoms. 2nd generation antipsychotics preferably used to treat negative schizophrenia symptoms. Antipsychotic drugs side effect: Extra pyramidal Symptoms (EPS), Parkinsonism, sexual dysfunction, anticholinergic side effects, tardive dyskinesia, and sedation. The first-generation antipsychotics have high extra pyramidal side effects, and second generation have high weight gain side effects. Question Alerts! Tardive dyskinesia (abnormal, involuntary movements of the eye, face, and tongue) is NOT a symptom of Parkinson’s disease. WEIGHT GAIN DYSLIPIDEMIA T2DM EPS OLANZAPINE +++ +++ +++ CLOZAPINE +++ +++ +++ 0 RISPERIDONE ++ ++ ++ ZIPRASIDONE 0 0 0 ARIPIRPRAZOLE 0 0 0 Drug interactions: Smoking inducer CYP 1A2. Smoking increase clearance (40%) of haloperidol and decrease serum concentration (70%). Olanzapine and clozapine metabolism is increase by smoking by inducing CYP1A2. CNS stimulants: Methylphenidate, amphetamines Methylation: Amphetamine --> Methyl amphetamine Dehydroxylation: Ephedrine or pseudoephedrine --> Methyl amphetamine OH NH2

NH2

CH3 Amphetamine

CH3

Dextroamphetamine

H N

OH CH3

CH3 Ephedrine

H N

H CH3

N

CH3 Pseudoephedrine

A C D B (meth amphetamine street name crystals meth, glass, ice)

CH3

CH3 Methamphetamine

E

SAR of CNS stimulants Pseudoephedrine and ephedrine are optical isomers Pseudoephedrine have one C-methyl, one N-methyl, one OH group Amphetamine and dextroamphetamine are optical isomers (enantiomers). Amphetamine and dextroamphetamine have one C-methyl Methamphetamine have one C-methyl and one N-methyl

Benzodiazepines Neurotransmitters that play important role in sleep or hypnotic actions are GABA, histamine, acetylcholine, catecholamine and adenosine.

Copyright © 2000-2018 TIPS Inc. Unauthorized reproduction of this manual is strictly prohibited and it is illegal to 27-7 reproduce without permission. This manual is being used during review sessions conducted by PharmacyPrep.

www.pharmacyprep.com

Medicinal Chemistry of CNS Drugs 2

Short acting

Midazolam Triazolam

Intermediate acting Alprazolam Lorazepam Oxazepam Temazepam

Long acting Diazepam Flurazepam Clonazepam Chlordiazepoxide

9

O

1

N

8

B

7

B

8 6

7

4

N5 1'

1'

6'

6' 5'

D X3 N

A

3

N 4

5

6

10 9

2

A

N

1

2'

C

5' 3'

4'

C

2' 3'

4'

5-Phenyl-1,4-benzodiazepine Annelated benzodiazepine

Annelated benzodiazepine. Example. Alprazolam, and triazolam The ring B in benzodiazepine is "diazepine" Dealkylation · Diazepam and chlordiazepoxide. Metabolized to desmethyl diazepam or desmethyl chlordiazepoxide have long half life (>50 hr). · Flurazepam metabolized to desalkyl flurazepam. Oxidation · Alprazolam and triazolam oxidized to short and intermediate acting. Rapid conjugation · Oxazepam and lorazepam metabolite have no intrinsic activity. · Benzodiazepines such as lorazepam, Oxazepam and temazepam (LOT) have OH group at position 3 easily undergo phase II glucuronidation conjugation, and have short half-life. · Benzodiazepine without OH group at position 3, must undergo phase I hydroxylation reaction and then phase II metabolism thus have long half life (long acting). Benzodiazepine pharmacokinetics Long acting benzodiazepines include Diazepam (longest half-life), flurazepam, clonazepam, chlordiazepoxide. Intermediate acting: Alprazolam, Lorazepam, Oxazepam, temazepam and nitrazepam. Short acting: Triazolam, midazolam (shortest half life). Short acing benzodiazepine have no phase I metabolism, or extra hepatic metabolism. Benzodiazepine pharmacology Benzodiazepine receptors BZ 1 and BZ 2 are found in brain, benzodiazepines bind to BZ 1 and BZ 2 that are parallel to GABA receptors. Benzodiazepine therapeutic use Benzodiazepine are minor tranquilizers are used to treat insomnia, anxiety and seizure. Benzodiazepine are used to help panic attacks and insomnia, benzodiazepines should be administered for short-term use only as they become ineffective and cause memory loss. Continuous R.E.M. sleep deprivation makes people more irritable and less able to concentrate. Benzodiazepine side effects: The benzodiazepines in some patients, especially with long-term use, experience drowsiness; tolerance dependency and withdrawal symptoms can be problematic. Even with short-term use, some patients experience tolerance and dependency. Copyright © 2000-2018 TIPS Inc. Unauthorized reproduction of this manual is strictly prohibited and it is illegal to 27-8 reproduce without permission. This manual is being used during review sessions conducted by PharmacyPrep.

www.pharmacyprep.com

Medicinal Chemistry of CNS Drugs

Barbiturates

Ultra-short action Thiopental

Short acting Amobarbital Secobarbital Pentobarbital

Long action Phenobarbital

Barbiturates chemistry · 5, 5-disubstituted derivatives of barbituric acid, a saturated diketopyramidine. · Two side chains in position 5 are essential for sedative hypnotic activity and long acting has a phenyl and an ethyl group in position 5. Barbiturates have structural similarity with hydantoins (phenytoin), however, hydantoins have five membered ring structure. · Primedone is prodrug, and active metabolite is phenobarbital. R1 O

R2 O

HN

O N

NH O

N CH2CH2CH2CH2

N

N N

O

General Barbiturate Structure

Buspirone (Buspar)

Barbiturates pharmacology Barbiturates interfere with Na+ and K+ transport across membrane potentiates GABA A action on Cl- entry. These drugs do not bind to BZ 1 and BZ 2 benzodiazepine receptors. Barbiturates Therapeutic Use They are effective only for a few weeks since they alter the length of time spent in R.E.M. sleep. They should only be used for short-term therapy as sedative or hypnotics. Tuinal (a combination of secobarbital and amobarbital), and secobarbital are reportable controlled drugs. Phenobarbital is used to control seizure disorders, often in combination with primidone and/or phenytoin. Barbiturates Side effects Drowsiness the next morning, tolerance, and dependence. Non benzodiazepine GABA-A agonist · Act at GABA A receptors and have high affinity than benzodiazepines. · Short acting · No rebound effect on withdrawal · Low physical dependence Imidazopyridines · Zopiclone · Zolpidem (ambien) · Pyrazolopyridine · Zeloplon (Sonata)

Copyright © 2000-2018 TIPS Inc. Unauthorized reproduction of this manual is strictly prohibited and it is illegal to 27-9 reproduce without permission. This manual is being used during review sessions conducted by PharmacyPrep.

www.pharmacyprep.com

Medicinal Chemistry of CNS Drugs

Anti-Parkinson’s Drugs Drugs to treat Parkinson’s disease COMT inhibitors

Dopamine precursor Peripheral dopa decarboxylase inhibitor Levodopa

Carbidopa

Dopamine agonist

MAO B inhibitor

Entacapone

Selegiline, rasagiline

D2 Selective Pramipexole Ropinirole

Combination

Ergot alkaloids D1 and D2 Non-selective Bromocriptine Pergolide

Anticholinergic Anti-viral drug Amantadine Benztropine

Levodopa/carbidopa (Sinemet) COMT = Catecholamine O-Methyltransferase Inhibitors: DA= Dopamine

Anti-Parkinson’s drugs chemistry COOH

HO HO

NCH2CH2

OH

NH2 Trihexyphenidyl

Levodopa

S

N CH3

Benztropine

O

N CH2CH3 CH2CH N CH2CH3 CH3

Question Alerts! 1) Levodopa to dopamine is catalyzed by? Dopa decarboxylase (decarboxylation). 2) Levodopa to 3-O-methyldopa is catalyzed by? 3) Example of COMTi ? 4) Selegiline and Rasagiline are? 5) Pramipexole and Ropinirole are? S

Ethopropazine

Direct acting dopamine agonist: (Bromocriptine, Pergolide, Pramipexole, and Ropinirole), Direct acting D 2 agonist. Inhibiting the secretion of the hormone prolactin from pituitary gland. Dopamine precursors (levodopa): High protein content meals interfere with transport of levodopa into the CNS. Levodopa should be taken on an empty stomach, typically 45 min before a meal. Dopamine precursors and decarboxylase inhibitors: Levodopa (L-dopa)/carbidopa is the single most effective anti-Parkinson drug. It is changed into dopamine in the brain. Antiviral agent with antiparkinsonian properties (Amantadine). Indicated in drug induced Parkinson’s Disease because levodopa will reverse the beneficial effect of the drug. Anti-Parkinson’s drugs therapeutic use Anti-Parkinson’s drugs side effects · Levodopa /carbidopa. Nausea, vomiting, orthostatic hypotension, dyskinesia, hallucinations, confusion Long term use of levodopa/carbidopa therapy can produce mydriasis & precipitation of glaucoma, and melanoma. · Amantadine. anticholinergic side effects, hallucinations, edema of feet & ankles. Copyright © 2000-2018 TIPS Inc. Unauthorized reproduction of this manual is strictly prohibited and it is illegal to 27-10 reproduce without permission. This manual is being used during review sessions conducted by PharmacyPrep.

www.pharmacyprep.com · ·

Medicinal Chemistry of CNS Drugs

Anticholinergic drugs, avoid in elderly. It aggravates glaucoma, memory impairment COMT inhibitors. dyskinesias, nausea, sleep disorders, anorexia, diarrhea, hallucinations.

Direct acting dopamine agonist. (Bromocriptine, Pergolide, Pramipexole, Ropinirole) nausea, vomiting, orthostatic hypotension, psychosis, pleural fibrosis (chest x ray before initiating therapy). Non selective dopamine agonist. Bromocriptine and pergolide are ergot alkaloids that have effects on partial dopamine (D 1 ) and full on D 2 receptors and other receptors. Selective (Full) dopamine agonist. Pramipexole, and Ropinirole selective full agonist on D 2 and D 3 receptors. These drugs do not have effect on non-dopaminergic receptors; thereby there are no peripheral effects associated with these drugs. Catechol-O-methyl transferase (COMT), (entacapone, and tolcapone) an enzyme that helps metabolize levodopa, is found in both the brain and in the peripheral nervous system. prevent peripheral metabolism of levodopa, which increases its availability to the brain.

Antiseizure drugs by mechanism of action

Reduces NMDA Receptor activation

*Felbamate

Affect GABA receptors

Reduce Na+ conductance in hyperactive neurons

Reduce Ca2+ current through Tchannels

Phenobarbital Primidone Ethosuximide

Diazepam Vigabatrin Topiramate Tiagabine Gabapentine Pregabalin

Carbamazepine Phenytoin Fosphenytoin Lamotrigine Valproate Valproic acid

Anti-epileptics chemistry Anti seizure drugs (Antiepileptics, anticonvulsants) Question Alerts! · Phenytoin (Dilantin). Hydantoins Phenytoin structurally related · Carbamazepine . Iminostilbenes (hydantoins) to phenobarbitals · Valproic acid Dialkylacetate. divalproex and valproic acid are related chemicals. Divalproex is a mixture of valproic acid and sodium valproate. In the body, they are metabolized to separate compounds, and both exert anticonvulsant effects. Copyright © 2000-2018 TIPS Inc. Unauthorized reproduction of this manual is strictly prohibited and it is illegal to 27-11 reproduce without permission. This manual is being used during review sessions conducted by PharmacyPrep.

www.pharmacyprep.com

Medicinal Chemistry of CNS Drugs

Primidone. Phenobarbital and primidone are chemically related. Primidone is metabolized to phenobarbital and another compound in the body, both of which have anticonvulsants properties. Ethosuximide (Zarontin). Suxinamides

· ·

GABA analogs: Gabapentin, vigabatrin, pregabalin and baclofen (a muscle relaxant). Gabapentin (structurally similar to GABA neurotransmitter), Lamotrigine. Phenyltriazine derivative. NH2 O

Gabapentin

OH

H

NH2 O

Pregabalin OH

carbamazepine is an iminostilbene derivative that is related chemically to the tricyclic antidepressants and is structurally similar to phenytoin. Oxcarbazepine: It is prodrug of carbamazepine. Oxcarbazepine is has minimal drug interaction with OCP. (It is not autoinducer).

CH3

O O

CH2CH3

HN

NH

HN O

O Phenytoin (Dilantin)

Ethosuximide (Zarontin)

H2N

COOH

N CONH2 Carbamazepine (Tegretol)

GABA Gabapentin (Primary amine)

Question Alerts! 1) Phenytoin is metabolized by phydroxylation followed by conjugation similar to phenobarbital. 2) Fosphenytoin is phosphate ester of phenytoin, rapidly hydrolysed to phenytoin in vivo. Fosphenytoin is neutral (pH-7) so it is less irritating. 3) Phenytoin Na must be buffered to an alkaline pH to maintain solubility thus very irritating when injected.

Anti-epileptics pharmacology Anticonvulsants are CNS acting drugs that are used to treat seizures. Epilepsy is a disorder of electrical conduction in the brain and results in loss of consciousness, seizures and convulsions. The electrical activity causes nerve cells to become hyper-excitable and to discharge uncontrollably. Phenytoin decreases the sodium content of nerve in the brain and thereby decreases the hyper excitability of the cells that are involved in initiating seizures. Carbamazepine blocks Na channels thereby reducing abnormal impulses in brain. Anti-epileptics side effects Phenytoin: can cause overgrowth of the gums (gingival hyperplasia) and proper mouth hygiene is necessary for people taking this drug. Encephalopathy, blood dyscrasias, Nystagmus (toxic symptoms), Hirsutism. Birth defects like cleft palate, and Stevens Johnson syndrome. Phenytoin structure has hydantoins. Thus, it gives hydantoins syndrome. This can give congenital defect in children as cleft palate. Carbamazepine: Rash, ↑ liver enzymes, neutropenia, aplastic anemia. Copyright © 2000-2018 TIPS Inc. Unauthorized reproduction of this manual is strictly prohibited and it is illegal to 27-12 reproduce without permission. This manual is being used during review sessions conducted by PharmacyPrep.

www.pharmacyprep.com

Medicinal Chemistry of CNS Drugs

Chronic: drowsiness, vertigo (sensation of dizziness and confused, disoriented state of mind). Aplastic anemia, Stevens Johnson syndrome (skin hypersensitivity reactions). Acute intoxication: Coma, hyperirritability, convulsions and respiratory depression. Topiramate. Cognitive dysfunction, headache, kidney stones, and weight loss. Clobazam. Tolerance to therapeutic effects, insomnia, depression, dizziness, drowsiness, Light headedness, ataxia.

Local anaesthetics Local anaesthetics chemistry. Most local anesthetics are structurally similar to the alkaloid cocaine; however, these structures consist of a hydrophilic amino group through ester or amide functional groups. Ester Type: Cocaine, procaine, chloroprocaine, and benzocain. Short acting due to rapid hydrolysis. Ester type of local anesthetic hydrolysis produces para amino benzoic acid (PABA). Amide Type: Lidocaine, dibucaine, prilocaine, mepivacaine, bupivacaine, and etidocaine. Long acting, and metabolized in liver. Procainamide is long acting amide type local anesthetic than procaine an ester type, because isosteric replacement of ester oxygen with a nitrogen atom. Lidocaine (tertiary amine)

CH3

O N H

CH3

Procaine (aromatic amine)

CH2CH3 N

CH2CH3

O O

CH2CH3

N CH2CH3

H2N

Amide Type

Ester Type

Local anesthetics pharmacology Inhibit sensory nerves that carry stimuli to CNS, block nerve fibre conduction. Blockade is reversible, must continue administration of the drug for effects to continue.

Question Alerts! What type of local anesthetics gives PABA metabolites? Benzocaine, Procaine and tetracaine

Local anesthetics therapeutic use Procainamide à Antiarrhythmic drug (Na+ channel blocker). Lidocaine is used for à antiarrhythmic drug and amide type local anesthetic.

General anesthetics Add classification from pharmacology General anesthetics chemistry

Copyright © 2000-2018 TIPS Inc. Unauthorized reproduction of this manual is strictly prohibited and it is illegal to 27-13 reproduce without permission. This manual is being used during review sessions conducted by PharmacyPrep.

www.pharmacyprep.com

Medicinal Chemistry of CNS Drugs O Cl NH

O

O

HN

N

NH2CH3 ClO

-

S Na+

Thiopental Sodium

Ketamine Hydrochloride

H2C6H5O7- N+ CH2CH2 H

Fentanyl Cytrate

General anesthetics pharmacology Ketamine. Short acting non-barbiturate anesthetic induces a dissociated state in which the patient appears awake but is unconscious and does not feel pain. Propofol. Sedative or hypnotic action, used in induction and maintenance of anesthesia. Fast onset (40 seconds of administration). Thiopental is rapid acting barbiturates. General anaesthetics therapeutic use · · · · ·

Halothane is used in pediatric anaesthesia; it is infrequently used in adults. Isoflurane (Forane), enflurane, sevoflurane and desflurane are frequently used in adults. Enflurane is used as an inhalation agent for adults, but is not widely used for pediatric cases. Isoflurane may be the most widely used inhalation agent. Fentanyl is a narcotic, available as transdermal patch. Good analgesic property

General anaesthetics side effects Ketamine. Causes sedation, immobility, amnesia, and nightmares. Halothane. is associated with malignant hyperthermia.

Tips · · · · · · · · · · · · · · · ·

SSRI onset of action is à Fluoxetine washout period à Depression with sexual dysfunction à To treat depression in insomnia patient à Depression with diabetes à Venlafaxine at higher dose act on à Higher dose of venlafaxine (225 mg/day) have effect on à Patient on antidepressants and shows with dilated pupil, may be due toà TCA onset of action is à A substance found commonly in fermented foods, which can be toxic when MAO inhibitors are, usedà MAO is classified as à Avoid taking cheese with à Milk + MAOI à St. Johns wort is à Selegiline and Rasagiline à Venlafaxine is classified as à

Copyright © 2000-2018 TIPS Inc. Unauthorized reproduction of this manual is strictly prohibited and it is illegal to 27-14 reproduce without permission. This manual is being used during review sessions conducted by PharmacyPrep.

www.pharmacyprep.com · · · · · · · · · · ·

Medicinal Chemistry of CNS Drugs

Drug of choice against bipolar disorders and manic depression à Lithium toxicity symptoms --> CNS--> Thiopental à Lithium blood levels in adults à Serotoninergic syndrome à Fast or quick acting benzodiazepine is à GABA analogs are à Antiseizure drugs that gives Stevenson Johnson syndrome à What antiseizure drugs that is also used as weight loss therapy? Nystagmus à

Select TRUE OR FALSE Statements______________ · · · · · · · · ·

Lithium concentration varies with Na+ ions (T/F) Li+ conc. increases with decrease Na+ (T/F) Li+ conc. decreases with increase in Na+( T/F) ACEI decrease Na+ and increase Li+( T/F) NSAID decrease Na+ renal perfusion is less (T/F) Thiazide deplete Na+( T/F) Fluoxetine (SSRI) increase Li+ toxicity (T/F) Renal dysfunction à Li+ increase( T/F) Dehydrationà Li+ increases (T/F)

Copyright © 2000-2018 TIPS Inc. Unauthorized reproduction of this manual is strictly prohibited and it is illegal to 27-15 reproduce without permission. This manual is being used during review sessions conducted by PharmacyPrep.

www.pharmacyprep.com

Medicinal Chemistry of CNS Drugs

Copyright © 2000-2018 TIPS Inc. Unauthorized reproduction of this manual is strictly prohibited and it is illegal to 27-16 reproduce without permission. This manual is being used during review sessions conducted by PharmacyPrep.

www.Pharmacyprep.com

Medicinal Chemistry

28 Medicinal Chemistry and Pharmacology of Endocrine Drugs Questions Alerts! Common questions in pharmacy exam is to ask! · Insulin structure activities and duration of action and peak effect. · Structures activity of steroidal hormone such as estrogen (phenolic group), progesterone (ethinyl group) and testosterones. · Thyroid hormones thyroxine (T4) and triiodothyronine (T3) · Structure activity of Adrenal corticoids mineral corticoids and glucocorticoids such as hydrocortisone (11- position hydroxyl group). Topical glucocorticoids ranking of potency. · Classification hypoglycemic drugs like 1st & 2nd gen sulfonylureas. The second generation such as glyburide, gliclazide and glimipride. · Insulin and incretin hormones functions and analog liraglutide and Exenatide. · DPP-4 inhibitors such as sitagliptine and saxigliptine.

Antidiabetic drugs pharmacology Sitagliptine. Prolong the half life of an endogenously produced glucagon like peptide-1 (GLP-1). In DM-2, the GLP-1 levels are deficient. GLP-1 are inactivated by DPP-4. Thereby DPP-4 inhibitors partially reduce the inappropriately elevated glucagon post prandially and stimulate glucose dependant insulin secretion. Sulfonylureas (Chlorpropamide, Gliclazide, Glimepiride, Glyburide, Tolbutamide). Increased secretion of insulin by action on β-cells of pancreas. Glyburide may produce a mild diuresis by enhancing renal free water clearance. Question Alerts!

1) Glyburide all action is due to insulin secretion (most hypoglycemic). 2) Glimepiride is quick onset of action and long acting. It has extra pancreatic activity thus has less hypoglycemia SE. Binds to different proteins in the putative sulfonylurea receptors. Exerts hypoglycemic effect with less secretion of insulin. Copyright © 2000-2018 TIPS Inc. Unauthorized reproduction of this manual is strictly prohibited and it is illegal to 28-1 reproduce without permission. This manual is being used during review sessions conducted by PharmacyPrep.

www.Pharmacyprep.com

Medicinal Chemistry

Chlorpropamide has antidiuretic activity possibly due to potentiating of antidiuretic hormone (ADH) in the renal tubules. First generation: Tolbutamide, Chlorpropamide, Second generation: Glyburide, Gliclazide. Glyburide all action is due to insulin secretion (most hypoglycemic). Third generation: Glimepiride Glimepiride is quick onset of action and long acting. It has extra pancreatic activity thus has less hypoglycemia SE. Binds to a different protein in the putative sulfonylurea receptors. Exerts hypoglycaemic effect with less secretion of insulin. Second generation agents have larger R 1 substituent's, because of large substituent's these are more lipids soluble and more potent compared to first generation. Sulfonylureas areas acidic compounds attached to aromatic ring. R1

H O H N N HS O O

R2

General Sulfonylurea srtucture T 1/2

Duration

Glyburide

1.5 – 3 h

Up to 24 h

Gliclazide

2-4 h

Up to 24 h

Glimepiride

5-9 h

Up to 24 h

Highest hypoglycemia, Pancreatic action Pancreatic and Extra pancreatic (delta cell). Faster onset of action and less hypoglycemia

R1 = Halogen, amino, acetyl, methyl, trifluro groups potentiate activity R2 = governs the duration of activity Meglitinides (Nateglinide, Repaglinide): Increased secretion of insulin by action on β-cells of pancreas. Structurally similar to sulfonylureas but substituted by benzoic acid derivatives (meglitinides). Has no sulfa allergy. Greater decrease in post prandial blood glucose. Safer in renal disease patient. Metformin is basic compound. Meglitinides are acidic compounds. Rosiglitazone and pioglitazone are acidic compounds. Repaglinide Half-life is 1 hrs and rapidly eliminated. Thus, used for post prandial blood glucose levels. Alpha-glucosidase Inhibitors (Acarbose) Inhibits alpha glucosidase in intestinal border thus decreasing the absorption of starch and disaccharides. Biguanides (Metformin hydrochloride): Increased peripheral utilization of glucose by decreased gluconeogenesis. Do not cause hypoglycemia in monotherapy.

Copyright © 2000-2018 TIPS Inc. Unauthorized reproduction of this manual is strictly prohibited and it is illegal to 28-2 reproduce without permission. This manual is being used during review sessions conducted by PharmacyPrep.

www.Pharmacyprep.com METFORMIN

Medicinal Chemistry

¯ GLUCONEOGENESIS (LIVER) ¯ GLYCOGENOLYSIS (LIVER) - GLUCOSE UPTAKE

TATOMERISM Thiazolidinedione’s (Pioglitazone and Rosiglitazone): Decreases insulin resistance in the periphery and liver. Improves glycemic control while reducing circulating insulin levels. THIAZOLIDINEDIONE

¯ GLUCONEOGENESIS (LIVER) ¯ GLYCOGENOLYSIS (LIVER) - GLUCOSE UPTAKE

Orlistat. Blocks the action of stomach and pancreatic enzymes (lipases) that digest fats, so fats and other fatsoluble vitamins ADEK are not absorbed in the body but pass through and excreted in the feces. Antidiabetic drug Therapeutic uses Sulfonylureas (Chlorpropamide, Gliclazide, Glimepiride, Glyburide, Tolbutamide) Disulfiram like reaction is caused by which type of antidiabetic drugs, when taken with alcohol. Chlorpropamide has antidiuretic activity possibly due to potentiating of ADH in the renal tubules; there is some evidence that chlorpropamide may actually stimulate ADH secretion. Meglitinides (Nateglinide, Repaglinide): Nateglinide used for type 2 diabetes. It is imperative that there be rigid attention to diet and careful adjustment of dosage. When metformin is combined with a sulfonylurea, instruct the patient on hypoglycemic reactions and their control. If vomiting occurs, withdraw drug temporarily, exclude lactic acidosis, then resume dosage cautiously. Biguanides (Metformin). Metformin can be of value for the treatment of obese diabetic patients. May cause hypoglycaemia when combined with sulfonylureas (glyburide). Metformin: basic compound. Meglitinides; acidic compounds. Rosiglitazone and pioglitazone; acidic compounds Alpha-glucosidase Inhibitors (Acarbose). For type II Diabetes Mellitus in combination with other antidiabetic drugs. In combination oral hypoglycemic therapy, always use agents from different class of oral hypoglycemics. Thiazolidinedione’s (Pioglitazone, Rosiglitazone): These agents depend on the presence of insulin for its mechanism of action. Contraindicated in patients with serious hepatic impairment, acute heart failure. Increasing insulin sensitivity in type 2 diabetes. It improves sensitivity to insulin in muscle and adipose tissue and inhibits hepatic gluconeogenesis. Agonist of the peroxisome-proliferator activated receptor- g (PPAR g). The PPAR g binds to DNA activating transcription of wide variety of metabolic regulators (antihyperglycemic). THIAZOLIDINEDIONES AGONIST OF PPAR g

¯ GLUCONEOGENESIS (LIVER) ¯ GLYCOGENOLYSIS (LIVER) - GLUCOSE UPTAKE

- EDEMA AND CHF

Copyright © 2000-2018 TIPS Inc. Unauthorized reproduction of this manual is strictly prohibited and it is illegal to 28-3 reproduce without permission. This manual is being used during review sessions conducted by PharmacyPrep.

www.Pharmacyprep.com

Medicinal Chemistry

Side effects: Rosiglitazone + Metformin: Indicated for use when diet, exercise, and metformin or rosiglitazone alone do not result in adequate glycemic control. Rosiglitazone+metformin is the combination used in case of insulin resistance. Rosiglitazone + Metformin: If acidosis of any kind develops, Avandamet should be discontinued immediately. The use of rosiglitazone in combination therapy with insulin is not indicated due to its side effects, therefore, Avandamet is not indicated for use in combination with insulin. 2,4-thiazolidinedione is pharmacophore of glitazones. Orlistat (Xenical). Reduces fats stores and produce weight loss. It is an intestinal lipase inhibitor. DPP4 Inhibitors are Quinozolinone or cyanopyrrolidine based structures. Sitagliptine,

saxigliptine

DPP4 GLP-1 ------------------------------à PANCREATIC ISLET CELL

GLP-1 RELEASED FROM GI DPP4, INACTIVATES GLP-1

linagliptine. Safe in renal disease

- INSULIN ¯ GLUCOGON ¯ POST PRANDIAL BLOOD GLUCOSE ¯ FASTING GLUCOSE

Antidiabetic drugs Side effects Sulfonylureas (Chlorpropamide, Gliclazide, Glimepiride, Glyburide, Tolbutamide). GIT disturbance (metallic taste), hypoglycemia, weight gain, hepatic x renal insufficiency. (Jaundice), tachycardia, headache, rash, increased ADH. Structure activity of biguanide (metformin) Biguanides (Metformin hydrochloride) · · · ·

GI: Diarrhea, nausea, vomiting, abdominal bloating, flatulence, and anorexia (weight loss) are the most common reactions to metformin and are approximately 30% more frequent in patients. Occasionally, temporary dose reduction may be helpful. Decreased vitamin B 12 (cyanocobalamine) absorption thus can cause megaloblastic anemia. Unpleasant or metallic taste, which usually resolves spontaneously. A rare and serious side effect is lactic acidosis.

Alpha-glucosidase Inhibitors (Acarbose): GI. Flatulence, diarrhea, abdominal pain, cramps, nausea. Thiazolidinediones (Pioglitazone, Rosiglitazone): Weight gain, fluid retention, congestive heart failure and hemodilution, varying effects on lipids, increase HDL, increase LDL, pioglitazone decrease TG. As a consequence of their improved insulin sensitivity, these patients may be at risk of pregnancy if adequate contraception is not used. Orlistat. Stethorrhea (oily leakage), and abdominal discomfort.

Copyright © 2000-2018 TIPS Inc. Unauthorized reproduction of this manual is strictly prohibited and it is illegal to 28-4 reproduce without permission. This manual is being used during review sessions conducted by PharmacyPrep.

www.Pharmacyprep.com

Medicinal Chemistry

Increase glucose excretion in urine. Act as Sodium-glucose transport protein 2 (SGLT 2 ) inhibitor. Canagliflozin (Invokana), Dapagliflozin and Empagliflozin DPP4 inh Sitagliptine Saxigliptine Linaglptine

GLP-1 analog Exenatide Liraglutide (long acting)

SGLT2 Canagliflozin Empagliflozin Dapagliflozin

Oral only Weight neutral Nasopharyngitis

SC injection only Weight loss Pancreatitis

Renal/hepatic Linagliptine is safe in renal disease

Renal elimination Suppress glucagon release and reduce appetite.

Oral only Weight loss DKA, Hyperkalemia, polyuria, polydipsia. Renal elimination

Ketoacidosis: Mainly Type 1DM Acetyl CoA (produced in liver) --> acetoacetic acid Acetone

Beta-hydroxybutyric acid

Thyroid disorders

Thyroid gland secretes levothyroxine, liothyronine or triiodothyronine and calcitonin Medicinal chemistry of thyroid disorders

Drugs Used In Thyroid Disease

Hypothyroids

Thyroxine (T4) Synthroid Eltroxin

Hyperthyroids

Triidothyronine (T3)

Thioamides: Methimazole Propylthiouracil

Iodide Lugol solution: (KI+I)

131

I

Ipodate

Important Concept! 1) levothyroxine (T4) have 4 iodines and Levothyronine have 3 iodine. 2) Deiodination reaction is catalyzes T4 to T3 by deiodinase enzyme which takes place in peripheral tissues and liver. 3) Lugol's solution is 10% KI + 5% I, is taken as oral drops. It can stain. 4) Drugs that affects thyroid functions? Warfarin, heparin, lithium, amiodarone. 5) Amiodarone structure is similar to thyroxine. Thus amiodarone interferes with peripheral conversion of T4 to T3. Where as dronedarone has NO iodines. Copyright © 2000-2018 TIPS Inc. Unauthorized reproduction of this manual is strictly prohibited and it is illegal to 28-5 reproduce without permission. This manual is being used during review sessions conducted by PharmacyPrep.

www.Pharmacyprep.com

Medicinal Chemistry

Deionidation HO

HO I

I

H NH2 COONa

I

I H NH2 COONa

I O

O I

Sodium Levothyroxine (T4) 3,5, 3’, 5’ thyroxine (T 4 )

I

Sodium Liothyronine (T3) 3,5,3’ Triiodothyronine (T 3 )

5-monodeiodinase enzymes found in extra thyroidal peripheral tissues catalyzes T 4 to T 3 . Thyroid hormone pharmacology: Levothyroxine a major hormone of thyroid gland is liothyronine and thyroid, desiccated. Thyroid hormones therapeutic use. Levothyroxine, liothyronine, thyroid, desiccated is used for the treatment of Goiter and thyroid cancer. Thyroid hormones Side effects. Levothyroxine. Rare side effects such as anxiety, diarrhea, weight loss, sweating, insomnia, and muscle cramps. Thyroid hormones drug interactions

Important Concept! 1) Overdose symptoms of thyroid hormone cause hyperthyroid symptoms 2) Levothyroxin taken empty stomach. 3) Hypothyroidism: Increase serum TSH, decrease FT4 and TT3

Thyroid hormones increase catabolism of vitamin K dependent clotting factors thereby increase the effect of warfarin. Glycemic control may decline with initiation of levothyroxine, potentially requirement of antihyperglycemic agents. Antiepileptic, cholestyramine, and sucralfate may reduce the absorption of levothyroxine because these agents bind with levothyroxine.

Drug and Food interactions. May decrease absorption of levothyroxine by iron salts, cholestyramine, colestipol and sucralfate. Thyroid hormones monitoring Levothyroxine: Periodic tests of thyroid function, monitor serum TSH levels to adjust initial Dosage after 6 to 8 weeks then as required or annually adrenal insufficiency may adrenal insufficiency may have to be increased during pregnancy to maintain TSH in desired range, check TSH each trimester and 4 to 6 wk after any dosage adjustment. Treatment of hyperthyroidism: Antithyroid drugs Chemistry: Thioamides such as methimazole, and propylthiouracil. The iodides, Lugol's solution (KI+I) oral drops and radioactive iodine.

Question Alerts! Mechanism of antithyroid drugs

Copyright © 2000-2018 TIPS Inc. Unauthorized reproduction of this manual is strictly prohibited and it is illegal to 28-6 reproduce without permission. This manual is being used during review sessions conducted by PharmacyPrep.

www.Pharmacyprep.com

Medicinal Chemistry

Antithyroid drugs Pharmacology Antithyroid drugs such as methimazole and propylthiouracil (PTU) act by. · Decreasing thyroid hormone production · Decreasing response to thyroid hormone Antithyroid drugs structure: thiocarbamide (R-(C=S)-NH-R) is essential group for antithyroid activity. Thiocarbamide are similar to thiourea. Antithyroid drug mechanism: TPO Iodine transport into thyroid gland ------> thyroglobulins ----> T 4 , T 3 released into blood TPO; THYROID PEROXIDASE ENZYME. · Inhibit the synthesis of T 3 and T 4 by inhibiting the iodination of tyrosine in the thyroglobulin. (Inhibits thyroid peroxidase or TPO). · Blocks the coupling of the iodo thyroxin. · Inhibits the conversion of T 4 to T 3 thereby thyroid hormone synthesis is decreased. · Obvious effects are very slow since it takes 3 to 4 weeks before the hormone levels show a decrease. · Do not prevent the uptake of IODINE by the gland (do not effect the extensive amount of hormone stored in thyroidal thyroglobulin). THIOAMIDES

INHIBIT TPO INHIBIT COUPLING OF IODOTHYROXIN INHIBIT DEIODINATION (T 4 àT 3 ) DO NOT AFFECT ON IODINE UPTAKE

Propylthiouracil. Manage the overactive thyroid gland Iodides. (Lugol's solution) · · · ·

Lugol solution is KI+I Inhibits the uptake of I 2 by a tyrosine. Inhibit hormone release by inhibiting thyroglobulin degradation. Decrease the size of the gland, decrease blood supply to the enlarged gland therefore it is used in preparation for surgery

Radioactive iodine · · · ·

Effects of iodide on the thyroid gland. Emission of B rays. Gets incorporated into the storage facility Very effective as it concentrates in the thyroid destroys the gland within a few weeks

Antithyroid drugs side effects Antithyroid drugs (Methimazole and propylthiouracil). Reduction in white blood cells leading to the risk of infection, nausea and vomiting, joint pain, headache, rashes and itching, Jaundice, fever. Iodides: Uncommon, reverses when the drug is discontinued, outbreak of acne

Copyright © 2000-2018 TIPS Inc. Unauthorized reproduction of this manual is strictly prohibited and it is illegal to 28-7 reproduce without permission. This manual is being used during review sessions conducted by PharmacyPrep.

www.Pharmacyprep.com

Medicinal Chemistry

Swollen salivary gland, ulceration of the mucous membranes, conjunctivitis, rhinorrhea, and metallic taste, bleeding disorder, anaphylactic reaction. Radioactive iodide: induced genetic damage, leukemia, neoplasia. Cannot be administered to pregnant and nursing mothers cross placental barrier secreted into the breast milk Therapeutic uses · Antithyroid drugs (Methimazole Propylthiouracil) · Methimazole: used to treat hyperthyroidism · Propylthiouracil: used to treat hyperthyroidism · Iodides: Used in the treatment of thyroid storm · Radioactive iodine: I131 isomer of iodine is used in the treatment of thyrotoxicosis Antithyroid drugs Contraindications / Precautions · ·

Propylthiouracil: Prescribed with caution in pregnant women. There is a risk of goiter and hypothyroidism in the newborn infant if too high dose is used. Reduce dose to infant and children Iodides: Should be avoided in pregnancy as it crosses the placental barrier thus causing fetal goiter.

Antithyroid drugs Pharmacokinetics Methimazole: Well-absorbed, Slow excretion, t 1/2 is 6 hours Propylthiouracil · Propylthiouracil – rapid absorption after oral administration · Peak serum levels seen after 1 hour, and t 1/2 is only 2 hours · Extensive first pass metabolism · Excreted by the kidneys as glucuronide (inactive) · Preferred in pregnancy for it does not cross the placental barrier · Strongly protein bound. Secreted in breast milk less than methimazole Radioactive iodine: Sodium I131 given orally and well absorbed from the GIT, t 1/2 is 5 days Medicinal Chemistry Steroid Hormones: The hormones that consist of fused 17-carbon atom ring system are classified as steroids. Examples of hormones that have steroidal structure. Vitamin D, adrenocorticoids or corticosteroids, and gonadal hormones or sex hormones (estrogen, progesterone, and testosterones). Chemically these are derivatives of cyclohepanoperhydrophenanthrene, which also similar to aromatic phenanthrene ring structure. Steroid skeleton consist of three 6 member cyclohexane rings and 1 cyclopentane ring. 17 11 1

D

C

16 A

B

4

6

7

3

Question Alert! All steroid hormones like estrogen, progesterone, testosterone, aldosterones, adrenal corticoids and vitamin D have 3 cyclohexane rings, and 1 cyclopentane ring.

Steroid Skeleton

Adrenocorticoids: Derived from C-21 pregnane steroidal nucleus. Cortisone and hydrocortisone. Adrneocroticosteroids are classified as glucocorticoids and mineral corticoids. Copyright © 2000-2018 TIPS Inc. Unauthorized reproduction of this manual is strictly prohibited and it is illegal to 28-8 reproduce without permission. This manual is being used during review sessions conducted by PharmacyPrep.

www.Pharmacyprep.com · · · · · · · ·

Medicinal Chemistry

Glucocorticoides formed and secreted from the middle layer of adrenal cortex. 17-b-keto side chain (COCH 2 OH), the 4-ene, and the 3-ketone structures. Oxygen atom at C-11 is essential for glucocorticoid activity. Double bond between C-1 and C-2 increases glucocorticoid activity without increasing mineral corticoid activity. Fluorination at C-9 increases both mineral corticoid and glucocorticoid activity. Fluorination at C-6 increases glucocorticoid activity and less on mineral corticoid activity. Hydroxyl group at C-17 and methyl group at C-16 enhances glucocorticoid activity and abolishes mineral corticoid activity. An acetate ester at C-21 or 16a, 17a-isopropylidenedioxy groups enhances topical absorption.

O O

O OH OH

OH OH

HO

Cortisone

O

Hydrocortisone

O

Mineral corticoids are aldosterone · Desoxycorticosterone acetate and fludrocortisone acetate. Formed in the outer layer of the adrenal cortex, a prototypical mineral corticoid (aldosterone) and middle layer gives cortisones. O

O OAc

OH

HO

HO

F

Desoxycorticosterone

O

OAc

Fludrocortisone

O CH3

Glucocorticoids potency ranking Topical glucocorticoids potency ranking · Very high potency. Betamethasone (0.05%) dipropionate, Clobetasol (0.05) · High potency. amcinonide, Triamcinolone 0.5% · Medium potency. Betamethasone benzoate (0.025), Triamcinolone 0.025 to 0.1 · Low potency. dexamethasone, and hydrocortisone

Inhaled and intranasal corticosteroids · Beclomethasone · Budesonide (highly potent nonhalogenated steroid) · Flunisolide · Triamcinolone · Fluticasone · Mometasone (quick onset, more local absorption and lowest systemic absorption, consequently fewest systemic SEs).

Copyright © 2000-2018 TIPS Inc. Unauthorized reproduction of this manual is strictly prohibited and it is illegal to 28-9 reproduce without permission. This manual is being used during review sessions conducted by PharmacyPrep.

www.Pharmacyprep.com

Medicinal Chemistry

Gonadal Hormones O

Question Alert! 1) Estrogen has phenolic ring. 2) Presence of 17 ethinyl group results in orally effective drug by preventing formation of estradiol to estrone.

Phenolic ring

HO Estrogen

OH

OH C CH

O

O Progesterone

Testosterone

Although there is a new chiral group at C 7 . It is considered antiestrogen since there is no functional group at carbon 7. Gonadal Hormones

Testes

Ovary

Estrogen

Estradiol Ethinyl estradiol Mestranol (prodrug)

Estrogen antagonist Tamoxifen Clomiphene Estrogen agonist + antagonist (SERM) Raloxifine

Testosterone

Progesterone

Norethidrone Norgestrel Norethynodrel Norgestimate Desogestrel Medroxy proge Progesterone antagonist Mifepristone (RU-486)

Testosterone Anabolic steroids

Testosterone Antagonist Finasteride Dutasteride Cyproterone acetate Nonsteroidal Flutamide Bicalutamide nilutamide

Estrogens Estrogen chemistry: Ovaries produce 17b estradiol and estrone. These hormones have 18 carbons for four rings. Three 6 membered rings and one 5-member ring. Estrogen exist as estradiol in body in equilibrium with oxidized form of estrone and further biochemically modifies water soluble of estriol that let excrete estrogen. Estrogen A ring is phenolic ring (aromatic), basic nucleus is known as estrane with methyl group designated as C-18 on position C-13 cyclopentano-perhydrophenanthrene. Ethinyl estradiol à 17 alpha estradiol. Naturally occurring estrogen is estradiol Synthetic estrogen is ethinyl estradiol Copyright © 2000-2018 TIPS Inc. Unauthorized reproduction of this manual is strictly prohibited and it is illegal to 28-10 reproduce without permission. This manual is being used during review sessions conducted by PharmacyPrep.

www.Pharmacyprep.com

Medicinal Chemistry

Diethylstilbestrol à Non-steroidal synthetic estrogen (stilbene derivatives) OH

HO

OH

O

Phenolic ring

OH

HO

HO Estradiol

Estrone

Estriol

Estrogen Pharmacology Estrogens are female sex hormones that are used primarily to decrease bone loss and to treat the symptoms of menopause. Estrogen is used to reduce or prevent osteoporosis in susceptible women. Estrogens decrease the frequency and severity of hot flashes as well as the dryness in the vagina that many post-menopausal women experiences. Estrogen therapeutic uses Estrogen is the component of all brands of oral contraceptive pills. Estradiol and conjugated estrogens are available in tablet form; estradiol is available in a patch and injection. Conjugated estrogens are available in a vaginal cream. Estrogen is component of Diane 35 and Alesse oral contraceptive that are used for the treatment of acne. Estrogen side effects. Feminization (Breast tenderness), GI: Nausea, stomach upset, depression, weight gain, CVS: increased blood clotting (Increased risk of thromboembolism diseases), edema, hypertension, stroke, and MI. The most frequent adverse effect is nausea. Prolonged used of unopposed estrogens (estrogen given without progesterone) in postmenstrual women increases the risk of endometrial cancer.

Antiestrogens The two-drug tamoxifen and clomiphene are categorized as full antiestrogens. Raloxifene is a Selective estrogen receptor modulator (SERM) has partial antiestrogen and estrogenic actions. Antiestrogen chemistry. These drugs are non-steroidal antiestrogenic compounds equally effective in oral or injection forms. N

O

N

O

O

N

O Cl Tamoxifen (NOLVADEX)

Clomiphene

OH HO

S Raloxifen (EVISTA)

Tamoxifen and clomiphene are bioisoster. Tamoxifen is an example of geometric isomer. Antiestrogen pharmacology Tamoxifen competes for binding to the estrogen receptors thereby inhibits the action of estrogen. Clomiphene Interferes with negative feedback of estrogen on hypothalamus and pituitary thereby increases the secretion of gonadotropin releasing hormone (GnRH) and causes stimulation of ovulation. Copyright © 2000-2018 TIPS Inc. Unauthorized reproduction of this manual is strictly prohibited and it is illegal to 28-11 reproduce without permission. This manual is being used during review sessions conducted by PharmacyPrep.

www.Pharmacyprep.com

Medicinal Chemistry

Antiestrogen therapeutic use: Tamoxifen is indicated in advanced breast cancer in postmenopausal women. Clomiphene is used to treat infertility associated with anovulatory cycles. Antiestrogen side effects: Tamoxifen: hot flashes, vaginal bleeding, menstrual irregularities, risk of endometrial cancer, nausea, vomitingà Least nauseating anticancer drugs. Clomiphene: ovarian enlargement, vasomotor flushes, visual disturbances. Raloxifene Pharmacology: Reduces bone resorption thereby decrease bone turnover. Exhibit estrogen like (agonist) effect on bones and lipid metabolism. Exhibit estrogen antagonist action on uterine and breast tissues. Raloxifene therapeutic use: Used in prevention of osteoporosis. Raloxifene side effects: Hot flushes

Progesterone Progesterone chemistry. Progesterone is a C-21 steroid, its basic nucleus is known as pregnane. Two types of progesterone · 17-alpha hydroxyprogesterone such as Question Alert! medroxyprogesterone acetate and megestrol acetate. 17-alpha ethinyl androgen are potent 17-alpha ethinylandrogens (more potent). Norethindrone and progesterone. Norethynodrel are commonly used in OCs, because potent oral activity, more lipids soluble and less first pass metabolism. 17-alpha hydroxyprogesterone Injections Medroxyprogesterone, megestrol acetate

17-alpha ethinyl androgens (androgenic progesterone's) Oral stable Potent Norgesterol, norethindrone, levonorgesterl

Progesterone therapeutic use Progesterone is used alone for the treatment of amenorrhea, abnormal uterine bleeding, and endometriosis. Levonorgestrel is progesterone, which is available as a sub-dermal implant for long-term contraception. Six capsules are implanted on the inside of the upper arm. Contraception begins within 24 hours and may last up to 5 years. Side effects nausea, depression, liver failure, cancer and high blood cholesterol. Medroxyprogesterone acetate (Depo-Provera injection) is used to treat amenorrhea (cessation of menstrual periods) and abnormal uterine bleeding. It is also used as a contraceptive. As a contraceptive, 150 mg is injected once every three months. (no aromatic ring and ketone at position 3

C C

O

O O Progesterone

O OC - CH3

CH3

Medroxy Progesyterone Acetate HO

O

O

C

C-H

Noethindrone

Copyright © 2000-2018 TIPS Inc. Unauthorized reproduction of this manual is strictly prohibited and it is illegal to 28-12 reproduce without permission. This manual is being used during review sessions conducted by PharmacyPrep.

www.Pharmacyprep.com

Medicinal Chemistry

Progesterone products Prometrium (micronized progesterone) 100 mg capsules powder is suspended in peanut oil, glycerine. Progesterone side effects Medroxyprogesterone acetate unexpected or increased flow of breast milk, depression, loss or changes in speech, coordination, stomach pain, swelling of face, ankles or feet, headaches, mood changes, and unusual fatigue.

Anti Progestins Anti Progesterone: Mifepristone (RU – 486) Anti Progesterone pharmacology Progestin antagonist with partial agonist activity. Mifepristone also has anti glucocorticoid property. Causes abortion if administered in early pregnancy (85%) due to interference with progesterone and decrease in production of human Chorionic Gonadotropin (hCG). Anti Progesterone therapeutics use Abortifacient: Administration of mifepristone used as contraceptive given once a month when progestin levels are high (Prostaglandin E 1 and misoprostol orally after single dose of mifepristone effectively terminates the gestation.

Androgens Danazol, Nandrolone, Stanozolol, and Fluoxymesterone Testosterone. C-19 steroid. All androgens have anabolic activity. Androgenic effect. Testosterone 17-enantahne, resemble estradiol esters that increases duration of action when given I.M. Agents with 17-methyl or ester groups are orally active O O C (CH2)5 CH3

OH

O

O

Testosterone enanthate

Androgen Testosterone (no ester or ethinyl group) O O

O

C

CH2CH3

C

R

H3C O

O Dromostanolone

N H H

Finasteride (presence of Heterocyclic ring)

Antiandrogen

Androgen pharmacology Testosterone is the androgen that leads to the development of male secondary sexual characteristics and maintains the male reproductive system.

Copyright © 2000-2018 TIPS Inc. Unauthorized reproduction of this manual is strictly prohibited and it is illegal to 28-13 reproduce without permission. This manual is being used during review sessions conducted by PharmacyPrep.

www.Pharmacyprep.com

Medicinal Chemistry

AROMATASE 5-ALPHA REDUCTASE 17 B ESTRADIOL Insulin hormone antagonist à Glucogan hormone causeà Diabetic ketoacidosis (DKA) mainly occurs in à Sulfonylureas: Chlorpropamide, Gliclazide, glimepiride (Amaryl),glyburide and tobutamide. Chlorpropamide + alcohol can cause à Meglitinides – Nateglinide, repaglinide Biguanides - metformin Dose adjustment in renal impairment, SE: lactic acidosis, diarrhea (most), VB 12 ↓ ONLY oral, NO Wt↑ → good for obese Pt, NO hypoglycaemia on its own DI: alcohol (potentiates hypoglycemic effect) CI: hepatic impairment, renal impairment, CHF, hypoxemic states Pts Metformin + glyburide may causeà Thiazolidinediones (TZDs) pioglitazone, rosiglitazone and combination rosiglitazone with glimepiride or metformin. Thiazolidinediones SEs: Wt↑(most), fluid retention, ↑ HDL, NO hypoglycaemia on its own DIs: gemifibrozil ↑repaglinide concentration thereby avoid combination. CIs: congestive heart failure. Can use renal impairment patients. Alpha-glucosidase Inhibitors: Acarbose, SE: flatulence, diarrhea With meal, NO use as monotherapy. Incretin hormone or glucagon like peptide-1 (GLP-1) are inactivated by? Dipeptidyl peptidase-4 enzyme (DPP-4s). Sitagliptine inhibits? DPP-4 Sitagliptine SEs: Nasopharyngitis, DI: low potential (NO inhibit CYP P450), Take with metformin, with or without food, NO potentiates hypoglycaemia Intestinal Lipase Inhibitors – orlistat, SE: Diarrhea, Stethorrhea, abdominal discomfort, and oily leakage. Take with food; impair absorption of fat-soluble vitamins (A, D, E, K) Thyroid Hormones-levothyroxine (Eltroxin, Euthyrox, Synthroid): T4

Copyright © 2000-2018 TIPS Inc. Unauthorized reproduction of this manual is strictly prohibited and it is illegal to 28-15 reproduce without permission. This manual is being used during review sessions conducted by PharmacyPrep.

www.Pharmacyprep.com · · · · · · · · · · · · · · · · · · · · · · · · · · · ·

Medicinal Chemistry

Dosage: 1.6 µg/kg/day (adults), 12.5-25 µg/day (pts with coronary artery disease or elderly) Example of antithyroid drugs à Myxedema is malfunction of à Antithyroid Agents - methimazole: MMI (Tapazole), propylthiouracil: PTU (Propyl-Thyracil). SE: agranulocytosis, monitor WBC or CBC Stop about 5days prior to a thyroid scan, RAIU or treatment with 131I Increase in cortisone cause (Hypercorticoids) à Decrease in cortisone cause (Hypocorticoids) à Glutathione is à During ovulation increase of à Corpus luteum is stimulated by à What steroidal hormone structure have phenolic ring à Finaseride is à Vitamin D 3 acts as à The effect of vasopressin on kidney à Deficiency (absence) of ADH cause à Glutathione protectsà The endocrine gland plays important rule in calcium metabolism à The major factor that controls Na excretion in kidneyà ) The effect of the antidiuretic hormone is to à In postmenopausal therapy, which drugs have, risk of endometrial cancer à Oral contraceptives completely contraindicated inà Oxytosin is used à Calcium reabsorption of distal convoluted tubule due toà Testosterone to 5-hydroxy testosterone is catalyzed by? à Example of antiandrogenic drug à Finasteride mechanism of action à Steroid structures are common in hormones, such estrogen, progesterone, and testosterones. Steroid contain how many cyclohexane and cyclopentane respectively in it skeleton.

Copyright © 2000-2018 TIPS Inc. Unauthorized reproduction of this manual is strictly prohibited and it is illegal to 28-16 reproduce without permission. This manual is being used during review sessions conducted by PharmacyPrep.

www.Pharmacyprep.com

29 Medicinal Chemistry and Pharmacology of Respiratory Drugs Questions Alerts! Common questions in pharmacy exam is to ask! · Pharmacology of beta2 agonist · Side effects of oral steroids like prednisone · In children asthma use short acting steroids like prednisone and avoid long acting dexamethasone. Short acting have less side effects on growth suppression. Asthma Chronic inflammatory disorder of the airways, ↑ airways responsiveness, causes reversible obstruction. In asthma esinophils, mast cells and T lymphocytes plays significant role. ↑ Sensitivity, and hypersensitivity of airways to specific and non-specific stimuli, such as air, odour, allergens, and virus etc. Asthma Airway (Bronchus)

Inflammatory disease Esinophilic Coughing with wheezing during breath, and SOB. DOC for acute asthma attacks is salbutamol, antiinflammatory Inhaled corticosteroids (ICS).

COPD Progressive, partially reversible airway limitation. Increases frequency and severity of exacerbations. COPD is preventable and treatable. The cardinal symptoms are SOB and activity limitation, fatigue, and barrel chest. Terminal alveoli Emphysema (permanent enlargement of Chronic bronchitis alveoli) Neutrophil Neutrophil SOB (dyspnea), fatigue SOB (dyspnea) + sputum DOC bronchodilator ipratropium, tiotropium, salbutamol, salmeterol, formoterol (LABA). Inhaled corticosteroids are LEAST likely used. PSO2 12 yr age and only oral available. Advantage of combinations. More convenient, enhance adherence, less expensive. Disadvantage. Loss in dosing flexibility Chronic Obstructive Pulmonary Diseases Chronic obstructive pulmonary diseases (COPD) is due to chronic obstruction of airway passage. COPD is two types. Emphysema (high altitude sickness) and chronic bronchitis. Emphysema is a disease in which the small air exchange sacs (alveoli) in the lungs become permanently enlarged and damaged (alveoli walls destroyed) decreasing oxygen absorption and resulting in shortness of breathe. Chronic bronchitis is an inflammation of the airways in the lungs that causes lungs to produce excessive amounts of mucus (phlegm), associated with chronic productive cough. This reduces the flow of air to the lungs. Drug used for the treatment of COPD: Anticholinergics. Ipratropium and tiotropium are muscarinic blocker and act as bronchodilator. Beta adrenergic agonists, Corticosteroids and theophylline. Community acquired pneumonia (CAP) without risk factors: Copyright © 2000-2018 TIPS Inc. Unauthorized reproduction of this manual is strictly prohibited and it is illegal to 29-3 reproduce without permission. This manual is being used during review sessions conducted by PharmacyPrep.

www.Pharmacyprep.com Pneumonia in COPD patient is treated by amoxicillin, doxycycline, cotrimoxazole, azithromycin, or clarithromycin. COPD with risk factors (FEV 1 1 A drug is hydrophilic if partition coefficient is Internalization of drug molecules (cell drinking) PASSIVE DIFFUSION CONCENTRATION GRADIENT IS ESSENTIAL Topical drugs, eye drops, placenta

ACTIVE TRANSPORT No concentration gradient Tubular secretion Calcium & vit.D

FACILITATED

ENDO/EXOCYTOSIS

Glucose uptake by insulin

Copyright © 2000-2018 TIPS Inc. Unauthorized reproduction of this manual is strictly prohibited and it is illegal to 33-4 reproduce without permission. This manual is being used during review sessions conducted by PharmacyPrep.

www.Pharmacyprep.com

Biopharmaceutics

Factors that determines the absorption Oral drug

Question Alerts!

1) Ficks Law determines "rate of absorption". 2) Noyes-Whitney determines "rate of dissolution".

Disintegration

Dissolution

Rate of Dissolution Noyes-Whitney

Absorption

Ficks Law

Hasselback & Handerson

- Rate of absorption

Factors that affect the rate of absorption * pH effect * ionization & unionization

Question Alerts! Ranking of various oral dosage (fastest → slowest) · Solutions (ready for absorption) · Suspensions (wetted and ready for dissolution) · Powder [(dispersed + GI fluid → wet) absorbed] · Capsules (dissolve gelatine cap first → then powder) · Tablets (disintegrate from tablet to smaller granulate then → powder. · Sustained release tab (barrier of coating materials).

The fastest oral dosage form? Solution

The rate of absorption of solid drugs. Solid drug -->(disintegration) and then (dissolution) --> Drug in solution --> (absorption) in to blood. Dissolution is the rate-limiting (slowest) step in sequence. · Disintegration. Breaking down of drug so as to facilitate dissolution. This usually occurs in oral routes. · Dissolution. Dissolving substances in the GIT fluids to facilitate absorption of the drug. · Rate limiting step. It is the time it takes for a drug to dissolve and become available for absorption. Factor that effect rate of absorption: Degree of ionization, solubility, pH (acidity or alkalinity of the substance). Degree of ionization: Drugs will pass thru a membrane at a faster rate if they are unionized. The size of an ion increases due to dipole-to-dipole attraction especially water. Ionized portion of drug is less soluble in lipid but more in water. DRUG IONIZED EXCRETE DRUG UN-IONIZED REABSORBS (EXCRETE SLOWLY) Solubility · Extremes of either water or oil solubility is associated with poor absorption. · Hydrophilic = water soluble (water lover), poorly absorbed. Removed faster from the body. Copyright © 2000-2018 TIPS Inc. Unauthorized reproduction of this manual is strictly prohibited and it is illegal to 33-5 reproduce without permission. This manual is being used during review sessions conducted by PharmacyPrep.

www.Pharmacyprep.com ·

Biopharmaceutics

Hydrophobic or lipophilic = oil or lipid soluble, or oil and lipid lover.

pH (acidity or alkalinity of the substance) · Most drugs are weak acid or weak bases. Dissociation of weak acids or bases is directly affected by pH therefore absorption is directly affected by pH. · As pH rises (H+ decreases), the amount of weakly acidic drug in the non-ionized state decreases · As pH falls (H+ increases) the amount of weakly acidic drug in the non-ionized state increases; i.e. diffusion pressure increases. · For weak bases the effect is opposite. NOTE. One changes the pH by 1 unit, the ratio of non-ionized to ionized changes by factor of 10. The direction of change and ration can be calculated in one’s head when the pH changes are 1 full unit Henderson-Hasselbalch Equation. Helps to determine pH effect on absorption. (Determining the drug absorption by pKa). If the pKa of drug and pH of the medium are known, the fraction the molecules in the ionized state can be predicted by means of the Henderson-Hasselbalch equation. How much of drug is found on either side of membrane. For an acid: pH = pKa + log (Salt)/ (Acid) For base:

pKa = pH – log (Salt)/ (Acid)

pH = pKa + log (Acid)/ (Salt) You may also see the above as: pH = pKw – pKb + log (base) (salt) Since pKw = pKa + pKb Weak acid Weak Base

RCOOH (Cross membrane) à RCOO + H+ R-NH 3 à RNH 2 + H+ (cross membrane)

Predicting Percent ionized (To calculate percent of a weak acid or base that is ionized).

Where charge -1 if acid drug or 1 if basic drug Notes on ionization · Ionized drugs are water soluble and poor absorption through stomach, blood brain barrier, and placenta, no reabsorption across membrane and excrete faster. · Non-ionized (unionized) are lipid soluble and absorbed well in membrane (cell membranes are composed of lipids), and have higher reabsorption. · An acid in an acid solution will not ionize (acid + acid = nonionized). · An acid in a basic solution will ionize (acid + base = ionized). · A base in a basic solution will not ionize (base + base = nonCopyright © 2000-2018 TIPS Inc. Unauthorized reproduction of this manual is strictly prohibited and it is illegal to 33-6 reproduce without permission. This manual is being used during review sessions conducted by PharmacyPrep.

www.Pharmacyprep.com

· · · · · · ·

Biopharmaceutics

ionized). A base in an acid solution will ionize (base + acid = ionized). PKa = pH drug exists as 50% ionized and 50% unionized. If pH-pKa = 0, then 50% of drug is ionized and 50% is unionized. If the acid/base ratio is 1:1, then the log of that number will be zero, and pH = pK a . If pH - pKa = 0.5, then the solution is 75% ionized/25% unionized or 75% unionized/25% ionized. If pH - pKa >1 then the solution is 99 to 100% ionized or 99 to 100% unionized. If pH - pKa >2 then the solution is 100% ionized or unionized.

Surface area (Ficks law of diffusion) Ficks law predicts the rate of movement of molecules across barrier. Formula: Rate of diffusion = D x A x (C 1 – C 2 ) L D = Diffusion coefficient A = Surface area of solid C 1 = Concentration near to stagnant layer C 2 = Concentration of solute to other side of stagnant layer L = The length of the stagnant layer This relationship demonstrates the drug absorption is faster from organs with large surface areas. The rate of diffusion is directly proportional to the area of the solid, the concentration difference between the concentration of solute in the stagnant layer at the surface of solid and its concentration on the farthest side of the stagnant layer and diffusion coefficient. It is inversely proportional to the length of stagnant layer. The driving force behind the movement of the solute molecules through the stagnant layer is the difference in concentration of solute at C 1 and its concentration at C 2 . Solubility Defined as the concentration of solute in a saturated solution under specific conditions of temperature and pressure. It may be viewed as an equilibrium condition in which solute molecules are leaving the solid (or undissolved phase) at the same extent as solute returning to it. Saturated solution: Is a state in which solute is at equilibrium with solid phase Supersaturated Solution: Contains more dissolve solute then it normally would contain at a specific temperature if there were undissolved solute present. If upon cooling, the excess solute fails to crystallize from the lower temperature, the solution is supersaturated. Unsaturated solution: Contains dissolved solute in a concentration below that necessary for complete saturation Compendia expressions of approximate · Very soluble………………………..Less than 1 part · Freely soluble……………………...1-10 parts · Soluble…………………………….. 10-30 parts · Sparingly soluble ………………… 30-100 parts · Slightly soluble …………………… 100-1,000 parts · Very slightly soluble ……………… 1,000-10,000 parts

Copyright © 2000-2018 TIPS Inc. Unauthorized reproduction of this manual is strictly prohibited and it is illegal to 33-7 reproduce without permission. This manual is being used during review sessions conducted by PharmacyPrep.

www.Pharmacyprep.com

Biopharmaceutics

Types of Solvents Polar solvents These consist of strongly dipolar molecules having high dielectric constants, e.g., water, and methyl alcohol, ethyl alcohol. Semi polar solvents These are strongly dipolar molecules, but which do not form hydrogen bonds. Examples are ketones and certain alcohols. They may induce a degree of polarity in nonpolar solvent molecules. Nonpolar solvents These solvents have a small or no dipolar character. Theses include hydrocarbons, fixed oils, and mineral oil. They have a low dielectric constant and possess little tendency to reduce the attractive forces between ions of strong and weak electrolytes. Factor Affecting solubility and Rate of Solution Noyes-Whitney Equation A thin layer of solvent, which behaves as an integral part of the particle and is referred to as the diffusion layer, C 1 -C 2 , surrounds a particle of solute dispersed in a solvent medium. The diffusion layer remains a part of the solute particle regardless of extent of agitation of the bulk solution. The dissolution rate of the solute is expressed by the Noyes-Whiteney equation: Dc/dt = KA (C 1 -C 2 ) Where dc/dt is the change in concentration of solute in solution with respect to time. Effect of Temperature Solubility of a solid in a liquid is dependent on the temperature. If heat is absorbed in the solution process, solubility of solute will increase with increase in temperature. Effect of electrolytes on the solubility of non electrolytes Salting out and salting in. The addition of a salt may either increase or decrease the solubility of nonelectrolyte. When the solubility is decreased, the effect is referred to as “salting out”. When the solubility is increased, it is known as “salting in”. Action potential across cell membrane: Neuronal excitability depends on the influx of ions through specific channels in membranes. Membrane depolarization: Excitation = increase Na entry (influx), decrease K exit (efflux) Membrane repolarization. Inhibition = increase Cl- enter in and increase K+ exit. Agent Botulinus toxin Curare

Action Blocks release of ACh from presynaptic terminals Competes with ACh for receptors on motor end plate

Neostigmin

anticholinesterase

Hemicholinium

Blocks reuptake of choline into presynaptic terminal

Results Total blockade Decrease size of end plate potential: maximal doses produce paralysis of respiratory muscles and death. Prolongs and enhances action of ACh at muscle end plate Depletes ACh stores from presynaptic terminal.

Copyright © 2000-2018 TIPS Inc. Unauthorized reproduction of this manual is strictly prohibited and it is illegal to 33-8 reproduce without permission. This manual is being used during review sessions conducted by PharmacyPrep.

www.Pharmacyprep.com

Biopharmaceutics

Tips

· · · · · · · · · · · · · · ·

1.

Oral

2

rectal

3

4. 7. 10

first pass metabolism nasal mucosa transdermal

5 8

sublingual intramuscularly

6 9

in the stomach and intestine but mainly in the liver lungs intravenous

The metabolism of the drug before the drug reaches the general circulation ( ) Where does the first pass metabolism occur? ( ) Which route of administration is most likely to subject a drug to a first-pass effect? ( ) What sites of absorption have low first pass metabolism? ( ) What factors affect the bioavailability of a drug? ( ) What is responsible for the different phases of a two-compartment model of drug elimination? ( ) What is meant by first pass metabolism? Drugs are metabolized before reaching systemic circulations Where does the first pass metabolism usually occur? liver Which routes of administration are most likely to subject a drug to a first pass effect? Oral (drug that by pass liver have no first pass metabolism). What sites of absorption have low first pass metabolism? Rectal. Bioavailability of sustained release and immediate release preps, what is the same! AUC. Fick's law describes? ( rate of absorption) The rate limiting step in ophthalmic drops? (cornea) Partition coefficient is described as? ( predicting drug solubility) Write the sequence of surface area in GIT (small intestine>stomach> large intestine).

TRUE OR FALSE · Sustained release and immediate release formulations of a drug have different rate of bioavailability however the same extent. · After oral administration of iron, it is absorbed from duodenum by an active transportation (T/F) · Whenever a drug is more rapidly and more completely absorbed from a solid form, the rate-limiting factor is the dissolution process (slowest). · Write sequence of absorption for oral dosage, from higher to lower: solution > suspension > liquid gel caps > powder>

Copyright © 2000-2018 TIPS Inc. Unauthorized reproduction of this manual is strictly prohibited and it is illegal to 33-9 reproduce without permission. This manual is being used during review sessions conducted by PharmacyPrep.

www.Pharmacyprep.com

Biopharmaceutics

Copyright © 2000-2018 TIPS Inc. Unauthorized reproduction of this manual is strictly prohibited and it is illegal to 33-10 reproduce without permission. This manual is being used during review sessions conducted by PharmacyPrep.

www.Pharmacyprep.com

Physical Pharmacy

34 Physical Pharmacy Questions Alerts! Common questions in pharmacy exam is to ask! · United States Pharmacopeia/National Formulary (USP/NF) standards of alcohol and temperatures, Compounding Formula · Colligative properties. Osmotic pressure AND OSMOLARITY CALCULATIONS. · Arrhenius equation · Polymorphism · Critical solution temperature

State of the matter

Gases

Liquids

Solids

Changes in state. Increase in temperature of a substance increases its heat content, or enthalpy. Melting. Solid to liquid state. Vaporization. Liquid to gaseous state. Question Alerts! Volatile liquids used as inhalation anesthetics e.g. ether, halothane, and Sublimation is? methoxyflurane also used in vasodilatation in acute angina e.g. amile nitrite. · Sublimation: Solid heated directly to gaseous or vapor state without passing through the liquid state e.g. camphor and iodine. Examples of process like lyophilisation (freeze drying). · Deposition. The reverse process to the sublimation i.e. direct transition from the vapor state to the solid state. Example colloidal silicon dioxide and some form of sulphur. Solids.

· · · ·

Polymorphism: Polymorphism is the occurrence or existence of the same substance in different crystalline forms. Ability of a substance or drug to exist in different crystalline forms. Different properties such as melting points, solubility, dissolution rate, density, and stability. Examples of polymorphism include theobroma oil or cocoa butter exhibit polymorphism. Amorphous materials. Solids and liquids differ from crystals in that they do not possess long-range periodicity of packing. They will therefore be isotropic. Window glass, basically SiO 2 is a common example. Many plastics like PVCs are also amorphous.

Copyright © 2000-2018 TIPS Inc. Unauthorized reproduction of this manual is strictly prohibited and it is illegal to 34-1 reproduce without permission. This manual is being used during review sessions conducted by PharmacyPrep.

www.Pharmacyprep.com

Physical Pharmacy

Crystalline versus amorphous form. The amorphous form of a compound is usually more soluble than the crystalline form. Different polymorphic forms of the same compound may demonstrate different physical properties including water solubility. Question Alerts! Interfacial Phenomena Wetting Phenomenon? The Interface: Interfacial phenomena are attributable to the effect of the properties of molecules located at or close to the boundary between immiscible phases. The region of influence is referred to as the interface. Interface may exist between a liquid and a gas (a foam) between two immiscible liquids (an emulsion), between a solid and liquid (a suspension), between solids, solid and gas, etc. Wetting Phenomena: A solid is said to be wetted by a liquid if the liquid spontaneously spreads over the solid. A solid is not wetted by a liquid if the latter cannot spread over the former spontaneously. The contact angle is an important parameter reflective of the degree of wetting of a solid by a liquid. This is the angle that a droplet of the liquid makes with the solid surface at the point of contact. Liquids: With few exceptions, most organic solvents are irritating or toxic. Aromatic hydrocarbons cause paralysis of the central nervous system and are irritating to the skin. Methyl alcohol (methanol) and isopropyl alcohol, ethylene glycol is toxic, and butyl and amyl alcohol are irritating. Volatile ethers paralyse the central nervous system, and are irritating to mucous membrane increases. Ketones are mildly irritating and the low molecular weight esters are irritating. Toxicity and irritation limit the many solvents internal use except, Glycerine, ethyl alcohol, and propylene glycol can be employed for internal use as pharmaceutical solvents. Aliphatic hydrocarbons, ether, and glyceryl esters of aliphatic acids can be employed for external use as pharmaceutical solvents. Propylene glycol has been employed as a solvent for oral and parenteral solutions of drugs such as antihistamines, barbiturates and vitamins. Hydrogen bonding (H….N or H….O) increases the likelihood of cohesion in liquids and further affects their physiochemical behaviour. Van der Waals forces impose regular arrangement among molecules. London forces in molecules are weak intermolecular forces in liquid hydrocarbon are not true chemical bonds. Critical solution temperature: It is the maximum temperature above which homogenous liquid is formed regardless to any concentration of phenol. Viscosity is an internal property of a fluid that offers resistance to flow. Not, all liquids are the same. Some are thin and flow easily. Others are thick and gooey. Honey or corn syrup will pour more slowly than water. A liquid's resistance to flowing is called its viscosity. Gases: The intermolecular forces of attraction in gases are virtually non existence at room temperature. Pressure. Random collision of molecules with boundaries of the system is responsible for pressure. Ideal gas law: The interrelation among Volume (v), pressure (P) and the absolute temperature (T) is given by ideal gas law: PV = nRT, where n = number of moles of gas and R = molar gas constant (0.08205 L atm/mole deg). Copyright © 2000-2018 TIPS Inc. Unauthorized reproduction of this manual is strictly prohibited and it is illegal to 34-2 reproduce without permission. This manual is being used during review sessions conducted by PharmacyPrep.

www.Pharmacyprep.com

Physical Pharmacy

Pharmaceutical gases: · Anaesthetic gases: Nitrous oxide and halothane · Compressed gases: Oxygen, nitrogen, carbon dioxide · Liquefiable gases: used as propellants in aerosol (pressurized package) products, eg: ethylene oxide is gas used to sterilize or disinfect heat-labile objects. Chemical kinetic & Drug stability Factors that affect chemical stability: The factors that affect chemical stability include: temperature, pH, moisture, air (oxygen) and light. Effect of temperatures on drug degradation. The Arrhenius equation describes the effect of temperature on the rate of drug degradation reaction. Heat increases rate of chemical reaction. Every 10oC increase normally 2 to 3 times rate of reaction increases. Arrhenius equation k = S x e-Ea/RT Ea =Arrhenius activation energy T = absolute temperature e-Ea/RT = Boltzman factor S = Frequency factor R= Gas constant

Question Alerts! Arrhenius equation determine factors effecting on chemical stability

Logarithm Log k = log S-Ea/2.303 RT Integration between two limits k 1 and k 2 at temperature T 1 and T 2 Log K 2 /K 1 = Ea/2.303 R x (T 2 -T 1 /T 1 T 2 ) Change pH effect on degradation of drugs. The magnitude of the rate of hydrolytic reaction catalyzed by acid (H+) and base (OH-) can change with pH. Acid (H+) catalysis predominates at lower pH, whereas base (OH-) catalysis operates at higher pH. To determine the effect of pH on degradation kinetics, decomposition is measured at several H+ concentrations. The pH of optimum stability can be determined by plotting the logarithm of the rate constant (k) as function of pH. The point of inflection of the plot is the pH of optimum stability. The value is useful in the development of a stable drug formulation. At pH 1 to 3 (strong acidic) more susceptible to H+ (acidic). At pH 5 to 14 (weak acid and base) more susceptible to OH- (base). Change in pH: Hydrolysis reactions are catalyzed by H and OH ions, can change with pH · Acid (H+) catalysis predominates at lower pH · Base (OH-) catalysis predominates at higher pH The effect of pH on degradation kinetics, decomposition is measured by plotting the log of the rate constant as function of pH. The point of inflection on the plot is the pH of optimum stability. This value is useful in development of stable drug formulation. Modes of pharmaceutical degradation: Hydrolysis, Oxidation and free radicals results in degradation and photolysis. Antioxidants prevents free radical propagation (hydrogen peroxide, .OH, and benzoyl peroxides).

Copyright © 2000-2018 TIPS Inc. Unauthorized reproduction of this manual is strictly prohibited and it is illegal to 34-3 reproduce without permission. This manual is being used during review sessions conducted by PharmacyPrep.

www.Pharmacyprep.com

Physical Pharmacy

Antioxidants: · Water soluble: Ascorbic acid, sodium bisulfate, and sodium sulfite. · Lipid soluble: Butylated hydroxyl anisole (BHA), butylated hydroxyl toluene (BHT), propyl gallate, and the tocopherol (vitamin E 1 ). Photolysis: Exposure to light wavelength less than 400 nm. Protect using amber glass or opaque storage. Sodium nitroprusside has a shelf life only 4 hour, if exposed to normal room light, when protected form light, the solution is stable for at least one year. Stability, kinetics and shelf Life: Most commonly zero order and first order reactions are encountered in pharmacy. Zero order

-dC = ko dt Where Ko is zero-order rate constant [C/t]

C = -Kot + Co Where Co is the initial concentration of drug

t = C-Co -Ko

First order

-dC dt

= kC

Where C is concentration of intact drug remaining, t is time, (-dC/dt) is the rate at which the intact drug degrades, and k is the specific reaction rate constant.

log C =

-kt +log Co 2.303

Where Co is the initial concentration of drug In natural log form:

Question Alerts! Shelf life t10% or t90% = 0.105/k

In C = -Kt + In Co Shelf life t 10% or t 90% = 0.105/k

Copyright © 2000-2018 TIPS Inc. Unauthorized reproduction of this manual is strictly prohibited and it is illegal to 34-4 reproduce without permission. This manual is being used during review sessions conducted by PharmacyPrep.

www.Pharmacyprep.com

Physical Pharmacy

Buffers and Buffer Calculations: A buffer is a compound or a mixture of compounds that has the ability to resist changes in pH when limited amounts of acid or base are added to the solution of the buffer or when the solution is diluted with solvent. Generally, a buffer system consists of a weak acid and its salt of the weak acid, or a weak base a salt of the weak base. An example of the former is acetic acid and sodium acetate and of the latter is ammonium hydroxide and ammonium chloride. Mechanism of action: In the example of the acetic acid-sodium acetate buffer combination the acetic acid is essentially unionized and the sodium acetate is completely ionized. When acid is added to the buffer system, the hydronium ion reacts with acetate ion to form more unionized acetic acid CH 3 COOH + H 3 O+ -------> CH 3 COOH + H 2 O And when base is added to the buffered solution, it will react with acetic acid to form more acetate ion. CH 3 COOH + OH- --------> CH 3 COO + H 2 O Other Types of Systems: The combination of certain salts may function as a buffer system, as for example, the combination of monobasic potassium phosphate and dibasic potassium phosphate in the appropriate molar ratio. A study of the mechanism will reveal that the buffer behaviour is essentially the same as the previous cases mentioned above. Buffer Calculations; The calculations involving buffer systems are based on the Henderson-Hasselbalch equation. pH = pKa + log [base]/ [acid] The above equation applies to all buffer systems involving a single proton transfer (conjugate acid-base pair). If a buffer system, for example, is composed of 0.1 molar acetic acid and 0.1 molar sodium acetate (Ka for acetic acid is 1.75 x 10-5). The pH of a solution will be pKa is 4.5 pH = pKa + log [base]/[acid] pH = 4.75 + log (0.1/ 0.1) = 4.75 pKa = -log of Ka If the concentrations of acetic acid and sodium acetate are equimolar, the pH of the solution will always be the same, 4.74. However, the higher the concentration of buffer compounds, the greater will be the buffer capacity or the greater the ability to resist change in pH. Buffer Capacity: This is a quantitative expression of the ability of a buffer system to resist change in pH. β = ∆A/∆ pH where β is the buffer capacity and ∆ A is the addition in gram equivalents per litre of strong acid (or strong base) to buffered solution to produce a pH change of ∆pH. Colligative properties: The Colligative properties depend primarily upon the number of particles in solution. Example adding solute to solution of these properties includes. · Lowering vapor pressure Question Alerts! · Increase in boiling point 1) Examples of colligative properties? · Decrease in freezing point 2) Freezing point depression is used to calculate · Osmotic pressure molecular weight of non-ionic molecule. Copyright © 2000-2018 TIPS Inc. Unauthorized reproduction of this manual is strictly prohibited and it is illegal to 34-5 reproduce without permission. This manual is being used during review sessions conducted by PharmacyPrep.

www.Pharmacyprep.com · · · · · ·

Physical Pharmacy

Lowering of vapour pressure. When a solute is added to a liquid, it will decrease the vapor pressure of the liquid. Increase in boiling point. The effect on the boiling point is just the opposite. That is the boiling point of a liquid is increased if something is dissolved in it. Boiling is the vapour pressure of liquid not more than the atmospheric pressure. Decrease in freezing point. When a solute or salt is added to liquid, it will decrease the freezing point. Osmotic pressure. Isotonic. The solute concentration is the same on both sides of the membrane. The solutions are said to be isotonic compared to one another. Hypotonic. The clinical significance of all this is to insure that isotonic or iso-osmotic solutions do not damage tissue or produce pain when administered.

Tips · · · · · · · · · · · · · · · ·

Arrhenius equation is used for? à Water-soluble antioxidants? à Fat-soluble antioxidants? à Theobroma oil and cocoa butter are? à Polymorphs have à Arrhenius equation is used for? à Water soluble antioxidants? à Polymorphs are different in crystalline forms of the same drug, will differ in à USP official temperature is à Protect from light indicates storage in light resistant container that reduces light transmission in the range of à Pycnometer is used for à Hydrometer is used for à Boric acid is à Tannic acid is à Acetic acid is à Freon is à

USP (United States Pharmacopea): · Alcohol USP 94.9% ethanol V/V 92.3% ethanol W/V · Diluted alcohol 49% ethanol V/V · Rubbing alcohol 70% V/V absolute alcohol (denatured)

Copyright © 2000-2018 TIPS Inc. Unauthorized reproduction of this manual is strictly prohibited and it is illegal to 34-6 reproduce without permission. This manual is being used during review sessions conducted by PharmacyPrep.

www.Pharmacyprep.com

Pharmaceutical Excipients

35 Pharmaceutical Excipients Questions Alerts! Common questions in pharmacy exam is to ask! · Types of water in parenteral preps · Examples of alkalinizing agents, antioxidants, surfactants, levigating agents, bacterial and fungal preservatives.

Question alerts! 1) What type of water is used to manufacturing parenteral preps? 2) What type of water is used for multiple use containers?

Water for injection USP · Prepared by double distillation or reverse osmosis · Free from pyrogen · Used as solvent for parenteral solutions in manufacturing. Sterile water for injection USP · not >1 liter · Sterilized and packed in single dose container of type I and II glass. · Limitation of total solids depends on size of the container. · For compounding Bacteriostatic water for injection USP · Not >30 ml · Contain 1 or more antimicrobial agent Packed in single or multiple use dose containers. Sterile water for irrigation USP à >1 liter Sterilized and suitably packed It contain no antimicrobial agents or other added substance

Copyright © 2000-2018 TIPS Inc. Unauthorized reproduction of this manual is strictly prohibited and it is illegal to 35-1 reproduce without permission. This manual is being used during review sessions conducted by PharmacyPrep.

www.Pharmacyprep.com

Pharmaceutical Excipients

Purified water USP Prepared by distillation, reverse osmosis, or ion exchange Should not contain 10 ppm solid particle Should have pH between 5 and 7 Used in prescription and finished manufactured product Not used in parenteral and ophthalmic products Sterile purified water USP It is purified water sterilized and packed It does not contain antimicrobial agents It is NOT intended for parenteral preparations Water for injection USP Manufacturing

IV, IM, SC and ophthalmic

STERILE water for injection Parenteral or IV admixture, ophthalmic Compounding Single dose containers

Bacteriostatic

Distilled water

Multi dose container

Oral, topical and all other preps

Vitamin B12 inj

The method of preparation of purified water USP must be indicated on the label. [Sterile water for injection (USP method)]. Excipient Type Acidifying agent

Use Used in liquid preparations to provide medium for product stability.

Alkalizing agent

Used in liquid preparation to provide alkaline medium for product stability. Also used for acidic drug (ASA) overdose.

Adsorbent

An agent capable of holding other molecule onto its surface (adsorption) by physical or chemical (chemsorbtion) means. An agent responsible for developing

Aerosol propellant

Example

Acetic acid Ammonium chloride Ascorbic acid Citric acid Fumaric acid Hydrochloric acid Nitric acid Ammonia solution Ammonium carbonate Diethanolamine Monoethanolamine Potassium hydroxide Sodium borate Sodium carbonate Sodium hydroxide Triethnolamine Trolamine Sodium bicarbonate Powdered cellulose Activated charcoal Carbon dioxide

Copyright © 2000-2018 TIPS Inc. Unauthorized reproduction of this manual is strictly prohibited and it is illegal to 35-2 reproduce without permission. This manual is being used during review sessions conducted by PharmacyPrep.

www.Pharmacyprep.com

Pharmaceutical Excipients

the pressure within an aerosol container and expelling the product when valve is opened. Ozone oxidation mechanism is? free radical formation. Air displacement Antifungal preservative

Antimicrobial preservative

An agent which is employed to displace air in a hermetically sealed contained to enhance stability. Used in liquid and semi solid preparations to prevent the growth of fungi.

Used in liquid and semi solid preparations to prevent the growth of microorganisms. Ophthalmic preps use benzalkonium chloride concentration 0.004%.

Antioxidant

An agent which inhibits oxidation and thus is used to prevent the deterioration of preparations by oxidative process. All of the following are antioxidants, except? a. vitamin C (ascorbic acid) b. vitamin E (Tocoferal) C. EDTA d.Sodium bisulphite E. Sodium benzoate Ans. E

Buffering agent

Used to resist change in pH upon dilution or addition of acid or alkali. Buffers are made with Acid and salt of acid or base and salt of base.

Dichlorodifluoromethane, Dichlorotetrafluoroethane, Trichloromonofluoromethane (CFCs) also known as freon gas, it is not safe. Hydrofluroalkenes (HFA) are safe. Topical sprays are n-butane and propane. Nitrogen or inert gases (Ar, Ne, Xe, He) Benzoic acid Butyl paraben Ethyl paraben Methyl paraben Propyl paraben Sodium benzoate Sodium propionate Benzalkonium chloride Benzothonium chloride Benzyl alcohol Cetylpyridinium chloride Chlorobutanol Phenol Phenylethyl alcohol Phenylmercuric nitrate Thimerosal Water soluble (aqueous): Ascorbic acid Sodium ascorbate Sodium bisulphite Sodium formaldehyde Solfoxylate Sodium metabisulfite Hypophosphorous acid Lipid soluble (non aqueous): Ascorbyl palmitate Butylated hydroxyanisole (BHA) Butyllated hydroxytoluene (BHT) Monothiglycerol Propyl gallate Tocoferal (vitamin E 1 ) Potassium metaphosphate Potassium phosphate Monobasic Sodium acetate Sodium citrate anhydrous and dehydrate

Copyright © 2000-2018 TIPS Inc. Unauthorized reproduction of this manual is strictly prohibited and it is illegal to 35-3 reproduce without permission. This manual is being used during review sessions conducted by PharmacyPrep.

www.Pharmacyprep.com Chelating agent

Colorant

Clarifying agent Emulsifying agent (Surfactant)

Encapsulating agent

Flavorant

Humectants

Levigating agent

Pharmaceutical Excipients

A substance that forms stable, water soluble complex (chelates) with metals. Chelating agents are used in some liquid pharmaceuticals as stabilizers to complex heavy metals, which might promote instability. In such a case they are also sequestering agents. Used to impart color to liquid

Used as filtering aid because of adsorbent Used to promote and maintain the dispersion of finely subdivided particles of a liquid in a vehicle in which it is immiscible. The end product may be a liquid emulsion or semisolid emulsion (e.g. cream). Emulsifying agents also known as surfactants.

Used to form the shells for the purpose of enclosing a drug substance or drug formulation for ease of administration. Used to impart a pleasant flavor and often odor.

Used to prevent the drying out of preparation, particularly ointments and creams due to the agent’s ability to retain moisture. A liquid used as an intervening agent to reduce the particle size of drug

EDTA Edate disodium Edetic acid

FD&C Caramel Ferric oxide gives red color Tartazine gives yellow color Bentonate Acacia Cetomacrogol Cetyl alcohol Anionic Sodium lauryl sulfate Non ionic Tweens (Polyoxyethylene 50 stearate) Glycerl monoestearate and Spans (Sorbital monooleate) Cationic Benzalkonium chloride Gelatin Cellulose acetate phthalate (CAP), enteric coated. Sorbitol Anise oil Cinnamon oil Cocoa Menthol Orange oil Peppermint oil Vanillin Strawberry Glycerin Propylene glycol Sorbitol Mineral oil Glycerin

Copyright © 2000-2018 TIPS Inc. Unauthorized reproduction of this manual is strictly prohibited and it is illegal to 35-4 reproduce without permission. This manual is being used during review sessions conducted by PharmacyPrep.

www.Pharmacyprep.com

Ointment base

Plasticizer

Solvent

Pharmaceutical Excipients

powder by grinding together, usually in mortal. The semisolid vehicle into which drug substance may be incorporated in preparing medicated ointment.

Used as a component of film coating solutions to enhance the spread of the coat over tablets, beads and granules. An agent used dissolves another pharmaceutical substance.

Question Alerts! 1) What solvents are NOT used in parenteral preps? Mineral oil, theobroma oil

Stiffening agent

Used to increase the thickness or hardness of ointment

Water soluble base Hydrophilic ointment Rose water ointment Polyethylene glycol (PEG) ointment Lipid soluble Lanolin Propylene glycol Petrolatum (Occlusive base) Hydrophilic petrolatum White ointment Yellow ointment Diethyl phthalate Glycerin Sorbitol Solvent that are used in parenteral prep. Olive oil Corn oil Canola oil Cottonseed oil Peanut oil Water for injection USP Sterile water for injection USP Normal saline, D5W, Ringer solution. Solvent in oral prep. Purified water Sterile water for irrigation Ethyl alcohol Glycerin Mineral oil Oleic acid Cetyl alcohol Cetyl esters wax Microcrystalline wax Paraffin Stearyl alcohol White wax Yellow wax

Copyright © 2000-2018 TIPS Inc. Unauthorized reproduction of this manual is strictly prohibited and it is illegal to 35-5 reproduce without permission. This manual is being used during review sessions conducted by PharmacyPrep.

www.Pharmacyprep.com

Pharmaceutical Excipients

Surfactant

Substance, which absorbs to surfaces or interfaces to reduce surface or interfacial tension. May be used as wetting agents, detergents or emulsifying agents

Suspending agent

A viscosity increasing agent used to reduce the rate of sedimentation of particles dispersed throughout a vehicle in which they are not soluble.

Question Alerts! SUSPENDING AGENT - VISCOSITY ¯ SEDIMENTATION Tablet anti adherent

Agent, which prevent the sticking of tablet formulation ingredients.

Tablet coating agent

Used to coat a formed tablet for the purpose of protecting against drug decomposition by atmospheric oxygen or humidity to provide desired release pattern for drug substance after administration, to mask the taste and odor or the drug. The coating may be various type: Sugar coating Film coating or enteric coating Sugar coated is water based and results thickened covering around a formed tablet. Sugar coating generally starts to break up in stomach. A film coated is thin cover around a formed tablet or bead. Unless it is an enteric coated the film coat will dissolve in the stomach. An enteric-coated tablet or bead will pass through the stomach and break up in the intestine. Some coating that is water insoluble may be used to coat tablets or bead to slow the release of drug as they pass through the gastrointestinal tract. Used in direct compression tablet formulation

Tablet direct compressing exipient

Benzylalkonium chloride Nonoxynol 10 Oxtoxynol 9 Polysorbate 80 Sodium lauryl sulfate Sorbitan monpalmitate Agar Bentonite Carbomer Carboxymethylcellulose sodium (CMC) Hydroxymethyl cellulose Hydroxypropyl cellulose Hydroxypropyl methyl cellulose (HPMC) Kaoline Methyl cellulose Tragacanth Veegum Magnesium stearate Talc Silicon dioxide Sugar coating: Liquid glucose Sucrose Film coating: Hydroxethyl cellulose Hydroxypropyl cellulose Hydroxyproply methylcellulose Methylcellulose (e.g. Methocel) Ethyl cellulose (e.g. Ethocel) Enteric coating Cellulose acetate phthalate Shellac (35% in alcohol, “pharmaceutical glaze”). Methacrylic acid

Dibasic calcium phosphate (e.g. Ditab)

Copyright © 2000-2018 TIPS Inc. Unauthorized reproduction of this manual is strictly prohibited and it is illegal to 35-6 reproduce without permission. This manual is being used during review sessions conducted by PharmacyPrep.

www.Pharmacyprep.com Tablet /Capsule opaquant Tablet polishing Tonicity agents

Pharmaceutical Excipients

Used to render a capsule of a tablet coating opaque. May be used alone or in combination with colorant Used to impart an attractive sheen to coated tablet Used to prepare isotonic solutions in parenteral, ophthalmic and irrigation solutions.

Titanium oxide Carnauba wax White wax Sodium chloride and dextrose.

Filler/ diluents

Filler functions: Increases the bulk volume so that the final product has the proper volume for patient handling. Filler requirements: inert, compatible, non-hydroscopic, soluble, cheap, compactable, and tasteful. Fillers: Lactose, sucrose, glucose, mannitol, sorbitol, calcium phosphate, calcium carbonate, and cellulose.

Binder

Binders cause the adhesion of the powdered drug and inactive ingredients. Dry powder added to the mixture prior to the wet granulation process solution that is used in the wet granulation process. Binder types. Wet/Solution Binders. Gelatine, cellulose, cellulose derivatives, polyvinyl pyrrolidone (PVP), starch, sucrose, and polyethylene glycol. Dry Binders: Cellulose, methyl cellulose, polyvinyl pyrrolidone, polyethylene glycol, and starch. Disintegrant are used to ensure that when tablets are in contact with water, they are rapidly breaking into smaller fragments, facilitating their dissolution. Disintegrates facilitate water uptake, rupture the tablets. Disintegrate: Starch, cellulose, crosses linked polyvinyl pyrrolidone, sodium starch glycolate, sodium carboxymethyl cellulose. To improve the "flow" ability of the powder or granules or both. Example Corn starch, silica derivatives (silicon dioxide colloidal), and talc. Glidants properties are measured by “Angle of repose” and X-ray photo electron. To ensure that tablet formation and ejection can occur with "low friction" between the solid and the die wall. Example polyethylene glycols, stearic acid, stearic acid salts (calcium, zinc and magnesium stearate). It is used to "reduce the adhesion" between the powder (granules) and the punch faces and thus prevent tablet sticking to the punches. Example talc, and starch. Limited fluid sorbing in dry state.

Disintegrant

Glidants Lubricant Antiadherent Sorbent

Tips 1 4 7

Anti-adherent lubricant stearic acid

improve flow ability of granules 10 sorbic acid ester

2. 5.

Polysorbate Water for injection USP

3. 6.

alkalizing agent Anionic surfactant

8.

ascorbic acid

9.

Sodium bisulfate

11.

hydrofluoroalkenes HFA

Copyright © 2000-2018 TIPS Inc. Unauthorized reproduction of this manual is strictly prohibited and it is illegal to 35-7 reproduce without permission. This manual is being used during review sessions conducted by PharmacyPrep.

www.Pharmacyprep.com · · · · · · · · · ·

Pharmaceutical Excipients

Triethanol amine is? ( ) Prevent sticking in die wall is referred as? ( ) Glidant is? ( ) Example of water soluble antioxidants are? ( ) Sodium lauryl sulfate is? ( ) The type of sterile water used in parenteral preparation in manufacturing? ( In aerosol propellant action is given by? ( ) Magnesium stearate is? ( ) Tween is? ( ) Span is? ( )

)

Copyright © 2000-2018 TIPS Inc. Unauthorized reproduction of this manual is strictly prohibited and it is illegal to 35-8 reproduce without permission. This manual is being used during review sessions conducted by PharmacyPrep.

www.Pharmacyprep.com

Rheology

36 Rheology Rheology is study of deformation and flow of matter. Newtonian Fluids · The rate of shear (flow) should be the directly proportional to the shearing stress. · The reciprocal of viscosity is defined as fluidity. · The units of viscosity are poise. (gm/cm sec).

Questions Alerts! Common questions in pharmacy exam is to ask! · Thixotropy systems (gel to solution) · Antithixotropy systems (solutions to gel)

Non-Newtonian Fluids The fluids that do not obey the Newton’s law are described as non-Newtonian fluids. Non-Newtonian flow is characterized into three types: plastic, pseudo plastic and dilatants. Plastic Flow · The substance that exhibits plastic flow and does not pass through the origin, it normally intersects the shearing stress axis. · The point at which it intersects the shearing stress axis is known as yield value. · Plastic flow is also known as Bingham bodies. · Plastic flow materials. They do not start to flow until the applied shearing stress equals the yield value. · At stress below the yield value, material act as an elastic material. Pseudo plastic flow · Most of the pharmaceutical follow pseudo plastic flow. Pseudo plastic flow also known as shear thinning system. · Flow begins at the origin. · The viscosity of material decreases with increase rate of shear force.

Copyright © 2000-2018 TIPS Inc. Unauthorized reproduction of this manual is strictly prohibited and it is illegal to 36-1 reproduce without permission. This manual is being used during review sessions conducted by PharmacyPrep.

www.Pharmacyprep.com

Rheology

It is thixotropy. Examples suspending agents such as ethyl cellulose, carboxymethylcellulose and its derivatives. Thixotropy systems: Two types of systems associated with thixotropy. Products that have thixotropy with shear stress, decrease in viscosity and increase flow ability. · Pseudo plastic flow · Plastic systems

·

Thixotropy occurs when transformation of gel to solution. Thixotropy is used in formulation of suspensions dosage form where system remains gel form upon resting and by application of shear stress this can be converted to solution form. Gel-------------------->Solution ↓ Viscosity ↑ Flow Dilatants flow · Also known as shear thickening systems, it is just opposite to pseudo plastic flow. · The increase in rate of shear force normally increases resistant to flow. · Normally suspension with high percentage of dispersed solid particles does follow dilatants flow. · Examples are suspension containing high concentration of small-deflocculated particles. Anti-thixotropy systems (RHEOPEXY): Products that exhibit opposite action of thixotropy are referred to as antithixotropy. Anti-thixotropy occurs when solutions to gel transformation. Example dilatants flow. · Upon shear stress decrease flow and increase viscosity. · Increase in viscosity thereby decrease flow rate. · Example products fusidic acid ophthalmic and timolol ophthalmic drops. or Excessive suspending agent. Solution ----------------> Gel ↑Viscosity ↓Flow THIXOTROPY Pseudo plastic flow (shear thinning) Gel-------------------->Solution ↓ Viscosity ↑ Flow ↓ resistance to flow

ANTI-THIXOTROPY Dilatant flow (shear thickening) Solution ----------------> Gel ↑Viscosity ↓Flow ↑ resistance to flow

Tips 1.

Bentonite at high conc

2.

4. 7.

increase flow decrease flow

5. 8.

10

Solution to gel

is increase viscosity with increase shear stress decrease viscosity thixotropy increases flow and decrease viscosity

3. dilatants flow 6. antithixotropy 9. Gel to solution

Copyright © 2000-2018 TIPS Inc. Unauthorized reproduction of this manual is strictly prohibited and it is illegal to 36-2 reproduce without permission. This manual is being used during review sessions conducted by PharmacyPrep.

www.Pharmacyprep.com · · · · · · · ·

Rheology

Term that referred to non Newtonian flow, increase force will increase difficulty in suspension flow? ( Pseudo plastic flow ( ) Thixotropy systems ( ) Anti-thixotropy systems ( ) Dilatants flow ( ) Rheopexy is? ( ) Agents to prepare dilatants flow ( ) Shear thickening is? ( )

)

Copyright © 2000-2018 TIPS Inc. Unauthorized reproduction of this manual is strictly prohibited and it is illegal to 36-3 reproduce without permission. This manual is being used during review sessions conducted by PharmacyPrep.

www.Pharmacyprep.com

Rheology

Copyright © 2000-2018 TIPS Inc. Unauthorized reproduction of this manual is strictly prohibited and it is illegal to 36-4 reproduce without permission. This manual is being used during review sessions conducted by PharmacyPrep.

Pharmacyprep.com

Pharmaceutical Dosage Forms

37 Pharmaceutical Dosage Forms Questions Alerts! Common questions in pharmacy exam is to ask! · Tablet manufacturing methods (for ASA) and problems · Soft gelatin and hard gelatin capsules · Methods of powdering (levigation, trituration, pulverisation) · Suppository calculation · Suspending agents and flocculating agents · HLB for w/o and o/w

Solid Dosage

Tablets Pills Caplets Lozenges

Powder Capsules Soft gelatine shell Hard shell

Coarse Dispersion Dosage

Solution Dosage

Internal Use Suspensions Gels Emulsions Lotions Liniments Creams Ointments Vitamin drops

Miscellaneous

Inhalants Aerosols

External Use

Suppository Pellet/Bead

Copyright © 2000-2018 TIPS Inc. Unauthorized reproduction of this manual is strictly prohibited and it is illegal to 37-1 reproduce without permission. This manual is being used during review sessions conducted by PharmacyPrep.

Pharmacyprep.com

Pharmaceutical Dosage Forms

Solid Dosage Form Tablets

Most commonly known dosage form is the tablet.

Advantages

Accurate dosage Lesser manufacturing cost Easy to pack and ship, Easy to identify, Easy to swallow Appropriate for special-release forms Best for large-scale production Most stable of all oral dosage forms Tampered proof Some drugs are hard to compress into tablet Some drugs may be difficult to formulate to have adequate bioavailability. Some drugs has foul odour or disgusting taste, so they are preferably done in forms of capsules.

Disadvantages

Methods of tablet preparation. There are 3 methods of tablets preparation

Question Alerts! 1) What method is NOT suitable for ASA tablet manufacturing? wet granulation 2) Direct compression is used for chemical like potassium chloride 3) Wurster Process is used for coating tablets

Wet granulation method, Dry granulation method and direct compressions method. Wet granulation

This is a widely employed method for the production of compressed tablets. This method NOT suitable for moisture sensitive drugs, like ASA, this drug can undergo hydrolysis in moisture.

Dry granulation

In the dry granulation method the granulation is formed by compacting large masses of the mixture and subsequently crushing and sizing these pieces into smaller granules. By this method either the active ingredient or the diluents must have cohesive properties in order for the large masses to be formed. Some granular chemicals like potassium chloride and methenamine possess free flowing as well as cohesive properties that enables them to compress directly in a tablet machine without need of either wet or dry granulation. Vehicle should be compressible and have good flow. Example dried lactose, mannitol, and starch.

Direct compression

Important Concept! 1) Poorly manufactured tablets may have small “Pinholes” on the surface which occurs when the tablet powder stick to punch face-picking 2) Air entrapment can causes? Lamination 3) Excessive moisture can cause? sticking 4) Too much pressure or excessive pressure in punching may cause? Capping

Copyright © 2000-2018 TIPS Inc. Unauthorized reproduction of this manual is strictly prohibited and it is illegal to 37-2 reproduce without permission. This manual is being used during review sessions conducted by PharmacyPrep.

Pharmacyprep.com

Pharmaceutical Dosage Forms

Problems in tablet manufacturing Capping The partial or complete separation of the top or bottom crown from the main body of the tablet. Too much pressure or excessive pressure in punching. Lamination Separation of tablet into two or more distinct layers. Air entrapment results from excess powders which traps air in the tablet, deep marking or tablet punches. Warm or imperfect punches. Too much pressure. Moist and soft granulation or unsuitable formula. Picking The removal of the surface material of tablet laid a punch. Sticking Mottling

An adhesion of tablet material to a die. Picking and sticking results from or these problems are caused by excessive moisture or the inclusion of substance with low melting temperature in the formulation. The uneven distribution of colour. Degradation of active ingredients can give rise to mottling.

Evaluation Tests for tablet (quality control) contain parameter: general appearance, weight variation, disintegration test, dissolution test, friability, hardness and thickness. Hardness Measures the degree of force required to break a tablet and also indicates tensile strength of tablet. Hardness of tablet greatly effects dissolution and disintegration. Thickness

Measured by capillary scale (Vernier calipers)

Friability

Ability of the tablet to withstand abrasion in packing, handling and shipping which is defined as loss in weight of tablet due to chipping or fragmentation in the form of fine particles. Tumbler method is used for testing friability.

Weight Measuring weight of tablet. USP methods include Friability, dissolution, disintegration, hardness and weight variation. Thickness is unofficial test. (Thickness of tablet is inversely proportional to hardness i.e. increase hardness decrease the thickness. USP OFFICIAL TESTS Weight variation Disintegration Dissolution Drug content

USP UN-OFFICIAL TESTS Hardness Friability Thickness

Capsules A solid dosage form in which medicinal Question Alerts! or inert substances are put inside a 1) Uses of soft gelatine and hard gelatine capsules? small gelatin shell. 2) Plasticizer used in capsule shell? Capsule shell sizes (5 to 000). 000 is 3) You have to fill 500 mg powder in capsules. Choose? the largest size of capsule, its capacity is 600 mg. The smallest capsule size is 5 is 30 mg. Manufacturer also makes available number 10, 11, and 12 for veterinary use. The gelatin shell dissolved in 10 to 20 minutes after ingestion.

Copyright © 2000-2018 TIPS Inc. Unauthorized reproduction of this manual is strictly prohibited and it is illegal to 37-3 reproduce without permission. This manual is being used during review sessions conducted by PharmacyPrep.

Pharmacyprep.com CAP # 5 000

Pharmaceutical Dosage Forms

CONTENT 30 mg 600 mg

Capsule comes in two types. Soft gelatine shell manufactured in one piece with drug usually in liquid form inside the shell, e.g. fat-soluble vitamins A and E, Procardia (nifedipine), etc. soft-shell capsule are made from gelatine to which glycerin or some other polyol, such as sorbitol or propylene glycol has been added as plasticizer. Spherical or ovoid capsules are sometimes called pearl or globules. Soft gelatin capsules: Made up of gelatin shell, glycerin or polyhydric alcohol are added to make the shell elastic and plastic-like. It also contains preservatives to prevent microbe’s growth. Advantages of capsule dosage form are good for drugs with objectionable taste or odour and easy to swallow. Hard gelatine shell: The hard-shell gelatin capsules are made from the mixture of gelatin, sugar and water, with without suitable coloring agent. Hard gel caps contain powder and cannot fill liquid. Sulfur dioxide is used as a preservative. Capsules are made opaque with titanium dioxide. Hard capsule are available in variety of sizes and designated by numbers 000 to 5. Manufacturer also makes available number 10, 11, and 12 for veterinary use.

Powders Powders generally range from 0.1 to 10 micron (0.1mM to 10 mM) in size. The screen size indicates the number of openings in the mesh screen per inch. For example, a # 40 sieve has 40 openings per inch in the screen mesh. Particles that can sift through that mesh are said to be "40 mesh" size. Very small particles (below 1 mM) posses high surface free energy that results in absolute solubility. Higher the mesh size, smaller the particle. Advantages Flexibility in compounding Good chemical stability Rapid dispersion of ingredient Disadvantages Time consuming in preparing powder The dose is inaccurate Unsuitable for hygroscopic, deliquescent drugs and unpleasant tasting. Comminution. The process of reducing the particle size have 3 methods. Particle size

Trituration

The continuous rubbing or grinding of the powder in a mortar with a pestle. This method is used when working with hard, fractural powders.

Levigation

This method is also used to reduce the particle size of insoluble materials when compounding ointments and suspensions. Reduces the particle size by triturating it in a mortar or spatulating it on an ointment slab or pad with a small amount of a liquid in which the solid is not soluble. The solvent should be somewhat viscous such as Mineral oil or glycerin.

Pulverization

By intervention it is the process of reducing a substance to a fine powder by means of utilizing solvent, which can evaporate easily. Used with hard crystalline powders that do not crush or triturate easily, or gummy-type substances.

Copyright © 2000-2018 TIPS Inc. Unauthorized reproduction of this manual is strictly prohibited and it is illegal to 37-4 reproduce without permission. This manual is being used during review sessions conducted by PharmacyPrep.

Pharmacyprep.com Mechanical Comminution

Pharmaceutical Dosage Forms

Ball or pebble mills, wiley mill, hammer mill fluid energy mills.

Powder mixing methods Spatulation Small amounts of powder, having the same range of particle sized and densities, may be conveniently mixed on a sheet or paper or tile with spatula. Trituration

Powders may be mixed in a mortar by gentle trituration with a pestle.

Sifting

Where free-flowing, light powders are desired, the ingredients may be brushed through a sieve. Ordinary household sifters may be used in sifting pharmaceutical powders.

Tumbling

When simple mixing of powders is desired without reduction in particle size.

Geometric dilution

Dissolving in small proportions.

Question Watch? Geometric dilution?

Special Powders Hygroscopic A substance that absorbs moisture from the air is termed hygroscopic. (Absorb H 2 O). Deliquescent Hygroscopic substances, which absorb moisture from the air to the extent that they liquefy by partially, or wholly forming solution are termed deliquescent. (Absorption H 2 O). Efflorescent

Crystalline substances, which become powdery and liberate their water of crystallization are said to be efflorescent. (Liberate H 2 O).

Effervescent salts

Granules or powders consisting of sodium bicarbonate, a suitable organic or inorganic acid, and medicinal ingredients are known as effervescent salts. In the presence of water, the acid and base react to liberate carbon dioxide, thereby producing effervescence. (Liberate CO 2 ). Examples of effervescent salts include Alka Seltzer (ASA with NaHCO 3 ), and Calcitral.

Eutectic mixtures

A eutectic mixture is defined as that proportion of components, which will give the lowest melting point. Example menthol, and camphor. Compound A 50oC and compound B 80oC after combining mixture 20oC.

Suppositories Types of suppositories Rectal

Vaginal

Urethral

Solid or semi solid dosage form intended to be inserted into body orifice. The most common method of suppository preparation is fusion Method. Bullet-like shape to moves it inward when rectum contracts, 2 g adult, children smaller than adult. Tapered shape and rectal suppository can provide systemic medication Ova shape and 5 g. Have local absorption, but systemic absorption may occur. Variable size, cylindrical shape, often contain poly ethylene glycol (PEG), water soluble base Long and tapered. Has local effect.

Copyright © 2000-2018 TIPS Inc. Unauthorized reproduction of this manual is strictly prohibited and it is illegal to 37-5 reproduce without permission. This manual is being used during review sessions conducted by PharmacyPrep.

Pharmacyprep.com Suppository Bases

Pharmaceutical Dosage Forms

Criteria: It should have a narrow or sharp melting range. It should yield a clear melt just below body temperature or it should dissolve or disintegrate readily in the cavity fluid. It should be inert and compatible with wide variety of drugs. It should be non-irritating and non-sensitizing

Question Alerts! 1) Most commercial vaginal suppositories use a base of polyethylene glycol o/w. 2) An excellent choice of diluents for a compressed vaginal tablet would be Lactose. 3) Melting point temperature of suppository base?

Types of Suppository Bases Lipid Cocoa butter USP (Theobroma oil, cocoa butter). At 34 to 35oC, it melts to produce a thin, soluble oily liquid. It is a good base for rectal suppositories, but less ideal for vaginal suppositories. Witepsol bases (Lauric acid): Do not exhibit polymorphism. High melting witepsol can be mixed low melting witepsol to produce 34 to 44oC. Contain emulsifiers. Q (surfactants) in suppositories enhance rate of absorption. Wecobee bases (coconut oil): These bases are derived from coconut oil. The incorporation of glyceryl monostearate and propylene glycol monosterate them emulsifiable. Water soluble

Polymer of ethylene oxide and water, molecular weight range 400-6000 Polyethylene glycol polymers e.g. carbowaxes. Usually anhydrous, water soluble and washable, non-greasy, non-occlusive lipid free. Glycerin suppositories USP consist of 91% glycerin gelled with 9% sodium stearate. They are available as adult and infant suppositories to evacuate the lower bowel.

Displacement value. Displacement Value = number of grams of salt to replace one gram of cocoa butter. Example. ZnO displaces cocoa butter. Two parts of ZnO displaces one part of cocoa butter. ZnO 1 part

ZnO 1 part

Cocoa butter 1part

Preparation of suppositories. To prepare 15 suppositories, each containing 300 mg ASA are required what amount of cocoa butter? Density factor (displacement value) of ASA is 1.1. Each, mold is 2 ml. Solution. 16 x 2 ml = 32 ml 16 x 300 mg = 4.8 g ASA 4.8 g/1.1 g/ml = 4.4 ml 32ml - 4.4 ml = 27.6 ml of cocoa butter Copyright © 2000-2018 TIPS Inc. Unauthorized reproduction of this manual is strictly prohibited and it is illegal to 37-6 reproduce without permission. This manual is being used during review sessions conducted by PharmacyPrep.

Pharmacyprep.com

Pharmaceutical Dosage Forms

Suppose 12 suppositories, each containing 300 mg ASA, are required. Given the density factor of ASA is 1.1. What is amount of cocoa butter required for the preparation? Each mold is 2 ml. a. 18 g B. 20 g C. 20. 73 g C. 20.04 g Ans. C

Suspension Suspension is a two-phase system in which the internal or dispersed phase is solid external or continuous phase is liquid. Physical and chemical properties of suspension Colloidal suspension. A suspension containing particles between 1 nm to 0.5 µm in size. Coarse suspension. The particle size is between 1 to 100 µm, the suspension. Purpose. Sustaining effect it necessitates drug dissolution prior to absorption. Stability. Drug degradation occurs more slowly with suspension compared to solution form. It improves the taste. Basic solubility, it provides alternative solvents. Question Alerts! 1) Stoke's law relation of the rate of sedimentation Ideal Suspensions with particle size, and viscosity. · A uniform particle size-uniformly distributes 2) Affects of flocculating agents and deflocculation? · Suspension that have no particle, particle interaction · Suspension that have no sedimentation. LARGE PARTICLE SIZE - SEDIMENTATION Factors for an ideal suspension · Sedimentation HIGH VEHICLE DENSITY ¯ SEDIMETATION · Suspending Agent - VISCOSITY · Flocculating Agent FLOCULATING AGENTS

· · ·

Sedimentation: The Stoke’s law can express the relationship of the rate of sedimentation with various parameters. V = 2r2 (P1 – P 2 )g 9n V = velocity of sedimentation in cm/sec P 1 = density of disperse phase in g/cm3 P 2 = density of dispersion medium in g/cm3 R = radius of the particles in cm n = viscosity of dispersion medium g = gravity acceleration 980.7 cm/sec2 The rate of sedimentation is independent of the lipophilic nature of particles. Summary of Stokes equation. The velocity of sedimentation of particles in a suspension can be determined by using the Stoke's equation.

Copyright © 2000-2018 TIPS Inc. Unauthorized reproduction of this manual is strictly prohibited and it is illegal to 37-7 reproduce without permission. This manual is being used during review sessions conducted by PharmacyPrep.

Pharmacyprep.com

Pharmaceutical Dosage Forms

Particle size: Larger particles will settle faster at the bottom of the container and too fine particles will easily form hard cake at the bottom of the container. Larger the particle sizeà increase in sedimentation. In most good pharmaceutical suspension, the particle diameter is between 1 and 50 mm. Particle size reduction is done by dry milling method. Density of particles: The settling decreases as (p 1 –p 2 ) approaches zero. Density of the vehicle: Adding the following substances either alone or in combination can increase the density of the vehicle of a suspension polyethylene glycol, polyvinyl pyrolidone, glycerine, sorbitol, and sugar. Increase vehicle density à decreases sedimentation. However, the density of the dispersion medium cannot be altered thereby density of particle is changed. Viscosity of the vehicle: Adding the suspending agents or viscosity enhancers increases the viscosity of a suspension. Q Increase in viscosity of vehicleà decrease in sedimentation rate. Suspending agents (viscosity enhancers): Natural hydrocolloids; acacia, tragacanth, alginic acid, locust bean gum, guar gum, gelatin and cellulose. Semi synthetic hydrocolloids. Methylcellulose, sodium carboxymethylcellulose, and carbomen. Synthetic hydrocolloids. Carbopol. Clays. Bentonite, veegum Problems in suspension. Sedimentation, and caking Flocculating Agents: The addition of flocculating agents to enhance particle "dispersability". Flocculating agents are electrolytes, which carry an electrical charge opposite that of the net zeta potential of the suspended particles. The addition of the flocculating agent, at some critical concentration, negates the surface charge on the suspended particles and allows the formation of floccules or clusters as particles are held loosely together by weak Van der Waals forces. The particles are linked together only loosely. They will not cake and may be easily re-dispersed by shaking the suspension. Floccules have approximately the same size particles. Examples of flocculating agents are potassium stearate, potassium laurate, acryl polymers and surfactants. Addition of viscosity enhancers to reduce sedimentation rate in the flocculated suspension. Higher volume of sedimentation. NO clear boundary is seen when the particles settle. COMPARISON BETWEEK DEFLOCCULATED AND FLOCCULATED SYSTEMS DEFLOCCULATED SYSTEM FLOCCULATED SYSTEM Clear boundaries No clear boundaries Small volume of sedimentation Higher dispersion volume of sedimentation - SEDIMENTATION ¯ SEDMENTATION ¯ DISPERSION - DISPERSION ¯ VISCOSITY Not ideal suspension Ideal for suspension The hard cake cannot be redispersed and The sediment is easy to redisperse and Easily Not easy to disperse dispersible Pleasant appearance, because of uniform Somewhat unsightly sediment. dispersion of particles. Supernatant remains cloudy. Supernatant is clear Copyright © 2000-2018 TIPS Inc. Unauthorized reproduction of this manual is strictly prohibited and it is illegal to 37-8 reproduce without permission. This manual is being used during review sessions conducted by PharmacyPrep.

Pharmacyprep.com

Pharmaceutical Dosage Forms

Particles exist as separate entities Rate of sedimentation is slow, as the size of particles are small. The sedimentation is closely packed and form a hard cake. Bioavailability is higher due to large specific surface area.

Particles form loose aggregates Rate is high, as flocculation are the collection of smaller particles having a larger size. Sediment is a loosely packed network and hard cake cannot form. Bioavailability is comparatively less due to small specific surface area.

Emulsions Emulsion is a two phase system consisting of at least two immiscible liquids. Internal or discontinuous phase. The dispersed liquid, external or continuous phase is the dispersion medium. Emulsion is classified as five different categories: Water in Oil (W/O): Oil is a continuous phase and water is a disperse phase, i.e., lotions and liniments. Example. Lotions and liniments. Oil in Water (O/W): Water is a continuous phase and oil is a dispersed phase, i.e., most of the oral emulsions to unmask the oily taste of a medication. Microemulsion. Unlike emulsions, microemulsion is transparent with a small particle size. It is believed to be thermodynamically unstable. The particle size of microemulsion lies between 10 to 200 nm. It is generally used for the solubilization of the drug in pharmaceutical dosage form. Nanoparticles. Micro-emulsion- droplets size range 0.01 to 0.1 mm, the particle size of this kind of emulsion is limited to nanograms. They are useful for the preparation of globulins and toxoids. Tetanus toxoid and human immunoglobulin G are examples of nanoparticles emulsion. * Multiple emulsions: Water in Oil in ware (W/O/W), Oil in Water in Oil (O/W/O). The w/o/w emulsions are generally more preferable for preparation of various pharmaceutical dosage forms. They are used to prolong the duration of action of various drugs, to localize drug in the body and to prepare cosmetics Purpose of emulsion · Increase drug solubility · Increase drug stability · Prolonged drug action · Improve taste · Improved appearance

Question Alerts? 1) Emulsifying agents such as Tween and Span are? Surfactant 2) HLB is used to classify surfactant 3) Sodium lauryl sulfate is? Anionic surfactant 4) Creaming in emulsions is due to? Increased in droplet size. (Reversible separation a layer of emulsified particle). 5) The process of dispersed oil globule in aqueous vehicle called? Emulsion 6) Liquid droplets dispersed in another immiscible liquid is called? Emulsion 7) Irreversible phase separation in emulsion may cause? Cracking, or breaking 8) What is NOT a problem of emulsion? A) Cracking B) Creaming C) Aggregation D) Coalescence E) Flocculation

Stability of emulsion: Protect emulsions against the extremes of cold and heat. Emulsions may be adversely affected by microbial contamination.

Copyright © 2000-2018 TIPS Inc. Unauthorized reproduction of this manual is strictly prohibited and it is illegal to 37-9 reproduce without permission. This manual is being used during review sessions conducted by PharmacyPrep.

Pharmacyprep.com

Pharmaceutical Dosage Forms

Emulsifying agents (surfactants) are categorized as: · · ·

Anionic agent: Sodium lauryl sulphate. Cationic agent: Benzalkonium chloride. Non-ionic: Tween and Spans. Tween is polysorbate and span is sorbital esters. Anionic agent surfactants Sodium lauryl sulphate

Non-ionic surfactants Tween (POLYSORBATE) Span (SORBITAL ESTER)

Cationic surfactants Benzalkonium chloride

Problems in emulsion categories. Creaming, breaking (cracking) phase inversion and coalescence. Creaming: It occurs due to flocculation of globules of the internal phase. It is not a potential cause of instability of emulsion, however, occurrence of creaming is a potential step towards complete breaking of emulsion. The rate of creaming can be expressed by Stoke’s law. V = d2 (Ps – Po) x g 18n d = diameter of particle in cm Ps = density of disperse phase Po = density of dispersion medium g = gravitational force n = viscosity of medium Breaking. Breaking generally results in separation of the internal and external phase. It cannot be reformulated. Hydrophilic lipophilic balance (HLB) measures the surfactants mixability in water and lipids. Classification of surfactants based on HLB values and uses. 0-3----------------------- Antifoaming Question Alert! 4-6-- .................w/o emulsifying 1) TWeen HLB? O/W 7-9----------------------- wetting 2) Span HLB? W/O 8-18------------- - o/w emulsifying 13-15-------------------- detergent 10-18-------------------- solubilizing agent Combinations of emulsifiers can produce more stable emulsions than using a single emulsifier with the same HLB number. The HLB value of a combination of emulsifiers can be calculated as follows.

Example. What is the HLB value of a surfactant system composed of 20 g span 20 (HLB = 8.6) and 5 g Tween 21 (HLB = 13.3)?

Copyright © 2000-2018 TIPS Inc. Unauthorized reproduction of this manual is strictly prohibited and it is illegal to 37-10 reproduce without permission. This manual is being used during review sessions conducted by PharmacyPrep.

Pharmacyprep.com

Pharmaceutical Dosage Forms

Creams

Cold cream Preservatives used Barrier creams

Cream examples

Ointments Mechanism

Therapeutic use

Advantages Disadvantage Storage Ointment Preparation

Cream (W/O: A semisolid emulsion of oil, e.g. lanolin or petrolatum, and water. These are either water-miscible or readily washed-off, or oily and not so easily washed off. Cold Cream (W/O) emulsion and vanishing cream (O/W) emulsion. Chlorocresol and Hydroxybenzoates, both of which may cause skin allergies. Barrier creams often contain water-repellent substances such as dimethicone or other silicones. They give protection against irritation or repeated hydration and is useful in the treatment of Question Alerts! napkin rash and bedsores, 1) Cold cream is? etc. 2) Vanishing cream? 3) Water washable cream is? Creams should be stored in cool place and supplied in well closed containers that prevent evaporation and contamination of the contents. Eumovate, Elocom, Tridesilon Cream. Substances such as precipitated sulphur, salicylic acid, menthol and camphor, hydrocortisone powder, hydroquinone, to mention a few, may be incorporated into creams and/or ointment bases extemporaneously.

Ointments are semisolid substances that are greasy, normally anhydrous, and insoluble in water, and intended for external use. The most commonly used ointment bases consist of soft paraffin or a combination of soft paraffin with liquid paraffin and hard paraffin. Due to their anhydrous nature ointments do not require any preservatives. They are typically used as emollient that makes skin more pliable Protective barriers prevent contact to skin from harmful substances. It moisturizes, more occlusive than creams and forming a protective film over the skin. The occlusive effect tends to prolong and enhance drug penetration. They are messy to use. They should be kept in well-closed container that prevents evaporation and contamination in a temperature not exceeding 25oC. The material making up the container should be resistant to absorption or diffusion of the contents. Levigation to reduce particle size, most commonly used method for pharmaceutical compounding. Levigating agents. Levigating agents used for wet & disperse powder Main agents: mineral oil, cottonseed oil, and castor oil. Glycerin: (propylene glycol PEG 400). Surfactants: polysorbate 80 (Tween 80). Not all surfactants are compatible Fusion method: used method if the base contains solid that has higher melting

Copyright © 2000-2018 TIPS Inc. Unauthorized reproduction of this manual is strictly prohibited and it is illegal to 37-11 reproduce without permission. This manual is being used during review sessions conducted by PharmacyPrep.

Pharmacyprep.com

Pharmaceutical Dosage Forms

point. Some examples ointments are generally used for treatment of hemorrhoids. Preparations H, Anusol, Anusol HC and Anugesic. Water and lipid soluble base. Important Concept! Poly ethylene glycol (PEG) 1) Occlusive bases effectively prevent Lipid soluble base Lanoline water evaporation from the skin. Tends to Occlusive bases: Petrolatum prolong and enhance drug absorption.

Ointment bases

1) Hydrophilic Petrolatum, USP · Contains cholesterol, stearyl alcohol, white wax and white petrolatum · Forms water-in-oil emulsions Cholesterol is the emulsifying agent CREAM o/w vanishing Washable Good for dry skin

w/o Cold cream Washable Good for dry skin

OINTMENT Greasy, insoluble anhydrous Not washable

GEL NO WATER GOOD FOR OILY SKIN

Pastes Mechanism Therapeutic use Examples

Pastes are stiff preparations containing a high proportion of finely powdered solids such as zinc oxide and starch. They are less occlusive than ointments and can be used in subacute, lichenified, or excoriated skin conditions. Due to the stiff nature, they can be applied accurately to a particular lesion such as chronic eczema and psoriasis, and are therefore useful for the local application of irritating drugs. Anthralin OTC paste 0.025% and 0.2% for seborrhea and psoriasis. Benzoyl peroxide paste for acne.

Gels Mechanism Therapeutic use

Examples

Gels are semisolids or solids prepared from high molecular weight polymers in an aqueous or alcoholic base. They are easy to apply and wash off (example hairy places). Gels are useful for promoting wound granulation (example Actovegin Jelly), in treatment of acne (example Panoxyl gel) and Scalp psoriasis (example Synalar gel mix). Due to their drying effect and especially the alcoholic ones, they may cause irritation to the skin. Acne gel preparations. Made of synthetic polymers such as carbo vinyl and polyoxyethylene laurel ether in hydroalcoholic liquids are used as bases for benzoyl peroxide in the treatment of acne. Topical gels. Tretinoin or tretinoin + clindamycin (clindagel), tretinoin + erythromycin gel should be stored in refrigerator and protect from light.

Copyright © 2000-2018 TIPS Inc. Unauthorized reproduction of this manual is strictly prohibited and it is illegal to 37-12 reproduce without permission. This manual is being used during review sessions conducted by PharmacyPrep.

Pharmacyprep.com

Pharmaceutical Dosage Forms

Panoxyl®, Benazagel®-10, Hormone replacement (Androgel®, Estrogel®). Diffusimax is a commercially prepared pluronic gel that easily penetrates the skin and is used as a vehicle to apply such drugs as Diclofenac sodium to ease muscle pains. Hydrophilic Petrolatum, USP · · ·

Contains cholesterol, stearyl alcohol, white wax and white petrolatum Forms water-in-oil emulsions Cholesterol is the emulsifying agent

Lotions Mechanism Therapeutic use

Examples Counseling

Lotions are aqueous solutions or suspensions that cool diffusely inflamed unbroken skin. Finely powdered drugs are suspended in a thin, semi-solid base and applied to the skin. Cool skin by evaporation and should be applied frequently. Lotions are also used to apply drugs to the skin when only a thin layer of the preparation is intended to be applied over a large surface area. Shake lotions (e.g. calamine lotion) that contain insoluble powders are applied to less acute, scabbed, and dry lesions. In addition to cooling, they leave a deposit of inert powder on the skin surface. Benoxyl, valisone and scalp lotion, calamine lotion (Zinc oxide + Ferric oxide). Lotions and suspensions require a ‘Shake Well” label and if intended for topical use. An “External use only” label.

Liquid Dosage Form Spirits

Spirits or essences are alcoholic or hydroalcoholic solutions of volatile substances prepared usually by simple solutions or by admixture of the ingredients (contain 50% to 90% alcohol). Spirits require storage in tight, light-resistant containers to prevent loss by evaporation and to limit oxidative changes. Some of spirits are medicinal but mostly are used as flavouring agents. Tinctures: Natural products or herbal extracts taken orally. They are extracted in alcohol. Tinctures are alcoholic or hydroalcoholic solutions prepared by mixing chemical substances like iodine. The alcohol content of the official tinctures varies from 10% to 21% with opium tincture USP; 74% to 80% with benzoin tincture USP. Glycerin may be added to hydroalcoholic solvent to increase solubility of the active content and reduce precipitation during storage. Tincture is categorized as protectant. It is used to protect and toughen skin in the treatment of bedsores, ulcers, cracked nipples, and fissures of lips and anus. Tinctures require storage in tight, light resistant containers, away from direct sunlight and excessive heat (may undergo photochemical change). Topical tinctures: Iodine tincture, compound benzoin tincture and themerosal tincture. Copyright © 2000-2018 TIPS Inc. Unauthorized reproduction of this manual is strictly prohibited and it is illegal to 37-13 reproduce without permission. This manual is being used during review sessions conducted by PharmacyPrep.

Pharmacyprep.com

Pharmaceutical Dosage Forms

Iodine tincture. The iodine tincture is prepared by dissolving 2% of iodine crystals and 2.4% of sodium iodide to an amount of alcohol equal to half the volume of tincture to be prepared. Benzoin tincture Prepared by the maceration in alcohol 10% of benzoin and lesser amounts of aloe, storax, and tolu balsam totalling about 24% of starting material.

Astringents

Common astringents Calamine lotion

Locally applied solutions that precipitate protein. The protein precipitates which forms serve as a protective coat, allowing new tissue to regenerate underneath. It causes constriction and reduces secretions therefore they can be used as astringent. These substances that stop oozing, discharge or bleeding. Zinc oxide used for diaper rash treatment and hemorrhoids. Calamine lotion used for cold sores or fever blister treatment and poison ivy. Burrow's solution (Aluminum acetate) used for otitis externa, and dermatitis treatment. Calamine lotion is mixture of zinc carbonate or zinc oxide colored with ferric oxide.

Collodions Collodions are liquid preparations consisting of a solution of proxylin in a mixture of ether and alcohol. When collodions are painted on the skin and allowed to dry they leave a flexible film over the site of application. Collodions may be used to seal minor cuts and wounds or as a mean of holding a dissolved drug in contact with the skin for a long time. Keep away from fire

Therapeutic use Counselling

Emollients

Therapeutic use

Emollients are derived from animal or vegetable fats or petroleum products, used to soften or protect internal or external body surfaces. Emollients are fats or oils in a two-phase system (one liquid is dispersed in the form of small droplets throughout another liquid). Emollients soften the skin by forming an occlusive oil film on the stratum corneum, thus preventing drying from evaporation from the deeper layers of skin. Emollients are employed as protective and as agents for softening the skin and rendering it more pliable in conditions like dry eczema, ichthyosis and psoriasis. They also serve as vehicles for more active drugs.

Gargles Therapeutic use Monitoring

Gargles are aqueous solutions, usually in concentrated forms intended for use, after dilution, for treatment of infections of oral cavity and throat. A gargle does not, however, act as a protective covering to mucous membranes. In the treatment of mucositis, try to avoid a gargle that contains a high

Copyright © 2000-2018 TIPS Inc. Unauthorized reproduction of this manual is strictly prohibited and it is illegal to 37-14 reproduce without permission. This manual is being used during review sessions conducted by PharmacyPrep.

Pharmacyprep.com

Pharmaceutical Dosage Forms

concentration of alcohol as it may produce irritation. Examples of gargles are thymol gargle, chlorhexidine gargle and difflam gargle. Gargles may contain a drug to relieve sore throat such as Tantum (Benzydamine): commercially prepared mouthwashes or saline solution (0.9% sodium chloride) may be used for stomatitis. Chlorhexidine: used for stomatitis, mucositis, and gingivitis. Caution: stains teeth after excessive use. Use chlorhexidine 30 min before or after use of toothpaste. Chlorhexidine can interact with fluoride and can stain teeth.

Examples

Humectants Mechanism

Humectants promote water retention due to their hygroscopic. They act by being absorbed into the skin and attract water from atmosphere and serve as a reservoir for the stratum corneum. Commonly used humectants are propylene glycol and glycerin.

Liniments Mechanism

Liniments are viscous liquid containing substances possessing analgesic, soothing or stimulating properties when applied on the skin. They are usually made with a base of oil, alcohol, or soap solutions. Example. Methyl salicylate liniment. Liniments should not be applied to broken skin

Syrups (Drug + sugar + water) Sugar in water is syrup. Generally syrup contain sugars solutions, caution diabetic patients and recommend syrup containing artificial sweeteners such as aspartame, in place of sugar. Elixir: Contain alcohol. Elixir are clear, sweetened, hydroalcoholic solutions intended for oral use and usually flavored to enhance palatability. Elixir is prepared for products that are soluble in water and alcohol. Elixir contains less sugar, less viscous than syrup. Rubbing alcohol: Rubbing alcohol contains about 70% of ethyl alcohol by volume, the remainder consisting of water, denaturants with or without color additives and perfume oils and stabilizers. In each 100 mL it must contain not less than 335 mg of sucrose octa acetate of 1.4 mg of denatonium bentoate, a bitter substances that discourage accidental or abusive oral ingestion. It is used as rubefacient externally germicide for instruments and skin cleanser prior to injection. This product is flammable stored in tight container far from fire. Isopropyl rubbing alcohol: Isopropyl rubbing alcohol contains about 70% of ethyl alcohol by volume, the remainder consisting of water. Can be used as disinfectant.

Copyright © 2000-2018 TIPS Inc. Unauthorized reproduction of this manual is strictly prohibited and it is illegal to 37-15 reproduce without permission. This manual is being used during review sessions conducted by PharmacyPrep.

Pharmacyprep.com

Pharmaceutical Dosage Forms

Tips 1

Has low accumulation

4.

disintegrating agents

7

Stokes equation

1 0 1 3 1 6 1 9 2 2 · · · · · · · · · · · · · · · · · · · · · · ·

alcoholic and hydroalcoholic liquid mixtures disintegrating agents small intestine>oral>mucus membrane>stomach mixing of 2 subs. Leads to lowering in melting point, example includes camphor and menthol o/w emulsion

2 . 5 . 8 .

Rate of diffusion

3.

Burrow solution

Pc=octanol water o/w

6.

first pass effect

stability & solubility

9

all oils except mineral oil

11

Tocoferol alpha

anionic surfactant

14

sodium lauryl sulfate

1 2 1 5 1 8 2 1

17 20

stratum corneum glycerin

The definition of tincture is? ( ) Sodium lauryl sulphate is? ( ) High HLB values associated with? ( ) What problem does not occur in suspension? ( ) Fick's law describes? ( ) The rate limiting step in ophthalmic drops ? ( ) Partition coefficient is described as? ( ) Drug with low volume of distribution? ( ) The most commonly used humectants is? ( ) Aluminum acetate is? ( ) Eutectic mixture is? ( ) Oils used for parenteral preparations? ( ) Write the sequence of surface area in GIT ( ) The largest size of capsule is? ( ) The rate limiting step for the topical drugs ( ) Water used in parenteral preparation is? ( ) What is avoided in transdermal patch? ( ) What is an example of anionic surfactant( ) The tablets are disintegrated by? ( ) Relation of sedimentation and particle size can be explained by? ( What type of tocoferol has the strongest antioxidant properties? ( Complexation can modify which two properties in a drug? ( ) An example of biphasic include ( )

)

Sterile Water for injection USP capsule size 000 crystallization

)

Copyright © 2000-2018 TIPS Inc. Unauthorized reproduction of this manual is strictly prohibited and it is illegal to 37-16 reproduce without permission. This manual is being used during review sessions conducted by PharmacyPrep.

www.pharmacyprep.com

Drug delivery Systems

38 Drug Delivery Systems Questions Alerts! Common questions in pharmacy exam is to ask! · Solvents in parenteral preparations like types water, and sterilization methods · Asthma devices in children meter dose inhaler + spacer and nebulizers · Ophthalmic prep preservatives, viscosity enhancers like HPMC Routes of drug administration Route

Site

Oral (PO)

By mouth. Whenever possible, safest and most convenient route

Sublingual (SL) Parenteral Intravenous

Under the tongue. When rapid effect is desired. Other than the gastrointestinal tract (by injection) Administered directly in vein. Aqueous solutions are used. Single small volume (bolus) or large volume slow iv drip infusion. SUSPENSION ARE NOT USED IN IV. Artery. IM are given in deep into skeletal muscles, generally Gluteal or lumbar muscles. Drugs that irritate subcutaneous may be administered by IM. Volume used is 2 to 5 ml. If require more than 5 ml, administered in divided dosages at two different sites. Faster than subcutaneous. Angle of injection is 90°. Needle gauge is 23 (22-25), and length is 1.5 inch. Heart Spine. CHEMOTHERAPY DRUGS ARE NOT USED AS INTRATHECAL.

Intraarterial IM

Intracardiac Instraspinal or intrathecal Intraosseous Intraarticular Inrasynovial Intracutaneous or Intradermal or Subcutaneous

Bone Joint Joint-fluid area Beneath the skin loose subcutaneous (hypodermic) tissue. Subcutaneous Less than 100 mL). Heparin, KCl, lidocaine. Gloss bottles. Nitroglycerin.

Intrapulmonary drugs Intrapulmonary drug delivery devices Components of aerosols

Metered dose inhalers (MDI), diskus, turbuhaler, handihaler and nebulizers.

Formulation of aerosols Inhalation Therapy Metered dose inhalers (MDI)

Solutions, suspensions, emulsions

Dry powder inhalers (DPI) or turbuhaler

Propellants, valves, containers

Deposition of particles in the lungs. Contain suspension or liquid. Suspension containing metered dose inhaler (MDI) should shake before use. Safe propellant hydrofluorocarbons (HFA) or HFC. Aerosol flow rate 30 m/s or 100 km/h. Shake before use. Do not exceed prescribed dose. Good for under 5 year old. Dry powder inhalers also known as turbuhaler No propellant and no need to shake. Requires patient’s own inspiratory effort to form aerosol Powder is delivered only when the patient inhales. Useful in children above 5 years, teenagers & arthritic patients. Can be used open mouth and closed mouth technique.

Copyright © 2000-2018 TIPS Inc. Unauthorized reproduction of this manual is strictly prohibited and it is illegal to 38-3 reproduce without permission. This manual is being used during review sessions conducted by PharmacyPrep.

www.pharmacyprep.com Nebulizers

Example compositions Inhalation aerosol

Topical spray

Vaginal foam

Drug delivery Systems

Turns an aqueous solution of drug into fine mist Drug will be inhaled with normal respiration Medication reaches lower airways more effectively Two types. jet & ultrasonic Jet: Cools during operation, Small aerosol particle size, Less expensive, More noise Ultrasonic: Heats up during operation, Larger aerosol particle, More expensive, Less noise Choice of inhalation therapy Infants -----------------Nebulizer Children < 4 years----Nebulizer and MDI 4 year or above--------DPI/MDI/Spacer 7 years-------DPI/MDI Adults-------------------MDI/DPI Acute episodes-------Nebulizer Drug…………………………. Albuterol (bronchodilator) Surfactant…………………… Oleic acid, and propylene Propellant…………………… HFC (HFA) Co-solvent………………….. None Type of output……………… Dry mist Drug…………………………. Miconazole Surfactant……………………. Glycol Propellant……………………..Propane or n-butane Co-solvent…………………… Isopropyl alcohol Type of Drug output……Wet mist Drug……………………………Nonoxenol-9 (Contraceptive) Surfactant……………………. Triethanolamine Propellant…………………….. n-butane, and propane Co-solvent……………………..None Type of output……………..… Stable foam

Rectal Dosage forms Rectal route may be preferred over oral route in order to avoid liver by first pass (less first pass). · Safe route if patient is vomiting, unconscious or unable to swallow (mainly applies to rectal). · Often used for local effects e.g. hemorrhoids, local infections. · Suppositories can provide systemic medication (diffuse through mucosa and transport the veins and lymph vessels into systemic fluids or tissue. (By-pass liver). Surfactants in suppositories increase drug absorption. Rectal ointments: Ointments are generally used for the treatment of hemorrhoids. Some examples are Preparations H, Anusol, Anusol HC, and Anugesic.

Copyright © 2000-2018 TIPS Inc. Unauthorized reproduction of this manual is strictly prohibited and it is illegal to 38-4 reproduce without permission. This manual is being used during review sessions conducted by PharmacyPrep.

www.pharmacyprep.com

Drug delivery Systems

Rectal suppositories. Suppositories are topical dosage forms designed to soften when placed in the rectum. They may be cylindrical or egg-shaped and are used for local action as laxative. Suppositories should be protected from heat. Some examples Dulcolax or glycerin, analgesic, Anusol, Preparation H. Anti-inflammatory suppositories indomethacin or naproxen. Suppositories are also used for systemic use when the oral route is either impossible or not desirable. Some examples of drugs available for systemic use rectally antiemetics such as Gravol, and Stemetil. Analgesics. Acetaminophen and ASA. Rectal solutions or rectal suspensions (enemas) Enema is administered in a syringe or disposable squeeze bottle with an applicator tip. Some examples: Systemic anti-inflammatory effect: 5-ASA (Salofalk)

Vaginal dosage forms Vaginal dosage forms include tablets, creams, aerosol foams, jellies, solutions and ovules (suppositories) and the sponge. The ovule is shaped differently from that of a rectal suppository-the laxative fleet enema are larger, more oval in shape, and the base is water-soluble. Vaginal preparations: Most commercial vaginal suppositories use a base of polyethylene glycol o/w. An excellent choice of diluents for a compressed vaginal tablet would be lactose. Vaginal products Vaginal dosage forms for contraception: Delfen Foam, and the sponge Protect Aid. Some example of anti-infective: Monistat, Canesten, Flagystatin, douches, and Hormone replacements. Metronidazole (Flagystatin): Partner should be treated same time, it is important to avoid alcohol. Metronidazole indicated for trichomonas associated vaginitis. Miconazole: drug of choice in vaginal candidiasis in pregnancy. Cream is preferable over suppositories and ovules. Cream effective decrease itching associated with vaginitis.

Topical Dosage forms Important concept! Four bases are used: Hydrocarbons, 1) Some products may be used in both the ear and eye like Sofracort. which are greasy to the touch While otic preps have a glycerin or PEG base cannot be used in eye. (petroleum jelly). 2) What type of topical preps requires sterilized preps? Ophthalmic Absorption bases, which are also 3) Ophthalmic ointments have high contact time so high absorption greasy but allow the addition of than drops. liquid usually water or dissolved 4) Percentage of bioavailability from ophthalmic preps is? 5% chemicals in aqueous solution. Water removable bases, which are oil-in-water mixtures, easily removed form the skin with water (Glaxal Base) and water-soluble bases, which can be mixed with substances that will dissolve in water. (Polyethylene glycol bases). Ophthalmic Ointments/Solutions/Suspensions: Copyright © 2000-2018 TIPS Inc. Unauthorized reproduction of this manual is strictly prohibited and it is illegal to 38-5 reproduce without permission. This manual is being used during review sessions conducted by PharmacyPrep.

www.pharmacyprep.com

Drug delivery Systems

Cornea as a barrier to ophthalmic drug absorption. Formulation: Isotonic, sterile and pH additive (should be formulated at a pH equivalent to tear fluid value of 7.4. Ophthalmic ointments are sterile ointments containing antibiotics or substances to relieve dry eyes. Only sterile products are used in the eye. The advantage of using an ointment rather than a solution is that the ointment allows increased contact time in the eye. Ophthalmic ointments are packaged in 3 or 3.5 g tubes. Examples Cetamide, Tobra-Dex, Garamycin. Sterile solutions and suspensions are used in the eye (ophthalmic preparations) to treat infections, allergies, and inflammation and dry eyes in the ears (otic operations) to treat infection. Examples of ophthalmic preparations used in the eye are Sulamyd, Opticrom, Inflamase, prednisolone, and Isopto Tears. Ophthalmic drops · Voltaren. Cause stinging and burning, and blurred vision. · Ketorolac. Cause stinging and burning · Trifluridine. Store in refrigerator · Pilocarpine. cause miosis · Latanoprost (Xalatan). Causes pigmentation, and enlargement of eyelashes. · Latanoprost + Timolol (Xalacom). · Timolol · Some products may be used in both the eye and the ear, such as Sofracort, Cortisporin while otic preparations have a glycerine or propylene glycol base and cannot be used in the eye. · Examples of otic preparations: Cortisporin Otic, Garamycin Otic, Garasone Otic, Locacorten Vioform, and Otic Drops. Modified drug delivery system: Modified release drug delivery system Sustain release dosage forms Provides medication over extended period of time Do NOT attain zero order release kinetics SR for 12 hrs, XL for 24 hrs Bupropion SR Q12hr Bupropion XL Q24hr Ritalin 3-4 hr Ritalin SR 6-8 hr Ritalin LA 8-10 hr Concerta 12 hrs

Controlled dosage forms Maintain constant drug levels in blood or tissue Release the drug in zero order pattern CR, MR, CD, continuous

Controlled/targeted delivery: An ideal controlled release mechanism for a device is the one, which exhibits a zero order drug release. i.e. the release of drug is independent of time. Delayed release or sustain release (SR) --> repetitive intermittent dosing of IR. (e.g. Bupropion SR). Extended release (XL)--> maintain therapeutic levels prolong time (e.g. Bupropion XL) Site specific target-->target to one organ location (enteric coated) Copyright © 2000-2018 TIPS Inc. Unauthorized reproduction of this manual is strictly prohibited and it is illegal to 38-6 reproduce without permission. This manual is being used during review sessions conducted by PharmacyPrep.

www.pharmacyprep.com

Drug delivery Systems

Receptor targeting--> target a drug receptor. Conventional dosage forms, drug concentration raises rapidly, peaks, and then falls until the next dose is taken. Controlled delivery systems Advantages of controlled delivery systems. Maintenance of optimum therapeutic drug concentration in the blood or in a cell predictable and reproducible release rates for extended periods of time enhancement of activity duration for short half-life drugs the elimination of side effects, frequent dosing, waste of drug, optimized therapy better patient compliance. Drug may be coated on small inert beads of sugar & starch. Some beads can then be coated with lipids to delay release. Beads with different coating thickness can then be combined in a capsule to achieve sustained release, e.g., the Spansule technology (Contac, Dexedrine). Technological methods Drug can be embedded in slowly eroding matrix, e.g., SLOW-K (KCl in a wax matrix), SLOW-Fe etc. Two-layered or press-coated tablets for sustained release. Drug embedded in inert plastic matrix, e.g. Gradumet Drug complexes with ion exchange resins, e.g. Pennkinetic systems Floating capsules or tablets, e.g. slow release diazepam (Valrelease), a hydrodynamically balanced (HBS) drug delivery system. Coating or sustained release tablets should not be crushed or chewed. Osmotic release: Nifedipine (Adalat), and Concerta (Methylphenidate). Liposomes for drug delivery systems Liposomes are lyotropic liquid crystals composed mainly of amphiphilic bilayers. Liposomes have the advantage of primarily consisting of lecithin and cholesterol, which are materials that occur naturally in the human body. Lecithin and cholesterol are also present in the body in large amounts, and thus demand good bioacceptability. This liposome’s can entrap drugs and may be used for drug targeting, sustained release, or reduced drug toxicity. Phospholipid can spontaneously form concentric, bilayer lipid vesicles when dispersed in water. Can be processed into various types & sizes. PEGylating Transdermal delivery involves diffusion of the drug through the skin. The drug properties that are suitable for transdermal preps are drugs with short half life, and lipid soluble drugs. Nicotine patch. Volatile (may evaporate), Androderm, Estraderm, Habitrol, Nicoderm, Nitro-Dur, and Estalis. (Moisture and photosensitive) Nitroglycerin patch: Use nitrate free period to reduce tolerance (apply 12 hours and 12 hours without patch). Can be applied on chest. Avoid applying on lower extremities. Fentanyl patch. Effective for 72 hours. Estrogen patch. Do not apply on the chest. If it falls frequently apply new patch. Scopolamine patch: Apply behind the ear, effective for 3 days. Oxybutynin Patch: Anticholinergic drug for the treatment of urinary incontinence Tolterodine Patch: Anticholinergic drug. Clonidine Patch: Centrally acting antihypertensive drugs. Copyright © 2000-2018 TIPS Inc. Unauthorized reproduction of this manual is strictly prohibited and it is illegal to 38-7 reproduce without permission. This manual is being used during review sessions conducted by PharmacyPrep.

www.pharmacyprep.com

Drug delivery Systems

Implants for drug delivery Implantable drug delivery systems are being developed to take the place of traditional drug delivery systems, such as pills and hypodermic injection. Implantable systems that are currently available include Norplant and various pumps, such as insulin pumps. The systems are designed to deliver drugs directly into the bloodstream at a controlled rate of transmission.

Tips 1.

it is rapid onset & no first pass metabolism

2.

8 to 12 hrs

3.

4.

for drugs that cause GI irritation and N, and uncooperative or unconscious patient hydrofluoroalkane hfa

5.

no shaking but prime

6.

8.

iv

9.

10.

the tissue mass, extent of ionization

11.

it is the fastest acting method

12.

13.

im

14.

15.

16.

salbutamol, fluticasone, budesonide, salmeterol

17.

ophthalmic and parenteral the taste and or smell and possible first pass metabolism and slow

they provide continuous drug administration and no first pass metabolism injection in joints

18.

into the spinal column

7.

· · · · · · · · · · · · · · · · ·

drugs that act directly on the bronchi & inhalation anesthetics it is the most convenient to both the patient and physician the physical and chemical characteristics of the drug

What are the advantages of oral administration? ( ) What dosage forms require sterile? ( ) Intra articular injections ( ) What propellant used in meter dose inhaler? ( ) Nitroglycerin patch require nitrate free period for? ( ) Meter dose inhalers that require shaking ( ) Nitroglycerin spray require ( ) What are the disadvantages of oral administration? ( ) What is the advantage of IV drug administration? ( ) What are the advantages of sublingual administration of a drug? ( ) What is meant by the intrathecal administration of a drug? ( ) Drug distribution into different tissues depends on which factors? ( ) What is the most direct route of drug administration? ( ) What factors must be considered in choosing a specific route of administration? ( Which provides a more rapid absorption, IM or SC administration? ( ) What type of drugs can be given by inhalation? ( ) What are the advantages of rectal administration of drugs? ( )

)

Copyright © 2000-2018 TIPS Inc. Unauthorized reproduction of this manual is strictly prohibited and it is illegal to 38-8 reproduce without permission. This manual is being used during review sessions conducted by PharmacyPrep.

www.Pharmacyprep.com

Pharmaceutical Analysis

39 Pharmaceutical Analysis Questions Alerts! Common questions in pharmacy exam is to ask! · Chromatography methods like HPLC, GC, TLC · Spectroscopic methods like mass spectroscopy, or NMR. · Bioassays like ELISA, gel electrophoresis, western blot, eastern blot and Polymer chain reaction (PCR) and real time PCR (RtPCR). Chemical separation or purification techniques include, distillation, chromatography, extractions and centrifuge.

Chromatography Chromatography is a method of separation of mixture of chemicals that relies on differences in partitioning behaviour between a flowing mobile phase and a stationary phase to separate the components in a mixture. The commonly used chromatographic methods of analysis include: Gas chromatography, HPLC, TLC, and paper chromatography.

Chromatography instrumental procedures Liquid

Gas Column GLC

Planar

GSC HPLC TLC

Normal phase (NR-HPLC)

Paper

Reversed phase (RP-HPLC)

Copyright © 2000-2018 TIPS Inc. Unauthorized reproduction of this manual is strictly prohibited and it is illegal to 39-1 reproduce without permission. This manual is being used during review sessions conducted by PharmacyPrep.

www.Pharmacyprep.com

Pharmaceutical Analysis

Liquid Chromatography Liquid chromatography (LC) is an analytical chromatographic technique that is useful for separating ions or molecules that are dissolved in a solvent. High Performance Liquid Chromatography (HPLC) or High-Pressure Liquid Chromatography: Used for non-volatile and thermally stable compounds to separates macromolecules, ionic species, labile natural products, polymeric materials and high-molecular weight compounds. Diagram.

Solvent

Injector

Columns and Pump

Detector

Records chromatogram

Mobile phase. Solvent such as methanol, water, ethanol and CCl 4 . Stationary phase. Silica gel column. Column. 4 to 5 mm heavy wall, glass-lined metal tubing or 10 to 30 cm. Detectors · · · · · ·

UV-VIS photometers (diode array detector) Fluoviometric detector Electrochemical Refractometers (RI) Conductivity for ion chromatography Radiodetector

Question Alerts! Mobile phase are solvents; these are two types 1) Organic (methanol, ethanol, acetonitrile, CCl4) are normal phase 2) Aqueous solvents (water) are reversed phase HPLC 3) Silica gel is? Stationary phase.

Parameter that affects resolution. · Mobile phase. Nature of solvent should be compatible with substance. · Solubility’s of mixture in mobile phase. · Concentration of mixture substance. · Stationary phase, thickness of silica gel, column size and pressure in HPLC. · Temperature (gas and column chromatography). · pH of solvent and flow rate. · Detectors does NOT enhance resolution of separation of mixtures.

Gas Chromatography (GC): The gAS CHROMATOGRAPHY IS A CHROMATOGRAPHIC TECHNIQUE THAT CAN BE USED TO SEPARATE VOLATILE ORGANIC COMPOUNDS. EXAMPLE VOLATILE LIQUIDS, OILS AND ALCOHOLS ETC. Mechanism. The organic compounds are separated due to differences in their partitioning behaviour between the mobile gas phase and the stationary phase in the column. GLC (Gas Liquid Chromatography) · Mobile phase is gas for example, helium, argon, or nitrogen · Stationary phase is liquid surface on solid GSC (gas solid chromatography) · Mobile phase: gas for example helium, argon, or nitrogen Copyright © 2000-2018 TIPS Inc. Unauthorized reproduction of this manual is strictly prohibited and it is illegal to 39-2 reproduce without permission. This manual is being used during review sessions conducted by PharmacyPrep.

www.Pharmacyprep.com

Pharmaceutical Analysis

· Stationary Phase (solid) · Used: For volatile and thermally stable compounds Components of gas chromatography; Gas chromatography columns known as capillary columns, Oven, Detector, and Recorders Types of detectors in GC. · TCDàThermal conductivity Detectors · FIDàFlame ion detectors · ECDàElectron conductivity detectors. Thin Layer Chromatography (TLC) · A simple and rapid method to monitor the extent of a reaction or to check the purity of organic compounds. The mobile phase is a solvent and the stationary phase is a solid adsorbent on a flat support. · Mechanism: Relies on capillary action TLC Spray Reagent · Ninhydrin is used for detection of amino acids, amines, and aminosugars. · Ehrlich's reagent is dimethyl aminobenzaldehyde/hydrochloric acid reagent for detection of amines, indole derivatives. Paper Chromatography · Stationary phase. The stationary phase is cellulose paper (paper is made from cotton fibres and highly purified about 90% alpha cellulose). · Properties of stationary phase: Highly hydroxylated polysaccharide, this has great affinity for water and other polar solvents. · The tightly bound water is actual stationary phase and as mobile phase passes over the surface of paper. · Mobile phase: The solvents used for paper chromatography analysis are similar to those employed in other forms of chromatography. Spectroscopic Methods of Analysis Spectroscopic methods: Instrumental methods that are used to identify chemical structure and analysis of drugs. The following instrumental techniques are commonly used in drug analysis and structure determination of new and unknown chemical structures. Spectroscopic Methods Mass spectroscopy (MS)

Ultraviolet light spectroscopy (UV/Vis)

Nuclear Magnetic Resonance (NMR)

Other spectroscopy Infrared (IR) Atomic Absorption (AA)

Mass Spectroscopy (MS): Mechanism. The basics mass spectrometry is that a charged particle passing through a magnetic field is deflected along a circular path on a radius that is proportional to the mass to charge ratio, and m/e. (Electronic ionization method).

Copyright © 2000-2018 TIPS Inc. Unauthorized reproduction of this manual is strictly prohibited and it is illegal to 39-3 reproduce without permission. This manual is being used during review sessions conducted by PharmacyPrep.

www.Pharmacyprep.com

Pharmaceutical Analysis

General Diagram of a mass spectrometer

Collection & recorder

electron beam Radius of curvative

ionized mixed ion beam Neutral molecules Molecules

Gas

magnetic field Sample

Inlet System

separated ion beam

Vaporized

Inlet Source

Vacuum System

Mass Analyze

Ion Collection System

Data Handling System

Used in structural determination of unknown chemical structures. Detect molecular weight of substance. Advantage. This can detect trace amount of substances. · Blood sample analysis (drugs or alcohol in blood) à GC-MS · Pharmacokinetics analysis of drug samples. · Detection of environmental samplesàGC-MS · Probably the most useful information you should be able to obtain from a MS spectrum is the molecular weight of the sample. · Used in detection and analysis of unknown chemical structures molecular weight of substances or drugs. Type of detectors in mass spectroscopy · CI-MS; Chemical Ionization mass spectroscopy · FAB-MS. Fast atomic bombardment mass spectroscopy Question Alerts! 1) What is used to determine molecular weight? 2) What spectrometer is used to determine number of hydrogen carbons and functional groups by resonating nucleus or spinning technique? 3) What spectroscopic method shows pharmacophores? 4) What spectroscopic method is used to identify enantiomers? 5) What spectroscopic methods breakdown chemical structure? ·

EI-MS. Electron impact mass spectroscopy.

Copyright © 2000-2018 TIPS Inc. Unauthorized reproduction of this manual is strictly prohibited and it is illegal to 39-4 reproduce without permission. This manual is being used during review sessions conducted by PharmacyPrep.

www.Pharmacyprep.com

Pharmaceutical Analysis

Ultra Violet/Visible light spectrophotometer: Question Alerts! · Wavelengths (l) 190 and 800 nm Bathochromic shift is? · Ultraviolet radiation 700 nm · Infrared light lies between the visible and microwave portions of the electromagnetic spectrum. · Infrared light has a range of wavelengths, just like visible light has wavelengths that range from red light to violet. Atomic-absorption spectroscopy Atomic-absorption (AA) spectroscopy uses the absorption of light to measure the concentration of gas-phase atoms. The analyte concentration is determined from the amount of absorption. The Beer-Lambert law

·

· · · · ·

The Beer-Lambert law (or Beer's law) is the linear relationship between absorbance and concentration of an absorbing species. A = a (l) * b * c Where A is the measured absorbance a (l) is a wavelength-dependent absorptive coefficient b is the path length c is the analyte concentration.

Nuclear Magnetic Resonance Spectroscopy (NMR): NMR techniques are used to identify unknown chemical structure. Functional groups and isomers of structure can be determined by NMR. Magnetic Resonance Imaging (MRI): MRI is used to scan tumours, minor blood clots etc. CAT scan (computer assisted tomography).

Copyright © 2000-2018 TIPS Inc. Unauthorized reproduction of this manual is strictly prohibited and it is illegal to 39-5 reproduce without permission. This manual is being used during review sessions conducted by PharmacyPrep.

www.Pharmacyprep.com

Pharmaceutical Analysis

Bio and Immunoassay Methods Gel electrophoresis, ELISA, Western blot, and PCR.

Immunoassay are an assay (test) that detects antigens (Ag) or antibodies (Ab). Immunoassays described into various types based on their different techniques such as radioimmunoassay, enzyme immunoassays etc. Radio immunoassay are more sensitive assay (0.0001 to 0.001 µg/ml, 0.1 to 1 ng/ml). This makes it suitable for measuring hapten size drugs and hormones in the blood, things you can't get in large concentrations needed for precipitation or agglutination. Enzyme Immunoassay (EIA) is a laboratory test that detects specific antigens or antibodies utilizing enzyme tagged antigens or antibodies, and in the presence of a specific substrate, it produces a colour change that indicates a positive reaction. ELISA: The name stands for Enzyme- Linked Immunosorbent Assay (ELISA): It is a useful and powerful method in estimating ng/ml to pg/ml antigens (Ag) or antibodies (Ab) in the solution, such as serum, urine and culture supernatant. Used for diagnosing HIV viral infections. Gel electrophoresis: It is a method that separates macromolecules either nucleic acids or proteins on the basis of size, electric charge, and other physical properties. 1. Restriction enzyme cleave the DNA into smaller segments of various sizes. 2. DNA segments are loaded in to gel. This gel floats in buffer. 3. When apply electric current DNA fragments move toward the positive charge cathode. 4. Small segments move faster and farther than larger DNA segments. Western blot Using antibodies separate mixture of proteins. Blot consisting of a cellulose derivative that contains spots of antibodies for identification by suitable molecular probe.

Easter blot For analysis of post translational proteins, Lipids. Blot consisting of a cellulose derivative that contains spots of proteins for identification by suitable molecular probe.

Northern blot For detection of RNA

Southern blot For detection DNA

Blot consisting of a cellulose derivative that contains spots of RNAs for identification by suitable molecular probe.

Blot consisting of a cellulose derivative that contains spots of DNAs for identification by suitable molecular probe.

PAGE: Polyacrylamide gel electrophoresis system is used for protein and protein mixture separations. PCR is Polymer chain reaction is used for magnifying DNA. Question Alerts! 1) ELISA stands for? Enzyme Linked Immunosorbent Assay 2) Purpose for ELISA is? Diagnosis of HIV 3) Western blot is used for? Using antibodies detects proteins and nucleic acids 4) Eastern blot is used for? Analyze protein post translational modifications like lipids, phosphates, and glycoconjugates. 5) PCR is? Polymerase chain reaction magnifies DNA utilizes. Taq polymerase and this produced by Thermos aquaticas.

Copyright © 2000-2018 TIPS Inc. Unauthorized reproduction of this manual is strictly prohibited and it is illegal to 39-6 reproduce without permission. This manual is being used during review sessions conducted by PharmacyPrep.

www.Pharmacyprep.com · · ·

Pharmaceutical Analysis

Polymerase chain reaction (PCR) is utilizes heat stable DNA polymerase such Q Taq polymerase (this enzyme is isolated from bacteria Q Thermus aquaticus). The Taq polymerase enzymatically assembles a new DNA building blocks using single stranded DNA nucleotides as a template. It is also known as DNA primers. Polymerase chain reaction includes 3 major steps? 1) Denaturation at 94-96◦C 2) Annealing at -68 ◦C 3) Elongation at 72 ◦C

Bioassay PCR Gel electrophoresis Western blot ELISA

use

Titration’s Non-aqueous solvents. Acid base titration in non-aqueous solvents. Three types of solvents: Amphiprotic, nonionisable, and aprotic or inert. Amphiprotic: Autoprotolysis produce both an acid and base species, or solvent dissociates such that it produces cation and anion species. Example: H 2 O, methanol, ethanol and acetic acid H 2 O à H+ + OHCH 3 OH à CH 3 O- + H+ Non-ionisable-with basic properties: No autoprotolysis, but solvent has a group that can react with acids. No reaction bases. Only act to transport on pairs. E.g. pyridine, ethers, benzene, esters, ketenes, and aldehyde Protic Upon reaction provides H+ ion. Example: Methanol, Acetic acid. Water etc. Aprotic or inert There is no reaction with acids or bases, they simply provide medium in which the sample species or titrant are soluble (only contribute solubility). Example: CCl 4 (Carbon tetrachloride) Copyright © 2000-2018 TIPS Inc. Unauthorized reproduction of this manual is strictly prohibited and it is illegal to 39-7 reproduce without permission. This manual is being used during review sessions conducted by PharmacyPrep.

www.Pharmacyprep.com

Pharmaceutical Analysis

Levelling effect Regardless of the type of acid and basis, the actual acid strength is actually determined by the strength of H 3 O+. Equilibrium reactions · At equilibrium hydrolytic reaction · The rate of forward reaction is equal to backward reaction · Example. In aproteolytic reaction of acetic acid with water, the rate of forward reaction increased as the hydronium ions are depleted or when acetate ions are depleted

Gravimetric Analysis The quantitative determination of a substance by precipitation followed by isolation and weighing of the precipitate. The basic method of gravimetric analysis A weighed sample is dissolved after which an excess of a precipitating agent is added. The precipitate which forms is filtered dried or ignited and weighed. From the mass and known composition of the precipitate, the amount of the original ion can be determined. Criteria for successful determinations The desired substance must be completely precipitated. In most determinations the precipitate is of such low solubility that losses from dissolution are negligible. An additional factor is the common ion effect; this further reduces the solubility of the precipitate. Example: When Ag+ is precipitated out by addition of Cl- the (low) solubility of AgCl is reduced still further by the excess of Cl-, which is added, pushing the equilibrium to the right. Ag+ + Cl- = AgCl (s)

Extraction Methods Liquid/liquid · Examples of liquid/liquid extraction methods are fractional distillation, which functions based on boiling points of solvents · Mixture of two volatile liquids can be separated using fractional distillation techniques · Example: Ethanol in water, Hexanes in Chloroform · Immiscible liquids: this mixture can be separated by using separating funnel. · Example: Hexanes in water (Organic solvents in water) Solid phase extraction (SPE) · A solid sorbent material, typically an alkyl bonded silica, is packed into a cartridge or imbedded in a disk and performs essentially the same function as the organic solvent in liquid-liquid extraction. · Example,

Copyright © 2000-2018 TIPS Inc. Unauthorized reproduction of this manual is strictly prohibited and it is illegal to 39-8 reproduce without permission. This manual is being used during review sessions conducted by PharmacyPrep.

www.Pharmacyprep.com

Pharmaceutical Analysis

Reverse-phase SPE employed to extract non-polar compounds, pesticides for instance, from polar samples such as water generally utilize a solid sorbent containing non-polar functional groups such as octadecyl (C 18 ) or octyl (C 8 ) bonded silicas.

·

Tips 1.

to dry wet powders, which are heat sensitive drugs

2.

4.

Analysis and detection of volatile chemical in samples Enzyme linked immunosorbent assay Gas Chromatography Increase in solubility by addition of salt Detection of HIV infection

7. 10. 13. 16.

· · · · · · · · · · · · · · · ·

3.

purification and analysis of pharmaceuticals

5.

Electronic ionization (EI) results in well-established fragmentation pattern that are useful in identification of unknown protein separation

6.

protein isolation

8.

HPLC

9.

UV, DAD, Fluorescent & RI

11.

capillary action

12.

14.

Decrease in solubility by addition of salt Mass Spectroscopy

15.

amine, amino sugars and proteins Detectors, printers

17.

Pump is present in what type of chromatography? ( ) Types of detectors used in HPLC? ( ) Drugs present in blood samples can be detected by? ( ) Salting in is? ( ) Salting out is? ( ) Mechanism of TLC is based on? ( ) Ninhydrin is used to detect? ( ) What are the factors that does not affect resolution in HPLC? ( GC-MS is? ( ) Lyophilization (freezing) is used for? ( ) Electron impact (EI) ionization mass spectroscopy is? ( HPLC is used for? ( ) ELISA is? ( ) ELISA is used for? ( ) Western blot test for? ( ) Gel electrophoresis separates? ( )

) )

Copyright © 2000-2018 TIPS Inc. Unauthorized reproduction of this manual is strictly prohibited and it is illegal to 39-9 reproduce without permission. This manual is being used during review sessions conducted by PharmacyPrep.

www.Pharmacyprep.com

Pharmaceutical Analysis

Copyright © 2000-2018 TIPS Inc. Unauthorized reproduction of this manual is strictly prohibited and it is illegal to 39-10 reproduce without permission. This manual is being used during review sessions conducted by PharmacyPrep.

www.Pharmacyprep.com

Canadian Healthcare System

40 The Canadian HealthCare System Questions Alerts! Common questions in pharmacy exam is to ask! · Role of governments in healthcare (Health Care system components) · Canadian Health Act (CHA) five principles: Comprehensiveness, Universality, Portability, Public administration and Accessibility. · Drug Benefit Programs. Federal drug benefit programs and provincial drug benefit programs. · Federal drug benefit programs covers natives, veterans, inmates, refugees and RCMP. · Provincial drug benefit programs covers seniors over 65 yo, and social welfare recipients. · Financial support for healthcare in Canada. Federal, and provincial taxes. This chapter provides information about Canadian healthcare system; explore key concepts around health care delivery system and health care professionals. This chapter is focus on Canadian health act (CHA), the role of federal, provincial, & territorial government in health care. FEDERAL Making federal law (e.g. CDSA, office of control substance) F&DA, marijuana for medical purpose use) and bill. International affairs Canadian safety. Federal Health Portfolio. 1.Health Canada (Drug Approvals in Canada DIN; NPN; DIN-HN; PIN) 2.Public health Agency (PHA) 3. Canadian Institute of Health Information. (CIHI). 4. Patented Medicine Price Review Board (PMPRB) 5. Canadian Food safety agency Royal Canadian Mounted Police (RCMP)

PROVINCIAL Provincial HealthCare Administration (Medicare) Education Flu vaccination Physician salaries Decentralized administration healthcare system

MUNICIPALITIES Police/Fire/ Roads/Parks/ Libraries etc District health officer BY-LAWS

Provincial police

City/regional

Copyright © 2000-2018 TIPS Inc. Unauthorized reproduction of this manual is strictly prohibited and it is illegal to 40-1 reproduce without permission. This manual is being used during review sessions conducted by PharmacyPrep.

www.Pharmacyprep.com Drug trafficking & CDSA Health Canada determines the manufacturing conditions of drugs.(GMP)

Canadian Healthcare System

DIDFA: Drug Interchangeability Dispensing Fee Act.

Primary care: This is first contact a person makes a with the system when a person feels the necessity of health care. This usually occurs through the family physician, pharmacist, or nurse at medical centers. Secondary care. This is specialized service from a specialist. This requires referral from primary health care levels. Tertiary care. This is specialized in diagnosing and highly technical care and treating complicated or unusual health problems. This generally takes place in hospital setting where generally diagnostic and complicated therapies can take place. Health care system components: Health care team members: Physician, pharmacist, nurses, dentist, physiotherapist etc. Physician agencies: CMA, Royal College, Pharmacy agencies: NAPRA, provincial college of pharmacy, Canadian Hospital Pharmacy Society, Canadian Pharmacy Association. Primary Care; Family health care; Community Health Centres; Hospitals; Public Health Care Systems. Health Canada is a federal agency: it is responsible for drug quality, safety and efficacy The Health Protection Branch (HPB) or The Health protection Food Branch Inspectorate (HPFBI) of Health Canada regulates drugs imported into and manufactured for sale in Canada. The Canadian health care system The Canadian health care system is universal health care (Medicare: Essential healthcare services are insured by the government to all Canadian), this mean that all citizens and immigrants will have access to health care regardless of their ability to pay. All Canadian are insured on equality basis and offered health in all ten provinces and three territories. Canadian health act (CHA): Canada health act (1984) unanimously passed in parliament, with adherence to the 5 principles, enforced by threat of withholding funds. Extra billing is banned as a restriction on access. Canadian health act five principles · Universality · Public administration · Portability · Accessibility · Comprehensiveness

Question Alerts! Five principles of the CHA!

Universalityà All insured parties are entitled to equal access to essential services.

Copyright © 2000-2018 TIPS Inc. Unauthorized reproduction of this manual is strictly prohibited and it is illegal to 40-2 reproduce without permission. This manual is being used during review sessions conducted by PharmacyPrep.

www.Pharmacyprep.com

Canadian Healthcare System

Public administrationà Healthcare insurance is to be administered on a non-profit basis by a public authority responsible to the province and subject to audit. The Canadian healthcare delivery is decentralized and offered for provincial delivery of care. Comprehensiveness à The insurance must cover all insured services supplied by hospitals, medical practitioners and “essential services” dentists. Each province determines which services are insured. Funded health care services are NOT provided equally across the Country. (Decentralized healthcare system). Portability à A series of obligations on provinces which essentially guarantee any Canadian resident (after a maximum wait of three months upon first becoming a resident) access anywhere in Canada on the same basis as local residents. Accessibility à Charges or other obstacles must not impede access to insured services. The CHA covers · Medically necessary hospital services. · Medically required physician’s services. · Medically or dentally required surgical-dental services requiring a hospital for proper performance. · The coverage reflects the two-stage evolution of public health care insurance in Canada. The 1959. Hospital Insurance and Diagnostic Services Act and the 1966 Medicare Act respectively brought hospital and medical insurance to the federal level. CHA does not cover The following services are not covered under the CHA. Services delivered by health-care professionals other than doctors, particularly outside of hospitals (some provinces do cover some of these services, but are not obligated to do so under the CHA). Services in sectors outside the hospital. These include long-term care facilities and home care. Pharmaceuticals, rehabilitation services and dental care are also not covered when provided outside of hospitals. Drug benefit program: Provincial drug benefit programs and Federal drug benefit programs. Provincial Drug Benefit Programs covers: People age over 65-year-old and Social Assistance (welfare), disabilities and long-term illness. Federal Drug Benefit Programs covers · Natives or aboriginals (covered by NIHB) · Inmates · Refugees · Veterans · Royal Canadian Mounted Police (RCMP) Canadian are likely to hold private health insurance. Non-insured health benefit (NIHB) programs: The role of federal government in health care. The federal government sets and administers national principles of healthcare system through Canada health act. Federal government gives funds to provincial and territorial health care services through fiscal transfers. Copyright © 2000-2018 TIPS Inc. Unauthorized reproduction of this manual is strictly prohibited and it is illegal to 40-3 reproduce without permission. This manual is being used during review sessions conducted by PharmacyPrep.

www.Pharmacyprep.com

Canadian Healthcare System

Delivers health care services to specific groups e.g. first nations (aboriginal), Inuit’s. Canadian forces and veterans, refugee claimants and penitentiary inmates, and RCMP. Provides other health related functions such as public health/health promotion programs and health research. Role of provincial and territorial health care as CHA principle is public administration this means the administration and delivery of health care services is the responsibility of each province and territory. Provinces and territories fund health services with assistance from the federal government in the form of transfer payments and some times equalization payments. Some examples of essential services that offered by provinces and territories are Physicians, diagnosis, and other health care services in primary clinics and emergency services in hospitals. Drugs coverage plans for specific groups, such as seniors, and social assistance. Some provinces have supplementary health benefits. Federal Health Portfolio The Minister of Health is responsible for maintaining and improving the health of Canadians. This is supported by the Health Portfolio which comprises; · Health Canada · Public health agency of Canada · Canadian Institute of health research · The Patented Medicine Prices Review Board · Canadian Food Inspection Agency Public health Agency of Canada Health promotion and disease prevention. Health promotion is the process of enabling people to increase control over and improve their health. Disease prevention focuses on efforts to avoid disease and injury. CIHI: gather and provide health information to the federal and provincial agencies. PMPRB: Determines the prices of brand or patented medicines in Canada. Canadian Food Inspection Agency (CFIA): monitored food safety across Canada. Provincial plans Some examples of the disease prevention include smoking cessation programs, breast cancer screening (mammogram), Pap smear screening, prostate specific antigens (PSA) colonoscopy and colorectal cancer screening. Cost of Health Care system: Health care financing: Provincial, Federal, health premium and charities. Distribution of health Expenditure in Canada Hospital Other institutions Physician salaries Other professionals Drugs (Rx and OTC) Other health spending

% 29.9% 9.9% 12.8% 10.7% 17.5% 6.1%

Copyright © 2000-2018 TIPS Inc. Unauthorized reproduction of this manual is strictly prohibited and it is illegal to 40-4 reproduce without permission. This manual is being used during review sessions conducted by PharmacyPrep.

www.Pharmacyprep.com

Canadian Healthcare System

Public health Capital Administration

5.5% 4.2% 4.1%

Canadian health human resources 2010 Total physicians in Canada ~80,000 The third highest public sector expenditure Total registered nurses in Canada ~240,000 Dentist/dental hygienist 45,554 The highest private section expenditure Pharmacist 42,000 Optometrists 4,841 Source National Health Expenditure Database, Canadian Institute for Health Information

Tips 1. 4. 7. 10 13 · · · ·

Pharmaceuticals Provincial or territories Portability Natives or aboriginals Veterans

2. 5. 8. 11 14

Non essential services Comprehensiveness Public administration Inmates RCMP

What are the five Canadian health act principles? ( Federal Drug Benefit Program covers ( ) What is not covered in CHA? ( ) Funding for healthcare system is paid by? ( )

3. 6. 9. 12

Federal Universality Accessibility Refugees

)

Copyright © 2000-2018 TIPS Inc. Unauthorized reproduction of this manual is strictly prohibited and it is illegal to 40-5 reproduce without permission. This manual is being used during review sessions conducted by PharmacyPrep.

www.Pharmacyprep.com

Canadian Healthcare System

Copyright © 2000-2018 TIPS Inc. Unauthorized reproduction of this manual is strictly prohibited and it is illegal to 40-6 reproduce without permission. This manual is being used during review sessions conducted by PharmacyPrep.

www.pharmacyprep.com

Canadian Pharmacy Regulations

41 Pharmacy Regulations Questions Alerts! Common questions in pharmacy exam is to ask! · National Drug Model or Harmonized Drug Model. NAPRA categorized drugs into Schedule I, II, III and unscheduled drug. · Food and Drug Act (F&DA) and Control Drug Substance Act (CDSA) regulates narcotics, benzodiazepine & targeted substance, and control substances. · Narcotics are categorized as straight narcotics (single opioids), narcotic preps (1+1+1)(verbal), and exempted narcotics (1+1+1). · Benzodiazepine and targeted substances like all benzodiazepine · Control substances have 3 parts. The part 1. CNS stimulants part 2. Barbiturates part 3: anabolic steroids. Canadian Federal Regulations Control Drugs and Substance Act (CDSA)

Food and Drug Act (F&DA)

National Association of pharmacy regulatory authority (NAPRA)

Narcotics, BZDs, Control drugs, sales, Rx and disposing.

Manufacturing conditions and prescription drug advertising.

Schedule I Schedule II Schedule III Schedule IV Schedule V Schedule VI

Schedule A Schedule B Schedule C Schedule D Schedule E Schedule F part 1

Pharmacy professional standards sales. Set standards of professional qualifications and competencies.

Schedule F Part 2 Schedule VII

Schedule 1 (require prescription)

Tips

Schedule F part1. Prescription drug advertisement standards FDA part 2. Veterinary drug

Schedule G

Copyright © 2000-2018 TIPS Inc. Unauthorized reproduction of this manual is strictly prohibited and it is illegal to 41-1 reproduce without permission. This manual is being used during review sessions conducted by PharmacyPrep.

www.pharmacyprep.com Schedule VIII

Schedule H

Canadian Pharmacy Regulations

Schedule 2 (behind the counter) dispensed by pharmacist without prescription Schedule 3: OTC Unscheduled

Behind the counter (BTC), under the counter. INSIDE PHARMACY AREA. Over the counter (OTC) or self-selection area or 10meter rule Corner store

Federal regulations Three federal regulations are F&DA, CDSA, and NAPRA applies to pharmaceuticals in Canada. Food and Drug Act (F&DA). This federal legislation controls the manufacture of all drugs in Canada. Also, the act controls manufacturing conditions, packaging, advertising standards and the sale of foods, drugs, cosmetics and therapeutic devices. As with all the laws in Canada, the law exists to protect the consumer or the public. Drugs regulated by the F&DA are grouped into A to H schedules. · Schedule A. Disease which treatment may not permit to public. · Schedule B. Describe official standard. · Schedule C. Radiopharmaceuticals. Drugs other than radionuclides for use in preparation of radiopharmaceuticals. · Schedule D. Allergic substances, vaccine, blood and blood derivatives. · Schedule F: Food and Question Alerts! prescription drugs. Schedule F have all prescription drugs that are present in Advertisement standards. NAPRA schedule 1 · Schedule G. Controlled drugs Schedule F have prescription drug advertisement · Schedule H. Restricted drugs standards. The Controlled Drugs and Substances Act (CDSA): The Controlled Drugs and Substances Act, 1997, is an act sets standards for the control of narcotics, controlled drugs and targeted substances (benzodiazepines). It is a federal act and the strictest of all the acts that govern the pharmacy industry. The CDSA is administered by office of control substance. Drugs regulated by the CDSA are grouped into 8 schedules. · Schedule I. Narcotics, opium poppy, cocaine, phenylpiperidine (pthedine) · Schedule II. Cannabis and cannabis preparations · Schedule III. Amphetamines, methylphenidate, LSD plus other listed psychoactive substances. · Schedule IV. Barbiturates, specific anorexiants, benzodiazepines, and anabolic steroids. · Schedule V. Phenylpropanolamine and others · Schedule VI. Ephedrine, ergotamine, pseudoephedrine and others. · Schedule VII. Cannabis that serves enforcement purposes regarding possession and trafficking. · Schedule VIII. Cannabis that serves enforcement purposes regarding possession and trafficking Narcotics regulations have categorized as straight narcotics, narcotic prep, and exempted narcotics.

Copyright © 2000-2018 TIPS Inc. Unauthorized reproduction of this manual is strictly prohibited and it is illegal to 41-2 reproduce without permission. This manual is being used during review sessions conducted by PharmacyPrep.

www.pharmacyprep.com

Canadian Pharmacy Regulations

Straight narcotics: Opioids like morphine, codeine, hydromorphone, oxycodone, methadone, fentanyl, Suboxone, OxyNeo, meperidine, buprenorphine, Tylenol # 4 (acetaminophen 300 mg + codeine 60 mg), ketamine, pentazocine, nabilone etc. · Given by written Rx only. · No repeats · No Rx transfers · Sales report require · Rx hard copies for stored 2 yrs (digital for minimum 7 yr) Narcotic preparations or verbal narcotics (1+1+1): opioid + 2 non-opioid. Tylenol # 2, (acetaminophen 300 mg+ codeine 15 mg + caffeine 15 mg). Tylenol # 3 (acetaminophen 300 mg + codeine 30 mg + caffeine 15 mg), Fiorinal c 1/4 (butalbital + ASA+ codeine 15 mg) Fiorinal C1/2 (butalbital + ASA+ codeine 30 mg) · Given by verbal or phone · No Repeats or NO refills · No Rx transfers · Rx hard copies store for 2 yrs Exempted narcotics or OTC narcotics: (1+1+1): Opioid (30 mg) · Lactulose · Vitamin D 3 drops · Vitamin B 12 inj. Schedule III: Over the counter · The lowest risk however, may present risks to certain populations. · Prescription is not required. Need direct pharmacist supervision. · These drugs may be stored in a self-selection area of the pharmacy

Copyright © 2000-2018 TIPS Inc. Unauthorized reproduction of this manual is strictly prohibited and it is illegal to 41-5 reproduce without permission. This manual is being used during review sessions conducted by PharmacyPrep.

www.pharmacyprep.com

Canadian Pharmacy Regulations

The pharmacist should be accessible, and approachable to assist the patient in making an appropriate selfmedication selection or to refer physician. Example. Plan B. Emergency contraception, bacitracin and its salts for ophthalmic

· ·

Unscheduled Drugs · May be sold from any corner store. · Labelling is considered sufficient to ensure safety of drug. · No pharmacy knowledge required. Example: Bacitracin and its salts for topical use Pharmacy related professional associations in Canada · Canadian Pharmacist Association (CPhA) (www.pharmacists.ca) · Voluntary national association. · Identify, respond to emerging issues of importance to the profession, assisting and acquiring new technologies and using information. · Create educational and professional development tools. CPhA publications include: · Canadian Pharmacy Journal (CPJ) · Compendium of Pharmaceutical Specialties (CPS) · Therapeutic choices (TC) · Patient selfcare (PSC) or Therapeutic Choice for Minor Ailments. · Compendium of patient self-care products (CPSP) · E-therapeutics

·

Canadian Society of Hospital Pharmacist (CSHP) · Voluntary national association. · Develop continuing education programs, residency training programs.

Tips

Find answers from the table. 1 4

· · · · · · · ·

Institute of Safe Medication Practices Canada (ISMP) Narcotics

2

NAPRA

3

CPhA

5

Benzodiazepine & Targeted Substance

6

Control Substances

National Association of Pharmacy Regulatory Authority ( ) Examples of pharmacy associations, where pharmacist can be a member? ( Examples of medication incident reporting systems in Canada? ( ) Who regulates pharmacy profession in Canada? à Who regulates pharmacist and its members in Canada? à Morphine, codeine, meperidine are regulated under ( ) Lorazepam, and Diazepam are regulated under ( ) Methylphenidate and amphetamine are regulated under ( )

)

Copyright © 2000-2018 TIPS Inc. Unauthorized reproduction of this manual is strictly prohibited and it is illegal to 41-6 reproduce without permission. This manual is being used during review sessions conducted by PharmacyPrep.

www.Pharmacyprep.com

Social, Behavioral and Administrative Sciences

42 Social, Behavioral, Economics Aspects of Pharmacy Profession Questions Alerts! Common questions in pharmacy exam is to ask! · Professional Misconducts · Pharmacist should always think as professionals · What factors that effects on patient compliance on medication therapy? Patient behavior on medication compliance While pharmacists are still responsible for managing the overall pharmacy environment and overseeing dispensing, they now focus more on reviewing prescriptions for therapeutic appropriateness, performing medication reconciliation and medication reviews, developing care plans and monitoring patients. The care plan is often focused on taking steps to resolve or prevent drug-related or health-related problems. Depending on the province or territory, the pharmacist may accomplish this by performing expanded scope activities such as adaptation of prescriptions, prescribing for minor ailments, ordering and/or interpreting laboratory tests and administering drugs, including injections for immunization or other purposes. Although the scope of practice can be slightly different in different provinces and territories, pharmacists must always assume responsibility for their own actions, be accountable to the public and manage the overall functioning of the pharmacy to ensure a safe and healthy environment. Pharmacy technicians recently became a regulated profession in some Canadian provinces. The other provinces and territories are also moving towards the regulation of pharmacy technicians. Along with regulation, pharmacy technicians have been given an expanded scope of practice that allows them to more effectively assist pharmacists so that the pharmacist can take on more clinical, patient-focused services. The scope of practice of pharmacy technicians can be slightly different in different provinces. The pharmacy technician may gather patient information for the pharmacist to review; order, receive and manage inventory, perform computer order entry and prepare products including compounding, counting, measuring and labelling. Depending on the province or territory, they may also perform expanded scope activities such as medical device demonstrations, transcribing verbal orders, transferring prescriptions, checking the technical aspects of a prescription and releasing the final product.

Copyright © 2000-2018 TIPS Inc. Unauthorized reproduction of this manual is strictly prohibited and it is illegal to 42-1 reproduce without permission. This manual is being used during review sessions conducted by PharmacyPrep.

www.Pharmacyprep.com

Social, Behavioral and Administrative Sciences

Regulated pharmacy technicians assume responsibility for their own actions, are accountable to the public and contribute to the overall functioning of the pharmacy to ensure a safe and healthy environment. Pharmacy technicians do not currently exist in Québec. Pharmacy assistants are other members of the pharmacy team who help support pharmacy technicians and pharmacists. They can perform certain tasks assigned to them by the pharmacy technician or pharmacist, but always do so under the direct supervision of the pharmacy technician or pharmacist. These may include tasks such as filing of pharmacy records, replenishing supplies and putting away drug orders. This may also include more advanced tasks such as computer order entry, counting, packaging, labelling and sometimes compounding of products if the pharmacist or pharmacy technician supervising them assigns them these tasks and verifies their work. Pharmacy assistants cannot transfer prescriptions, take verbal orders, check prescriptions or release products. Professionalism is described as the competence and skill expected and required of a professional. Professions have a formal knowledge base that is continually upgraded and practitioners usually require a long period of preparation and hands on training before they become independent practitioners. To become professional a candidate must meet certain educational standards, usually these set by regulatory bodies. Professions are committed to the public through their code of ethics: Healthcare professions are constantly changed with latest developmental technologies, identification of new diseases & treatments consequently there is emphasis on adopt to change and meet the rising expectations. ·

Personal attributes of professionals · Practice ethics and high moral standards · Reflection and self-awareness. · Responsibility and accountability of actions.

·

Cooperative attributes of professionals · Respect for patients. · Working as team. · Taking social responsibility.

A report on professionalism in medicine, the CMA (2001) states that, professionalism is: · A strong commitment to the well being of others. · High moral, ethical, integrity standards. · Mastery of body of knowledge and skills and continue education, or lifelong learning. · A high degree of autonomy. LAW BREAKING LAW CAUSE ILLEGAL AND HAS CONSEQUENCE. TAKEN CARE BY LAW ENFORCEMENT

ETHICS OR CONDUCT BREAKING ETHICS CAUSE MISCONDUCT. REGULATED AGENCY DISCIPLINE MISCONDUCT

Copyright © 2000-2018 TIPS Inc. Unauthorized reproduction of this manual is strictly prohibited and it is illegal to 42-2 reproduce without permission. This manual is being used during review sessions conducted by PharmacyPrep.

www.Pharmacyprep.com

Social, Behavioral and Administrative Sciences

Scope of pharmacist and pharmacy technician Pharmacist (R.Ph) Regulated Pharmacy technician (R.PhT) Patient care and therapeutic drug Solving technical problems (Rx receiving, related problem data entry, preparing, labelling, packaging and dispensing, insurance codes for billing). Counseling No Recommending therapies No Resolving DTPs (Drug interactions, side No effects, therapy monitoring) Assessing and recommending therapies No for minor ailments Liability insurance Liability insurance Narcotics ordering NO, pharmacist can delegate for inventory count. Receiving verbal narcotic (CDSA) Rx NO Receiving verbal Rx orders Yes Independent compounding of sterile Yes and non-sterile products. Schedule II recommendations NO Demonstrating medical devices Yes only for OTC devices (blood glucose monitor, blood pressure monitor, inhaler, aero chambers, peak flow meter, Epipen, thermometers, and pregnancy test kit. Immunization NO EXPANDED SCOPE (PROVINCIAL) ADAPTATION NO IMMUNIZATIONS NO ORDERING LAB TEST NO PHARMACEUTICAL OPINION NO ADVANCING NO

ASSISTANT Same as tech

NO LIABILITY

NO NO NO

NO

Professional boundaries: Professional boundaries are the defining lines that separate the professional relationship from any other behavior. The professional relationship is a purposeful relationship in which the client’s healthcare needs are priority. There are warning signs, which can prompt professional. It is health professional responsibility identifies and deals with boundary of violations if they arise. Patient and pharmacist relation is COVENENT. (Trust worthy, privacy, confidential and integrity). · Some warning signs that professional boundaries are being crossed are: · Noticing sexual content in interactions with the client. · Favouring one client’s care at the expense of another’s. · Giving/receiving gifts or continued or continued contact after discharge. · Acting and or feeling possessive about the client. Giving special attention, treatment to this client, which differs from that given to other clients. · Denying the fact that you have crossed the boundaries from professional relationship to non-professional relationship. Copyright © 2000-2018 TIPS Inc. Unauthorized reproduction of this manual is strictly prohibited and it is illegal to 42-3 reproduce without permission. This manual is being used during review sessions conducted by PharmacyPrep.

www.Pharmacyprep.com

Social, Behavioral and Administrative Sciences

Reporting requirements. Reporting wrong behavior of healthcare professional protect public and reputation of healthcare system. Each regulatory body have set different requirement for reporting by the members of the profession. Professional expectations: Public expect professional to offer activities with responsibilities and the best care. Here are some situations you may be expected to report: · Sexual abuse of patients · Misconduct, incapacity, incompetence · Unsafe practice Reporting requirement about abused patients: Sexually abused adult patient, pharmacist should report or give contact number of support groups or support agencies to patient. Reporting requirement about sexually abused child. Contact child associate society (CAS). Patients centred care values: · Respect. Respecting patients, wishes, concerns and strength · Human dignity. Caring for patients as whole or unique · Experts. Patient are experts of their own lives Prescription scope of practice of some healthcare · Timelines. Needs deserve a prompt response professions Regulated health professions in Canada · Pharmacist · Pharmacist · Physicians · Physician · Veterinarians · Physiotherapist · Dentists · Chiropractor (spine) · Nurse practitioners (NOT registered nurse, or registered practical nurse) · Respiratory therapist · Midwife · Podiatrist (Chiropodist) or Foot doctor · Optometrist · Registered Massage Therapist (RMT) Question Alerts! · Optometrist · Who can prescribe narcotics? · Midwife · Doctor, Dentist, veterinarians, midwife, podiatrist, · Veterinarian and nurse practitioner. · Dentist Pharmaceutical care delivery system · The major pharmaceutical care activities take place in the following systems · Community pharmacy · Hospital pharmacy · Long term care facilities · Specialty hospital units Community Pharmacies ·

Community pharmacies are considered one of the important components of the pharmaceutical care delivery system. However, health related services are primarily limited to dispensing medications and patient counselling.

Copyright © 2000-2018 TIPS Inc. Unauthorized reproduction of this manual is strictly prohibited and it is illegal to 42-4 reproduce without permission. This manual is being used during review sessions conducted by PharmacyPrep.

www.Pharmacyprep.com ·

·

Social, Behavioral and Administrative Sciences

They can be subdivided into three categories: 1. Chain-retail pharmacy services, 2. Individually owned pharmacy 3. Internet pharmacies (mail order pharmacies) Retail pharmacy and individually owned pharmacies work in a similar fashion. However, mail order pharmacy service is a little bit different in aspect of retail pharmacy. The latter lacks face-to-face patient counselling and OTC services.

Health Belief Model (HBM) · It was first proposed by Rodenstock and later modified by Becker. According to this model, the authors have hypothesized that people generally do not engage in preventive healthcare practices or participate in health detection and screening programs unless they view themselves vulnerable and/or have certain kinds of health relevant problems. · There are three categories of behavior related to healthcare, these include: 1. Health behavior 2. Illness behavior 3. Sick-role behavior Health behavior: It is defined as any activity undertaken by an individual who believes himself or herself to be healthy, for the purpose of preventing illness. · Weight reduction screening program · Exercise program · Stress reduction · Regular self-examination for breast or testicular cancer · Change in diet to reduce fat or cholesterol consumption Illness behavior: It is defining as any activity undertaken by an individual who believes he/she may be ill. · Discussing health problems with a family member, friend or pharmacist · Making an appointment to see physician · Self testing to determine blood pressure or blood sugar level · Experimenting with OTC products Sick-role behavior: It is defined as an activity undertaken by an individual who considers them to be ill or who have been diagnosed by a health professional as being ill. · Following medical advice · Taking medication as prescribed · Selecting an appropriate OTC product · Staying home from work or school

Tips____________________________________________________________________________ · · ·

The transtheoretical model of behavior change is starts with precontemplation & ends up with termination. Health behavior is defined as any activity undertaken by an individual who believes himself or herself to be healthy, for the purpose of preventing illness. Professionalism is described as the competence and skill expected and required of a professional.

Copyright © 2000-2018 TIPS Inc. Unauthorized reproduction of this manual is strictly prohibited and it is illegal to 42-5 reproduce without permission. This manual is being used during review sessions conducted by PharmacyPrep.

www.Pharmacyprep.com

Social, Behavioral and Administrative Sciences

Copyright © 2000-2018 TIPS Inc. Unauthorized reproduction of this manual is strictly prohibited and it is illegal to 42-6 reproduce without permission. This manual is being used during review sessions conducted by PharmacyPrep.

PharmacyPrep.Com

Pharmacy Management

43 Pharmacy Management Questions Alerts! Common questions in pharmacy exam is to ask! · Types of pharmacy ownership and formats of pharmacy · Financial statements like income statement and balance sheet · Human resources management and delegation · Inventory management (calculating turnover rate) Pharmacy management in the community pharmacy Pharmacy management in retail stores comprises several business issues, those discussed include: terminology commonly used in pharmacy business. Starting and managing pharmacy business, financial management, human resource management, and merchandise & inventory management. Starting and managing a pharmacy business Starting any business requires clear understanding and knowledge of the business. However, it is clear that lay people start pharmacy business. Like any other business, starting pharmacy business requires; business plan, organizing, staffing, and budgeting. Business plan: Business plan comprises: Business structure, Market area analysis, Business products and services, Competitive strategy, Positioning, Financing, Human resources, Operation and monitoring of performance. Types of business ownership Sole proprietorship · Advantage. Sole owner, low start up cost · Disadvantage. Unlimited liabilities Partnership · Advantage. Skills and knowledge can be shared · Disadvantage. Rate of conflicts is high.

Question Alerts! What type of pharmacy business structure is easy to start?

Corporations and Limited Liability Companies (Inc). The most common business form, business name often ends with “Inc”. · Advantage: Legal entity, several directors (several owners), and limited liabilities. · Disadvantage: Higher government involvement.

Copyright © 2000-2018 TIPS Inc. Unauthorized reproduction of this manual is strictly prohibited and it is illegal to 43-1 reproduce without permission. This manual is being used during review sessions conducted by PharmacyPrep.

PharmacyPrep.Com

Pharmacy Management

FORMATS OF PHARMACY CARPORATE FRANCHISE LABLAW (DRUG SHOPPERS DRUG STORE), Rexall MART (associates) WAL-MART No initial capital investment Fixed salaries for Manager

London Drugs, No initial investment Fixed salaries + bonus (PROFIT SHARING)

BANNER IDA, PHARMASAVE, GUARDIAN, Remedy Rx, Gives ownership Yes, need initial capital investment No fixed salaries You pay royalties to banners

INDEPENDENT MY PHARMACY

Yes, need initial capital investment No fixed salaries, and no royalties

Franchises: Associate franchises Associate have fixed term and payment plan from franchise corporations. · Owner of specific franchise location · Fixed and secure salary · Additional bonus or commission on performance. · No capital investment (do not own physical assets) · Central distribution Banner: own locations and pay franchises fee or commission to corporations. · Capital investment require because you own physical assets. · No fixed salary. · Franchising company provide name and marketing. · No central purchase require. Associate franchise No capital investment Central distribution Example: Shopper drug mart

Banner Capital investment No central distribution Example: IDA

TYPES OF PHARMACY OWNERSHIP STRUCTURES INCLUDE Retail pharmacy, Banner pharmacy, Chain pharmacy, Franchise pharmacy, Food store pharmacy, Mass merchandise, Specialty pharmacy, mail order pharmacy and central fill facilities (in hospital). BUSINESS LOCATION ANALYSIS. Methods applied in locating community pharmacy decision focused on. Region: Broad geographical area example country, and provinces.

Question Alerts! What is the most important in business location analysis? Market area analysis

Market area analysis: Basic information about market like population, sites consideration, and trading areas. Population. Trading area, business area, and market area analysis. Population is of interest to business and professional practice.

Copyright © 2000-2018 TIPS Inc. Unauthorized reproduction of this manual is strictly prohibited and it is illegal to 43-2 reproduce without permission. This manual is being used during review sessions conducted by PharmacyPrep.

PharmacyPrep.Com

Pharmacy Management

Trading area: Once location decisions are made regarding regional and market area, it is necessary to select particular trading area, type of retail operation desired. Example outlets, supermarkets, discount stores, national departmental stores (already established stores). Site considerations: There are scientific methods (techniques) to assess the site location. Site considerations: Important consideration in site selection is the relationship of cost to productivity. Physical characteristics of space in a building under consideration should be scrutinized. The shape of the space, its width and depth, exposed pipe and ductwork. Parking is a key concern. Techniques in assessing site locations. The use of ratios as rule of thumb is fairly common. Another rule of thumb deals with convenience and distance. Example sales per square feet. Financial Statements: There are three basic financial statements. A balance sheet, an income statement and a statement of retained earnings. Income Statement or profit and loss statements: Income statement is an indicator of sales, total sales, cost of the good sold, gross margin, expenses, total expenses, profit (net income), and profit and loss statements. · Unit price x unit volume = Sales % gross profit = · Unit volume x unit cost = Cost of goods sold · Sales – cost of goods sold = gross margin or gross profit Sales – Cost x 100 · Gross margin – expenses = Net profit Sales Balance sheet: Balance sheet is an important indicator of assets and liabilities. Balance sheet is an indicator of current assets (cash, current inventory, prepaid expenses), total current assets (fixture, furniture), liabilities (account payable), and long-term liabilities (over one year). Total liabilities. Net worth = assets – liabilities. · Cash + Account receivable + Inventory + Prepaid expenses = Total Current Assets · Total current assets + Fixed assets = Total Assets · Accounts payable + Notes payable + Accrued expenses = Total current liabilities · Total Current liabilities + Long term liabilities = Total liabilities · Total liabilities + net worth (owner equity) = Total liability and net worth Retained Earnings Statement. Retained earnings statement indicates business retained earning that includes dividend payments (share profit to share holders) that will reduce retained earnings, and net income that will increase retained earnings. Basic Management Principles Basic Management Functions includes · Recruiting/Staffing/Human Resource Management · Accounting · Strategic Planning/Organizing · Purchasing/ Negotiating · Inventory/Merchandising · Marketing · Dealing with regulatory officials Copyright © 2000-2018 TIPS Inc. Unauthorized reproduction of this manual is strictly prohibited and it is illegal to 43-3 reproduce without permission. This manual is being used during review sessions conducted by PharmacyPrep.

PharmacyPrep.Com

Pharmacy Management

Recruiting/Staffing/ Human Resource Management Described three commonly used steps in human resources. Job analysis, position description and job description. Job Analysis Job analysis is a concise and factual study of pharmacy’s staffing need. The scope of each job must be delineated, anticipated problems outlined, and hierarchy of position established. A thorough job analysis will possess all areas of the work to be done, and alert the pharmacy owner on duplication of functions. On long run, it will determine the responsibility of each employee and help prevent conflict. Position Description (Job advertisement). The position description outlines the main components of each position. A good position description should be relatively short (no more than two pages). General description of the job includes. Nature and the scope of the position, the main areas of responsibility, required qualification, experience, and other job related skills. Job description: The detailed job description should list, the entire task to be accomplished, in order of importance and described in detail. Pharmacists manager are responsible for the supervision of the activities of pharmacy and non-pharmacist staff in their activities of pharmaceutical services. This is useful in establishing the priorities of job functions within each position. Job analysis Determine how many staff need?

Position advertisement Advertise and interview

Job description Describe list of activities of staff.

A pharmacist manager maintains appropriate job description. Ensure adequate staff coverage for pharmacy activity levels. Delegation: Authorizing job to others. The principle of delegation consists of Question Alerts! three components responsibility, authority, and accountability. Delegation · Responsibility. Assigning a task or a project. · Authority. Given certain authority to operate a business. Like hiring personnel’s. · Accountability. Manager or staff pharmacist is accountable for completion of task or project. What can or cannot delegated to tech?

Scope of pharmacy technician

Counseling Verbal Rx order from prescriber Verbal narcotics on phone Counseling on natural products Witness to dispose narcotics or BZD OTC devices demonstration Independent sterile preparation and non-sterile compounders.

Regulated Technician no Yes NO NO Yes Yes Yes

Pharm Assistant (NOT regulated) NO NO NO NO NO NO NO

Copyright © 2000-2018 TIPS Inc. Unauthorized reproduction of this manual is strictly prohibited and it is illegal to 43-4 reproduce without permission. This manual is being used during review sessions conducted by PharmacyPrep.

PharmacyPrep.Com Rx transfer Data entry Monitor storage conditions Package (blister packs) and labelling Can they recommend sugar free vs sugar cough syrup

Pharmacy Management Yes Yes Yes Yes yes

NO YES YES YES NO

Employee motivation: The manager who supervises, controls, and motivates a management team. Basic principles of employee relation and motivations aspects are described by Maslow’s hierarchy. Maslow’s theory describes that every person has five basic levels of needs. Physiological needs, Safety and Security, Social needs, Esteem Needs, Self-actualization needs. Physiological needsàthe most basic level Safety and SecurityàPerformance report Social needsà every employee wants to become part of the group. An employer may ask the following questions: What degree of sharing of information about the business and it goals will be? Esteem Needsà If the three levels are satisfied, employees will become interested in addressing higher levels, through recognition of groups as leader or experts in a particular area. Self-actualization needsàthis is the highest level of needs, wherein employees will strive toward greater accomplishment and responsibility, because it gives them personal satisfaction. Workplace safety Workplace safety and Insurance board (WSIB) Workplace Hazardous Materials Information Systems (WHMIS) Material Safety Data Sheet (MSDS) Spill kits Risk Management; Strategic Planning SWOT Analysis (Strength, Weaknesses, Opportunities, Threat) analysis. STRENGTH WEAKNESS How can you maximize, SERVICES, LANGUAGE.. SPECIALTIES.. DRIVE THROUGH… FREE BP CHECK, FREE DELIVERY.. INTERNET REFILL..FOLLOW UP… 24 HRS… WEEKENDS…WEIGHING SCALE…DIABETIC CLINIC.. DIETICIAN…SMOKING, VACCINE..TRAVEL CLINIC.. BLOOD SAMPLE COLLECTION… OFFER DISCOUNT.. OPPORTUNITIES

How can you minimize, THEFT, POOR MANAGEMENT, NOT UPTODATE.. POOR CUSTOMERS SERVICES…

How can you maximize, SPECIALTIES.. SERVICES..

How can you minimize, competition,

THREAT

Copyright © 2000-2018 TIPS Inc. Unauthorized reproduction of this manual is strictly prohibited and it is illegal to 43-5 reproduce without permission. This manual is being used during review sessions conducted by PharmacyPrep.

PharmacyPrep.Com IF ANY SERVICES NOT AVAILABLE IN THE AREA..

Pharmacy Management regulations change,

Project Management: Project management functions, risk management, human resource management, and communication management. Quality Management: Four principles of quality management "P-D-C-A" Plan, Do, Check and Act. Merchandising: Visual selling or visual display merchandise using floor plan (Planogram). Question Alerts! What is merchandising? Visual display of item in store Inventory and merchandise management · SKUs: Stock keeping unit of each size, strength, format of stock items in one unit. Individual items of inventory are referred to SKU. · UPC: universal Product Code. ABC analysis (Pareto’s law): (20:80 LAW) · Category A. 20% of products stocked represent 80% of the inventory cost. · Category B. 15% of the products represent 15% of Question Alerts! the inventory cost. Pareto's law. 20% of products stocked · Category C. 65% of the products stocked represent 80% of the inventory cost. represent 5% of the inventory cost. · By closely monitoring the 20% of the items in category A, control is gained over 80% of the inventory costs. Identify your top 20% SKUs and maximize inventory control efforts on these. Some of the items in the 80% category of SKU should be probably not be stocked or sold on a special order basis. Third Party Insurance. The payment system can be classified into two different categories. Retrospective payment and Prospective payment. Retrospective payment (reimbursement) System. In this type of payment service, payment is generally made after the service has been provided by the hospital. The payment depends on the service actually provided by the hospital. The payment depends on the service actually provided by the hospital during the patient’s stay. There is little incentive for a hospital to keep a patient in the hospital for long because payment increases as more costs are incurred. It is not preferred by third party insurances. Prospective payment (reimbursement) system (PPS). This was introduced in 1982. A form of payment (reimbursement) system usually pays the hospital on the basis of DRG (Diagnostic Related Group) services. DRG includes lists of different kinds of illnesses and a fair amount of cost is required to treat such illnesses. Under this payment service, the hospital will reimburse a predetermined amount specific to the DRG in which the patient is classified. This single payment will cover the entire episode of care regardless of the patient’s stay in the hospital, the number of tests performed, or the number of drugs that are used.

Copyright © 2000-2018 TIPS Inc. Unauthorized reproduction of this manual is strictly prohibited and it is illegal to 43-6 reproduce without permission. This manual is being used during review sessions conducted by PharmacyPrep.

PharmacyPrep.Com

Pharmacy Management

Commercial third party insurances: They offer a variety of services with the condition that an initial certain amount of money should be spent by the consumer which is known as “deductible”. Also, most of them do not cover charges for certain types of services. They also collect the prepaid fixed monthly fees from the enrolled customers. Maximum allowable cost (MAC): The maximum amount that will be paid by a third party to a pharmacy when the drug is available from more than one source. Estimated acquisition cost (EAC): the third party estimate of the prices paid by a pharmacy for a particular drug product. Actual acquisition cost (AAC): The actual price paid by the pharmacy after all trade, volume and cash discounts. Average wholesale price (AWP): The published list price of a particular product. Cash flow: a summary of cash receipts and disbursements of a business, for a defined period of time. Cash management: freeing up funds for operating purposes by minimizing assets and maximizing liabilities and specifically accounts payable. Coinsurance: It is one type of cost sharing plan in which patient pay a specific percent of all expenses. Co-payment: It is one type of cost sharing plan in which patient has to pay fixed amount each time a service is provided. OR patient pays fixed amount for each prescription. Deductible: It is one type of cost sharing plan in which patient has to pay a specified amount during a specific period of time. Before benefits are paid by third party. DEDUCTIBLE Pay onetime in the beginning of every year. A 65yo patient of your pharmacy presents with 4 prescriptions. Total cost of Rx is $156. The senior’s insurance deductible for the year is $100. For this transaction. Patient pays? $100

CO-PAYMENT Pay each time fixed amount for every prescription A 65yo patient of your pharmacy presents with 4 prescriptions. Total cost of Rx is $156. Patient insurance pays total Rx cost. Your pharmacy bills $2 for each prescription. Patient pays? $8

CO-INSURANCE Each time you share the expenses with insurance A 50 yo women presents prescription with 4 medications. The total cost of Rx is $156. Her coinsurance plan covers 50%. For this transaction. Patient pays? $78

Business Indicators and Financial analysis Functions of ratios that indicates overall financial position of pharmacy: Ratios indicating profitability Ratio indicating efficiency Ratio indicating liquidity and solvency Ratio indicating financial position Ratios indicating profitability: · Net profit to net sales (NP:NS): The normal ratio lies between 3 to 7%. Copyright © 2000-2018 TIPS Inc. Unauthorized reproduction of this manual is strictly prohibited and it is illegal to 43-7 reproduce without permission. This manual is being used during review sessions conducted by PharmacyPrep.

PharmacyPrep.Com · · ·

Pharmacy Management

Net profit to total assets (NP:TA): The normal acceptable ratio lies between 10 to 15%. New profit to inventory (NP:IN): The normal acceptable ratio lies between $0.21 to $0.27. Net profit to net worth (NP:NW): The ratio lies between 20 to 25%, and 15% is acceptable for older pharmacies and 40% is attainable for newer pharmacies.

Ratio indicating efficiency Question Alerts! Inventory turnover rate (IN: TOR): It is 1) Turnover rate indicates? Inventory efficiency normally calculated by dividing the cost of 4) TOR is days, inventory in hand. Examples TOR 4 is, in a goods sold by an average beginning and year 4 times inventory bought and sold (365/4). ending inventory. The inventory turnover rate should be 4 as a minimum, with a target of 6 or higher. Theoretical number of times during a special period, usually one year, that inventory is bought and completely sold. · Average turnover rate is for retail pharmacy 4 min and 6 max or more. For hospitals turnover rate is 8 to 10. · Inventory turnover rate = Cost of goods sold/average inventory capital. or

Inventory turnover rate = Cost of goods sold Beginning inventory (opening stock) + End of inventory (closing stock) 2 Example if inventory purchased two times a year, one at beginning of inventory and end of inventory.

TOR 6 frequent shortage

Net sales to networking capital (NS:NWC); The normal ratio range is 4 to 8. Ratios greater than 8 are considered inadequate capitalization or overtrading. A value below 4 indicates under trading or too much capitalization. Net sales to inventory (NS:IN). The ratio normally ranges from 6 to 9. Net sales to net worth (NS: NW): The normal ration range is from 3 to 8. Greater than 8 is considered under capitalization and overtrading while below 3 indicates under trading. Net Sales to net worth = Net sales/ Net worth Account receivable collection time (A/R CT): This ratio is a direct measure of efficient credit management. Normally, a 30-day collection period is a reasonable target. A/R =

Year end account receivable/ Mean credit sales per day

Account payable remittance time (A/P RT): This is normally calculated by dividing year end accounts payable divided by mean credit purchase per day. A/P =

Year end account payable Mean credit purchase per day

RATIO INDICATING LIQUIDITY AND SOLVENCY:

Copyright © 2000-2018 TIPS Inc. Unauthorized reproduction of this manual is strictly prohibited and it is illegal to 43-8 reproduce without permission. This manual is being used during review sessions conducted by PharmacyPrep.

PharmacyPrep.Com

Pharmacy Management

Liquidity normally measure’s a pharmacy’s ability to meet its current liabilities with little or no interruption in the regular conduct of business. Solvency measures a pharmacy’s ability to meet current liabilities with moderate change in the composition of current assets. Acid test ratio: It is also known as quick ratio. The normal ratio is 1:1.

Question Alerts!

Definition of liquidity and solvency (bankruptcy)

Current ratio: The minimum standard value is 2:1. Inventory to networking capital (IN: NWC). Mean inventory is the average of the beginning and ending inventory for the accounting period. This ratio is an indirect measure of liquidity and solvency. A high ratio indicates low liquidity and too much inventory. A ratio if 80% is a reasonable target. Ratio indicating financial position Total liabilities to net worth (TL: NW). It is the most direct measure of the financial position of the pharmacy. A ratio of 50% or lower is acceptable. Founded debt to networking capital FD: NWC). It is also expressed as a percentage. Long term liabilities are defined as liabilities extending longer than one year. The normal acceptable value of a ratio is 20 to 25. Fixed assets to net worth (FA:NW). This helps to identify over investment in fixed assets. A high value indicates over investment in fixed assets while a low value indicates there is a need for remodelling. The target value would be a 20% or less. Business Math: Calculating markup% = [(sales price-cost)/cost] x 100 Calculating sales price = Cost+ (Cost x %markup) Calculating cost = Sales price/(1+%markup) % Gross margin = [(Total SALES- COST)/SALES] x 100 Cost Sales price/(1+%markup)

Mark up (gross profit) [(sales price-cost)/cost]x100

Sales Cost+(Cost x %markup)

Marketing in pharmacy “4 Ps “of marketing management. These activities, which are under the direct control of the business, were known as the “4 Ps” of marketing product, place, price, and promotion. Promotion: The methods of promotions are door-to-door flyers, local radio, TV, posters and news papers. Management use of Structure-Process-Outcome component (SPO) Measure of SPO · Structure component. Examples. Facilities, equipment, staffing, and personal qualification · Process component. Examples. Provider, healthcare system etc. · Outcome component. Examples. Mortality, morbidity, and consumer satisfaction

Copyright © 2000-2018 TIPS Inc. Unauthorized reproduction of this manual is strictly prohibited and it is illegal to 43-9 reproduce without permission. This manual is being used during review sessions conducted by PharmacyPrep.

PharmacyPrep.Com

Pharmacy Management

Tips 1. 4. 7.

POS 2. salaries Balance sheet 5. Turnover rate Nature and scope of 8. Market area analysis position 10 Sole proprietor 11 Partnership 13 Cash 14 Account receivable 16 Building · What are the forms of business structure in Canada? ( · What is banner pharmacy ownership? ( ) · What is the most important in business location analysis? ( ) · What financial statement includes sales of prescription drugs? ( · What financial statements include assets and liabilities? ( ) · Examples of current assets include? ( ) · Examples of fixed assets include? ( ) · What is the most expensive in pharmacy business? ( ) · Cost of goods sold/average inventory capital ( ) · The average turnover rate for a pharmacy ( ) · Staffing need is described as? ( ) · Position description includes ( ) · Point of sale system is ( )

3. 6. 9.

income statement 4 to 6 Corporation

12 Franchise 15 Furniture ) )

Select True or False Statements Backorder is (If an order is received, and an item has 'BO' means that item is out of stock at the supplier. The 'BO' signifies that the supplier will ship when the item becomes available or it may need to be re-order). A pharmacy has a net income of $90,000.00, what would be best purchasing offer if you want to have return on investment of 15%

Copyright © 2000-2018 TIPS Inc. Unauthorized reproduction of this manual is strictly prohibited and it is illegal to 43-10 reproduce without permission. This manual is being used during review sessions conducted by PharmacyPrep.

www.PharmacyPrep.com

Pharmacoeconomics

44 Pharmacoeconomics Questions Alerts! Common questions in pharmacy exam is to ask! · Definition. A study of affordability of drugs or therapy COST to SOCEITY. · Pharmacoeconomic methodologies like cost effective analysis (CEA), cost minimization analysis (CMA), cost utility analysis (CUA), and cost of illness analysis (COA). Pharmacoeconomics is “the description and analysis of the costs of drug therapy to healthcare systems and society.” Pharmacoeconomics research, identifies, measures, and compares the costs and consequences of pharmaceutical products and services. PE has been defined as the description and analysis of cost of drug therapy the health care system and society. It is process of identifying, measuring, and comparing the cost, risks/consequences and benefit of program, services, or therapies and determining which health alternative products the best health outcome for resource invested. Pharmacoeconomic Methodologies: There are some scientific methods are used to evaluate pharmacoeconomics. · Cost-benefit analysis (CBA) · Cost-effectiveness analysis (CEA) · Cost-minimization analysis (CMA) · Cost-utility analysis (CUA) · Cost-illness analysis (CIA) Methodology Cost-benefit analysis Cost-effectiveness analysis.

Cost (spend) Dollar Dollar

Cost-minimization analysis

Dollar

Outcome unit

Dollar Units such as blood pressure mm Hg, blood glucose or units of clinical effects (example costs per year of life saved). The most commonly used in PE calculations. Example the selection of drugs in hospital formularies. Also used to determine the first line therapy of clinical practice guidelines. Assumed to be equivalent in comparative groups. Only cost is compared, so cheap intervention will be chosen.

Copyright © 2000-2018 TIPS Inc. Unauthorized reproduction of this manual is strictly prohibited and it is illegal to 44-1 reproduce without permission. This manual is being used during review sessions conducted by PharmacyPrep.

www.PharmacyPrep.com Cost-utility analysis

Cost of illness analysis

Dollar

Pharmacoeconomics Quality-Adjusted Life Year (QALY). This is used when the impact a health-related quality of life is important outcome to treat a condition. No comparison is made. Choose the best one.

Quality-Adjusted Life Year (QALY): The results of CUA analysis are normally expressed in terms of quality adjusted life year gained. QALY includes both improvements in quantity of life and quality of life, QALY is used when, and quality of life is the only outcome. Quality and quantity of life are health outcomes. When intervention affects both mortality and morbidity and combined unit of outcome is desired. Cost-benefit analysis (CBA): It is a basic tool that helps to improve the decision-making process in the healthcare program. Cost and consequences: The outcome is measured in dollars. This study calculates all of the possible benefits that may occur from the program. All the benefits must be expressed in dollar value. Disadvantage: (require the economic evaluation of a human life.) It is very difficult to assign dollar values to non-financial benefits, e.g. benefits of the program that may improve a patient’s life. Cost-effective analysis (CEA) (dollars à clinical effects · This technique is used to make a decision in order to select the most cost effective intervention from the available alternative. · Cost and consequences · The output measure of this type of study is a health related measure rather than a financial, exampleà blood pressure, mm Hg, or blood glucose. Cost-minimization analysis (CMA) (dollars à equal in both groups · It is defined as: when two or more interventions are examined and assumed to be equivalent in terms of a given outcome. Cost associated with each intervention may be examined and compared. Example. The comparison of cost of two ca-channel blockers, which may successfully produce similar blood pressure reduction, patterns in a selected group of patients. Cost-utility analysis (CUA) (dollars à QALY) · It is an economic tool that measures the consequences in terms of outcome of the program in terms of quality and quantity of life QALY. · Cost and consequences · The outcome measured in CUA is cost per QALY (Quality Adjusted Life Year). · When the objective is to compare a gold standard intervention that already has the cost per QALY. QALY is calculated by multiplying the utility value obtained for the specific health condition with quantity of life year spent in that specific health condition. Comparison can be made for program and intervention. Cost of illness analysis (COI or CIA) à does not address both cost and consequences. · It is very important for evaluating new therapies. · No cost and consequences

Copyright © 2000-2018 TIPS Inc. Unauthorized reproduction of this manual is strictly prohibited and it is illegal to 44-2 reproduce without permission. This manual is being used during review sessions conducted by PharmacyPrep.

www.PharmacyPrep.com

Pharmacoeconomics

Healthcare Outcome Research · Outcome research (OR): The study of health care interventions (treatments, such drug therapy, surgery, palliative therapy etc) and healthcare quality that are evaluated to measure the extent to which optimum desirable outcome can be reached. · The purpose of OR is to assess the value of a program or therapy under study: · ECHO model: Provides framework for comprehensive evaluation of outcome. · Economic outcome: Acquisition cost associated with care, labor cost associated with care, treat side effect reactions, cost of treatment failure, hospital readmission, cost of emergency room, clinic visits. · Clinical outcomes: length of hospital stay, side effect reactions, hospital readmission and death. · Humanistic Outcome: Patient satisfaction, functional status of validated instruments, quality of life assessment. Health Related Quality of Life (HRQOL): · QALY focuses on all aspects of life. However, HRQOL only focuses on patient non- clinical information such as functional status, well being, perception of health, return to work from illness and other outcome that are effected by illness · Health Related Quality of Life (HRQOL): According to the WHO (world health organization), health is defined as complete physical, mental and social well-being. · HRQOL normally focuses on non-clinical components of healthcare such as functional status, well-being, and other important health- related outcomes. · HRQOL has a very large database. This database is prepared either by personal interviews, by telephone interviews, or by postal survey. · Personal interviews, telephone interviews and postal surveys are defined standardized questionnaires or instruments of HRQOL. Question Alerts! What is SF 36? Techniques of pharmacoeconomic assessment Short Form 36 (SF 36): The SF 36 was designed for use in clinical practice and research, health policy evaluation, and general population survey. Budget impact analysis (BIA) Multi-attribute Utility Theory (MAUT) · Used in assessing utilities. This include, clinical, financial effects as well as quality of life. Example. A hospital administrator may view clinical outcome 20%, financial outcome 70% and the quality of life 10%. The individual perspective will have a major impact on the final decision made, based on varying levels of priority chosen for evaluation. Willingness To Pay (WTP) · This technique is used to assess the perceived value or benefit of a product and service. Pharmacoeconomic Resources Methods for Economic Evaluation of Health Care Programmes, 3rd ed. Michael F. Drummond, Mark J. Sculpher, George W. Torrance, Bernie J. O'Brien, and Greg L. Stoddart. Oxford University Press.

Copyright © 2000-2018 TIPS Inc. Unauthorized reproduction of this manual is strictly prohibited and it is illegal to 44-3 reproduce without permission. This manual is being used during review sessions conducted by PharmacyPrep.

www.PharmacyPrep.com

Pharmacoeconomics

Review example of Cost Benefit Analysis (CBA) Cost

Benefit

drug Administration Monitoring Side effects Total cost Days at work ($) Extra months of life ($)

Drug A $1000 $100 $50 $75 $1225

Drug B $1100 $10 $0 $0 $1110

750

1200

C/B = 1 is loss Review example of Cost effective analysis (CEA): A marginal cost effectiveness ratio is calculated by determining the added cost divided the added benefit. Cost

Outcome

dose Hospitalization Stay require Total cost Extra years of life Cost effectiveness ratio

Drug A $300 $300 $600 $1200 2 600/2 $300

Drug B $300 $300 $600 $1200 6 600/6 $100

Review example Cost Minimization analysis (CMA): Assume the outcome to be equivalent in comparable groups. Cost

Outcome

Drug Administration Monitoring Side effects Sub total cost

Drug A $250 $75 $100 $100 $525

Drug B $430 $0 $10 $15 $455

Antibiotic effectiveness

80%

80%

Result = Cost Drug A>Cost of Drug B Cost utility analysis: A CUA takes patient preferences, also referred to as utilities, into account when measuring health consequences. The most common unit used in conducting CUA is QALYs, which incorporates both quality and quantity of life.

Copyright © 2000-2018 TIPS Inc. Unauthorized reproduction of this manual is strictly prohibited and it is illegal to 44-4 reproduce without permission. This manual is being used during review sessions conducted by PharmacyPrep.

www.PharmacyPrep.com

Cost

Utilities

Pharmacoeconomics Drug A

Drug B

Drug Administration Monitoring Side effects Total cost Extra years of life

$250 $75 $100 $100 $525 2

$430 $0 $10 $15 $455 3

Quality of life

0.33

0.25

QALY

0.5

0.40

Cost utility ratio

$525/0.5 $1050

$455/0.4 $1137

QALY = Quality Adjusted Life Years

Tips 1. 4 · · · · · · ·

Units 2. Society 3. Short Form 36 Cost Minimization 5. Quality Adjusted Life 6. Dollars Analysis Years 7 cost utility analysis What is QALY ( )? Pharmacoeconomics is the study that determine affordability of drugs to ? ( ) Cost minimization analysis outcome is measured in? ( ) Cost effectiveness analysis outcome is measured in? ( ) QALY is outcome of? ( ) What is SF 36? ( ) To compare cheaper intervention, what methodology is used? ( )

Copyright © 2000-2018 TIPS Inc. Unauthorized reproduction of this manual is strictly prohibited and it is illegal to 44-5 reproduce without permission. This manual is being used during review sessions conducted by PharmacyPrep.

www.PharmacyPrep.com

Pharmacoeconomics

Copyright © 2000-2018 TIPS Inc. Unauthorized reproduction of this manual is strictly prohibited and it is illegal to 44-6 reproduce without permission. This manual is being used during review sessions conducted by PharmacyPrep.

www.Pharmacyprep.com

Drug Development Process

45 New Drug Development Process Questions Alerts! Common questions in pharmacy exam is to ask! · Role of Health Canada. Drug market authorization in Canada is approved by Health Canada after review based on “safety, effectiveness, and manufacturing quality” · Drug approval process Clinical trials phase 1 to phase 4 · Adverse drug Reaction Reporting Drugs are approved by the therapeutic directorate of Health Canada. Pre market ……………………………………………………………………….. … Post-market Physician Prescribing

prescribing Pre-clinical studies

Clinical trials (phase I, II, and III Clinical trial applications

Regulatory product submission Submission review

New drug submission (NDS) or abbreviated NDS (generic products) or supplemental NDS (changes in existing products)

Market authorization By health Canada Notice of Compliance (NOC). Drug Product Database Labelling and product monograph

Physician

Physician

prescribing

CADTH Common Drug Review (CDR) Public and private drug plans/policies listing and reimbursement decisions

Public access Prescribing practices Real-world use studies (phase IV) by ADR Pharmacovigilance. Therapeutic costeffectiveness. DRUG UTILIZATION REVIEW Surveillance, inspection, and investigation for safety and regulatory compliance.

Copyright © 2000-2018 TIPS Inc. Unauthorized reproduction of this manual is strictly prohibited and it is illegal to 45-1 reproduce without permission. This manual is being used during review sessions conducted by PharmacyPrep.

www.Pharmacyprep.com

Drug Development Process

STEPS INVOLVED IN DRUG APPROVAL PROCESS Preclinical trials Clinical trial application Phase 1 to 3 clinical trials New drug submission review Notice of compliance (NOC); DIN, drug product database; drug monograph Patented Medicine Price Review Board (PMPRB) Post marketing surveillance (phase IV, Canadavigilance ADR) Common Drug Review (CDR) by the CADTH Drug included in provincial and private formularies Drug utilization review (DUR) CADTH: Canadian Agency of Drug and Technologies in Health. Conducts common drug review (CDR) after drug approved by health Canada. Notification issued indicating that manufacturer has complied with FDA regulations. Notice of compliance (NOC) is issued following satisfactory review of submission. Pre-clinical research (in animals) ¯ Phase I trials (small healthy human population, PK, Safety) ¯ Phase II trials (small disease human population, safety, effectiveness) ¯ Phase III trials (safety, dosage info, decisive phase) ¯ Trial review and approval ¯ Phase IV trials (post marketing surveillance), adverse drug reaction (ADR) monitored NO phase I trials for chemo preps. Drug identification number (DIN) A DIN is made up of exactly 8 digits. Only numbers allowed (0-9). Unlike other drug classes no meaning can be derived from the number itself. (Similar drugs are NOT clustered together at similar numbers). DIN is printed on the label. Preclinical Research Stage Pre-clinical research

# of patients None

Duration 18 months - 3 years

Phase I trials Stage # of patients Phase I 20-100 · Human Healthy volunteers · Small or limited population

Duration Up to 2 years

Purpose Laboratory investigation for efficacy and toxicity Purpose Safety and dosage

Copyright © 2000-2018 TIPS Inc. Unauthorized reproduction of this manual is strictly prohibited and it is illegal to 45-2 reproduce without permission. This manual is being used during review sessions conducted by PharmacyPrep.

www.Pharmacyprep.com ·

Designed to establish the effects of new drugs in humans, specifically determine a pharmacokinetic studies, drug toxicity, absorption, distribution and metabolism.

Phase II trials Stage Phase II · · ·

Drug Development Process

# of patients 100-300

Duration Several months-2yrs

Purpose Some shows term safety but mainly effective

Test on disease patients Slightly larger but limited population Tested safety and efficacy in slightly larger populations who afflicted with the disease or conditions, which the drug is developed.

Phase III Trials Stage Phase III

# of patients 100-3,000

Duration 2-3 years

Purpose Safety, relative effectiveness dosage

Tested on Disease patient Larger population than phase II Last pre-approval round of testing. Test the new drug with comparison of standard drug. The results of new trials usually provide the information that included in the package insert labelling. · After approval drug from phase III, this can be sold in market. Trail Review and Approval Stage # of patients Duration Purpose Review and None 1-2 years Safety, effectiveness, Approval dosage · · ·

Phase IV Study Stage # of patients Duration Purpose Phase IV 100- several thousand No limit Safety, effectiveness dosage After drug has been approved. Studies are conducted to compare the drug to a competitor. Explore addition side effects.

Tips 1.

Notice of Compliance

2

Phase III

3.

Disease patient, large number of patient, it is decisive phase

4.

Animals

5.

6.

Health Canada

7.

Patented Medicine Price Review Board PMPRB Disease patient and smaller number

8.

Post marketing, inspection of safety and regulatory compliance Pharmacy Manager/Owner

9.

Healthy volunteers, pharmacokinetics & safety

11

NPN

12

Natural Product Directorate

10

Copyright © 2000-2018 TIPS Inc. Unauthorized reproduction of this manual is strictly prohibited and it is illegal to 45-3 reproduce without permission. This manual is being used during review sessions conducted by PharmacyPrep.

www.Pharmacyprep.com · · · · · · · · · · · · · · · · ·

Drug Development Process

Who approves and authorizes the sale of medications in Canada? ( ) Who sets the prices of prescription drugs in Canada? ( ) Who sets the prices of over the counter drugs in Canada? ( ) Pre-clinical studies is done in? ( ) Phase I clinical studies is done in? ( ) Phase II clinical studies is done in? ( ) Phase III clinical studies is done in? ( ) Phase IV clinical studies is done in? ( ) What is notice of compliance (NOC)? ( ) Decisive Phase in clinical trials is ( ) Approves the prescription & OTC medications ( ) Natural Products Number is ( ) Terminology: Medline or Pubmed: Online source of indexes and abstracts. Red book: Pharmaceutical prices are described in this book. SOP = Standard Operating Procedures used in preparation and formulation of pharmaceuticals GMP = Good Manufacturing Practices or Procedure, the guidelines that determined by FDA for pharmaceutical preparations. GLP = Good Laboratory Practice

Copyright © 2000-2018 TIPS Inc. Unauthorized reproduction of this manual is strictly prohibited and it is illegal to 45-4 reproduce without permission. This manual is being used during review sessions conducted by PharmacyPrep.

www.PharmacyPrep.com

Epidemiology

46 Epidemiology Questions Alerts! Common questions in pharmacy exam is to ask! · Cross sectional, case control and cohort studies · Clinical study designs like parallel and cross over designs · Types of bias and confounders · Types of blinding · The credible clinical studies are randomized double blind General Design Elements: These are the several important tools that researchers use to improve the validity of a clinical trial, its ability to achieve the clinical endpoints, and its ability to provide the highest possible level of evidence.

Clinical Studies

Descriptive studies Case reports

Case series

Cross sectional (or) Prevalence (now) Collect the data after both disease and exposure. Determine prevalence rate No incidence rates

Explanatory Studies Meta analysis

Experimental (clinical trials)

Observational

Case control or retrospective or past. Good for rare disease, initial etiology. No incidence and prevalence. Outcome is measured by odds ratio method.(relative risk) Example. Antibiotic resistance pattern for the past 10 yrs.

Cohort or follow up Future or prospective Get incidence and estimate. Calculate the risk Example post marketing surveillance (ADR monitoring)

Copyright © 2000-2018 TIPS Inc. Unauthorized reproduction of this manual is strictly prohibited and it is illegal to 46-1 reproduce without permission. This manual is being used during review sessions conducted by PharmacyPrep.

www.PharmacyPrep.com CROSS SECTIONAL Present: Disease and exposure Prevalance only Asses the prevalence of an outcome in a broad population at one point in time.

Epidemiology CASE CONTROL Present: Disease Past: Exposure No prevalence and NO incidence. Measured by odd ratio Compares histories of people with a condition to a group without condition

COHORT Present: Exposure Future: Disease Incidence only Follows a group of people to track risk factors and outcomes over time.

Important concepts Epidemiological study designs to study drug use and outcomes in large populations. Prevalence: Collect data after both disease and exposure. or the number of cases of disease that are present in population at given time. Measured by cross sectional study. Incidence: Collecting data from exposed. The number of new cases that develop in given period of time. Measured by cohort studies. Prevalence Measures existing cases of diseases and expressed as proportion Measures all side effects of a drug Total all number of cases

Incidence Measures NEW cases of disease and is expressed in person, time, and units. Measure a NEW side effect of drug. Only new cases

Cross sectional examples · Determines or identifies the risk factors and etiological agents or disease or conditions. · To evaluate receiver characteristics of diagnostic procedure. · To evaluate a new laboratory tests. · Advantage: results appears at the time of study

Fig 55.1 Fig 55.1

Copyright © 2000-2018 TIPS Inc. Unauthorized reproduction of this manual is strictly prohibited and it is illegal to 46-2 reproduce without permission. This manual is being used during review sessions conducted by PharmacyPrep.

www.PharmacyPrep.com

Epidemiology

Case control study A case control study compares a population of patients with a specific pre-existing condition to a similar population without – to establish the influence of a disease or other event (e.g. Intervention, hospitalization, death). Data collection is usually done using medical records and patient interviews. Example. Studies of antibacterial resistant for the treatment of pneumonia. This study often the best way to study rare diseases that develops over a long period of time. It can be less reliable than randomized controlled trials or cohort studies because the difference that can be made about the influencing factors is limited by bias. Pros and Cons · (+) Requires fewer subjects than cross-sectional trials · (+) Only way to study long-term, rare disorders · (+) Fast and inexpensive · (-) Difficult to select control · (-) Potential bias form patient selection and recall · (-) Exposure status relies on records and recall Odds analysis: Probability that an event occurring vs the probability that will not occur. Odds ratio: measures association with exposed and outcome. Odd Ratio (OR): The odds represent the number of patients in a given group with an event divided by number of patient in the same group without it. OR = (A/C)/(B/D) OUTCOME The 2 x 2 table OR = AD/BC Exposure Yes No A= number of exposed cases B=Number of exposed NON cases Yes A B C=Number of unexposed cases NO C D D= Number of unexposed non case Tips. Always divide experiment by control (goes down in table) Sample of 100 people received flu vaccine 10 people has flu and sample of 100 people NOT received flu vaccine 40 people had flu during the same period. What is odd ratio? A. 1 C. 10 D. 40 E. 100 For disease

The odd of people taken flu vaccine 10 to 90 or 1:9 while The odd of people NOT taken flu vaccine 40 to 60 or 4:6

OR = odds of exposure among cases Odds of exposure among control

10/90 40/60 0.1/0.9 = 0.1 x 0.6 0.4/0.6 0.9 x 0.4 = 0.16 IF OR > 1 those who were exposed have an increased risk of the

outcome

IF OR < 1 those who were NOT exposed have an increased risk of the outcome

For treatment

Exposure

OUTCOME Yes No

Yes NO

A C

Exposur e Yes NO

B D

OUTCOME No Yes B D

A C

OR>1 those who were exposed have an increased risk of the outcome?

Copyright © 2000-2018 TIPS Inc. Unauthorized reproduction of this manual is strictly prohibited and it is illegal to 46-3 reproduce without permission. This manual is being used during review sessions conducted by PharmacyPrep.

www.PharmacyPrep.com

Epidemiology

Cohort Studies An observational study that allows a large population of patients with a specific pre-existing condition or treatment for a period of time, comparing them with another group unaffected by the affliction or the treatment. Cohort studies are employed when it would be unethical to test the effects of a condition or treatment on otherwise healthy patients (e.g. obesity in children), often in the etiology or prognosis of a disease. Pros and Cons · (+) Lower cost, easy administration (compared to RCTs). Question Alert! · (+)Can establish time and direction of events 1) Methods for continually monitoring · (+) Ethical side effects and safety of medication · (-) Not randomized, difficult to blind use in large populations. · (-) Control difficult to identify · (-) Differences can take a long time to develop. · (-) Participants can withdraw, develop other conditions, or die. CLINICAL TRIALS (EXPERIMENTAL TRIALS) Randomized Controlled Trials (RCTs): A clinical trial that answers questions about the effectiveness of different therapies by studying the effect of an intervention on randomized patients. This can be achieved by using 2 possible methodologies designed to reduce bias and promote comparison between the intervention group and 1 or more control groups (also known as ‘arms’) treated with a placebo and/or the current standard of care. RCTs are the standard trial design for answering clinical questions about therapy effectiveness. Example: A clinical trial of ACEi to determine the blood pressure lowering effect! Independent variable Dependant variable ACEi, lifestyle, SALT ETC. Blood pressure Predictor variable Outcome variables Randomization: Participants are selected by computer codes, randomization improves the validity of a trial’s results. Stratification: In some trial designs, patients may have important differences that researchers know will affect the outcome of the intervention, such as different stages in disease state or concurrent conditions. In such a case, patients may first be intentionally divided into 2 or more groups, example “smokers and non-smokers” or diabetics and non-diabetics”. Stratification enables researchers to evaluate how an intervention affects both groups by studying whether or not there is a difference in the subgroup. Population. Population, from which trial subjects will be selected. Sample size. The total number of participants included in a clinical trial is determined by the change the researchers want to observe in the chosen primary outcome. Placebo. A placebo is a device that imitates the active intervention but has no therapeutic value. The placebo effect is important when discussing adverse events (AEs) with a healthcare practitioner. AEs from the group taking the placebo can be compared with those from the group taking the intervention to determine to what degree adverse events are really attributed to the active agent. Copyright © 2000-2018 TIPS Inc. Unauthorized reproduction of this manual is strictly prohibited and it is illegal to 46-4 reproduce without permission. This manual is being used during review sessions conducted by PharmacyPrep.

www.PharmacyPrep.com

Epidemiology

Multi-Centre Trials Multi-centre trials are conducted at more than one location, often by more than one investigator, but using one central protocol. When conducted at multiple centres in more than one country they are referred to as multicentre, multi-national trials. Systematic Reviews Are critical assessment and evaluations of primary research trials that use rigorous methods to combine the current best evidence to answer a specific question on a clinical topic. · A comprehensive literature survey of the topic is conducted · Primary trials of sound methodology and the highest level of evidence are identified (eg. Randomized controlled trials) · The results of each trial are appraised · All results are summarized using predetermined, explicit, reproducible methodology. A systemic review shares many common characteristics with the first 4 steps of the evidence-based medicine process: phasing a clinical question (endpoint); researching the answer; evaluating the evidence; integrating and applying the evidence (conclusion). Meta-analysis Analysis of several clinical trials to answer clinical question or result of trial (endpoint) More credible Preferred for primary endpoint (most important). Example: a clinical trial of migraine with triptan gives direct outcome of how many patient migraines is relieved.

Systematic review Review all relevant studies to answer a clinical question (endpoint). Surrogate endpoint Example: Death from heart disease, is the endpoint of interest, but cholesterol is the surrogate marker. A clinical trial may show that a particular drug example statin is effective in reducing cholesterol without showing directly that statins prevent the death.

Primary endpoint: most important reference for study. Secondary endpoint: data analyzed indirectly from the study Surrogate (marker) endpoint: correlate with real endpoint Study designs Parallel group design: The more common of the 2RCT designs, the parallel group design, is often used for confirmatory trials. Participants are randomized into 2 or intervention, the other treated with placebo or current standard of care. Pros and Cons · (+) Randomized improves statistical analyses · (+) Unbiased distribution · (+) Can be blinded · (-) Requires extensive resources · (-) Can present ethical challenges · (-) Nature of volunteers can bias trial

Question Alerts! Parallel design studies often used for confirmatory trials

Copyright © 2000-2018 TIPS Inc. Unauthorized reproduction of this manual is strictly prohibited and it is illegal to 46-5 reproduce without permission. This manual is being used during review sessions conducted by PharmacyPrep.

www.PharmacyPrep.com

Epidemiology

Crossover Design: Participants are randomized into 2 or more groups. However, each group receives all the treatment (e.g. Intervention, standard, placebo) in a random order with a “washout” period in between, a period of time between two active treatments when the patient receives a placebo in order to remove the residual effect of the previous treatment before initiating the next active treatment. The basic crossover design is 2 x 2, in which 2 groups receive 2 consecutive treatments. Pros and Cons · (+) Reduced error variance, reduced sample size · (+) All participants serve as own control · (+) All participants treated · (+) Can be blinded · (+) Comparative studies · (-) All participants receive intervention, standard, and placebo · (-) Cannot be used for intervention with permanent outcome (like irreversible side effects) · (-) Length washout period · (-) Long duration

QAlerts! Cross over design all participants get treatment and placebo in crossover design. Reduce error variance

Meta Analysis: It thoroughly examines data from a number of valid trials that are similar enough to permit them to be combined and analyzed as one large trial. The evaluation of quantitative evidence from 2 or more trials involves combining the raw data or the summary of statistical results using a special statistical methodology. Question Alerts! Meta analysis is the top ranking The meta analysis trial design provides the highest level of evidence of clinical evidence any trial design because part of its methodology includes the analysis, critical appraisal, and summary of the results of many selected randomized controlled trials. Pros and Cons · (+) Top ranking in the levels of evidence · (-) Trials demonstrating a positive effect are published more often than those that don’t exposing a metaanalysis to a publication bias. · (-) Results from different trials do not always agree. · (-) Authors of a meta analysis could unconsciously or intentionally omit trials that may or may not support the clinical question. · (-) “Grouping” results from trials with different designs, statistical analyses, and patient populations may be problematic. Bias Bias is systemic error or distortion of a test measurement. Interview bias. Because of blinding of interviewer’s response may be influenced, known as interview bias. Recall bias: Differentials in the memory capabilities. Example case control studies. Lead time bias. The selection of cases from both of these groups introduces a form of non-random error known as lead-time bias.

Copyright © 2000-2018 TIPS Inc. Unauthorized reproduction of this manual is strictly prohibited and it is illegal to 46-6 reproduce without permission. This manual is being used during review sessions conducted by PharmacyPrep.

www.PharmacyPrep.com

Epidemiology

Berksons bias (admission rate bias): Distortion in risk ratios occurs as result of different hospital admissions. Confirmation bias: Linking directly to results. Selection bias: The selection of subjects into your sample or their allocation to treatment group produces a sample that is not representative of the population, or treatment groups that are systematically different.

Question Alerts! Confirmation bias! Look alike drugs, Look alike labels, Look alike shape drugs can result in confirmation bias.

Publication bias: Not publishing negative results. Confounder. A factor that is prognostically linked to the outcome of interest and is unevenly distributed between the study groups. Example. If study is examining the effect of age on ulcer relapse rate and several ulcer also smokers (smoking could be confounder). Example: A promising drug for the gastrointestinal protection is being developed. The study misses primary end point of GI bleeding symptom severity, but there does seem to be some effect at the higher doses. Blinding: It helps limit or eliminate factors that could unconsciously influence results. Blinding minimizes bias in several ways.

Single blind. One if the most basic forms of blinding in which trial participants have no knowledge of intervention (example patients cannot distinguish Important Concept! between the intervention, placebo, or active product used Who is NOT blinded? Biostatistician to compare different treatment regimens. Double blind. Neither the trial participants nor the investigators have knowledge of the assignment to the various trial groups of the interventions, placebo, or active product used to compare different treatment regimens. Triple blind. Neither the participants, the investigators, those monitoring the safety, nor any of the personnel involved in the selection of participants or evaluation of the outcomes have knowledge of the assignment of the interventions and placebo to the various trial groups.

Copyright © 2000-2018 TIPS Inc. Unauthorized reproduction of this manual is strictly prohibited and it is illegal to 46-7 reproduce without permission. This manual is being used during review sessions conducted by PharmacyPrep.

www.PharmacyPrep.com

Epidemiology

Tips format 002: Epidemiology 1.

case-control

2.

cross-sectional

3.

prospective

4.

Cohort

5.

Single-blind studies

6.

Double-blind

7.

Randomization of the sample Dependent variable

8.

Parallel Studies

9.

crossover studies

11

Meta analysis

12

Prevalence

Case report

14 17

10 13 16

15 18

· · ·

Factors reduces best bias ( ) The odds ratio associated with ( ) A study designed to determine the relationship between emotional stress and ulcers used the records of patients diagnosed with peptic ulcer disease versus controls over the period from May 2009-May 2010. This is an example of what kind of study? ( )

·

Twelve patients are given a drug or a placebo to determine the effect of medication on blood pressure. The dependent variable in this study is ( ) A study was undertaken to determine if prenatal exposure to marijuana is a cause of low-birth weight. Mothers of 50 infants weighing less than 5 lbs (low-birth weight) and 50 infants weighing more than 7 lbs (high-birth weight) were questioned about their use of marijuana during pregnancy. The study found that 20 mothers of low-birth weight infants and only 2 mothers of high-birth weight infants used the drug during pregnancy. This is an example of what kind of study? ( ) In this study, the odds ratio associated with smoking marijuana during pregnancy is (16) The odds ratio is 16 and is calculated as follows Mother smoked marijuana Mother did not smoke marijuana Low-birth weight babies A = 20 B = 30 High-birth weight babies (normal) C = 2 D = 48 960/60 = 16 Odds ratio = (A)(D) or (20)(48) = (B)(C) or (30)(2) Combining data from several studies (often via a literature search) to achieve greater statistical power. ( ) is the number of people who have an illness at a specific point of time ( )

·

· · · · · · · · ·

Copyright © 2000-2018 TIPS Inc. Unauthorized reproduction of this manual is strictly prohibited and it is illegal to 46-8 reproduce without permission. This manual is being used during review sessions conducted by PharmacyPrep.

www.Pharmacyprep.com

Biostatistics

47 Biostatistics Questions Alerts! Common questions in pharmacy exam is to ask! · Statistical significance: Probability of error (P) or level of significance (LOS) and confidence intervals (CI). · Hypothesis testing: Types of error like Type 1 (alpha) and Type 2 (beta) errors. · Calculating risk reductions like absolute and relative risk reductions · Calculating number needed to treat (NNT) or number needed harm (NNH). Characteristic of data: Descriptive statistical measurements are often used in medical literature to summarize data statistical distribution. Frequently used in clinical medicine are symmetrical distribution, measuring central tendency and measures of dispersion. Normal distribution: representation of normal phenomenon. Symmetrical distribution: The symmetrical distribution is also known as normal (Gaussian) distribution or bell shape curve. The dispersion or spread form the mean is represented by the standard deviation 68% (two thirds) of the value falls within one standard deviation of the mean. 95% of the values are found within two standard deviations of the mean. 99% of the values are found within three standard deviations of the mean. 99%

SD1 is 68%; SD2 is 95%; SD3 is 99%

95%

Bimodal = Two lumps Positive skew = Mean>median> mode: Tail on right handed side Negative skew = Mean < median 35

MEMORIZE: Height conversions: 1Foot = 12 inch 1 Foot = 30.48 cm 1meter = 100 cm 1 inch = 2.54 cm 1 meter = 3.28 ft

Example. 1) A person body weight is 180 lbs and height is 4 ft 6 in? What is right BMI? A) normal 4ft = 120 cm B) overweight 6 inch = 15 cm C) Obese 120+15 = 135 cm D) Morbid obese 135 Ans. 100

1 meter = 3.28 ft; 12 inch = 1 ft 1 m = 3.3ft (3.28 ft) i.e. the conversion factor from meters to feet is 3.28 or

3.3 ft = 1 m 1 ft = 12 inch 1 m = 39.37 inches

Height = 1.35 m Weight = 180/2.2 = 81 kg BMI = 81 kg 1.35 x 1.35 81/ (1.35x1.35) = 44 so BMI = 44 kg/m2

2. A doctor likes to prescribe Orlistat for a patient, weighing 150 lbs and height 4 ft 6 inch. Clinical practice guidelines recommends, Orlistat is used only for patient BMI >30. What is correct? A. Patient is not eligible for orlistat because BMI30 C. Orlistat is not related to BMI or Weight in Kg/height m x height m D. Orlistat is not used for weight loss Ans. B 4 ft x 30.5 cm = 122 cm Weight = 150/2.2 = 68 kg 6 inch x 2.54 cm = 15.4 cm Height= 137.4 cm/100 = 1.37 Total height in cm = 137 cm = 1.37 BMI = 68 kg/1.37 x 1.37 OR BMI = 36.2 kg/m2 4.5 ft /3.28 = 1.37 Tips. Orlistat is only suitable if BMI is 28 kg/m2 or greater and other conditions related to weight like blood pressure. or BMI >30 or treatment without any conditions and must be combined with low fat diet. A client BMI = weight in Kg/height m2 Body weight 90 kg and height 160 cm Pt. body weight 190 lbs and height 5.2 ft Pt. body weight 150 lbs and height 5 ft Pt. body weight 78 kg and height 5ft.4in

Calculate BMI?

Density (specific gravity): · Density = Weight/volume (D = W/V) · Volume = Weight/Density Copyright © 2000-2018 TIPS Inc. Unauthorized reproduction of this manual is strictly prohibited and it is illegal to 49-7 reproduce without permission. This manual is being used during review sessions conducted by PharmacyPrep.

www.pharmacyprep.com · · ·

Pharmaceutical sciences

Weight = Volume x Density Unit of density is g/ml Water density 1 g/ml D = W/V

V = W/D

Example: 1. If there is 1000 ml water. What is weight? A. 10 g B. 100 g C.1kg Ans. C

W=VXD

D. 10,000 g

2. Glycerine 1000 ml, and its density is 1.2. How many grams? A.1000 g Weight in g = volume x density B. 1200 g = 1000 ml x 1.2 C. 2000 g = 1200 g D. 2500 g Ans. B 3.An oil 1000 ml, and its density is 0.9 g/ml. How many grams? A.1000 g Weight = volume x density B. 1200 g Weight = 1000 ml x 0.9 g/ml = 900 g C. 900 g D. 9000 g Ans. C 3. A prescription order for glycerine 1200 ml, to prepare suppository in base. Glycerine density is 1.1. What is weight of glycerine in suppository? A. 1200 g B. 1000 g Weight = volume x density = 1200 ml x 1.1 g/ml = 1320 g C. 1320 g D.1500 g Ans. C Practice examples Given Theobroma oil 2.2 g and density is 0.9 g/ml Glycerine 1500 ml and density is 1.2 g/ml Winter green oil 900 ml and density is 0.89 g/ml

Want? How many mls? How many grams How many grams?

Teaspoons (tsp) and Tablespoons (Tbsp) The physician has prescribed 1 Tbsp of Riopan. The unit dose container available contains 30 ml of this medication. The correct dose is 15 ML The client has been taking 25 ml of a medication in the hospital. In preparing him to administer this medication in the home setting, you would teach him to take _5_ tsp of this drug. Copyright © 2000-2018 TIPS Inc. Unauthorized reproduction of this manual is strictly prohibited and it is illegal to 49-8 reproduce without permission. This manual is being used during review sessions conducted by PharmacyPrep.

www.pharmacyprep.com

Pharmaceutical sciences

Milligrams and Grains You are to administer phenobarbital gr ¾ . There are scored 60 mg tablets available in floor stock. You would administer __mg or _____ tablet(s). 65mg x 3/4 = 48 mg 60 mg/4= 15 mg per piece 3 x 15 mg = 45 mg dose 45 mg Morphine sulfate gr 1/8 IM is to be prepared from a solution containing 10 mg/1 ml. The volume to be given is _____ ml. Ans: 1 65 x 8 = 8 mg 10 mg.....................1 ml 8 mg...................? = 0.8 ml Millilitres and Ounces The physician has ordered ss ounce of a laxative. The correct dose is _____ tbsp ss = one half ounce = 30 ml tbsp = 15 ml Ans. 1 tbsp You are to administer 5 ml of ferrous gluconate that must be “diluted in” water to protect the client’s teeth and gastrointestinal track. The directions in your drug reference state that each ml of this medication must be diluted in 20 ml of water. Once diluted, the total volume to be administered is A. 100 ml Diluted in, added to, “added in” may increase total volume or weight. B.105 ml Dilute up to, quantity sufficient or qs, mark up to the volume should not increase C. 125 ml volume. D. 120 ml Ans. B

CCALCULATIONS INVOLVING PERCENTAGE, RATIOS, PROPORTIONS Percentage: To convert percent to fractions 25% = 25/100 = ¼ 0.25 . 100% = 25% 50% = 50/100 = ½ 200% = 200/100 = 2 To convert fraction to percentage: multiply by 100 3/4 . 100 = 75% RATIOS AND RATIO PERCENT: Relation between two quantities. A/B or A:B Copyright © 2000-2018 TIPS Inc. Unauthorized reproduction of this manual is strictly prohibited and it is illegal to 49-9 reproduce without permission. This manual is being used during review sessions conducted by PharmacyPrep.

www.pharmacyprep.com

Pharmaceutical sciences

W/W 1 gram in 100 grams = 1g:100 g W/V 1 gram in 100 ml = 1g:100 ml V/W 1 ml in 100 grams = 1mL:100 g V/V 1 ml in 100 ml = 1mL:100 mL Express Eatio to Ratio percent

Express ratio percent strength to ratio

5:100 = 10:100 = 5:1000 = 3:7000 =

0.1% = 1:1000 0.2% = 1:500 0.02% = 1:5000 0.5% =1:200 0.05% = 1:2000

5% 10% 0.5% 0.04%

What is the ratio percent? (5/100) x 100 = 5%

1:100 = (1/100) x 100 = 1% 1:200 = 0.5% 3:500 = 0.6% 1:2500 = 0.04% 1:10,000 = 0.01%

What is ratio? 0.1………….100 1…………….x 100 x 1 = 1:1000 0.1

100 = 1000 0.1 100 = 500 0.2 100 = 5000 0.02 5% ratio percent to ratio? A. 5:500 B. 5:1000 C. 5:5 D. 5:20 E. 1:20 Ans. 5% = 5:100 5:100 = 1:20

Express 0.02% as ratio strength Solution: 0.02à100 ml 2à10 000 than 1à5000 Ø Add 5 g of sugar in 100 mL to prepare a syrup? What is the ratio? A. 5:100 w/v B. 5:100 w/w C.5:100 v/w D.5:100 v/v ans. A Ø Dissolving 4.8 g of NaCl in water can make how many millilitres of a 1:1500 solution? Solution: We have 1 gram for every 1500 ml of solution So for 4.8 we will give x ml of solution X ml = 4.8 * 1500 /1=7200 ml

Copyright © 2000-2018 TIPS Inc. Unauthorized reproduction of this manual is strictly prohibited and it is illegal to 49-10 reproduce without permission. This manual is being used during review sessions conducted by PharmacyPrep.

www.pharmacyprep.com

PRODUCT PREPARATIONS An order calls for Cream A and Cream B 1:1 mitte 10 g Cream A and cream B 2:1 mitte 10 g 200 mg of 1:1:3 topical ointment 150 mg of custom 2:3:5 topical cream Cream A+ cream B + Cream C 1:2:1 mitte 12 mg

Pharmaceutical sciences

How much each ingredient needed? 1+1 = 2PARTS so 10 g/2 = 5 g 1X 5 g = 5 g 1X5 g = 5 g 6.6g : 3.3g 40 mg; 40 mg; 120 mg 30 mg; 45 mg; 75 mg 3 mg; 6 mg; 3 mg

PHARMACY PREP: RATIO STRENGTH PRACTICE EXERCISE For solids in semi solids; For solids in liquids For liquids in liquids (v/v) weight/weight (w/w) weight/volume (w/v) Weight in grams (g/g) Weight in gram and volume in milliliter (g/mL) EXAMPLES 2 g of hydrocortisone in 100 g of base powder. Express ratio strength 2:100 w/w OR 1:50 W/W Express percent strength? 2% w/w

2 g of amoxicillin powder in 100 ml water. Express ratio strength? 2:100 w/v OR 1:50 W/V Express percent strength? 2% w/v

For liquid in solid (V/W)

5 ml of glucose in 50 mL water. Express percent strength? 10% v/v

Ratio percent: W/W% · Example: 5 g of glucose in 50 mL water, what is ratio percentà 10% W/V (5/50)x 100 = 10%W/V · Examples: 50 g of glucose in a 1 kg glycerine, what is ratio percentà 5% W/W · Example: If 2% of glucose in water, what is ratio? 2 g/100 ml = 1/50 = 1:50 W/V · 12.5 g glucose in 100 ml Water, what is ratio percent?= 12.5% w/v · Example: 2.5% glucose in 80 ml of water? Glucose? 2.5g -------------- 100 ml ?--------------------80 ml = (80 ml x 2.5 g)/100 ml = 2 g of glucose is present in 80 ml Example: if 10% of dextrose in water, what is ratio? 1g :10ml W/V Copyright © 2000-2018 TIPS Inc. Unauthorized reproduction of this manual is strictly prohibited and it is illegal to 49-11 reproduce without permission. This manual is being used during review sessions conducted by PharmacyPrep.

www.pharmacyprep.com

Pharmaceutical sciences

Example: if 50g/L glucose, what is ratio percent? 5%W/V Example: 100g/1L water, what is ratio percent? 10% = 5g/50 mL = 5:50 or 1:10 Example: 50 grams of glucose in 1 L? = 50g/1L =50:1000 = 5:100 = 1:20 Example: Convert 5 grams of glucose in 100 mL into percent? · 5g/100mL. 100 = 5 W/V% · 5 grams of glucose in 1L water, what is ratio percent? · 5/1000 . 100 = 0.5 W/V% · 50 gram of glucose in 500 ml of water, what is ratio percent? · 50/500 .100 = 10 W/V% 5 grams of glucose in 500 mL water, what is ratio percent? 5/500 .100 = 1 W/V% Convert 10 grams of glucose in 10 ml into percent? 10 g/10 mL = 100/1 = 100 w/v% A drug x is 1 mg/10 ml present. What %W/V? 0.01% W/V Two ratios with the same value are equivalent. 1/3 x 2/2 = 2/6 = 1/3 If two ratios are equal, then their reciprocal are equal: If 1/3 = 2/6 then 3/1 = 6/2

PROPORTIONS: (TWO RATIOS) RELATION OF TWO RATIOS IS THE PROPORTION or two equal ratios forms proportion. A:B = C:D A/B = C/D or AD = BC From the four term one is unknown? Find the unknown Units for each term need to be correct Cross multiplying is used to solve X X/D = A/B ? = (D xA)/B Example: How many mg (?) of glucose in 500 mL water if this is equal to 1 g /L of water. Unknown X mg 500 ml X mg =

Known 1000 mg 1000 ml

500 ml x 1000 mg 1000 ml

X = 500 mg Copyright © 2000-2018 TIPS Inc. Unauthorized reproduction of this manual is strictly prohibited and it is illegal to 49-12 reproduce without permission. This manual is being used during review sessions conducted by PharmacyPrep.

www.pharmacyprep.com

Pharmaceutical sciences

Ampicillin 1 g drug vial. Add 6.8 ml sterile water to provide final concentration volume 8 mL. What is concentration of the reconstituted solution in the vial in mg/mL? A. 8 mg/mL Unknown Known B. 125 mg/mL x mg/mL 1000 mg/8mL C. 100 mg/mL 125 mg/mL D. 150 mg/mL What is the volume of ampicillin powder in vial? A. 8 mL B. 1.2 mL 8 mL – 6.8 mL = 1.2 mL C. 6.8 mL D. 100 mL E. 2 mL Given ampicillin 1 g vial. If the final concentration has been calculated to 125 mg/mL. What is the volume taken if dose of the ampicillin is 500 mg? A. 8 mL 125 mg...........1 mL B. 4 mL 500 mg.............? C. 2 mL or D.1 mL (unknown) (Known) Ans. B X ml/500 mg 1 ml/125 mg = 4mL

A sterile methylprednisone powder vial contain 125 mg drug. Reconstituted by adding 1.8 mL of sterile water to final volume 2 mL. What is final concentration of reconstituted solution in the per mg/mL? A. 125 mg/mL 2 mL ..................125 mg (known) B. 62.5 mg/mL 1 mL .................? (Unknown) C. 31.5 mg/mL OR D.16.5 mg/mL Unknown known Ans. B x mg/1mL 125 mg/2mL x mg = (1 mL x 125 mg)/2 mL An order for product preparation to maintain alkaline = 62.5 mg/mL urine. Final concentration. Sodium citrate 1% W/V Citric acid 6% w/w Sterile water for irrigation added up final volume 1000 mL. How many grams of sodium citrate and citric acid powder needed? A. 100 g and 10 g B. 10 g and 60 g C. 60 g and 10 g D. 50 g and 10 g E. 10 g and 10 g Ans. B Xg/1000 mL

1g/100 mL

Xg = (1000 mL x 1g)/100 mL = 10 g Copyright © 2000-2018 TIPS Inc. Unauthorized reproduction of this manual is strictly prohibited and it is illegal to 49-13 reproduce without permission. This manual is being used during review sessions conducted by PharmacyPrep.

www.pharmacyprep.com

Pharmaceutical sciences

OR 1 g ……… 100 ml ? g…….......1000 ml A 15 g of cream contains 1.45 g of Zinc oxide and 0.4 g of bismuth oxide. i). How much Zinc oxide will there be in a 180 g tube? ii). How much bismuth oxide will there be in a 180 g tube? A. 17.4 g of ZnO and 4.8 g of bismuth oxide B. 4.8 g of ZnO and 17.4 g of bismuth oxide C. 15 g of ZnO and 0.4 g of bismuth oxide D. 0.4 g of ZnO and 15 g of bismuth oxide Ans. A x g/180 g = 1.45 g/15 g 1.45 g x 180 g 15 g = 17.4 g of ZnO Bismuth oxide 0.4g x 180 g 15 g =4.8 g of bismuth oxide Product order to prepare 100 ml of hydrocortisone oral suspension with final concentration 1 mg/mL. Available hydrocortisone 20 mg tablets. How many tablets needed to prepare 100 mL? A. 20 tablets B. 10 tablets C. 5 tablets D. 1 tablet Ans. C Proportion and Percent Calculations Weight / weight W/W% Number of grams of substance in 100 grams of solvent or mixture.

Weight / Volume W/V% Number of grams of constituent per 100 ml of solvent.

Volume / Volume V / V% Number of milliliters of a substance in 100 ml of the solvent

1. How much drug should be added to 30 ml of water to make 10% w/w solution? Solution: The mass of the final solution in this case in unknown, we only have the mass of solvent (water) as 30 ml will weight 30 grams. So know we need a solution which contains 10 grams (10%) of drug in 100 grams of the solution (solvent + drug). In this case the solvent will represent 100 –10 = 90% So if 90% à30 grams, how much is 10 %? The weight of the drug is = 10* 30/90 = 3.33 g 10 g + 100 g (w/w) 110 g/30 g = 3.6 g Copyright © 2000-2018 TIPS Inc. Unauthorized reproduction of this manual is strictly prohibited and it is illegal to 49-14 reproduce without permission. This manual is being used during review sessions conducted by PharmacyPrep.

www.pharmacyprep.com

Pharmaceutical sciences

90 ml/30 g = 3 g

2-What is the percentage strength of an injection that contains 50 mg of pentobarbital sodium in each millilitre of solution? Solution: [(50 mg / 1 mL) x 1 g /1,000 mg] x 100 = 5 % w/v 3-If an injection contains 0.5% w/v of diltiazem hydrochloride; calculate the number of milligrams of the drug in 25 mL of injection. Solution: [(0.5 g / 100 mL) x ( 1,000 mg/ 1 g)] x 25 mL = 125 mg 4-How many grams of potassium permanganate should be used in compounding the following prescription? Rx Potassium permanganate 0.02% w/v Purified water ad 250 mL Solution: (0.02 g / 100 mL) x 250 mL = 0.05g = 50 mg = 50, 000 mg Parts Per Million (ppm) Occasionally, you will see a number followed by the term "ppm." This stands for "parts per million" and is most often used to indicate the amount of trace substances in water. The standard dilution for fluoride added to a municipal water source, for example, is 1ppm. In every 1,000,000 ml of water, therefore, there is 1 g of fluoride. 1) Express 5 ppm of iron in water as ratio strength and in percentage strength Solution: 5 ppm = 5 parts in 1,000,000 parts Ratio strength: 5:1,000,000 = 1:200,000 Percent strength: (5 /1,000,000) x 100 = 0.0005% 2) Express 10 ppm, in percent strength? 0.001% 10:1,000,000 = 0.001% 3) Express 0.0001% in how many ppm? 1ppm 4) Express 0.0005% of iron in water as parts per million (ppm)? (0.0005 x 1000000)/ 100 = 5:1000, 000 = 5 ppm 5) Express 0.023% of iron in water as ppm? [0.023 x 1,000,000]/100 = 230 ppm ____________________________________________________________________ 1-A parenteral solutions used in TIPS pharmacy. If 100 g of parenteral solution dissolved in 900 mL of petrolatum (Density of petrolatum is 0.9 g/mL), the concentration of parenteral solution is: D = W/V 900 mL x 0.9 g/mL = 810 g 810g+100g = 910g 100g/910g .100 = 10.9% w/w Copyright © 2000-2018 TIPS Inc. Unauthorized reproduction of this manual is strictly prohibited and it is illegal to 49-15 reproduce without permission. This manual is being used during review sessions conducted by PharmacyPrep.

www.pharmacyprep.com

Pharmaceutical sciences

% concentration =

solute

x 100

Solute+ solvent 100 g

x 100

100 g + 810 g = 10.9% w/w ____________________________________________________________________ 2) If 500 mL of a 15% v/v solution is diluted to 1,500 mL, what is the resultant percentage strength? Q 1 (quantity) x C 1 (concentration) = Q 2 (quantity) x C 2 (concentration) 500 (mL) x 15 (%) = 1,500 (mL) x? (%) 1,500 x = 7,500 x = 7,500 / 1,500 x = 5% v/v ___________________________________________________________________ 3. If syrup containing 65% w/v of sucrose is evaporated to 85% of its volume, what percent of sucrose will it contain? Note any convenient volume of syrup may be selected, say 100 mL. Then, 85% x 100 mL = 85 mL (V 2 ) C1 = 65 V1 = 100 C2 = ? V2=85 C2 = ( C1V1)/V2 C2= 76.47% w/v _____________________________________________________________________________ If 1 gallon of a 30% w/v solution is evaporated so that the solution has strength of 50% w/v, what is its volume in milliliters? C 1 = 30 V1 = 3800 mL C2 = 50 V2=? V2 = (C1V1)/C2 = (30 x 3800 mL)/50 = 2280 mL 5) A pharmacist mixed 100 mL of 37% w/w concentrated hydrochloric acid (specific gravity, 1.20) with enough purified water to make 360 mL of diluted acid. Calculate the percentage strength (w/v) of the diluted acid. 100 mL x 1.20 (specific gravity) = 120 g of concentrated acid 120 g x 37% w/w = 44.4 g HCI Copyright © 2000-2018 TIPS Inc. Unauthorized reproduction of this manual is strictly prohibited and it is illegal to 49-16 reproduce without permission. This manual is being used during review sessions conducted by PharmacyPrep.

www.pharmacyprep.com

Pharmaceutical sciences

(44.4 g / 360 mL) x 100 = 12.33% w/v ________________________________________________________________________ Drug Strength Expressions. Units (International units) or IU. Insulin, Penicillin G, Vitamin K, Vitamin E, Vitamin A, Vitamin D, Heparin, LMWH (Fragmin), interferon alpha, Nystatin, Polymixin, and Bacitracin. Example. U-100 insulin contains 100 units/mL Rx Insulin 40units For 60 days and bid. How many ml of insulin you dispense? Solution: (1 mL/100 units) x 40 units = 0.4 mL For 60 days and two times a day = 48 mL Q.If U-500 insulin (Humulin R U-500) is ordered for patient, a 1 ml syringe should be used. Dr prescribed 40 U of Humulin R U-500 insulin. How many ml of insulin is drawn in syringe? 500 .............. 1 mL 40.................? = 0.08 mL _______________________________________________________________________ 2.How many millilitres of a heparin sodium injection containing 200,000 heparin units in 10 mL should be used to obtain 5,000 heparin units? Solution: 200,000..........10 ml 5000............? (10 mL/200,000 units) x 5,000 units = 0.25 mL

Practice calculations 1. In dosing the drug gentamicin in pediatric patients, for every 1 mg/kg of gentamicin administered, serum drug concentrations are expected to increase by 2.5 µg/ml. What would be the expected serum drug concentration following an administration of a 2.5 mg/kg dose of gentamicin? A) 5 µg/ml B) 6.25 µg/ml C) 10 µg/ml D) 2.5 µg/ml 2) An elixir is to contain 250 mg of an alkaloid in each teaspoonful dose. How many grams of the alkaloid will be required to prepare 5 liters of the elixir? A) 0.25g B) 5g C) 250 g D) 2.5 g 3-A pediatric product contains 100 mg of erythromycin ethylsuccinate in each dropperful (2.5ml) of the product. How many kilograms of erythromycin ethylsuccinate would be required to prepare 5000 pint-size bottles? A-74.6 kg B-84.6 kg C-99.5 kg D-94.6 kg 4-A physician places a patient on a daily dose of 48 units of U 80 insulin (80units/mL). How many ml should the patient inject each day? A-0.4 mL B-0.5 mL C-0.6 mL D-0.25 mL 5-A 20-ml vial of biologic solution is labeled “2.0 megaunits.” How many units of drug are present in every ml of Copyright © 2000-2018 TIPS Inc. Unauthorized reproduction of this manual is strictly prohibited and it is illegal to 49-17 reproduce without permission. This manual is being used during review sessions conducted by PharmacyPrep.

www.pharmacyprep.com solution? A) 2000

B) 1000

Pharmaceutical sciences

C) 100,000

D) 10,000

6-A prescription calls for 10 units of a drug to be taken 3 times a day. How much will the patient have taken after 7 days? A-21.0 units B-0.21 units C-2.10 units D-210 units 7-A physician orders Meprobamate 0.2 g. How much is to be administered if the dose on hand is 400 mg. in each tablet? A-do not dispense B-give 2 tablets C-give 1 tablet D-give ½ tablet 8-The usual initial dose of chlorambucil is 150 µg per kg of body weight once a day. How many milligrams should be administered to a person weighing 154 lbs.? A-10.5 mg B-18 mg C-15 mg D-8 mg 9-An initial heparin dose of not less than 150 units/kg of body weight has been recommended of open heart surgery. How many ml of an injection containing 5000 heparin units per milliliter should be administered to a 300 pound patient? A-5.1 µl B-4.1 µl C-5.1 ml D4-.1 ml 10-The pediatric dose of cefadroxil is 30 mg/kg/day. If a child is given a daily dose of 2 teaspoonful of a suspension containing 125 mg of cefadroxil per 5 ml, what is the weight in lb. of the child? A-19.5 lbs. B-18.8 lbs. C-18.3 lbs. D-18.1 lbs. 11-If the loading dose of Kanamycin is 7 mg/kg of body weight, how many grams should be administered to a patient weighing 130 lbs.? A-0.492 g B-0.414 g C-414 g D-0.485 g 12-The adult dose of a liquid medication is 0.1 ml/kg of body weight as single dose. How many teaspoonfuls should be given to a patient weighing 220 lbs.? A-2 tsp. B-2.5 tsp. C-2 tbsp. D-2.5 tbsp. 13-If a prescription order requires 25 g of concentrated HCI (Density 1.18g/ml), what volume should the pharmacist measure? A-29.50 ml B-0.0212 ml C-23.0 ml D-21.2 ml 14-If the dose of a drug is 0.5mg/kg body weight/day, how many mg will a 35lb infant receive per 24hours? A-7.9 mg B-7.1 mg C-7.2 mg D-7.4 mg 15-What is the weight of 60 ml of oil whose density is 0.9624 g/ml? A-5.770 g B-57.7 g C-6.0 g D-0.577 g 16-A prescription calls for 0.3 g of phosphoric acid with a specific gravity of 1.71. How many milliliters should be used in compounding the prescription? A-0.5 B-0.7 C-0.18 D-0.3 17-How many ml of 0.9% (w/v) NaCl solution should be prepared from 250 ml of 25% (w/v) solution? A-3750 B-2500 C-6944.4 D-9 Copyright © 2000-2018 TIPS Inc. Unauthorized reproduction of this manual is strictly prohibited and it is illegal to 49-18 reproduce without permission. This manual is being used during review sessions conducted by PharmacyPrep.

www.pharmacyprep.com

Pharmaceutical sciences

18-A patient is determined to have 0.8 mg of glucose in each milliliter of blood. Express the concentration of glucose in the blood as mg%. A-800 mg% B-0.8 mg% C-8 mg% D-80 mg% 19-How many mL of a 1:400 (w/v) stock solution should be used to make 4 liters of a 1:2000 (w/v) solution? A-1000mL B-200 mL C-800 mL D-1600mL 20-If a patient is determined to have 100 mg % of blood glucose, what is the equivalent concentration in terms of mg/dL? A-1 B-10 C-40 D-100 21-Strong Iodine Solution USP contains 5% w/v iodine. How many mg of iodine are consumed daily if the usual dose is 0.3 mL t.i.d.? A-15 B-90 C-22.5 D-45 22-Express in percentage the fluoride concentration in drinking given in 0.6 ppm. A-0.06% B-0.00006% C-0.0006% 23-How many grams of dextrose are required to prepare 4 liters of a 5% solution? A-0.2g B-200g C-2g D-20g 24-Change to percent the number 1/300. A-3% B-33% C-3.3%

D-1/3%

25-A Pharmacy tech adds 75 mL of strong iodine solution USP (5.0% w/v) to 1 liter of sterile water for irrigation. What is the % w/v of iodine present? A-0.35% B-0.375% C-0.53% D-0.60% 26-How many grams of potassium citrate are needed to prepare 1 liter of 10%? A-1000 g B-50 g C-100 g D-10 g 27-How many grams of a drug are required to make 120 mL of a 25% solution? A-30 g B-10 g C-12.0 g D-12 g 28-Calcium Hydroxide Topical Solution contains 170 mg of calcium hydroxide per 100 mL at 15º C. Express this concentration as a ratio strength. A-1: 688 B-1: 888 C-1: 588 D-1: 788 29-How many mg of isofluorophate are contained in 15 g of a 1: 10,000 ophthalmic solution of isoflurophate in peanut oil? A-1.7 mg B-1.9 mg C-1.8 mg D-1.5 mg 30-Express 0.2 % in a ratio strength. A-1: 5000 B-1:50 C-1: 500

D-1:5

31-How much of a substance is needed to prepare 1L of a 1: 10,000 solution? A-0.1 g B-10 g C-0.01 g D-1.0 g 32-A cupric chloride injection (0.4 mg Cu/mL) is used as an additive to IV solution for TPN. What is the final Copyright © 2000-2018 TIPS Inc. Unauthorized reproduction of this manual is strictly prohibited and it is illegal to 49-19 reproduce without permission. This manual is being used during review sessions conducted by PharmacyPrep.

www.pharmacyprep.com

Pharmaceutical sciences

ration strength of copper in the TPN solution if 2.5 mL of the injection is added to enough of the IV solution to prepare 500 mL? A-1: 500 B-1:5000 C-1: 500,000 D-1: 50,000 33-How many milliliters of a 23.5% (w/v) concentrate of Sodium Chloride solution should be used in preparing 650 mL of a stock solution such that 30 mL diluted to liter will yield a 1: 5000 solution? A-0.2 mL B-4.33 mL C-18.44 mL D-11.75 mL 34-You have a stock solution of 50% Sodium citrate and you were asked to prepare 300 mL of a 10% solution. How many mL is needed? A-20 B-15 C-30 D-60 35-How many milliliters of 1:16 solution of sodium hypochlorite should be used in preparing 5,000 mL of a 5% solution of sodium hypochlorite for irrigation? A-800 ml B-2500 ml C-4000 ml D-300 ml 36) Prepare 1000 ml of KMnO 4 1:12,000 compresses out of KMnO 4 1: 8,000. A-Add 333.3 mL water to 1000 mL KMnO 4 1: 8,000 B-Add 666.6 mL water to 333.3 mL KMnO 4 1: 8,000 C-Add 333.3 mL KMnO 4 1: 8,000 and enough water to make final volume 1000 mL D-Add 333.3 mL water to 666.6 mL KMnO 4 1: 8,000 37-How many milliliters of 24% (w/v) concentrate of saline solution should be used in preparing 600 mL of a solution such that 10 mL diluted to a liter will yield a 0.09% solution? A-300 ml B-150 ml C-50.0 ml D-225 ml 38-The only source of Sodium Chloride is in the form of tablets, each containing 5.0 g. How many tablets should be used in preparing 3000 litres of a solution of such strength that 20 mL diluted to 100 mL with water will yield a 0.9% (w/v) solution? A) 60,000 tablets B) 27,000 tablets C) 12,000 tablets D) 9,000 tablets 39-How many grams of 10% (w/w) ammonia solution can be made from 1800 g of 28% (w/w) strong ammonia solution? A) 6428.57 g B) 5040 g C) 50,400 g D) 642.86 g 40. Griseofulvin HCl 50 g was added to 20ml of Glycerine to form a paste. Density of Glycerine is 1.25.

The final concentration of Griseofulvin in the paste dosage is: A) 80% w/w 20ml x 1.25 = 25gm of Glycerine B) 40% w/w 50% w/w = wt of solute / (wt of solute + wt of solvent) C) 66.7% w/w =[ 50 / (50 + 25)] x 100% = 5000 / 75= 66.7% w/w D) 33.3% w/w Ans. (C )

Copyright © 2000-2018 TIPS Inc. Unauthorized reproduction of this manual is strictly prohibited and it is illegal to 49-20 reproduce without permission. This manual is being used during review sessions conducted by PharmacyPrep.

50 Dosage Calculations Questions Alerts! Common questions in pharmacy exam is to ask! · Tapering dosage for prednisone, SSRI, BZD, Methadone, stimulants. · Number of tablets for prophylaxis therapies · Calculating dosages of loading dose of antibiotics · Calculating dosages for contraceptives pills One of the most common calculations in pharmacy practice is that of dosages. The available supply is usually labeled as a ratio of an active ingredient to a solution: Active ingredient (available)/solution (available) The prescription gives the amount of the active ingredient to be administered. The unknown quantity to be calculated is the amount of solution needed in order to achieve the desired dosage of the active ingredient. This yields another ratio: active ingredient (to be administered) / solution (needed) The amount of solution needed can be determined by setting the two ratios equal: [active drug/Solution available] = [active drug (to be administered)/ solution (needed)] When solving medication-dosing problems, use ratios to describe the amount of drug in a dosage form (tablet, capsule, or volume of solution). It is important to remember that the numerators and denominators of both fractions must be in the same units – for example, mg/ml, 5 mg/ml or mg/tablets = mg/tablets. CHILDREN DOSES: 1. Young’s Rule: Child dose = Age/Age + 12 x Adult’s dose 2. Cowlings Rule: Infant dose = [Age (at next birthday) x Adult’s dose]/ 24 3. Frieds Rule: Infant dose = [Age (in months) x Adult’s dose]/ 150 4. Clarks Rule: Child dose = [weight (in pounds) x Adult’s dose)]/150 Tips: Rarely applied now, generally dose for children is depending on weight of pt. & the technician should ask the agent or caregiver about the weight of pt. so that the pharmacist can check the dose. Example If the adult dose of X, is 5 mg. What is the dose for child of 8 years? Since we know according to Young’s rule: Child dose = AgexAdult’s dose Infusion rates: Flow rate (ml/hr) = total solution/number hrs to run Copyright © 2000-2018 TIPS Inc. Unauthorized reproduction of this manual is strictly prohibited and it is illegal to 50-1 reproduce without permission. This manual is being used during review sessions conducted by PharmacyPrep.

Drops per minute = (volume x drops factor)/time in minute

Q&A

1. A prescription calls for 10 units of a drug to be taken 3 times a day. How much will the patient have taken after 7 days? A-21.0 units B-0.21 units C-2.10 units D-210 units Ans: D 10 units x 3 x 7 = 210 units 2. A physician orders Meprobamate 0.2 g. How much is to be administered if the dose on hand is 400 mg. in each tablet? A-do not dispense B-give 2 tablets C-give 1 tablet D-give ½ tablet Ans: D Tips: 200 mg/400 mg = 1/2 tab 3) Prednisolone each tablet containing 5 mg. Start 35 mg and then taper by 5 mg every 1 day. How many tablets are needed? A) 20 tab B) 56 tab C) 14 tab D) 28 tab E) 8 tab Ans: D Tips: 7 + 6 + 5 + 4 + 3 + 2 + 1 = 28 tablets for 7 days 4) Prednisolone each tablet containing 5 mg. Start 35 mg and then taper by 5 mg every 2 day. How many tablets are needed? A) 20 tab B) 56 tab C) 14 tab D) 28 tab E) 8 tab Ans: B Tips: 7+7+6+6+5+5+4+4+3+3+2+2+1+1 = 56 tablets 5) Doctor prescribed a dose of 180 mg TID of Amoxicillin suspension for 10 days. Best Amoxicillin suspension size to pick up and reconstitute is (180 x 5 ml)/ 250 =3.6 ml A) Amoxicillin 250 mg/5 ml, 100 ml size 3.6 ml x 3 doses= 10.8 ml B) Amoxicillin 250 mg/5 ml, 150 ml size 10.8 x 10= 108 ml C) Amoxicillin 125 mg/5 ml, 100 ml size So it is better to pick out 250 ml/5 ml, 150 ml size. D) Amoxicillin 125 mg/5 ml, 150 ml size Ans.(B) 6) Using a vial containing 200,000 units of penicillin G potassium how many mL of solvent should be added to the dry powder to prepare a solution having a concentration of 25,000 units per mL. a) 8 b) 10 c) 15 d) 25 Ans: A Working: i. vial contains 200,000 units of penicillin G ii. Each mL of reconstituted solution will contain 25,000 units. Therefore, you will need (1/25,000) 200,000 = 8mL to produce a solution containing 25,000 units / mL Copyright © 2000-2018 TIPS Inc. Unauthorized reproduction of this manual is strictly prohibited and it is illegal to 50-2 reproduce without permission. This manual is being used during review sessions conducted by PharmacyPrep.

7) A patient who is 3 year old experiencing sore throat. His Dr. prescribed for him: Rx Zithromax 200 mg 1 QD for 5/7 days Best bottle of Azithromycin suspension to choose for reconstitution is: A) Azithromycin 300 mg which is 100mg/5ml B) Azithromycin 600 mg which is 200mg/5ml C) Azithromycin 900 mg which is 200mg/5ml D) None of the above Ans. (B) Tips: Since give for 5 days will constitute 1000mg so 2 bottles of Azithromycin 600mg is ok. 8) A 3-year-old boy with otitis media. Doctor prescribed Amoxicillin suspension in a dose of 90 mg/kg for 5 days. Child weight is 56 lbs. If the product in the pharmacy is 250 mg/ 5ml and the Doctor wants to give it in 3 divided doses. How much is the total volume required? A) 100 ml of 250 mg/5ml 56 lbs /2.2 lbs = 25 kg B) 150 ml of 250 mg/5ml 90 x 25 = 2,250 C) 200 ml of 250 mg/5ml 250 mg________5 ml D) 200 ml of 125 mg/5ml 2,250________X E) 225 ml of 250 mg/5ml X= 2250 x 5 / 250= 45 ml per day Ans. (E) 45 ml x 5 = 225 9) 3 parts per million of CO 2 in water is equal to A) 0.3 mg b) 3 mg C) 3 g Ans. (A) 3 parts g-------------1000 000 X-----------------100 X= 300/1000 000 = 3 x 10-4 g = 0.3 mg 10) 0.05 mg of glucose in 1L of water equal to: A) 50 parts per million B) 5 parts per million C) 0.5 parts per million D) 10 parts per million Ans. (A )

d) none of the above

0.05mg-------------------1000ml x----------------------------1000000 hence, 50 parts per million

11) A patient who is 3-year-old experiencing sore throat. His Dr. prescribed for him: Rx Zithromax 200 mg QD for 5/7 days Best bottle of Azithromycin suspension to choose for reconstitution is: E) Azithromycin 300mg which is 100mg/5ml Copyright © 2000-2018 TIPS Inc. Unauthorized reproduction of this manual is strictly prohibited and it is illegal to 50-3 reproduce without permission. This manual is being used during review sessions conducted by PharmacyPrep.

F) Azithromycin 600mg which is 200mg/5ml G) Azithromycin 900mg which is 200mg/5ml H) None of the above Ans. (B ) Since give for 5 days will constitute 1000mg so 2 bottles of Azithromycin 600mg is ok. 12) 20 mg of Gentamycin Sulfate inj. In 2 ml of water was added to 500ml glucose water. The concentration of Gentamycin is going to be: A) 0.08mg b) 0.16mg c) 0.09mg d) none of the above Ans. (A) (20mg) x (2ml) = (500ml) x (X) X= 40/500 =0.08 mg of Gentamycin Sulfate in 500 ml of glucose water. 13) Rx

40mg M.S contin I BID for 10days Available MS contin 20mg. The number of M.S contain given A) 20 B) 40 C) 10 D) None of the above Ans. (B ) 40 x 2 = 80 x 10 = 800 mg 800 ÷20 mg= 40 tab 14) Rx Ratio-prednisolone 1% ophthalmic drops Mitt: 5ml Sig. Gtt ii.o.s On the label the instruction should be: A) Instill 2 drops on the right eye B) Instill 2 drops on both eye C) Instill as directed D) Instill 2 drops on the left eye Ans. (D) 16. Syrup is an 85% w/v solution of sucrose in water. It has a density of 1.313 g/ml. How many grams of water

should be used to make 125 ml of syrup? (D = W/V) A. 85 g B. 106 g C. 125 g D. 164 g Ans. E 125 ml x 1.313 g/ml = 164 g 100 ........... 85 125.............? 125 x 85 100 = 106 g 164g -106g = 58 g of water

E. 58 g

Copyright © 2000-2018 TIPS Inc. Unauthorized reproduction of this manual is strictly prohibited and it is illegal to 50-4 reproduce without permission. This manual is being used during review sessions conducted by PharmacyPrep.

www.pharmacyprep.com

51 Dilution, Concentrations Questions Alerts! Common questions in pharmacy exam is to ask! · Dilution from Stock C1 x V1 = C2 x V2 or Q1 x C1 = Q2 x C 2 · Concentration = [quantity of solute/quantity of solution] x 100 · Using Allegation methods to prepare ointments mixture. · Electrolytes: Converting from milliequivalent to milligram mEq = [mg x valence/M. wt (mg)] · Converting from milligram to milliequivalent mg = [mEq x M. wt/Valence] · Isotonicity: MilliOsmole, isotonic preparation. · Sensitive requirement of balance.

Stock solution: Solutions of known concentration that are prepared in the most concentrated form. Sometimes a stock solution will be pure drug in powder or crystalline form. At other times it will be a liquid or a solid paste or cream. Stock solutions and additives. Here are some common IV stock solutions. A pharmacist or pharmacy technician will withdraw a calculated number of milliliters from the vial and place it into a bag of fluid, thereby diluting the original concentration of the stock solution. These stock solutions are also called "additives" because they are added to another IV solution. Most commonly dilution and concentration can be solved by inverse proportion method and by determination of percentage or ratio strength. The following formula can be used to calculate dilutions and concentrations: Q1 (quantity) X C1 (concentration) = Q2 (quantity) X C2 (concentration) Or We can use the equation = C 1 x V 1 = C 2 x V 2 C1x V1= C2 x V2 C 1 = stock concentration V 1 = stock volume C 2 = final concentration V 2 = final volume

C1 = (C2 V2)/V1 V2 = (C1V1)/C2 V1 = (C2 V2)/C1

Copyright © 2000-2018 TIPS Inc. Unauthorized reproduction of this manual is strictly prohibited and it is illegal to 51-1 reproduce without permission. This manual is being used during review sessions conducted by PharmacyPrep.

www.pharmacyprep.com Examples: 1) A prescription for hydrocortisone cream 0.1%. Pharmacy has 0.25% available in 30 g tube. How many grams diluents base (vanishing cream) should be added? A) 30 g B) 45 g C) 50 g D) 75 g E) 25 g V 2 = (C1 x V1)/C2 V 2 = (0.25% x 30 g)/0.1% V 2 = 75 g but the added tube contains 30 g so the used base = 75 g - 30 g = 45 g Dexamethasone is available as 4 mg/mL preparation. in infant is to receive 0.35 mg. Prepare a dilution so that the final concentration is 1 mg/mL. How much diluents will you need if the original product is in a 1mL vial and you use the full vial? A) 4 ml Step1: determine the volume of final product. Since dexamethasone is 4mg/mL, a 1 ml vial B) 3ml have 4mg of drug. X ml/ 4 mg = 1 ml/1 mg = 4 mL C) 1ml Step2: subtract the volume of concentrate from the total volume to determine the amount of D) 0.35ml diluents needed. 4 ml-1ml = 3 mL Ans: B If a 600 ml (v 1 ) of a 15% (v/v) (c 1 ) solution of methyl salicylates in alcohol are diluted to 1500 ml (v 2 ) what will be the percentage strength. A. 6% v/v C2 = (C1xV1)/V2 B. 12% v/v C2 = (15% x 600 ml) C. 6% w/v 1500 ml D. 6% v/w = 6% v/v E. 6% w/w A physician has prescribed 60 g (v 1 ) of 0.01% (c 1 ) fluocinolone acetonide cream. You have available the commercially prepared cream containing 0.2% (c 2 )fluocinolone acetonine (Synalar cream) and cream base (Dermabase ) for dilution. To prepare the Rx you would require: a) 1.2 g Synalar and 58.8 g Dermabase C1x V1= C2 x V2 b) 3 g Synalar and 60 g Dermabase 0.01% x 60 g = 0.2% . Q2 c) 0.3 g Synalar and 59.7 g Dermabase X = (60 g x 0.01)/ 0.2 = 3 g Synalar d) 3 g Synalar and 57 g Dermabase Order is for 60 g - 3 g = 57 g dermabase If 50 ml (v1) of 1:20 w/v (c1) solution are diluted to 1000 ml (v2), what is the ratio strength (w/v)? A. 1:20 1:20 = (1/20) x 100 = 5% B. 1:30 C2 = 5% x 50 C. 1:40 1000 ml D. 1:400 = 0.25% or = 25/10,000 E. 1:500 = 1:400 Ans. D Copyright © 2000-2018 TIPS Inc. Unauthorized reproduction of this manual is strictly prohibited and it is illegal to 51-2 reproduce without permission. This manual is being used during review sessions conducted by PharmacyPrep.

www.pharmacyprep.com If a potassium chloride elixir contained 20 mEq of potassium ion in each 15 mL of elixir, how many milliliters will provide 25 mEq of potassium ion to the patient? Solution/Answer: 20 mEq = 25 mEq 15 mL x mL x = 15 x 25 x = 18.75 mL 20 How many grams of dextrose are required to prepare 4,000 mL of a 5% w/v solution? Equivalent factor: a 5% w/v solution = 5 g in 100 mL of solution. Solution/Answer (5 g / 100 mL) x

4,000 mL = 200 g

Calculations involving dilution and concentration of Stock solutions A solution of known concentration that is prepared in the most concentrated form is referred as stock solutions. Sometimes a stock solution will be pure drug in powder or crystalline form. At other times it will be a liquid or a solid paste or cream. How many mL of a 1:500 (w/v) stock solution should be used to make 4 liters of 1:2000 (w/v) solution? Solution/Answer: 1:500 = 0.2% 4 liters = 4000 mL 1:2000 = 0.05% 0.2%/0.05% = 4000 mL / x mL = 1000 mL Concentrations: 2) A parenteral solution used in hospital pharmacy. If 250 g of parenteral solution dissolved in 1000 mL of glycerin (density of glycerin is 1.25 g/mL), the concentration of parenteral solution is? A) 8% w/w %concentration = solute x 100 B) 20% w/w D = M/V or W = D x V Solute+ solvent C) 24% w/w Solvent (glycerine)= W = 1.25 x 1000 mL = 1250 g 250g x 100 D) 16% w/w 250 g + 1250ml E) 32% w/w = 16% W/W

Allegation Method

Allegation medial: A method for calculating the average concentration of a mixture of two or more substances. 1) What is the final percentage of ZnO ointment made by mixing ZnO ointment of the following strengths? 200 g of 10% + 50 g of 20 % +100 g of 5% Solution: +200 * 10% = 20 + +50 * 20% =10 + +100 * 5% = 5+ Copyright © 2000-2018 TIPS Inc. Unauthorized reproduction of this manual is strictly prohibited and it is illegal to 51-3 reproduce without permission. This manual is being used during review sessions conducted by PharmacyPrep.

www.pharmacyprep.com Therefore, we have 350 g of the ointment which contain 35 g of ZnO Therefore, the percentage is 35/350=10% Ointment mixture of 15 g of 70%, 5 g of 90%, 10 g of 40%, 5 g of 10%, what is final concentration of this mixture? A. 15% 15 g + 5 g + 10 g + 5 g = 35 g B. 5% C. 55% (15x70)/100 + (5x 90)/100 + (10 x 40)/100 + (5x10)/100 = D. 70% 10.5 + 4.5 + 4 + 0.5 = 19.5 E. 40% (19.5 /35 g) x 100 = 55% Ans. C

ALLEGATION ALTERNATE: A method of calculation of the number of parts of two or more components of known concentration to be mixed when the final desired concentration is known.

Which proportion of 95% alcohol and 50% alcohol should be used to make a solution of 500 mL of 70% alcohol? Solution: 95% 50% 5

.................................. 20 parts of 95 % ............223 ml 70% (desired) ..............................

25 parts of 50%..............277 ml

The final proportion is 20+25 = total 45 parts Take 95%: (20/45) x 500 ml = 223 ml Take 50%: (25/45) x 500 ml = 277 ml Prescriber orders to prepare 1% hydrocortisone 60 g. Your pharmacy has stock of 0.5% hydrocortisone and 2.5% hydrocortisone. How many each grams is mixed? 1.5 parts (1.5/2)60 A. 15 g of 2.5% and 45 g of 0.5% HC 0.5% B. 45 g of 2.5% and 15 g of 0.5% HC 1% C. 15 g of 2.5% and 15 g of 0.5% HC D. 45 g of 2.5% and 45 g of 0.5% HC 0.5 2,5% E. 2.5 g of 2.5% and 0.5 g of 0.5% HC 0.5 2.5

1

1.5 0.5

2.5% of HC ..........( 0.5/2)x60 = 15 g 0.5% of HC...........(1.5/2)x60 = 45 g Rx A prescription for Hydrocortisone 1% 60 g. Your pharmacy has 2.5% hydrocortisone and petrolatum base (0%). What fraction of each hydrocortisone needed use to prepare above prescription A. Petrolatum 36 g & 2.5% HC 24 g B. Petrolatum 24 g & 2.5% HC 36 g 2.5% 1 part C. Petrolatum 60 g & 2.5% HC 24 g 1% D. Petrolatum 36 g & 2.5% HC 36 g 0% 1.5 part E. Petrolatum 24 g & 2.5% HC 24 g 2.5% HC = (1/2.5) x 60 g = 24 g Petrolatum base = (1.5/2.5)x60 g = 36 g Copyright © 2000-2018 TIPS Inc. Unauthorized reproduction of this manual is strictly prohibited and it is illegal to 51-4 reproduce without permission. This manual is being used during review sessions conducted by PharmacyPrep.

www.pharmacyprep.com Rx A prescription for Hydrocortisone 1% 60 g. Your pharmacy has 0.5% hydrocortisone and hydrocortisone powder (100%). What fraction of each hydrocortisone needed to use to prepare above prescription A. HC powder 0.3g and 0.5% HC 59.7 g 0.5% 99 B. HC powder 59.7 g and 0.5% HC 0.3 g C. HC powder 0.3 g and 0.5% HC 0.3 g 1% D. HC powder 59.7 g and 0.5% HC 59.7 g E. HC powder 59.7 g and 0.5% HC 0 g 100% 0.5 Ans. A 0.5% of HC = 59.7 g HC powder = 0.3 g Calculation involving electrolyte solutions Electrolyte solutions. These preparations used in treating electrolytes imbalance in body. The concentration of electrolytes is almost exclusively expressed in milliequivalents which reflect a unit of chemical activity. Mole: molecular weight in grams Reference atomic weights: Na = 23, C = 12, O = 16, K = 39, Cl = 35, Ca = 40 Eq. wt = Molecular Weight/Valence

One mole of NaCl = 58.5 g One mole of KCl = 74.5 g One mole of HCl = 36 g One mole of Na2CO3 = 106 g One mole of CaCl2 = 111 g

Converting between milligrams (mg) and milli equivalent (mEq) Number of mEq = Weight of substance in mg/mEq weight Molarity Molarity is the expression of the number of moles of solute is dissolved in litre of solution. Molarity can be calculated by diving the moles of solute by the volume of solution in litres. One mole dissolved in 1liter solution is 1M. 1M HCl = 36.5g of HCl dissolved in 1L 1M NaCl = 58.5g of NaCl dissolved in 1L One mole of substance dissolved in a liter solution Molarity = molecular weight in grams/ Litres 1molar NaCl = 58 grams in litre To prepare 100 mL of 1M NaCl, how many grams of NaCl (Mol. wt of NaCl = 58.5g) needed? Solution: 58.5 / 1000x 100 = 5.85g ________________________________________________________________ To prepare 28 mL of 0.5M NaCl, how many grams of NaCl needed? (M. Wt of Na = 58.5) Solution: 58.5/2 = 29 g Copyright © 2000-2018 TIPS Inc. Unauthorized reproduction of this manual is strictly prohibited and it is illegal to 51-5 reproduce without permission. This manual is being used during review sessions conducted by PharmacyPrep.

www.pharmacyprep.com 29/1000 x 28 mL = 0.8 g

Millimole Millimole (mmol)/L = Molecular Wt in milligrams/Litres 1 mmol of NaCl solution contain how many milligram of sodium chloride? 58.5 mg /L 1 mmol of 100 mL NaCl contain how many milligram of NaCl? 5. 8 mg 0.5 mmol of NaCl contain how many milligram of NaCl? 29 mg Magnesium has an atomic weight of 24, what is weight of 1 mmol? 1mM = 24 g/1000 = 0.024 g = 24 mg. Normality A method of dealing with acids, bases, and electrolytes which involves the use of equivalents. One equivalent of acid is the quantity of that acid that supplies or donates of mole of H+ ions. One equivalent of base is quantity that gives off one mole of OH- ions. One equivalent of acid (H+) reacts with one equivalent of base (OH-). Equivalent can be calculated for atoms or molecules. Equivalent wt dissolved in 1L Equivalent Wt = M. wt in g/Valence 1N HCl = 36 g of HCl dissolved in 1L The salts with valence 1 have the same molarity and normality. The valence in salts is referring to metal ions. The salts with valence 1: NaCl, HCl, KCl, Li 2 CO 3 , Na 2 CO 3 , NaHCO 3 The salts with valence 2: CaCO 3 , CaCl 2 , MgCl 2 , Mg (OH) 2 , ZnCl 2 The salts with valence 3: Al, citrate One mole of NaCl contain one equivalent of Na+ (Na mol. weight 23 g) One mole of NaCl contain one equivalent of Cl- (Cl mol. weight 35 g) 0.9 % NaCl contain 0.9 g of NaCl in every 100 mL 0.9% NaCl contain 9 g of NaCl in every 1liter MilliEquivalent: The amount in mg, of a solute equal to 1/1000 of its gram equivalent weight per unit volume. Converting milliequivalents per unit volume to weight per unit volume. Valence: Na, K, Li = 1; Ca, Mg = 2 and Al = 3 mEq = [mg x valence]/(Mol.Wt)

mg = [mEq x (mol.wt)]/valence

1. How many milliequivalents of sodium ion is in 92 mg/ml of NaCl salt? M.wt of Na = 23 A. 1 mEq mEq = 92 mg/ml x 1 B. 2 mEq 23 C. 3 mEq mEq = 4 mEq D. 4 mEq E. 5 mEq Copyright © 2000-2018 TIPS Inc. Unauthorized reproduction of this manual is strictly prohibited and it is illegal to 51-6 reproduce without permission. This manual is being used during review sessions conducted by PharmacyPrep.

www.pharmacyprep.com

How many mEq of magnesium sulphate are represented in 1 g anhydrous magnesium sulfate? (M. wt of MgSO 4 =120) A) 120 mEq mEq = 1000 mg x 2 B) 32 mEq 120 C) 16.6 mEq D) 33.2 mEq mEq = 16.6 mEq E) 66.6 mEq Ans: C A solution contain 10 mg% of Ca2+, describe this concentration in mEq/L. (Atomic weight = 40 and valence = 2 A. 5mEq/L B. 10 mEq/L 10 mg% is = 10 mg/100 ml C. 40 mEq/L 10 mg% for 1L is 100 mg/L D. 2 mEq/L mEq/L = [(mg/L) (valence)]/atomic weight E. 50 mEq/L [(100 mg/L) (2)] / 40 mg = 5 mEq/L Ans. A What is the concentration in g per ml of a solution containing 4mEq of calcium chloride (CaCl 2 x 2H 2 O) M. wt = 147 A. 294 mg Mg/L = (mEq x Mwt) B. 0.294 g/L Valence C. 0.0002 g/ml Mg/L = [(4 x 147)]/ 2 D. 147 mg = 294 mg/L = 0.294 g /L = 0.0002 g/mL E. 147 g Ans. C

mEq? mEq = [mg x valence]/(Mol.Wt) If valence is 1 (Na, K, Li) mEq = mg/Mol.wt How many mEq of sodium salt if 196 mg NaCl in 10 mL solution? (Na=23, Cl 35.5) 196/58.5= 3.35 mEq of Sodium ion present in 10 mL.

Mg/ml? Mg = [mEq x (mol.wt)]/valence If valence is 1 (Na, K, Li) Mg = mEq x mol.wt. A 100 mL bag contain 40 mEq of potassium chloride. What is the weight potassium is contained in IV bag? (K = 39, Cl=35.5) =74.5 40 mEqx 75 mg 3000 mg of K is present in 100 mL of IV bag. What is concentration in mg/ml of a solution containing 2 mEq of KCl per mL? 149 mg/ml What is the concentration in mg/ml of solution containing 4 mEq/ml of CaCl 2 .2H 2 0? Mol.weight = 147 (4x147)/2 = 294 mg/mL Potassium chloride 75 mg = 1mEq of KCl. (KCl MW: 75) Potassium citrate 108 mg = 1mEq of K.citrate (K.citrate MW

Copyright © 2000-2018 TIPS Inc. Unauthorized reproduction of this manual is strictly prohibited and it is illegal to 51-7 reproduce without permission. This manual is being used during review sessions conducted by PharmacyPrep.

www.pharmacyprep.com 108) Potassium chloride 75 mg = 1mEq of KCl. (KCl MW: 75)—K+ is 39 mg Potassium citrate 108 mg = 1mEq of K.citrate (K.citrate MW 108) K+ is 39 mg Rx Give K+ 39 mg. (atomic weight of K+ 39) Your pharmacy has potassium citrate salt (K+citrate MW = 108 mg). How many mg of potassium citrate is used to prepare this prescription? A.108 mg Rx Give K+ 39 mg. Your pharmacy has potassium chloride salt (KCl MW= 75 g). How many mg of potassium chloride is used to prepare this prescription? 75 mg Rx Give Ca2+ 40 mg Your pharmacy has calcium chloride salt. How many mg of calcium chloride (MW:111) is used to prepare this prescription? 111/2 = 55.5 mg Rx To prepare potassium ion 20 mEq and how many mg of potassium chloride needed (KCL MW = 75)? A.1500 mg B. 200 mg 20 x 75 = 1500 mg of KCl salt gives 20 mEq of K+ C. 750 mg D. 75 mg E. 7500 mg To prepare potassium ion 20 mEq and how many mg of potassium citrate needed (KCitrate MW = 108)? A.2160 mg 20 x 108 = 2160 mg of KCitrate salt gives 20 mEq of K+ B. 200 mg C. 750 mg D. 75 mg E. 7500 mg or potassium citrate? A. KCl = 75 mg x 20 = 1500 mg OR K.citrate = 108 mg x 20 = 2160 mg What is the concentration in mg/ml of a solution containing 2 mEq of KCl per millilitres (KCl M.Wt = 74.5)? A. 74.5 mg/mL B. 149 mg/mL 2 mEq. Of KCl= 74.5 mg × 2 = 149 mg/ml C. 39 mg/mL D. 75 mg/mL E. 58 mg/mL Ans. B OR Copyright © 2000-2018 TIPS Inc. Unauthorized reproduction of this manual is strictly prohibited and it is illegal to 51-8 reproduce without permission. This manual is being used during review sessions conducted by PharmacyPrep.

www.pharmacyprep.com mg/ml = 2 (mEq/ml) × 74.5 = 149 mg/ml What is the concentration, in grams per milliliter of a solution containing 4 mEq. Of CaCl 2 .2H 2 O per milliliter? (M. weight of CaCl 2 .2H 2 O = 147) Eq. wt. Of CaCl 2 .2H 2 O = 147/2 = 73.5 1 mEq. CaCl 2 .2H 2 O = (1/1000) × 73.5 g = .0735 g 4 mEq. Of CaCl 2 .2H 2 O = 0.0735 g × 4 = 0.294 g/ml OR by mg/ml = [4 x 147]/ 2 = 294 mg/ml = 0.294 g/ml Converting Milligram % to mEq/L 1-A solution contains 10 mg% of K+ ions. Express the conc. In terms of mEq/L? Solution: Atomic weight of K+ = 39 Equivalent weight of K+ = 39 1 mEq. Of K = (1/1000)x39 g = 0.039 g = 39 mg 10 mg% of K + = 10 mg K + per 100 ml = 100 mg per liter 100 mg x 39 = 2.56 mEq/L Or

MEq/L = [100 mg/L × 1] / 39 = 2.56 mEq/L 2-A solution contains 10 mg % of Ca ++ ions. Express this concentration in terms of mEq/Liter mEq/Liter = [100 mg /L ×2 ] / 40 (atomic weight of Ca ++) Converting weight to milliequivalents 1-How many mEq. Of KCL are represented in a 15 ml dose of 10 % w/v KCL elixir ? Mol. Weight KCl = 74.5 g Solution: Equivalent weight = 74.5 g 1 mEq of KCL = 0.0745 grams =74.5 mg 15 ml dose of 10 % w/v elixir = 1.5 g or 1500mg KCL 74.5 mg --à 1 mEq 1500 mg --àx x = 20.1mEq Milliosmole: The unit of osmotic activity. It is the unit of measuring the osmotic concentration. Osmotic pressure is directly proportional to the number of particles in the solution. Osmotic pressure µ Number of Particles in solution Milliosmole is a way of expressing number of particle in solution. i.e. if there is one particle then millimole = 1 milliosmole if there is two particle then millimole = 2 milliosmole Solute which dissociate (i.e. electrolytes) exert osmotic activity based on the number of particles present in solution after they have dissociated. 1 millimole of NaCl = 2 milliosmole Copyright © 2000-2018 TIPS Inc. Unauthorized reproduction of this manual is strictly prohibited and it is illegal to 51-9 reproduce without permission. This manual is being used during review sessions conducted by PharmacyPrep.

www.pharmacyprep.com 1 millimole of CaCl 2 = 3 milliosmole 1 millimole of NaH 2 PO 4 = 2 milliosmole Solutes which do not dissociates in solution, such as dextrose exerts 1 mOsm for each millimole. Osmol/L = wt. Of substance in g/L X number of species Molecular weight in g mOsmol/L = Wt. Of substance in g/L x number of species x 1000 Molecular weight in g

CALCULATING mOsmol FOR ELECTROLYTE SOLUTIONS Solutes, which dissociate exert osmotic pressure based on the number of particles present in the solution after they have dissociated. Some Example salts: For NaCl, the number of species = 2 NaCl à Na+ + ClFor CaCl 2 , the number of species = 3 CaCl 2 à Ca2+ + 2ClFor Li 2 CO 3 the number of species = 3 Li 2 CO 3 à 2Li + + CO 3 2For MgSO 4 the number of species = 2 MgSO 4 à Mg + SO 4 For Solutes which do not dissociate, the milliosmole = millimole

Copyright © 2000-2018 TIPS Inc. Unauthorized reproduction of this manual is strictly prohibited and it is illegal to 51-10 reproduce without permission. This manual is being used during review sessions conducted by PharmacyPrep.

www.pharmacyprep.com NON-IONIC SOLUTES (SPECIES IS ALWAYS 1)

Dextrose, gluconate are examples of substances that do not dissociates. However salts dissociates into ions. ________________________________________________________________ 1) Solution contains 5% anhydrous dextrose in water injection. How many milliOsmoles per litre are present in this concentration? (M. wt = 180 g). A. 278 mOsmol/L mOsmol/L = 50 g x 1 x 1000 = 278 mOsmol/L B. 180 mOsmol/L 180 C. 50 mOsmol/L D. 920 mOsmol/L

IONIC SOLUTS (SPECIES FOR NaCl = 2, CaCl 2 = 3; KCl = 2)

________________________________________________________________ 2) How many milliosmoles per litre are present in 0.9% NaCl solution (MW NaCl= 58.5 g? A. 278 mOsmol/L B. 307 mOsmol/L 0.9% NaCl is 0.9g in 100ml, however, solution in one litres, C. 58 mOsmol/L thereby NaCl concentration is 9 g D. 45 mOsmol/L (9 x 2 x1000) = 307 mOsmol/L 58.5

ISOTONIC SOLUTION PREPARATIONS

Isotonic à "Normal saline" and is 0.9% NaCl concentration Hypotonicà Less than 0.9% NaCl concentration Hypertonic à More than 0.9% NaCl concentration Tonicity is affected by number of particles in solution. Substances that dissociate have greater tonic effect than non-dissociated substances. Greater the dissociation greater the osmotic pressure and greater the tonic effect. Q. How much NaCl required to prepare? Rx Silver nitrate 1:500 (0.2%); NaCl equivalent = 0.34 Isotonic solution 60 ml A. 0.9 g NaCl B. 0.6 g NaCl C. 0.49 g NaCl D. 0.2 g NaCl E. 0.34 g NaCl Rx

(1/500) x 100 = 0.2% 100 ml………..0.2g 60 ml……..? = 0.12 g of Silver nitrate 0.12 g x 0.34 = 0.0408 g silver nitrate NaCl = 100 ml……….0.9 60 ml............? 0.9/100 = X/60 X = 0.54 g NaCl is isotonic Required NaCl = 0.54 - 0.04g = 0.50 g of NaCl should be added to make isotonic

ZnSO4 ----------1/4 % Phenylephrine--1/8 % NaCl----------------Q.S. Aq. Distilled ad to 30 mL

How much NaCl must be added to the following Rx to make it isotonic? NaCl equivalent of ZnSO 4 =0.16, NaCl Equivalent of phenylephrine=0.29. Copyright © 2000-2018 TIPS Inc. Unauthorized reproduction of this manual is strictly prohibited and it is illegal to 51-11 reproduce without permission. This manual is being used during review sessions conducted by PharmacyPrep.

www.pharmacyprep.com A. B. C. D. E.

247 mg NaCl 16 mg NaCl 290 mg NaCl 900 mg NaCl 40 mg NaCl

First step: Calculate the amount of each ingredient in the prescription so if we look at the prescription you will find that the final volume is 30 ml . so for ZnSO 4 , we need 0.25 for each 100 ml so for 30 ml we need 30 * 0.25/100=0.075 grams. now for phenylephrine do the same so we will need 30 * 0.125/100 = 0.0375 grams. Second step: By using the NaCl equivalent of each of them calculate the contribution of these salts to the isotonicity of the solution . now for ZnSo 4 : 1 gram of ZnSo 4 is equivalent to 0.16 grams of NaCl , but in our prescription we have only 0.075 so this makes the solution as if it contains 1 gram ------> 0.16 NaCl 0.075 gram -------> x NaCl x = 0.075 * 0.16 / 1= 0.012 NaCl Applying the same for phenylephrine: 0.0375 *0.29 / 1 = 0.0108 NaCl Therefore total contribution of the salts = 0.012 + 0.0108 = 0.022875 NaCl Third step: Find the amount of NaCl needed to make 30 ml - which is the volume of the final solution - isotonic. so we need 0.9 grams NaCl for every 100 ml ...so for 30 ml we need : 0.9 g --------100 ml x g ---------30 ml x= 30 * 0.9/100 = 0.27g NaCl Therefore the amount of NaCl needed = 0.27 - 0.022875 =0.247 grams = 247 mg NaCl You are given ZnCl 2 0.7%, phenylephrine 0.1% and boric acid 1.1% with E values 0.16, 0.32 and 0.5 respectively. This solution will be: [(0.7 x0.16] + [(0.1 x 0.32] + [(1.1 x 0.5)] A) Hypotonic = 0.112 + 0.032 + 0.55 = 0.69 g B) Hypertonic C) Isotonic = 0.9>0.69 = Hypotonic D) Non-isotonic Ans. A

0.9 - 0.69 = 0.21g

How much NaCl required to make above solution to isotonic solution? A. 0.21 g B. 0.69 g C. 0.9 g D. 2 g E. 9 g Ans. A

Copyright © 2000-2018 TIPS Inc. Unauthorized reproduction of this manual is strictly prohibited and it is illegal to 51-12 reproduce without permission. This manual is being used during review sessions conducted by PharmacyPrep.

www.pharmacyprep.com Dissociation factors: The dissociation factor is the measure of the number of particles resulted in when a

substance is placed in aqueous solution.

Non-electrolyte substances have low dissociation factor. Dissociation factor for non-electrolytes substances are assigned a value of 1. Substance dissociate into two ions dissociation factor (i) = 1.8 For three ions (i) = 2.6 For four ions (i) = 3.4 For five ions (i) = 4.2 Salts that dissociate into two ions: NaCl, KCl, LiCl, NaHCO 3 Salts that dissociate into three ions: Li 2 CO 3 , Na 2 CO 3 , ZnCl 2 CaCl 2 , Mg(OH) 2 Sodium chloride equivalent 1-Calculate the sodium chloride equivalent for fluorescein sodium, which dissociates into three ions and has a molecular weight of 376. i factor for sodium chloride = 1.8 i factor for fluorescein sodium = 2.6 Mol. wt of sodium chloride x i factor of substance = sodium chloride equivalent i factor of sodium chloride Mol. wt of substance 58.5 x 2.6 = 0.22 1.8 376 NaCl equivalent (E) = 0.22 1) Zinc sulfate is a two-ion electrolyte, dissociating 80% in weak solutions. Calculate its dissociation factor. A. 1.8 ZnSO4 à Zn + SO4 + B. 1.7 ZnSO4 C. 2 100 à 70 + 70 + D. 2.5 E. 1 EXPLAINED: On the basis of 70% dissociation, 100 particles of zinc sulphate (ZnSO 4 ) yield: 70 zinc ions 70 sulphate ions 30 undissociated particles 170 total particles Because 170 particles represent 1.7 times as many particles as were present before dissociation, the dissociation factor is 1.7.

CALCULATIONS INVOLVING BALANCE SENSITIVITY A prescription balance class III, balance has a sensitivity requirement of 6 mg, it means? A. Maximum weighing capacity of balance is 6 mg B. Minimum weighing capacity of balance is 6 mg C. As much as 6 mg could be added to or removed from pan before the balance marker move 1 division Copyright © 2000-2018 TIPS Inc. Unauthorized reproduction of this manual is strictly prohibited and it is illegal to 51-13 reproduce without permission. This manual is being used during review sessions conducted by PharmacyPrep.

www.pharmacyprep.com D. Have maximum capacity weight of balance 120 g Ans. C Pharmacy electronic balances typically have a less 10 mg sensitivity What is the minimum quantity that can be weight on a balance with sensitivity requirements of 10 mg of a 5% error is permissible? A. 6 mg B. 15 mg Sensitivity Requirement = Weight x Error C. 300 mg 10 = weight x 5/100 D. 600 mg Weight = (10/5) x100 = 200 mg E. 200 mg Ans. E

2) What is the sensitivity of a balance that can weight 120 mg of a substance and has a permissible error of 5%? A. 15 mg SR = Weight x Error B. 6 mg C. 5 mg SR = 120 mg x 5 /100 = 6 mg D. 120 mg E. 12 mg Ans. B SR = W x E W = 150 mg E = 5% 7.5 mg Error = sensitivity requirement/weight Weight = sensitivity requirement/error

E = SR/W SR = 10 mg W = 300 mg 3.3%

W = SR/E SR = 10 E = 5% 200 mg

The balance SR is 6 mg. A pharmacist makes an attempt to calculate 120 mg of codeine sulfate, calculate the percentage error? Error = sensitivity requirement/weight Practice Calculations 1) How many ml of 0.9% (w/v) NaCl solution should be prepared from 250 ml of 25% (w/v) solution? A-3750 ml B-2500 ml C-6944.4 ml D-9 ml Ans: C C 1 V1 = C 2V 2 (X)(0.9)= 250(25) X = 6944.4 ml 2) A Pharmacy tech adds 75 mL of strong iodine solution USP (5.0% w/v) to 1 litre of sterile water for irrigation. What is the % w/v of iodine present? A) 0.35% B) 0.475% C) 0.53% D) 0.60% Ans: A C 1V1 = C 2V2 Copyright © 2000-2018 TIPS Inc. Unauthorized reproduction of this manual is strictly prohibited and it is illegal to 51-14 reproduce without permission. This manual is being used during review sessions conducted by PharmacyPrep.

www.pharmacyprep.com 75 (5) = (1075) X X = 0.35% 3) Determine the specific gravity of a mixture of 900 mL of syrup with a sp. Gr. of 1.1898, 700 mL of elixir with a sp. Gr. of 0.975 and 1150 mL of glycerin with a sp. Gr. of 1.240. a) 1.1349 b) 1.1486 c) 1.1486 d) 1.1561 Ans: d 900 ml x 1.1898 = 1070.82 700 ml x 0.975 = 682.5 (1150 ml/3179 ) x 1.240 = 1426 3179./ 2750= 1.1561 4)What is the percentage alcohol in a mixture of 2000 mL of 50% (v/v) alcohol, 500 mL of 70% (v/v) alcohol and 2.5 L of 95% (v/v) alcohol? a) 71.67% b) 73.25% c) 72.50% d) 74.5% Ans: 2000 ml x 0.5 = 1000 500 ml x 0.7 = 350 (2500 ml /5000 ml) x (0.95 / 3725) = 2375 (3725 /5000) = (X/100) X = 74.5 5) If 800 g of 5% coal tar ointment is mixed with 1200 g of a 10% coal tar ointment. What is the concentration of coal tar in the finished product? a) 8.5 % b) 9.5% c) 8% d) 9% Ans: C

800 x 0.005 = 40 1200 x 0.001 = 120 160/ 2000 = X/100

X = 8%

Copyright © 2000-2018 TIPS Inc. Unauthorized reproduction of this manual is strictly prohibited and it is illegal to 51-15 reproduce without permission. This manual is being used during review sessions conducted by PharmacyPrep.

www.pharmacyprep.com

Copyright © 2000-2018 TIPS Inc. Unauthorized reproduction of this manual is strictly prohibited and it is illegal to 51-16 reproduce without permission. This manual is being used during review sessions conducted by PharmacyPrep.

www.PharmacyPrep.com

Generic and Brand Names

52 Generic and Brand Names Antiemetics: to treat vomiting Generic Name Brand Name Gravol (generics) Dimenhydrinate Meclizine Bonamine Ondansetron Zofran Prochlorperazine Stemetil (generics) Promethazine Phenergan Cardiovascular Drugs Antiarrhythmics: To treat irregular heart rhythms Generic Name Brand Name Amiodarone Cordarone (generics) Disopyramide Rhythmodan (generics) Lidocaine

Xylocaine

Generic Name Metoclopramide Scopolamine Domperidone Diclectin (vitamin B6 + doxylamine)

Brand Name Generics Generics

Generic Name procainamide Quinidine

Brand Name PronestylSR, Procan (generics) Biquin (generics)

Propafenone

Rythmol (generics)

Antihypertensive: to treat high blood pressure Diuretics (water pills): There are five types of diuretics i.e. Thiazides, Loop diuretics, potassium sparing, osmotic and carbonic ahnydrase diuretics Generic Name Brand Name Generic Name Brand Name Hydrochlorothiazide Generics Furosemide Lasix (generics) Metolazone Zaroxolyn Potassium Sparing Diuretics Generic Name Brand Name amiloride + Moduret (generics) hydrochlorothiazide spironolactone + Adactazide (generics) hydrochlorothiazide βeta Blockers (suffix "lol") Generic Name Brand Name Acebutolol Monitan, Sectral (generics) Atenolol Tenormin (generics) Bisoprolol Mococor Carvedilol Coreg Labetalol Trandate (generics) Metoprolol Lopresor, Betaloc (generics)

Generic Name Triamterene + hydrochlorothiazide

Generic Name Nadolol Pindolol Propanolol Sotalol Timolol

Brand Name generics

Brand Name Corgard (generics) Visken (generics) Inderal (generics) Sotacor (generics) generics

Copyright © 2000-2018 TIPS Inc. Unauthorized reproduction of this manual is strictly prohibited and it is illegal to 52-1 reproduce without permission. This manual is being used during review sessions conducted by PharmacyPrep.

www.PharmacyPrep.com

Generic and Brand Names

Angiotensin-Converting Enzyme Inhibitors (ACE Inhibitors) (suffix "pril") Generic Name Brand Name Generic Name Brand Name Benazepril Lotensin Lisinopril Zestril, Prinivil (generics) Captopril Capoten (generics) Perindropril Coversyl Cilazapril Inhibace Quinapril Accupril Enalapril Vasotec Ramipril Altace Fosinopril Monopril Trandolapril Mavik Angiotensin II, AT 1 Receptor Blockers (ARBs) Generic Name Brand Name Candesartan Atacand Eprosartan Teveten Irbesartan Avopro

(Suffix "sartan") Generic Name Losartan Telmisartan Valsartan

Brand Name Cozaar Micardis Diovan

Calcium-Channel Blockers (CCBs) (suffix "dipine, except verapamil and diltiazem) Generic Name Brand Name Generic Name Brand Name Amlodipine Norvasc Non-Dihydropyridines Nifedipine Adalat XL Diltiazem Cardizem CD (generics) Felodipine Plendil, Renedil ER Verapamil Isoptin SR (generics) Vasodilating Agents Generic Name Hydralazine Isosorbide dinitrate Isosorbide mononitrate Nitroglycerin SL tablet

Brand Name Apresoline (generics)

Generic Name Nitroglycerin SL spray

Cedocard SR (generics) Imdure Nitrostat

Minoxidil

Centrally Acting Antihypertensive Agents Generic Name Brand Name Methyldopa Aldomet (generics)

Generic Name Clonidine

Alpha-Adrenergic Blockers (alpha1 receptor blockers) (Suffix "zosin") Generic Name Brand Name Generic Name Alfuzosin Xatral Prazosin Doxazosin Cardura (generics) Tamsulosin Terazosin Hytrin (generics)

Brand Name Nitrolingual Spray (generics) Loniten

Brand Name Catapres (generics)

Brand Name Minipress (generics) Flomax

Antihyperlipidemic Agents (HMGCoA reductase inhibitors suffix "statin"): To treat high cholesterol, there are 4 categories of drugs i.e. statins, fibrates, niacin and resins Generic Name Brand Name Generic Name Brand Name Atorvastatin Lipitor Fluvastatin Lescol Bezafibrate Bezalip Gemfibrozil Lopid (generics) Cholestyramine Questran (generics) Lovastatin Mevacor (generics) resin Fenofibrate Lipidil (generics), Pravastatin Pravachol (generics) Lipidil Supra Rosuvastatin Crestor Simvastatin Zocor (generics)

Copyright © 2000-2018 TIPS Inc. Unauthorized reproduction of this manual is strictly prohibited and it is illegal to 52-2 reproduce without permission. This manual is being used during review sessions conducted by PharmacyPrep.

www.PharmacyPrep.com CNS drugs Opiates & Other Narcotics Generic Name Brand Name Codeine generics Dextropropoxyphene Darvon N, 642, (generics) Hydrocodone and Hycodan, Hycomine preps Novahistex DH, Novahistine DH, Tussionex Hydromorphone Dilaudid (generics) Fentanyl Duragesic

Generic and Brand Names

Generic Name Meperidine Morphine Oxycodone preps

Brand Name Demerol MS Contin and generics, M.O.S. Percocet (generics)

Pentazocine Oxycodone

Talwin Oxycontin

Anticonvulsants (antiepileptics or antiseizure drugs) Generic Name Brand Name Generic Name Carbamazepine Tegretol (generics) clobazam Phenytoin Dilantin clonazepam Gabapentin Neurontin (generics) diazepam vigabatrin Sabril lorazepam phenobarbital Generics primidone divalproex Epival (generics) lamotrigine valproic acid Depakene (generics) Antiparkinson Drugs: To treat Parkinson's disease Generic Name Brand Name levodopa/carbidopa Sinemet (generics) levodopa/benserazide Prolopa bromocriptine Parlodel (generics) trihexyphenidyl generics benztropine Cogentin (generics)

Generic Name Amantadine pergolide pramipexole ropinerole selegiline

Brand Name Frisium (generics) Rivotril (generics) Valium (generics) Ativan (generics) Mysoline (generics) Lamictal (generics)

Brand Name Symmetrel (generics) Permax Mirapex ReQuip Eldepryl (generics)

Antidepressants: SSRIs, TCAs, SNRIs and MAOIs Monoamine Oxidase Inhibitors (MAOIs) Generic Name Brand Name Generic Name phenelzine Nardil tranylcypromine Parnate

Brand Name

Tricyclic Antidepressants Generic Name Amitriptyline clomipramine desipramine doxepin imipramine

Generic Name maprotiline nortriptyline trazodone trimipramine

Brand Name Generics Aventyl (generics) Desyrel (generics) Surmontil (generics)

Generic Name fluvoxamine paroxetine

Brand Name Luvox (generics) Paxil

Brand Name generics Anafranil (generics) Norpramin (generics) Sinequan (generics) Tofranil (generics)

Selective Serotonin Re-uptake Inhibitors (SSRIs) Generic Name Brand Name Citalopram Celexa Fluoxetine Prozac (generics) sertraline Zoloft (generics)

Copyright © 2000-2018 TIPS Inc. Unauthorized reproduction of this manual is strictly prohibited and it is illegal to 52-3 reproduce without permission. This manual is being used during review sessions conducted by PharmacyPrep.

www.PharmacyPrep.com

Generic and Brand Names

Serotonin norepinephrine reuptake inhibitors (SNRI) and Dual action antidepressants Generic Name Brand Name Generic Name Brand Name Venlafaxine Effexor Bupropion Wellbutrin SR, Zyban Mirtazapine Remeron Buspirone BuSpar (generics) Reversible Inhibitors of Monoamine Oxidase (RIMAs) Moclobemide Manerix and generics Psychotropic (neuroleptic) agents (Antipsychotic drugs, antischizophrenia drugs) Generic Name Brand Name Generic Name Brand Name Chlorpromazine Novo-Chlorpromazine Hydroxyzine Atarax (generics) Fluphenazine Moditen (generics) Lithium Lithane, Duralith (generics) Haloperidol generics pericyazine Neuleptil loxapine generics perphenazine Trilafon (generics) pimozide Orap prochlorperazine Stemetil (generics) thioridazine generics trifluoperazine generics thiothixene Navane Second generation antipsychotics quetiapine Seroquel risperidone Risperdal olanzapine Zyprexa, Zyprexa Zydis OD Benzodiazepines (suffix "am") sleeping pill Generic Name Brand Name Alprazolam Xanax (generics) Chlordiazepoxide generics Diazepam Valium (generics) Flurazepam Dalmane (generics) Lorazepam Ativan (generics) Oxazepam Generics

Generic Name Temazepam Triazolam Clonazepam Bromazepam

Brand Name Restoril (generics) Halcion (generics)

Barbiturates (suffix "tal") Generic Name Brand Name Phenobarbital

Generic Name Thiopental

Brand Name

Stimulants Generic Name dextroamphetamine

Generic Name methylphenidate

Brand Name Ritalin (generics), Ritalin SR

Brand Name Dexedrine

Gastrointestinal Drugs Antacids Generic Name Aluminum hydroxide Aluminum and magnesium hydroxide Calcium carbonate Dihydroxy-aluminum sodium carbonate

Brand Name Amphogel Maalox, Mylanta, Gelusil Tums®

Generic Name Magnesium hydroxide Sodium/potassium bicarbonate Alginic acid/aluminum hydroxide

Brand Name Milk of Magnesia Alka seltzer Gaviscon

Rolaids®

H 2 -Receptor antagonists (Suffix "tidine") Generic Name Brand Name Cimetidine generics

Generic Name Nizatidine

Brand Name Axid (generics)

Copyright © 2000-2018 TIPS Inc. Unauthorized reproduction of this manual is strictly prohibited and it is illegal to 52-4 reproduce without permission. This manual is being used during review sessions conducted by PharmacyPrep.

www.PharmacyPrep.com Famotidine

Pepcid (generics)

Generic and Brand Names Ranitidine

Proton pump inhibitors (suffix "azole"): To treat ulcers Generic Name Brand Name Generic Name esomeprazole Nexium omeprazole lansoprazole Prevacid pantoprazole rabeprazole Pariet

Zantac (generics)

Brand Name Losec Pantoloc

Gastroduodenal Cytoprotective Agents Generic Name Brand Name sucralfate Sulcrate

Generic Name misoprostol

Brand Name Cytotec

Prokinetic Agents (antiemetics) Generic Name Brand Name metoclopramide generics

Generic Name domperidone

Brand Name generics

Generic Name Levothyroxine sod

Brand Name Cytomel

Brand Name Cyclomen

Generic Name Testosterone

Brand Name Andriol, Androgel, Androderm Depo-Testosterone, Delatestryl

Brand Name Estraderm®, Estrace®, Estalis®, Estrogel® Climera

Generic Name conjugated estrogens

Brand Name Premarin, C.E.S.

hormone replacement

Fem-HRT® Premplus®

Generic Name megastrol acetate norethindrone

Brand Name Megace® (generics) Norlutate® Micronor®

progesterone

Prometrium

Generic Name

Brand Name Demulen®

Hormones Thyroid Hormones Generic Name Levothyroxine sodium Thyroid Sex hormones Androgens Generic Name Danazol

Estrogen Generic Name estradiol-17β

estradiol-17β micronized estradiol Progesterone Generic Name levonorgestrel medroxyprogesterone acetate

Brand Name Synthroid, Eltroxin Thyroid

Estring, Vaginal Ring

Brand Name Mirena Provera (generics), Depo-Provera

Combined Oral Contraceptives Generic Name Brand Name Ethinyl estradiol and Alesse Levonorgestrel Brevicon® cyproterone acetate Diane-35 and ethinyl estradioL Ortho7/7/7

Marvelon® Ortho-Cept® Min-Ovral®

Copyright © 2000-2018 TIPS Inc. Unauthorized reproduction of this manual is strictly prohibited and it is illegal to 52-5 reproduce without permission. This manual is being used during review sessions conducted by PharmacyPrep.

www.PharmacyPrep.com

Generic and Brand Names

Triphasil® Diabetes: Insulin Generic Name Iletin® Novolin®

Brand Name Humulin R,N,U Mixtures of 30/70; 20/80; 50/50; 40/60

Synphasic®

Generic Name Humalog® NPH (intermediate)

Brand Name Humalog Mix25

Glargine (long acting)

Lantus

Oral Hypoglycemic Agents Generic Name Brand Name acarbose Prandase metformin Glucophage (generics) repaglinide GlucoNorm gliclazide Diamicron® Diamicron MR® (generics) glyburide Euglucon®, Diaβeta® (generics) Neuromuscular Blocking Agents Generic Name Brand Name pancuronium Pavulon bromide Anticholinergic Drugs Generic Name Atropine benztropine dicyclomine

Brand Name generics Cogentin (generics) Bentylol

Adrenergic Drugs (Decongestants) Generic Name Brand Name norepinephrine Levophed bitartrate (levarterenol) oxymetazoline Claritin Eye drops xylometazoline Otrivin Anti-infective agents: Penicillin's Generic Name Brand Name amoxicillin generics ampicillin generics penicillin V generics

Anti-infective agents: Cephalosporin's Generic Name Brand Name cefaclor Ceclor (generics) cefazolin Kefzol cefixime Suprax cephradine Velosef

Generic Name pioglitazone rosiglitazone chlorpropamide glimepiride

Brand Name Actos Avandia Generics Amaryl®

Generic Name succinylcholine

Brand Name Quelicin

Generic Name Ipratropium Oxybutinin Tiotropium

Brand Name Atrovent Ditropan Spiriva

Generic Name pseudoephedrine

Brand Name Sudafed

phenylephrine

Prefrin, Mydfrin

Generic Name bacampicillin cloxacilin pivampicillin

Brand Name Penglobe Generics Pondocillin®

Generic Name cefuroxime cephalexin cefprozil

Brand Name Ceftin (generics) generics Cefzil

Copyright © 2000-2018 TIPS Inc. Unauthorized reproduction of this manual is strictly prohibited and it is illegal to 52-6 reproduce without permission. This manual is being used during review sessions conducted by PharmacyPrep.

www.PharmacyPrep.com

Generic and Brand Names

Anti-infective agents: Macrolides Generic Name Brand Name azithromycin Zithromax erythromycin Eryc (generics)

Generic Name clarithromycin

Brand Name Biaxin

Anti-infective agents: Amino glycosides Generic Name Brand Name gentamicin Garamycin (generics) amikacin Amikin

Generic Name tobramycin

Brand Name Nebcin®

Anti-infective agents: Tetracycline Generic Name Brand Name minocycline Minocin (generics) doxycycline Vibra-Tabs (generics)

Generic Name tetracycline

Brand Name generics

Anti-infective agents: fluroquinolones Generic Name Brand Name Ciprofloxacin Norfloxacin Moxifloxacin

Generic Name Gatifloxacin Ofloxacin

Brand Name

Anti-infective agents: Others Generic Name Brand Name clindamycin Dalacin (generics) metronidazole Flagyl (generics)

Generic Name vancomycin

Brand Name Vancocin

Anti-infective agents: sulfa drugs Generic Name Brand Name co-trimoxazole Septra (generics)

Generic Name

Brand Name

Brand Name Fungizone® Canesten® (generics), Lotrimin- OTC Diflucan (generics) Single dose Monistat, Micatin® OTC Nilstat, Myocostatin (generics) Desemex OTC

Generic Name griseofulvin otraconazole

Brand Name Fulvicin® U/F Sporanox

ketoconazole

Nizoral (generics)

terbinafine

Lamisil (generics)

tolnaftate

Tinactin OTC

Brand Name Ziagsen® Agenerase® Symmetrel® (generics) Zovirax (generics)

Generic Name lamivudine nelfinavir nevirapine

Brand Name Heptovir® Viracept® Viramune®

oseltamivir

Tamiflu

Antifungals Generic Name Amphotericin B clotrimazole fluconazole Miconazole Nystatin undecylanate Anti-Viral Agents Generic Name abacavir amprenavir amantadine acyclovir

Copyright © 2000-2018 TIPS Inc. Unauthorized reproduction of this manual is strictly prohibited and it is illegal to 52-7 reproduce without permission. This manual is being used during review sessions conducted by PharmacyPrep.

www.PharmacyPrep.com delavirdine didanosine efavirenz famciclovir ganciclovir indinavir zidovudine Anti-neoplastic Drugs Generic Name bleomycin sulfate busulfan carboplatin chlorambucil cisplatin cyclophosphamide cytarabine dacarbazine daunorubicin doxorubicin fludaribine phosphate

Rescriptor® Videx® Sustiva® Famvir Cytovene® Crixivan® Retrovir®

Brand Name Blenoxane® Myleran® Paraplatin Leukeran® Cisplatin® Cytoxan®, Procytox® Cytosar®, ARA-C® DTIC® Cerubidine® Adriamycin® Fludara®

Antihistamine/Decongestant Products Generic Name Brand Name cetirizine Reactine (generics) chlorpheniramine Chlor-Tripolon® desloratadine Aerius®* dimenhydrinate Gravol® (generics) diphenhydramine Benadryl®, Allerdryl®

Generic and Brand Names ribavirin ritonavir/ lopinavir saquinavir stavudine valacyclovir zalcitabine zanamivir

Generic Name ifosfamide methotrexate mitomycin mitoxantrone paclitaxel tamoxifen vinblastine sulfate vincristine sulfate idarubicin 5-fluororoucil

Generic Name fexofenadine hydroxyzine loratadine meclizine

Rebetron® Kaletra® Invirase® Zerit® Valtrex® Hivid® Relenza®

Brand Name Ifex® generics Mutamycin® Novantrone® Taxol® Nalvodex® (generics) Vinblastine® Vincristine® Idamycin® Adrucil®, Efudex®, 5-FU

Brand Name Allegra Atarax (generics) Claritin Bonamine®

Coughs and Colds Drugs Generic Name Brand Name Neo Citran® Actifed® Drixoral® Contac®

Generic Name Drisdan®

Brand Name Tylenol Cold and Sinus®

Anti-emetics Generic Name droperidol metoclopramide ondansetron

Generic Name prochlorperazine promethazine

Brand Name Stemetil® (generics) Phenergan®

Brand Name Droperidol® generics Zofran®

Antipyretics, Analgesics; Non-steroidal Anti-inflammatory Drugs Generic Name Brand Name Generic Name acetylsalicylic acid, Aspirin® ibuprofen enteric coated ASA Entrophen®, indomethacin Novasen® celecoxib Celebrex® ketoprofen diclofenac Voltaren® (generics) ketorolac diflunisal generics meloxicam floctafenine Idarac® (generics) naproxen flurbiprofen Ansaid® (generics) piroxicam tolmetin Tolectin® sulindac tiaprofenic acid Surgam® (generics)

Brand Name Motrin®, Advil® (generics) Indocid® (generics) Orudis® (generics) Toradol® Mobicox® Anaprox®, Naprosyn® (generics) Feldene® (generics) generics

Copyright © 2000-2018 TIPS Inc. Unauthorized reproduction of this manual is strictly prohibited and it is illegal to 52-8 reproduce without permission. This manual is being used during review sessions conducted by PharmacyPrep.

www.PharmacyPrep.com Antidiarrheal Generic Name diphenoxylate/atropine attapulgite loperamide Laxatives Generic Name bisacodyl cascara sagrada castor oil docusate sodium docusate calcium lactulose senna

Generic and Brand Names

Brand Name Lomotil®

Generic Name psyllium mucilloid

Kaopectate® Imodium® (generics)

bismuth subsalicylate

Brand Name Dulcolax®

Acilac® (generics) Senokot®

Generic Name magnesium citrate magnesium hydroxide magnesium sulphate mineral oil (heavy) polyethylene glycol products psyllium

Brand Name Metamucil®, Prodiem®, Mucillium® Pepto-Bismol®

Brand Name Citro-Mag® Milk of Magnesia® Epsom Salts® GoLytely®, Colyte® Metamucil®, Prodiem®, Mucillium®

Copyright © 2000-2018 TIPS Inc. Unauthorized reproduction of this manual is strictly prohibited and it is illegal to 52-9 reproduce without permission. This manual is being used during review sessions conducted by PharmacyPrep.

www.PharmacyPrep.com

Generic and Brand Names

Copyright © 2000-2018 TIPS Inc. Unauthorized reproduction of this manual is strictly prohibited and it is illegal to 52-10 reproduce without permission. This manual is being used during review sessions conducted by PharmacyPrep.

www.pharmacyprep.com

Prescription Processing

53 Prescription Processing Questions Alerts! Common questions in pharmacy exam is to ask! · Definition of prescription · Essentials in prescription. Prescriber name and address, patient name, date and medication and directions, repeats (regulations). Prescriber signature. · Directions of ophthalmic and ear drop · Directions that often causes errors

Receiving prescription. (implied consent) Entering prescription in computer (KROLL, HEALTHWATCH, DELTA, NEXYSS) Preparing a prescription Labeling Dispensing (checked by the pharmacist) The prescription: Written direction from a registered medical practitioner to a pharmacist for preparing and dispensing a drug. Dr. Michael Chang MD. TIPS Medical Centre 4789 Yonge St., Toronto. 416-227-7737 Date: 00/00/00 Name: Laura Herzberg Age: 55 Gender: Female

1230 yonge street, Toronto 416 233 4545

HealthCare DIN 12345678 Rx: 1002345 Herzberg, Laura 4 Synthroid 112 mcg

00/00/00 Refills:

Levothyroxine 112 mcg Rx

Synthroid 0.112 mg

90 Tab Abbt

Dr. Chang Michael.

TAKE 1 TABLET TWICE DAIL

1 tablet qd. Mitte: 90 Repeat

4

times Sign: Dr. Michael Chang

PRESCRIPTION LABEL Copyright © 2000-2018 TIPS Inc. Unauthorized reproduction of this manual is strictly prohibited and it is illegal to 53-1 reproduce without permission. This manual is being used during review sessions conducted by PharmacyPrep.

www.pharmacyprep.com

Rx Abbreviation a ac Ad lib AD AS AU bid bx c cc dr dx dx g gr gtt h hr h.s. hx ID IM IU IV IVPB kg L lb mcg mEq mg ml oz p

Meaning Before Before meals As desired Right ear Left ear Both ear Twice a day Biopsy with With food Dram Diagnosis Fracture Gram Grain Drop Hour Hour At bed time History Intradermal Intramuscular International unit Intravenous IV piggyback Kilogram Litre Pound Microgram Milliequivalent Milligram Milliliter Ounce post

Prescription Processing

Latin Phrase Ante Ante cibum Ad libitum

bis in die cum

Hora somni

Abbreviation OD OS OU pc p.o. pr p.r.n. pt

Meaning

qt rx, Rx s ss stat supp sx T, Tbsp or tbs t, tsp

Right eye Left eye Both eyes After meal By mouth Per rectum As needed Pint Every Every morning Everyday Every other day Four times a day Every two hours… etc Quart Prescription Without One half Immediately Suppository Symptoms Tablespoon Teaspoon

t.i.d. T.O. tr tx ung VO Ex. aqua

3 times a day Telephone order Tincture Treatment Ointment Verbal order In water

Fl.oz

Fluid ounce

qam. qd qod qid Q2h, q3h, etc

Latin Phrase

Post cibum Per os Pro re nata quaque Quaque die Quarter in die

sine

ter in die

post

ISMP. The Institute of Safe Medication Practices dangerous abbreviations. Interpreting Directions (SIG) of prescriptions · caps ii tid pcà Take two capsules three times a day after meals (after food). · suppos I pr q6h prn à Unwrap and insert 1 suppository into the rectum every 6 hours as needed. · tabs ss stat. tabs I q6h ccà Take one half tablets now (at once), then take 1 tablet every 6 hours with food. Copyright © 2000-2018 TIPS Inc. Unauthorized reproduction of this manual is strictly prohibited and it is illegal to 53-2 reproduce without permission. This manual is being used during review sessions conducted by PharmacyPrep.

www.pharmacyprep.com · · · · · · · ·

Prescription Processing

fl.oz I tid cc àTake two tablespoonful three times a day with food gtts ii ou qid for 7 daysàInstil 2 drops into both eyes 4 times a day for 7 days For a child. 10 mL stat, then 5 mL tid for 10 daysàgive 2 teaspoonful at start then 1 teaspoonful three times a day for 10 days. gtts IV au qid for 7 daysà Instil 4 drops in both ears, 4 times a day for 7 days tabs ii qam ss at noon & tabs ii qhsàTake 2 tablets every morning, half tablet at noon and two tablets at bed time. fl.oz IV stat; fl.oz II q4h udàTake teacupful (120 mL) at start: four tablespoonful every 4 hours as directed. app ung sp aa tidà Apply ointment sparingly to affected area three times a day. 10 gtts x po q12h udàGive 10 drops orally every 12 hours as directed

Direction of administration of prescription order · For adults: · For eye drops, and nasal drops useà instil or place · For tabs and capsulesà take · For liquidà take · Suppositoriesà unwrap and insert 1 suppository into the rectum or into vagina · Ointment and cream, topicalà apply · Aerosolsàinhale · Sublingual tabletsà place or dissolve one tablet under the tongue · Effervescentàdissolve one tablet in water and take For children: Oral liquid, tablets, or capsules use "give" Chewable tablets use "chew" Quantities in prescriptions: 1/7 to denote days 10/7 = 10 days supply 1/52 = 1 week’s supply 3/52 = 3 weeks supply 1/12 = 1 month supply 3/12 = 3 month supply 1/24 = 1 hour

Rx Prednisone 5 mg 3/12 Sig. Take 1 tab daily How many 5 mg tablet is dispensed? A. 3 tablets B. 12 tablets C. 90 tablets D. 36 tablets Ans.

Prescription should consist of: Prescriber Prescriber name and title (number) Prescriber office address Prescriber phone number Prescriber signature Patient Patient name and address Patient age Copyright © 2000-2018 TIPS Inc. Unauthorized reproduction of this manual is strictly prohibited and it is illegal to 53-3 reproduce without permission. This manual is being used during review sessions conducted by PharmacyPrep.

www.pharmacyprep.com

Rx

Prescription Processing

Date on which prescription was written Drug name, strength Quantity to be dispensed Signa: direction to patient Refill instructions

Date of prescription Dispensing direction to pharmacist or subscription Direction for patient or signa (to be placed on label) Refill, special labeling or other instructions Prescriber signature and license or CPSO # Clarify with patient · The following information should be added to the prescription if missing or clarified. · Patient demographic information · The correct spelling of the patient’s full name (last and first) · The patient addresses · The patient home phone number · Payment · The age and date of birth (DOB) The physician full name (last and first) The physician address and telephone number Clarify with doctor · Drug interactions, contraindication or DRPs or DTPs. · Potential side effects to patient medical conditions. · Interchangeable. (except generic and brand) · To verify dose, missing information of prescription drug (dosage form, strength, duration) and route administration. · Doctor signature is illegible. Suspicious prescription of forged narcotic prescription. What if doctor signature is not legible? Expanded scope of practice include · Dispensing emergence medication (advancing) · Extending Prescription (Rx fee can be charged). · Pharmaceutical opinion · Ordering, Monitoring and interpreting Lab Test to optimize medication management. · Adaptations (changing dosage form, strength, therapeutic equivalent drug) What is prescription adaptation (expanded scope of practice)? Prescribed dosage form does not exist. Prescribed dosage release form is not available? The following Information added to the prescription at the time of dispensing: The prescription numbers. Retail price (cost + professional fee = total) Copyright © 2000-2018 TIPS Inc. Unauthorized reproduction of this manual is strictly prohibited and it is illegal to 53-4 reproduce without permission. This manual is being used during review sessions conducted by PharmacyPrep.

www.pharmacyprep.com

Prescription Processing

Label. The information, which must be on the prescription label include: · Patient full name (first and last) Question Alerts! · Prescription number What is NOT present on label of · Instruction for the patient use of the prescription drug container? medication · Name of drug · Manufacturer (if drug was ordered by generic name). · The quantity · The strength · Filling date · Physician name/initial · Drug Identification Number · Pharmacist name/initials NOT on label. Expiry dates Package insert The package insert is a document that is included in the medication's surrounding box or wrapper.

Tips

· · · · · ·

1. OD 2. pc 3. OS 4. ac 5. prescription 6. expiry dates 7. ex aqua 8. OU 9. AD What is prescription? à Written direction from a registered medical practitioner to a pharmacist for preparing and dispensing a drug ( ) The information which must be on the prescription label includes: (patients full name, prescription number, instruction for the patient use of medication, name of drug, manufacturer, quantity, strength, date, physician name or initial, drug identification number, pharmacist name/initial (optional) This should not be on the label ( ) Prepared in water ( ) before meals ( ); after meal ( ); right eye ( ), left eye ( ), both eyes ( ), right ear ( )

Select True or False Statement · · · ·

Abbreviation "AU" is mistaken for? "OU". Abbreviation "o.d" or "OD" is mistaken for "right eye" use daily Abbreviation "per os" intended meaning is? orally or by mouth Abbreviation "mg" when handwritten is misinterpreted as? microgram "mcg"

Copyright © 2000-2018 TIPS Inc. Unauthorized reproduction of this manual is strictly prohibited and it is illegal to 53-5 reproduce without permission. This manual is being used during review sessions conducted by PharmacyPrep.

www.pharmacyprep.com

Prescription Processing

Copyright © 2000-2018 TIPS Inc. Unauthorized reproduction of this manual is strictly prohibited and it is illegal to 53-6 reproduce without permission. This manual is being used during review sessions conducted by PharmacyPrep.

www.pharmacyprep.com

Safety of Medications in Special Populations

54 Safety of Medications in Special Populations Pregnant, breast feeding, Infants, and Seniors (geriatrics) Geriatric Population Pharmacokinetic factors Increase with age Gastric pH (basic) or achlorhydric Questions Alerts! Body fat or lean body mass ratio What pharmacokinetic factors Decrease with age increase with age? Catabolism Lean body mass (breakdown) Acid secretion GI motility Renal function (CrCl) Serum albumin Total body water Cytochrome P450 enzyme First pass metabolism Serum creatinine Decrease cardiac out Note. Serum creatinine is not a good predictor because creatinine production decreases with age. Pharmacokinetic changes related to aging (absorption, distribution, metabolism, elimination) Heart - Decrease Cardiac out put ¯ Renal-Decreased blood flow ¯ ¯ renal elimination (half-life -) Liver-Reduced enzyme production ¯ ¯ CYP metabolism or phase I metabolism, hepatic blood flow GI - pH increased (alkaline) Total body water 15% ¯ Copyright © 2000-2018 TIPS Inc. Unauthorized reproduction of this manual is strictly prohibited and it is illegal to 54-1 reproduce without permission. This manual is being used during review sessions conducted by PharmacyPrep.

www.pharmacyprep.com Body fat Drug excretion Absorption (drugs, nutrients, vitamins, supplements).

Safety of Medications in Special Populations ¯ ¯

50% women and 100% man ¯ gastric emptying time, ¯absorption surface

The Modified Cockcroft and Gault equation is a commonly used formula to estimate creatinine clearance (ClCr) using a stable serum creatinine level and patient demographics (e.g., age, gender, weight). 1.2 (140−age) (weight in kg) ClCr (mL/min) = Serum creatinine (µmol/L) For females, multiply the result by 0.85. Modification of creatinine clearance estimates may be required in some patients. The accuracy of using the serum creatinine value to predict creatinine clearance is influenced by diseases (e.g., cirrhosis), clinical conditions (e.g., malnutrition, obesity, spinal cord injuries) and dietary intake (e.g., high consumption of meat). Drug absorption · Rate of absorptionà may be altered in some patient · Extent of absorptionà No effect Calcium supplements: · Calcium supplements: 1500 mg/day and vitamin D 800 IU · Calcium carbonate is required acidic medium, thus it is not preferable in seniors. · Calcium citrate is recommended · Calcium carbonate takes in divided doses Distribution · Decrease in total body water. Decreases water distribution of water-soluble drugs. (e. g. acetaminophen). · Lipid soluble drugs (diazepam, propranolol) distribution increases. · Albumin levels are decreased with age therefore albumin bound drugs have greater free concentration. · Renal function (renal excretion) decreases with age, 50% decrease of renal function by age of 70. · Geriatric patients have sensitive reaction drugs cause anticholinergic effects and should be avoided. Vitamins in seniors Vitamin B 12 supplements is recommended Medications to use lower dosages (¯ clearance) due to decreased renal clearance in the elderly · Aminoglycosides · Vancomycin · Fluoroquinolones · Penicillins · Imipenem · Digoxin · ACE Inhibitors · Beta Blockers (Atenolol, Nadolol)

Decreased hepatic clearance in the older adults · · · · · · · ·

Benzodiazepines Calcium Channel Blockers Lidocaine Phenytoin Celecoxib Theophylline Imipramine or desipramine, and trazodone Isoniazid

Copyright © 2000-2018 TIPS Inc. Unauthorized reproduction of this manual is strictly prohibited and it is illegal to 54-2 reproduce without permission. This manual is being used during review sessions conducted by PharmacyPrep.

www.pharmacyprep.com · · · ·

Safety of Medications in Special Populations

Sotalol Glyburide Ranitidine, Cimetidine, Famotidine Lithium

·

Procainamide

Beers criteria: Describes the list potential inappropriate medications to elderly (drug to avoid in seniors). Anticholinergic drugs (antihistamines, TCA are not included) - BPH due to urinary retention. Benzodiazepine (- side effects due to low therapeutic index in elderly) Beta blocker (¯ drug effect due to ¯ beta receptors) Glyburide (- hypoglycemic effect) Antiarrhythmic drugs Alpha blockers Alpha2 agonist Digoxin >125 mcg/day · · · · · · · · · · · · · · · ·

Ticlopidine Methyldopa Reserpine Disopyramide (Norpace) Meperidine (Demerol) Propoxyphene (Darvon) Barbiturates (e.g. Fiorinal, Nembutal, Seconal) Benzodiazepine (e.g. Librium, Valium, Dalmane, Halcion) Meprobamate Sedating Antidepressants (Elavil, Doxepin, Imipramine) Methylphenidate (Ritalin) Antiemetics (Phenergan, Tigan) GI antispasmodics (e.g. Donnatal, Bentyl, Levsin) Antidiarrheals (Lomotil) Urinary antispasmodics (Ditropan) NSAIDs (especially indocin, toradol, ponstel, feldene)

Drugs in Pregnancy and Lactation PREGNANCY (Approximately 40 WKs. There are 3 trimesters i.e. each 13 wks.) Biogenesis Organogenesis Fetal period

The first 15-21 days after fertilization. Cleavage, and germ layer formation. 2-8 wks. The major organ starts developing. (Congenital defects or hereditary birth defects). e.g. down syndrome, cleft palate, septal defects. at 9th wk, the embryo referred to as fetus.

PREGNANCY TRIMESTERS 1st trimester EMBRYO (2-8 WKS) HIGH RISK WITH DRUG EXPOSURE.

2nd trimester FETUS (>9WKS)

3rd trimester

Most critical period week 2 to 8 wks least critical period week 1 to 2 wks

Copyright © 2000-2018 TIPS Inc. Unauthorized reproduction of this manual is strictly prohibited and it is illegal to 54-3 reproduce without permission. This manual is being used during review sessions conducted by PharmacyPrep.

www.pharmacyprep.com

Safety of Medications in Special Populations

Embryotoxic. Most critical period in pregnancy for drugs therapy 14 to 56 days (2 to 8 weeks). FETOTOXIC: Most critical period of fetotoxic drugs Ninth week to birth. (Effect on fetal growth or formed organs or functional maturation organs thus affects. On function of organ rather than gross structural damage. Example Behavioral teratogenicity due to phenytoin, antidepressants, or alcohol. Embryo toxic Drugs Embryo toxicity (Embryonic period from 18 to 60 d after conception) results in termination of pregnancy. Basic steps of organogenesis affected and damage irreparable. Can cause structural abnormality. Examples. Hormones (estrogen, progestin, androgen), oral contraceptives, plan B, ACEi, ARBs, statins, misoprostol, clomiphene, and antidiabetic drugs. Teratogenic Drugs Risk is highest in 1st trimester. Teratogenicity causes mental and physical deformation to the developing fetus. Examples. Isotretinoin, tretinoin, warfarin, tetracycline, finasteride, dutasteride, and quinolones. Drug Factors that affect on teratogenicity. Teratogens must reach the developing conceptus. Large molecules with Mol. Weight >1000 (e.g. heparin) do not cross placenta. PLACENTAL EXCHANGE: (SIZE, ELECTRICAL CHARGE, PROTEIN BINDING, AND LIPID SOLUBILITY) Factors related to drugs affects rate and extent of placental transfer include polarity, lipid solubility, and existence of specific carrier protein (P-glycoprotein) binding, Molecular weight, pH, and drug distribution. The transfer of drugs, nutrients and oxygen through placenta occurs via passive diffusion. Absorption Gastric emptying time Distribution Plasma albumin

Delayed ¯ (protein binding and more free drug - body fat

PK changes in pregnancy cause - progestin levels ¯ GI motility (-constipation) & ¯ esophageal sphincter pressure (-heartburn) Placenta - hCG - N&V - lung perfusion & alveoli drug transfer - absorption of pulmonary drugs ¯blood albumin - fraction of free drug molecule -plasma volume - V d , - renal blood flow, - GFR, - cardiac output Alter liver function Alter metabolism

FDA classification Category A à Safe. Adequate clinical controlled trials, has not evidence of harm. Copyright © 2000-2018 TIPS Inc. Unauthorized reproduction of this manual is strictly prohibited and it is illegal to 54-4 reproduce without permission. This manual is being used during review sessions conducted by PharmacyPrep.

www.pharmacyprep.com

Safety of Medications in Special Populations

Category B à Animal studies showed safe. Can be safe in human? Category C à Animal studies shows risk, but human data not available? Category D à Demonstrate risk to fetus Category X à Positive evidence of risk to fetus in clinical well controlled trials. Category X The positive evidence of risk to fetus. Contraindicated in woman who are pregnant or who may become pregnant. · Vitamin A derivative (isotretinoin, and tretinoin) · ACE inhibitors, ARBs, and statins · Warfarin causes Fetal Warfarin Syndrome (first trimester) · Estrogen and androgen, can cause genital tract mal formation. · Methimazole and carbimazole. · Leflunomide · Finasteride and dutasteride · Methotrexate and chemotherapeutic drug · Alcohol in large quantities can cause abnormalities in growth, cardiac, skeletal development. Fetal alcohol syndrome (FAS). · Misoprostol · · · ·

Drugs not used in pregnancy Tetracyclineà Mottling of teeth (taken by mother after 18 week of pregnancy). MetronidazoleàUse in first trimester must be evaluated carefully. Completely contraindicated for Trichomoniasis in 1st trimester. QuinolonesàNot recommended in pregnancyàcauses arthropathies (cartilage erosion) Lithium àcardiovascular malformations

Anticonvulsants contraindicated in first trimester. Phenytoin, Valproic acid; Sodium valproate (neural tubule defect). Phenytoin à cleft-palate, spina-bifida · Anticancer or antineoplastics are contraindicated in pregnancy (potential risk for toxicity). · Finasteride cause abnormalities in male genital, should avoid contact with men who are on this medication. The most teratogenic antibiotic · Nalidixic acid derivative as quinolone · Fluroquinolones · Amantadine · Tetracycline Can be used in Pregnancy · Erythromycin · Cephalexin · Ampicillin · Amoxicillin

Alteration of organ function in pregnancy Heart-increase CO and blood volume

Increases

Decreases

Copyright © 2000-2018 TIPS Inc. Unauthorized reproduction of this manual is strictly prohibited and it is illegal to 54-5 reproduce without permission. This manual is being used during review sessions conducted by PharmacyPrep.

www.pharmacyprep.com

Safety of Medications in Special Populations

Renal-increased blood flow Liver-placenta and fetal liver contribute to metabolism GI-Increased HCL production and N&V. Decreased peristalsis DRUG OF CHOICE IN PREGNANCY · Nausea and vomiting (morning sickness) à Diclectin (vitamin B 6 +doxylamine) · Anti-hypertension (pre-eclampsia) à Methyldopa, hydralazine, and labetalol · Diabetic Type I and II à Insulin · Epilepsy à Carbamazepine · Hyperlipidemia à Cholestyramine · Hyperthyroidism à Propylthiouracil (PTU) · Ulcerative colitis à 5ASA or sulphasalazine · Constipation à Psyllium (bulk laxative), and stool softener · Stomach ulcers à Antacids, H 2 blockers, and PPI. · Vulvovaginitis candida à Clotrimazole (except 1st trimester), miconazole or nystatin. · Migraine, fever and pain à Acetaminophen, NSAID’s (avoid full anti-inflammatory dose in 3rd trimester. · Depression à Fluoxetine or SSRIs · Urinary tract infections à Cephalosporin's (cephalexin), Nitrofurantoin, cotrimoxazole (Possible increase in the risk of neural tubule and cardiovascular, oral cleft in 1st trimester exposure, this can be minimized by folic acid supplements). · Drug of choice to treat Syphilis Penicillin G · Drug of choice to teat herpes? Acyclovir · Drug of choice to treat Chlamydia? Azithromycin · Drug of choice to treat gonorrhea? Cefixime, doxycycline Drugs in lactation Factors that effect on drug secretion into breast milk Lipid solubility Membrane permeation Low Molecular weight Base drugs have high excretion into breast milk because breast milk is weak acidic. This cause ionization increase excretion. Protein binding affinity Pediatric population Neonates (1st 4 postnatal weeks) Infants (weeks 5 to 52 postnatal) Children (1 to 12 years) Adolescents (12 to 16 years)

Copyright © 2000-2018 TIPS Inc. Unauthorized reproduction of this manual is strictly prohibited and it is illegal to 54-6 reproduce without permission. This manual is being used during review sessions conducted by PharmacyPrep.

www.pharmacyprep.com

Safety of Medications in Special Populations

Tips 1) Eat in small & frequent meals, avoid fat, oily and spicy food, avoid heavy meals 4) decreased renal clearance, 7) fiber diet 10)Calcium citrate

2) Urinary tract infection

3) Diclectin

5) Dimenhydrinate 8) stool softeners 11) Vitamin B 12

13)ginger root

14) There is decreased rate of absorption as well as change in drug distribution 17) Folic acid & multivitamins

6) body fat/lean muscle mass ratio 9) lactulose 12)Drugs must be able to diffuse across lipid barriers to enter the fetus 15) Neurotubular defect

16)Category X · · · · · · · · · · · · · · · ·

18) morning sickness

Pharmacokinetics factors that increase with age? ( ) Calcium supplements that are preferably given to seniors? ( ) What vitamin supplements are recommended to seniors? ( ) What therapy recommended for constipation in seniors? ( ) Drugs that should be discontinued in pregnancy? ( ) Supplements that should be recommended in pregnancy? ( ) Folic acid supplements in pregnancy prevents? ( ) Nausea and vomiting in pregnancy also referred as? ( ) Drug of choice therapy against nausea and vomiting in pregnancy is? ( ) OTC drug therapy against nausea and vomiting in pregnancy include? ( ) Self care that is recommended for nausea and vomiting in pregnancy? ( ) Cranberry juice is used against? ( ) How do pharmacokinetic characteristics in the very young differ from that of an adult? ( What pharmacokinetic characteristics change in elderly? ( ) What is the significance of the placental barrier? ( ) Herbal products that is recommended for nausea & vomiting in pregnancy? ( )

)

Copyright © 2000-2018 TIPS Inc. Unauthorized reproduction of this manual is strictly prohibited and it is illegal to 54-7 reproduce without permission. This manual is being used during review sessions conducted by PharmacyPrep.

www.pharmacyprep.com

Safety of Medications in Special Populations

Copyright © 2000-2018 TIPS Inc. Unauthorized reproduction of this manual is strictly prohibited and it is illegal to 54-8 reproduce without permission. This manual is being used during review sessions conducted by PharmacyPrep.

www.Pharmacyprep.com

Prevention and Intervention in Medication Overdose

55 Promoting Medication Adherence Medication works only when patient takes them as prescribed. Non-adherent definition Primary medication nonadherence is failing to fill or take a new prescription Steps to improve medication adherence. A patient is considered adherent if they take 80% of their prescribed medications. If patient takes less than 80% of their prescribed medications they are considered non-adherent. Eight steps to improve medication adherence. 1. Consider medication nonadherence first as the reason a patient's condition is not under control and especially when considering increasing therapy or adding another medication. Increasing or adding additional therapy when non-adherence is hidden can be dangerous. Case: A patient who is hospitalized and has started all medication according to their medication list. The hospital nurse is not aware that patient is NOT taking all of prescribed medications. 2. Develop a process for routinely asking about medication adherence. Gathering patient medication information and medication review. Potential medication non-adherence alerts the physician to discuss the potential issues. Case: 3. Create a blame-free environment to discuss medications with the patient. Asking patients non-judgemental way, “Why aren't you taking the medications I prescribed?” is confrontational and suggests that you think the patient's nonadherence is because they are defying your recommendations. Instead, try saying, “Many people have trouble taking their medications on a regular basis. Do you find this is the case for any of your medications?” This removes blame from the patient to allow them to open up about their particular situation.

Copyright © 2000-2018 TIPS Inc. Unauthorized reproduction of this manual is strictly prohibited and it is illegal to 55-1 reproduce without permission. This manual is being used during review sessions conducted by PharmacyPrep.

www.Pharmacyprep.com

Prevention and Intervention in Medication Overdose

4. Identify why the patient is not taking their medicine Most non-adherence is intentional, patient make conscious decision not to take their medication based on their knowledge, experience, and belief, here are some common reason for nonadherence. FEAR: Patient may be frightened for potential side effects, they had with same or similar medications or they might have witnessed side effects experienced by family member or friends. COST: Cost of medicine can be a barrier and may not fill in first place. TOO MANY MEDICATIONS: Greater the number and higher the frequency, will likely cause more non-adherence. LACK OF SYMPTOMS: Patient may not see any difference, in taking their medications. WORRY: Concern of depending on medication can cause non-adherence MISTRUST: Patient may not have trust on doctor motive to prescribe medication, believe that this pharmaceutical companies marketing efforts. DEPRESSION: Patient who are depressed less likely to take their medication. MISUNDERSTANDING: Patient may not understand the need for the medicine or expected time it takes to medication response. For chronic therapies, patient taking medication, do not see any significant changes and thinks medication do nothing. Failure to see immediate improvement may lead to premature discontinuation. 5. Respond positively and thank the patient for sharing their behavior. Good communication by physician and pharmacist improve patient medication adherence. inadequate physician communication with patients may account for 55% of medication nonadherence. 6. Tailor the adherence solution to the individual patient. Identifying and discussing their reasons for not taking medication may help to personalize their therapy. Case: Changing to less sedative. Changing to long acting medications to avoid withdrawal effect of medicaitons. 7. Involve the patient in developing their treatment plan 8. Set patients up for success

Copyright © 2000-2018 TIPS Inc. Unauthorized reproduction of this manual is strictly prohibited and it is illegal to 55-2 reproduce without permission. This manual is being used during review sessions conducted by PharmacyPrep.

www.Pharmacyprep.com

Pharmacy Practice and Management

56 Professional Communication Skills Questions Alerts! Common questions in pharmacy exam is to ask! · The best communications skills are verbal and writing · Barriers in communications are environmental barriers, personal barriers, and financial and administrative barrier. · Cultural competence or cultural diversity. · Communication with other healthcare professionals. Type of communication methods. · Verbal communication methods · Non-verbal communication methods Verbal communication · Verbal communication comprises speaking and listening · The sender. One who transmits a message to another person · The message. It is an element that is transmitted from one person to another · The receiver. One who receive message from sender · Feedback. It is the process of replying to sender. Barriers in communications. The interference that affects the receiving, sending transmitting of message. · Environmental barriers (noise, loud music, counter heights, poor lighting etc). · Personal barriers ( cultural diversity (language), no confidence, shy, incompetence) · Patient barriers (knowledge) · Administrative and financial barriers Environmental barriers: Distractions in environment often can result into environmental barriers like height of prescription counter separating the patient from the pharmacist. · Crowded and noisy prescription areas inhibit one to one communications. · Presence of support workers like technician who stands between pharmacist and patient. · Distraction or loud noise, telephone rings, music, and traffic. Copyright © 2000-2018 TIPS Inc. Unauthorized reproduction of this manual is strictly prohibited and it is illegal to 56-1 reproduce without permission. This manual is being used during review sessions conducted by PharmacyPrep.

www.Pharmacyprep.com

Pharmacy Practice and Management

The following potential factors that are associated with environmental barriers: · Is pharmacist visible? · Is counter top or stuff on counter tops blocking pharmacist visible? · Does it easy to get pharmacist attention? · Is it private counseling area available to conduct private interview? · Is that lost background noise or distractions? · Is it easy to get pharmacist attention? Recommendation to minimize environmental barriers · Place computers terminal near the patient counseling area to minimize distractions. · Create a quite private counseling area. · Make countertops wider to accommodate Question Alerts! computers, printers etc. 1) Your pharmacy is located in multicultural area. What Personal barriers is appropriate to do offer good customer service? · Low self confidence. a) have multicultural language signs in pharmacy · Cultural differences (cultural competent). b) hire multicultural staff, or staff speaks many · Discomfort to sensitive situations languages · Conflicting values of therapy c) Print information in different language. · Shyness d) Find a translator e) Sign language Patient barriers The following are the examples of patient 2) What is cultural competence? For language barrier? barrier. A. Find a translator or sign language · Patient perceive being as knowledgeable? B. Translate in internet · Patient perception about pharmacist C. Print in English language knowledge? D. Ask someone to translate · The perception of impersonal atmosphere · Patient perceptions about their medical conditions as minor? · Patient may be anxious about their conditions. Administrative and Financial barriers There are several factors of administrative and financial aspects effects pharmacy practice. Like bureaucracy can be administrative barrier. · Pharmacist are not paid directly for educating or communicating with patients, therefore many managers perceive the task of talking with pharmacist is expensive service and not a high priority. · Pharmacy policies that encourage minimum number of pharmacist. · Excessive tasks to pharmacist by typing label, count medications, talk on phone, and completing other tasks while communicating with patients. Time barriers · Setting inappropriate timing of appointment. · Large number of prescription needs to be filled in short time. Non-Verbal Communication · Communication does NOT require verbal language. · The elements of non-verbal communication kinesis and proxemics.

Copyright © 2000-2018 TIPS Inc. Unauthorized reproduction of this manual is strictly prohibited and it is illegal to 56-2 reproduce without permission. This manual is being used during review sessions conducted by PharmacyPrep.

www.Pharmacyprep.com

Pharmacy Practice and Management

Kinesics or body language The manner in which one uses her/his eye contact, arms, legs, hands head to convey a message to receiver. Example: Relaxed posture, slight lean toward the other person, eye contact, and smile. Proxemics The distance between two interactive people put more emphasis on content of communication, and it is defined as proxemics. Written. It is powerful nonverbal communication tool.

Tips 1. 4. 7. 10. · · · ·

Low self-confidence Promotion sales Verbal Discomfort to sensitive situations

2. 5. 8. 11.

Distractions Telephone Written Conflicting values to therapy

The best communication skills are? ( ) Examples of communication barriers includes? ( Examples of communication distractions includes? ( Examples of personal barriers includes? ( )

)

3. 6. 9. 12.

language Noise Cultural differences Shyness

)

Copyright © 2000-2018 TIPS Inc. Unauthorized reproduction of this manual is strictly prohibited and it is illegal to 56-3 reproduce without permission. This manual is being used during review sessions conducted by PharmacyPrep.

www.Pharmacyprep.com

Pharmacy Practice and Management

Copyright © 2000-2018 TIPS Inc. Unauthorized reproduction of this manual is strictly prohibited and it is illegal to 56-4 reproduce without permission. This manual is being used during review sessions conducted by PharmacyPrep.

www.Pharmacyprep.Com

Pharmacy practice and management

57 Bioethics/Professional Ethics This chapter provides basic understanding of ethical principles, code of ethics, professional responsibility and liability. You will learn how to make ethical decisions. Lecture presentation includes scenarios of ethical decisions and professional liability and legal issues. Questions Alerts! Common questions in pharmacy exam is to ask! · Beneficence = doing good or doing the very best to patient · Nonmaleficence = preventing harm · Autonomy = patient right to choose (paternalism breaks autonomy) · Veracity = honesty or telling the truth without deception. · Justice = equality or first come first service (fairly). Difference in ethics and regulations LAW and RULE (REGULATIONS) ETHICS (conducts/moral/character) uphold or followed or break? Described in constitution Professional judgement in the best of interest patient in the ethical principles Must be followed Followed or uphold/broken Cannot advance narcotics, benzodiazepine, Breaking ethics is misconduct and control drugs. Beneficence Beneficence: to do good or doing well or doing the very best to patient. The health professional should act in the best interest of the patient. Decisions made with perception are based on what patient needed. In other words:

· ·

Acting in the patient’s best interest. Current thinking is to involve patient letting the patient determine what is in their best interest.

Pharmacists demonstrate beneficence whenever they provide critically needed prescription drugs to their patients in emergency situations without regard to possible legal consequences.

Nonmaleficence Nonmaleficence is do no harm or prevent harm. Pharmacists who refuse to fill a prescription order because of their concern for patient safety or well being observed the principle of nonmaleficence. Copyright © 2000-2018 TIPS Inc. Unauthorized reproduction of this manual is strictly prohibited and it is illegal to 57-1 reproduce without permission. This manual is being used during review sessions conducted by PharmacyPrep.

www.Pharmacyprep.Com

Pharmacy practice and management

Professional interventions on drug related problem OR DRUG THERAPY PROBLEM that can prevent harm is action of nonmaleficence. Harm reduction services such dispensing syringes to drug addicts. Receiving narcotics to destroy from customers. Autonomy: Whether like to take or not like to take? Letting the patient have the final decision, even if it is not in their best interest i.e. refusing treatment, surgery, etc. In other words. Patient’s right to self-determination. To choose what will be done to them. Respect for life and autonomy of patient: Contraception, emergence contraception, abortion. Palliative care (care before 24 hr of death), pain management, and end-of-life care. Euthanasia = Assisted suicide or planned death or physician assisted suicide. Mental capacity to assess patient's capacity to make decision (advance directive). Terminology Consent (Mental Capacity) Substitute decision making Palliative care (end-of-life care) Living will (advanced directive) Last will Disclosure The action of telling new or secrete information known. Empathy Autonomy opposite is Paternalism. Paternalism: When one fails to respect another’s autonomy, and acts with disregard to the individual rights. Substitute their own beliefs, opinions and judgment to that of another. Claim they acted in the person’s best interest. Honesty and Veracity: Act with honesty without deception. The patient has the right to the truth medical condition, course of the disease and treatments. Code of ethics states that a pharmacist, “has the duty to tell the truth and to act with conviction of conscience” Rapport is built on trust, which is based on honesty. Rx Obecalp Fidelity (loyal) In other words, fidelity is the right of a patient to have health professionals provide services that promote patient interests rather than their own. The right of patients to have practitioners provide services that are in the patient’s best interest. Infidelity from a prescriber could be recommending vitamins that patients don’t need. Failing to confront a doctor with an inappropriate prescription out of fear that the doctor will direct his/her patients elsewhere.

Copyright © 2000-2018 TIPS Inc. Unauthorized reproduction of this manual is strictly prohibited and it is illegal to 57-2 reproduce without permission. This manual is being used during review sessions conducted by PharmacyPrep.

www.Pharmacyprep.Com

Pharmacy practice and management

Justice or Fairness First come first serve. It means providing services equally. Professional Ethics Informed Consent (permission) and decision making Three types of consents implied, express, and informed. Requires honesty and autonomy to exist. Patients have the right to full information of all relevant facts and must give explicit consent before treatment. Informed consent exists when. · All relevant information has been provided · The patient understands the information · Consent is freely given and there is no coercion · The patient is capable of understanding the information · Note. Often, practitioners rely only on the disclosure part of the list! Confidentiality Federal regulations of confidentiality is personal information protection and electronic documentation act (PIPEDA). Provincial regulations are personal health information protection act. (PHIPA). From the patient’s perspective, this is “self-disclosure” and they should be the ones making this decision. In other words, the principle of confidentiality serves to assure the patient that information about their health, medical condition, treatment will not be given to individuals without their permission. Confidentiality of patient Information. The pharmacist preserves the confidentiality of information about individual patient acquired in the course of his or her professional practice, and does not divulge this information except where authorized by the patient or required by law. Spouses: If someone is asking a copy of his or her spouse’s prescription information, get permission from the patient whose information is being released. Exceptions of confidentiality based on your reasoning? Cognitive impairment (advance directives require or care giver)? Patient decision making capacity should be assessed by health care professionals Unconscious patients (circle of care)? Mental illness (alert family) ? HIV patients (imply information) or contagious disease?

Copyright © 2000-2018 TIPS Inc. Unauthorized reproduction of this manual is strictly prohibited and it is illegal to 57-3 reproduce without permission. This manual is being used during review sessions conducted by PharmacyPrep.

www.Pharmacyprep.Com

Pharmacy practice and management

Tips 1. 4. 7. · · · · · · · · ·

Violating autonomy Beneficence preventing harm

2. 5. 8.

Doing good ( ) Non maleficence ( ) Autonomy ( ) Honesty ( ) Veracity ( ) Justice ( ) Fidelity ( ) Paternalism ( ) Code of conducts or morals (

Tell the truth Ethics Best interest of patient

3. 6. 9.

Right of determination Equality with everyone honesty without deception

)

Copyright © 2000-2018 TIPS Inc. Unauthorized reproduction of this manual is strictly prohibited and it is illegal to 57-4 reproduce without permission. This manual is being used during review sessions conducted by PharmacyPrep.

www.Pharmacyprep.com

Drug Information Resources

58 Drug Information Resources Questions Alerts! Common questions in pharmacy exam is to ask! Types of literature primary, secondary and tertiary. · Canadian drug references like compendium of pharmaceutical specialties (CPS). · Compendium of Therapeutic choices (CTC) and Compendium of Therapeutic for Minor Ailments (CTMA). · USP DI vol.1 drug information for health care professionals. · Motherisk program. Safety of drugs in pregnancy. · References like Cochrane data base (Cochrane Collaborative Library) an evidence base medicine database. · Medline is used for new drugs and therapies and off label use. · Martindale. Foreign drugs. Response to drug information request? What reference is appropriate for minor ailment treatment? CTMA selection of suitable references for foreign drug and information sources? Peer review: Assessment of a clinical trial by experts for scientific merit, participant safety, and ethical considerations. Literature: It is defined as an extensive, heterogeneous collection of resources, which provide information about drugs. Drug information sources can be categorized into primary literature, secondary literature and tertiary literature. Primary Sources: Consist of original information about clinical trials or research. Examples Scientific Journals containing clinical trial information. · Peer reviewed articles are published in scientific journals. The primary source literature provides latest and current information. · Examples. Canadian Medical Association Journal (CMAJ), Canadian Pharmacy Journal (CPJ), Journal of American Medical Association (JAMA). Pharmacy connection, Clinical research data, Journal of Informed Pharmacotherapy, Canadian Family Physician, Canadian Journal of Clinical Pharmacology, Etc. · Benefits: Gives most current information and keeps up latest development and research in pharmacy. It is good resource of continuing education. · Limitation: Does not guarantee that the article is accurate, however respected journal enhances the credibility of information contained in the article.

Copyright © 2000-2018 TIPS Inc. Unauthorized reproduction of this manual is strictly prohibited and it is illegal to 58-1 reproduce without permission. This manual is being used during review sessions conducted by PharmacyPrep.

www.Pharmacyprep.com

Drug Information Resources

Secondary Sources: Examples Indexes, bibliography and abstracts. · Information published in secondary sources obtained from various primary sources and compiled as abstracts and indexed into publications. · Example. Internet search, Medline. · Current Content (Abstracts), Index Medicus (Index); Monthly Biomedical research- results. · Clin-Alert (Index). Abstracting service as a newsletter edition semimonthly (bi weekly) adverse reaction. · Current content: Weekly-Clinical practice · International pharmaceutical abstracts, monthly and quarterly pharmacy practice · Online sources: MedLine or Pubmed (recent drug information, and off label use) · Benefits: It is an important resource for quick and selective screening of primary literature for specific information, or article. · Limitations. Each indexing service provider may provide specific list of journals, so this can limit and thoroughness of literature search. Tertiary Sources: Examples text books and compendia · Information published in tertiary sources such reference book and textbooks are obtained from primary and/or secondary sources. · Example: Compendium of pharmaceutical specialties (CPS). United States pharmacopeia (USP-DI), Martindale, Remington, Compendium of Therapeutics choice, all text books and reviews. · Benefits: Provide easy and comprehensive topics in one textbook. · Limitation: No recent information. PRIMARY LITERATURE ORIGINAL INFORMATION

SECONDARY LITERATURE INDEXES AS GROUP

CLINICAL TRIALS, NEW INVENTIONS DATA

SEARCH RESULTS IN INTERNET MEDLINE, PUBMED

TERTIARY LITERATUR COMPENDIUM, TEXT BOOKS BOOKS PUBLICATIONS

Compendium of Pharmaceuticals Specialties or CPS · Compendium of Therapeutic Choices (CTC) · Compendium of Therapeutic Minor Ailments (CTMA) · RxTx (CPS, CTC CTMA & CPMA) · RxFiles.ca · United States Pharmacopeia DI-Vol.1 (USP-DI Vol.1) · Martindale · Cochrane data base or reviews · Health Canada Drug Product database (DPD) · Drugs in pregnancy and lactation or Motherisk program REFERENCES Compendium of pharmaceutical speciality (CPS or RxTx) CTC CTMA USP-DI Vol.1 Cochrane data base Health Canada DPD Medline (Pubmed)

A prescriber requesting drug information. What drug information request, would you select? Drug monographs, side effects, clinical use, drug interactions, contraindications. Pharmacology. Dosing, storage conditions. Treatment options (1st line therapy or alternates) Self care and over the counter drugs FDA approved drugs in USA Evidence base medicine systematic review New drugs approved Off label drugs

Copyright © 2000-2018 TIPS Inc. Unauthorized reproduction of this manual is strictly prohibited and it is illegal to 58-2 reproduce without permission. This manual is being used during review sessions conducted by PharmacyPrep.

www.Pharmacyprep.com

Professional product monograph Health care professional use CPS

Drug Information Resources

Consumer product monograph Public use/patient information Package inserts

Pharmacy Practice Reference Resources Compendium of pharmaceuticals specialties · Discontinued products · Brand and generic name index (green pages) · Therapeutic guide (pink pages) · Product identification (pages containing photographs of the medicines) · Directory (Yellow pages) · Clini-info (Lilac pages) · Monographs (white pages) · Appendices (white pages at the end of the CPS) · Drugs that have peanut and soya proteins, lecithin’s preps, and ethanol. · Drugs that can be crushed and chewed. EVIDENCE BASED MEDICINE INFORMATION SOURCES: Cochrane reviews, database Drugs manufactured in United States · USP-DI volume I · The American Drug Index (updated annually) · Drug Facts and Comparison (updated monthly and bound annually) · Drug Topics Red Book (released monthly and bound annually) · Physician desk reference (PDR) (updated annually) · AHFS Drug Information. American Hospital Formulary Society (supplemented quarterly and updated annually). Drug Manufactured in foreign countries · Martindale. The complete drug references · Index Nominum · USP Dictionary · USAN For Investigational Drugs · Medline or Pubmed and Health Canada website · Martindale. The complete drug reference · Drug facts and comparison · Unlisted drugs · FDA website. The new drug application (NDA) pipeline · Health Canada

Copyright © 2000-2018 TIPS Inc. Unauthorized reproduction of this manual is strictly prohibited and it is illegal to 58-3 reproduce without permission. This manual is being used during review sessions conducted by PharmacyPrep.

www.Pharmacyprep.com

Drug Information Resources

For unknown drugs. Try to identify them by physical characteristics such as special marks, color, shape etc. and or recommend chemical analysis. · USP DI volume I · The PDR · Drug facts and comparison · Manufacture · Laboratory Side Effects · Compendium of pharmaceutical specialties · USP DI Vol. I: Drug Information for the Health Care Professional · Clin-Alert · Meyler's Side Effects of Drugs · AHFS Drug Information Drug-Drug Interactions · Drug Interaction Facts (Tatro) · Drug Interactions. Compendium of Pharmaceuticals and Specialties · Clin-Alert (Generali) · Hansten and Horn's Drug Interactions Analysis and Management · Drug-Drug Interactions. Handbook of Clinical Drug Data (Anderson) · Handbook of Adverse Drug Interactions · Drug Interactions: A Source Book of Adverse Interactions, Their Mechanisms, Clinical Importance and th Management (Stockley: 6 ed. 2003) · EDI: Evaluations of Drug Interactions · Concise Guide to Cytochrome P450 System: Drug Interaction Principles for Medical Practice Drug-Lab Test Interactions · American Hospital Formulary Society (AHFS) Drug Information · Effects of Drugs on Clinical Laboratory Tests (Young: 5th ed. - 2000) · Drug-Laboratory Test Interferences. In: Handbook of Clinical Drug Data (Anderson) Drug-Food Interactions · Drug Interaction Facts (Tatro) · USP-DI volume I · Reference Guide to Drug and Nutrient Interactions · Food Medications Interactions Handbook (Pronsky) · HIV Medication-Food Interactions Handbook (Pronsky) Drug-Herb Interactions · Database of Natural product · Herbs: Everyday Reference for Health Professionals (CphA) · Herb Contraindications and Drug Interactions (Brinker) · Herb-Drug Interactions Handbook (Herr) · Drug Interaction Facts: Herbal Supplements and Food

Copyright © 2000-2018 TIPS Inc. Unauthorized reproduction of this manual is strictly prohibited and it is illegal to 58-4 reproduce without permission. This manual is being used during review sessions conducted by PharmacyPrep.

www.Pharmacyprep.com ·

Drug Information Resources

Interactions between Drugs & Natural Medicines: What the Physician and Pharmacist Must Know About Vitamins, Minerals, Foods and Herbs (Meletis)

Compounding or Extemporaneous prep · Remington: The Science and Practice of Pharmacy · Merck index · A Practical Guide To Contemporary Pharmacy Practice · Pharmaceutical Dosage Forms and Drug Delivery Systems · Pharmaceutical Practice · The Art, Science, and Technology of Pharmaceutical Compounding · The Pharmaceutical Codex Formulas For Compounding · United States Pharmacopea-National Formulary (USP-NF) · Merck index · British Pharmacopoeia Vol. 2 · Contemporary Compounding Compendium Extemporaneous Oral Liquid Dosage Preparations (CSHP) · The Ghen and Rains Physicians Guide to Pharmaceutical Compounding · Extemporaneous Formulations (Children's hospital of Philadelphia) · Pediatric Drug Formulations · Pocket Book of Extemporaneous Formulations · Allen's Compounded Formulations: The US Pharmacist Collection · The Art, Science, and Technology of Pharmaceutical Compounding · Trissel's Stability of Compounded Formulations (Recommended for parenteral solution stabilities) · Minutes from manufacturer Characteristic Of Specific Chemicals/Drugs · The Merck Index · Martindale. The Complete Drug Reference · Remington. The Science and Practice of Pharmacy · Handbook of Pharmaceutical Excipient · The Pharmaceutical Codex · The United States Pharmacopeia/The National Formulary · Trissel's Stability of Compounded Formulations Compounding · Extemporaneous Ophthalmic Preparations · Guidelines for Preparation of Sterile Products in Pharmacies (CSHP) · Handbook on Injectable Drugs (Trissel) · Principles of Sterile Product Preparation (ASHP) · Sterile Dosage Forms (Turco) Question Alerts!

Compatibility IV mixture can be found in? Compatibility and Stability Of Parenteral Drugs · Handbook on Injectable Drugs (Trissel) · Parenteral Drug Therapy Manual (Ottawa General Hospital) · Trissel's Tables of Physical Compatibility · Pocket Guide to Injectable Drugs (Trissel)

Copyright © 2000-2018 TIPS Inc. Unauthorized reproduction of this manual is strictly prohibited and it is illegal to 58-5 reproduce without permission. This manual is being used during review sessions conducted by PharmacyPrep.

www.Pharmacyprep.com ·

Drug Information Resources

IV Index System (Micromedex) subscription required.

Pharmacokinetic Monographs · Compendium of pharmaceutical specialties · USP-DI Volume I. Information for the Health Care Professional · AHFS Drug Information · Handbook of Clinical Drug Data Patient Counseling · Compendium of Therapeutic Minor Ailments CTMA (CPhA) for over the counter products · USP DI: Volume II: Advice for the patient in lay language · Communication skills in pharmacy practice: a practical guide for students and practitioners (Tindall: 4th ed. 2002). · Communication skills for pharmacists: building relationships, improving patient care (Berger) Pregnancy and Lactation · Drugs in Pregnancy and Lactation (Briggs) · Mother Risk Program (Sickkids hospital, Ontario) · AHFS Drug Information · Clinical Therapy in Breastfeeding Patients (Hale) · Nausea and Vomiting of Pregnancy: State of the Art 2000 th · Teratogenicity and Drugs, in Breast Milk. In Applied Therapeutics: The Clinical Use of Drugs (7 ed. 2001) · USP DI - Volume I. Information for the Health Care Professional · Breast-Feeding Precaution Listing. USP DI-Vol. II (22nd ed.) nd · Pregnancy Precaution Listing. USP DI-Vol.II (22 ed. 2002) Pediatrics · Manual of Clinical Problems in Pediatrics (Roberts) · Manual of Pediatric Therapeutics (Graef) · Martindale: The Complete Drug Reference · Nelson Essentials of Pediatrics · Nelson's Textbook of Pediatrics · Neofax · Pediatric Dosage Handbook · Pediatric Pharmacology · Problems in Pediatric Drug Therapy · Red Book 2000: Report of the Committee on Infectious Diseases · Rudolph's Pediatrics · The Harriet Lane Handbook · The Pediatric Drug Handbook (Benitz) Poisoning and Toxicology · Refer to poison control centers: Addresses are found in Yellow pages CPS. Casarett and Doull's Toxicology: The Basic Science of Poisons (6th edition - 1996) · Clinical Management of Poisoning and Drug Overdose (3rd edition - 1998) · Comprehensive Review in Toxicology for Emergency Clinicians (3rd edition - 1998) · Drug Toxic kinetics · Ellenhorn's Medical Toxicology: Diagnosis and Treatment of Human Poisoning (2nd edition - 1997) Copyright © 2000-2018 TIPS Inc. Unauthorized reproduction of this manual is strictly prohibited and it is illegal to 58-6 reproduce without permission. This manual is being used during review sessions conducted by PharmacyPrep.

www.Pharmacyprep.com · ·

Drug Information Resources

Poisoning and Toxicology Handbook (APhA - 1997) Principles and Methods of Toxicology (4th edition - 2000)

Veterinary Medicine Compendium of Veterinary Products Current Therapy in Equine Medicine (Robinson) Current Veterinary Therapy 4: Food Animal Practice (Howard) Development and Formulation of Veterinary Dosage Forms (Hardee) The Exotic Animal Drug Compendium: An International Formulary Handbook of Comparative Veterinary Pharmacokinetics and Residues of Pesticides and Environmental Contaminants The Veterinary Formulary: Handbook of Medicines Used in Veterinary Medicine Veterinary Drug Handbook (Plumb) Doses And Drug Administration Physician desk reference (PDR) Drug facts and comparison Martindale: The extra pharmacopia Intravenous and Intramuscularly Compatibilities Handbook of injectables drug (Tresel) Martindale: The extra pharmacopia Teratogenicity Physician desk reference (PDR) Drugs in Pregnancy and Lactation (Briggs) Indexes Drug Interaction Index FDA/Manufacturer Alert Index First Report Index Legal Action Index Newly Marketed Drugs: 1997-2000 Drug Index Summaries of some important reference sources Martindale. The complete reference. Reference books contained with complete source of information about foreign drugs and approved and off labeled drugs. Martindale’s Extra Pharmacopoeia is probably one of the most comprehensive, international, single volume references on drugs and drug products. Martindale’s is divided into three parts: The first part consists of monographs on drugs and ancillary substances. (Although drugs that are manufactured in the England are stressed, generic and propriety products from many other countries are included). The monographs include chemical data, storage, incompatibilities, uses, doses, and toxic effects. The second part contains a supplementary discussion of new drugs, obsolete drugs, and miscellaneous substances. The third party lists formulas of OTC products sold in the England. There is also a directory of worldwide pharmaceutical manufacturers. Copyright © 2000-2018 TIPS Inc. Unauthorized reproduction of this manual is strictly prohibited and it is illegal to 58-7 reproduce without permission. This manual is being used during review sessions conducted by PharmacyPrep.

www.Pharmacyprep.com

Drug Information Resources

Merck Index: Contained information about chemical properties and compounding. Merck Manual: Basic information about anatomy, physiology and pathophysiological conditions. USP DI Available USP DI® Drug Reference Guides include: Volume I: Drug Information for the Health Care Professional Volume II: Advice for the Patient in lay language Volume III: Approved Drug Products and Legal Requirements USP DI Volume I USP DI® Volume I. Drug Information for the Health Care Professional contains · In-depth monographs cover dosing · Indications · Interactions · Pharmacology/pharmacokinetics · Side/adverse effects · Patient counselling guidelines · Labelled and off-label uses are discussed to facilitate third-party reimbursement. USP DI Volume II USP DI® Volume II: Advice for the Patient - Drug Information in Lay Language. Provides patient-oriented drug information to help patients understand and successfully follow their drug regimens. These include brand names, descriptions, proper use, and precautions, Side effects. Instructions for how to handle missed doses are included, as are guidelines for when to seek medical assistance or supervision. USP DI® Volume III USP DI® Volume III: Approved Drug Products and Legal Requirements. Staying apprised of federal guidelines and legislations related to prescribing and dispensing drugs are time consuming. This is a single, easy-to-use reference containing all the information you need including the complete FDA "Orange Book." USP DI Volume III helps you quickly identify a drug's chemical properties, determine if a drug has been discontinued, or select an appropriate generic substitute for a more expensive brand name drug. Excerpts from USP-NF offer data on quality, packaging, storage, and labeling requirements. Contains guidelines and laws governing the safe handling and distribution of drugs. USP Dictionary USP Dictionary Content Overview United States and international drug names The USP Dictionary is the authoritative source for generic drug names established by the United States Adopted Names (USAN) Council. The dictionary also provides other types of drug names used worldwide: Brand names. Chemical names. International Nonproprietary Names (INNs). British Approved Names (BANs). Japanese Accepted Names (JANs). Official USP-NF names. FDA-established names. The Medical Letter Its newsletters, The Medical Letter on Drugs and Therapeutics and Treatment Guidelines from The Medical Letter, publish critical appraisals of new drugs and comparative reviews of older drugs. Clin-Alert The adverse drug reaction events reported by Clin-Alert during the year 2000 have been compiled, organized, re-formatted, indexed, and published in a convenient one-volume reference book Copyright © 2000-2018 TIPS Inc. Unauthorized reproduction of this manual is strictly prohibited and it is illegal to 58-8 reproduce without permission. This manual is being used during review sessions conducted by PharmacyPrep.

www.Pharmacyprep.com

Drug Information Resources

Drug Interaction Facts Drug Interaction Facts loose-leaf edition provides drug-drug and drug-food interaction information in a quick reference format. Drug Interaction Facts covers more than 20,000 brand and generic drugs and more than 70 therapeutic classes. Drug significance ratings are also included. Drug Interaction Facts includes: Herbal Supplements and Food. Which covers over 100 monographs and discusses the interactions' significance, onset, severity, documentation, mechanism, and more. Updated quarterly. Remington. The Science and Practice Of Pharmacy This textbook has been the definitive reference for all aspects of the science and practice of pharmacy, and is used for pharmaceutics, therapeutics and pharmacy practice courses in primary curricula. Remington covers many education and practice issues, from the history of pharmacy and ethics, to industrial pharmacy and pharmacy practice.

Tips

· · · · · · · · · · · ·

1 Second use of drug or unapproved use of drug 4 Drug monographs.

2.

CPS

3.

5.

Immunization Guide of Health Canada

6.

7 Clinical practice guidelines.

8

Remington. The science and practice of pharmacy

9

Compendium of Therapeutic Choices Compendium of Therapeutic Minor Ailments Antibiotic recommendations or clinical practice trial recommendations

Children immunization schedule is found in? ( ) Dental prophylaxis clinical practice guidelines can be found in? ( ) Initial treatment can be recommended by using? ( ) Off label indication is? ( ) Compendium of pharmaceuticals specialties ( ) What is found in compendium of pharmaceuticals specialties? ( ) What is NOT found in CPS? OTC drugs and selfcare. (but vitamins, iron, folic acid present in CPS). What is found in compendium of therapeutic minor ailments? (Recommendation of OTC drugs therapy) What is not present in compendium of therapeutic minor ailment? ( ) Compounding reference ( ) Remington reference is used for? Buffer, compounding, excipients What is present in CTC? Treatment options (1st line, & alternate therapies).

Copyright © 2000-2018 TIPS Inc. Unauthorized reproduction of this manual is strictly prohibited and it is illegal to 58-9 reproduce without permission. This manual is being used during review sessions conducted by PharmacyPrep.

www.Pharmacyprep.com

Drug Information Resources

Copyright © 2000-2018 TIPS Inc. Unauthorized reproduction of this manual is strictly prohibited and it is illegal to 58-10 reproduce without permission. This manual is being used during review sessions conducted by PharmacyPrep.

www.Pharmacyprep.com

Medication Errors

59 Medication Errors Questions Alerts! Common questions in pharmacy exam is to ask! · Institute of Safe Medication Practices (ISMP). The ISMP is a non-profit independent agency established for the collection and analysis of medication error reports and development of recommendation of enhancement of patient safety. · Dangerous abbreviations and High alert drugs · Near missed errors THE CANADIAN ORGANIZATIONS INVOLVE IN MEDICATION SAFETY Organization Functions Health Canada's MedEffect Advisories, withdrawal, recall Health Canada's pharmacovigilance ADR in post marketing surveillance Canadian Institute of Health Information (CIHI)Medication incidence in hospital. Institute of safe medication practices (ISMP-Canada) medication incidence in community pharmacy Promotes innovative solutions and facilitate collaboration among Canadian patient safety institute (CPSI) government and stakeholders to enhance patient safety. Drug recalls: Class I (Type I) Strong likelihood that product will cause serious adverse effect or death.

Class II (Type II) May cause temporary but reversible adverse effects or in which little likelihood of serious adverse effects.

Class III (Type III) Product is not likelihood to cause adverse effect.

Example of recall. 0.9% Sodium Chloride Injection, 1000 mL (2017-05-10) Recall Starting date: May 10, 2017 Posting date: May 19, 2017 Type of communication: Drug Recall Subcategory: Drugs Hazard classification: Type I Source of recall: Health Canada Issue: Product Safety Audience: General Public, Healthcare Professionals, Hospitals Identification number: RA-63340 Copyright © 2000-2018 TIPS Inc. Unauthorized reproduction of this manual is strictly prohibited and it is illegal to 59-1 reproduce without permission. This manual is being used during review sessions conducted by PharmacyPrep.

www.Pharmacyprep.com

Medication Errors

Affected products Cefazolin for Injection DIN, NPN, DIN-HIM DIN 02237140 Dosage form Powder for solution Strength Cefazolin Sodium 10 g/vial Lot or serial number 303723 Companies Recalling Firm Adverse drug reaction (ADR)

Unintended effects, Side effects, new side effects

Adverse drug reaction (ADR) reporting; ADR form is reported to Canada Vigilance Program, of Therapeutic Directorate, Health Canada. Or can report to HealthCanada/MEDEFFECT Consumers/patients and health professionals can report adverse reactions (also known as side effects) to health products, including prescription and non-prescription medications, biologics, (includes biotechnology products, vaccines, fractionated blood products, human blood and blood components, as well as human cells, tissues and organs), natural health products and radiopharmaceuticals, to the Canada Vigilance Program. Health Product InfoWatch (old name was CARN) The Health Product InfoWatch is a monthly publication intended primarily for healthcare professionals. It provides clinically relevant safety information on pharmaceuticals, biologics, medical devices and natural health products. This chapter focuses on errors that occur during the medication use process that includes the prescribing, dispensing, and administration phases of medication use. Monitoring the patient for expected and unexpected drug related problems and patient compliance. It reviews common causes of medication errors and suggest measures to safe and effective use of medications/strategies to prevent dispensing errors. Types of medication errors: According ASHP guidelines on preventing medication errors in hospitals, medication errors can be categorized into 11 types. · Prescribing errors · Omission errors · Wrong time errors · Unauthorized drug errors · Improper dose errors · Wrong dosage form errors · Wrong drug preparation errors · Wrong administration errors · Deteriorated errors Copyright © 2000-2018 TIPS Inc. Unauthorized reproduction of this manual is strictly prohibited and it is illegal to 59-2 reproduce without permission. This manual is being used during review sessions conducted by PharmacyPrep.

www.Pharmacyprep.com · · ·

Medication Errors

Monitoring errors Compliance errors Other errors

Prescribing errors A prescribing error occurs at the time a prescriber orders a medication for specific patient. These errors may include the selection of incorrect drug, dose, dosage form, route of administration, length of therapy, or number of doses. Omission errors Failure to administer an ordered dose to a patient in hospital, nursing home, or other facility before the next scheduled dose is considered an omission error. Wrong Time Timing of administration is critical to effectiveness of some medications. Maintaining an adequate blood level of some drugs such as antibiotics, frequently depends on evenly spaced, around the clock dosing. Unauthorized drug error Administration of a medication to a patient without proper authorization by a prescriber is categorized as an authorized drug error Improper dose Improper dose error occur when a patient is given a dose that is greater or less than prescribed dose Wrong dosage form errors Doses administered or dispensed in a different form from the ordered by the prescribed are classified as wrong dosage form error Wrong drug preparation errors Drugs requiring reconstitution (adding liquid to dissolve a powdered drug), dilution or special preparation prior to dispensing or administration are subject to wrong drug preparation. Wrong Administration Technique Errors Doses that are administered using an inappropriate procedures or incorrect technique are categorized as wrong administration technique error. Preventing dispensing errors: In order to prevent dispensing errors, pharmacy should have policy and procedures in place. College of Pharmacists and professional regulatory agencies provide guidelines to prevent dispensing error. All pharmacy staff should obey and discuss those guidelines. Before you dispense any medication, check the followingà (7 point check) Name of patient Name of medication Name of doctor Pharmacist and technician initials (check expiration date) Check DIN Quantity of medication Number of refills 3 point check during preparation Read label when taking drug from shelf Read label before preparing label Read label when placing back on the shelf

Copyright © 2000-2018 TIPS Inc. Unauthorized reproduction of this manual is strictly prohibited and it is illegal to 59-3 reproduce without permission. This manual is being used during review sessions conducted by PharmacyPrep.

www.Pharmacyprep.com Incidents (errors) Near missed incidents

Medication Errors Medication errors, dispensing errors. Error did not reach to patient

Related agencies of medication incidences are ISMP and CIHI. Strategies in reducing dispensing errors · Procedures and policies must be developed to deal with these errors, keeping in mind that the patient safety is paramount. · Discuss commonly dispensing error drug with staff. · Bringing the common drugs that may have dispensing problems to others attention, that means everyone is extra careful as they are dispensing them. · Drugs that look similar separate and label them. Dispensing error occurred. Question Alerts! · When dispensing errors occur, rapid resolution of 1) In community pharmacy If dispensing error occur the problem is essential, including the recall of who should be informed first? the product. · Inform the pharmacist manager immediately for any misfills or other serious problems. Do not try to handle the situation yourself. · Inform patient · Inform manager · Inform doctor if patient have taken wrong medication. · Documenting all errors and discussing them with all staff helps keep everyone aware of mistakes that are easily made. Resolving dispensing errors. Pharmacy should have policies and protocol about resolving dispensing error. Follow the guidelines provided by the provincial college of pharmacists. However, this is a general discussion to help you get started understanding resolving dispensing errors. In case an error has occurred, the patient needs to be informed of the error that has occurred that the medication she is taking is not the correct medication as the pharmacy has made an error in dispensing the medication she was supposed be taking Diclectin as the doctor has prescribed but the pharmacy dispensed dicetel 7 by mistake. Tell her to stop taking the medication, Ask her how many dicetel 7 she has taken, apologize for the mistake that has happened, take reasonability for the mistake, Call the doctor and report the mistake, replace the medication with the right medication, speak to the staff about the error that has occurred. Examples of medication error that could occur if auxiliary labels are not used or used inappropriately: 1) Codeine containing medication auxiliary label: dizziness may occur be careful when operating machinery, A car accident may occur if the patient is not aware that this medication may cause dizziness. 2) Fosamax auxiliary labels: drink with plenty of water, remain upright for about ½ hour after taking the dose, take on an empty stomach, If the patient is to take the medication and lie down right after taking it then the patient may experience oesophageal adverse experiences 3) Ventolin auxiliary labels: shake well; don’t take too much of the medication If the patient is to take too much of the medication then the patient may experience adverse drug reactions such as palpitations, tachycardia, tremors, nervousness, hypokalemia. 4) Flovent auxiliary labels: shake well, rinse mouth after using this inhaler. If you don’t wash your month a pharyngeal candidiasis fungal infection in the mouth may occur in the mouth. Copyright © 2000-2018 TIPS Inc. Unauthorized reproduction of this manual is strictly prohibited and it is illegal to 59-4 reproduce without permission. This manual is being used during review sessions conducted by PharmacyPrep.

www.Pharmacyprep.com

Medication Errors

5) Lipitor auxiliary label: avoid grapefruit juice. Grapefruit juice may have the potential to increase plasma levels of HMG CoA reductase inhibitors metabolized by this isoenzyme causing increased potential for adverse effects such as muscle weakness and pain. Medications that should be taken with plenty of water: Sulphonamides: Recommended for sulphonamides to decrease likelihood of crystalluria. Expectorants: Expectorants to enhance viscosity reduction of bronchial secretions Bulk laxatives: Bulk laxatives to increase stool bulk and decrease the likelihood of compaction Irritating drugs: Such as potassium supplements, Chloral hydrate, theophylline and some antibiotics. Take with food or milk: Recommended for the drugs that cause stomach upset when this effect may be decreased by taking medication with food. Medication examples include: · NSAIDs, and ASA · Erythromycin · Nitrofurantoin · Valproic acid Take Medication on an empty stomach, 1 hour before or 2 to 3 hours after 3 hours after meals unless or otherwise directed physician. Recommended for drugs that have decreased absorption or increased destruction in stomach when taken with food. Examples of drugs that should be taken empty stomach like ampicillin and tetracycline. Actions that should be taken o resolve dispensing errors; · Discuss with all staff to keep everyone aware of mistake · Discuss and identify drugs that have similar names such as: · Biaxin à one tablet twice a day (500 mg) · Biaxin à 2 tablets once a day (500 mg) · Lasix (furosemide) · Losec (omeprazole) · Procedure and policies must be developed to deal with these problems. · Check DIN (Drug Identification Number) · Cautionàsame DIN for different quantity packaging · Expiry date · Check patient name and date of birth · Check allergy · Paediatric dose child weight Handling Returned Products · It is against the law to replace returned medication as stock bottle once it has been dispensed and given to the patient. Exceptions are hospital pharmacies. · Prescription drugs may not be returned by a customer and placed back in stock. · Sometimes, the manager’s attention needs to be drawn to a complaint. In other cases, policies will need to be developed e.g. for product recalls and dispensing errors. Expired drugs. Do not sell expired drugs.

Copyright © 2000-2018 TIPS Inc. Unauthorized reproduction of this manual is strictly prohibited and it is illegal to 59-5 reproduce without permission. This manual is being used during review sessions conducted by PharmacyPrep.

www.Pharmacyprep.com

Medication Errors

Medical incidents Institute of Safe Medication Practices (ISMP) is a non-profit independent agency established for the collection and analysis of medication error reports and development of recommendation of enhancement of patient safety. Reference. American Society of Hospital Pharmacists. ASHP guidelines on preventing medication errors in hospitals, Am. J Hosp Pharm. 1993; 50: 305 to 14. ISMPs dangerous drug abbreviations. · Use units instead "U" or "IU" · Use "daily" instead of "o.d" · Use mcg instead of "mg" · Use "0.5" instead of ".5" · Use "5" instead of "5.0" · Use "qd" instead of "q.d" · "D/C" is used for discharge but confused with discontinue · "CC" is intended for "cubic centimeter" mistaken for units · "@" is intended for "at", mistaken for 2 or 5 · ">" is intended for greater than mistaken for 7 Dangerous abbreviation IU OD "mg" ".5" "5.0" @ D/C > CC

Mistaken as IV or 10 Right eye mg 5 50 2 Discontinue 7 Units

Recommended Units Daily mcg 0.5 5 at discharge Greater Cubic centimeters

High alert drug: The drug errors that can cause serious harm to patient. Opioids, warfarin, digoxin, insulin, epinephrine, KCl solution, TCA, pregnancy x category drug, pediatric liquid formulation that require measurements. Anticancer drugs and Lithium. Home work: To avoid error. Label as “HIGH ALERT” Vincristine and vinblastine prednisone and prednisolone Sound alike names, look alike drugs: lisinopril and fosinopril Tallman letters are used for sound alike name drugs to avoid medication errors. DAUNOrubicin and DOXOrubicin BuPROPIon and BuSPIROne ALPRAZOLam and LORAZEPam ClomiPHENE and ClomiPRAMINE INFLIximab and RITUximab HydrALAzine and HydrOXYzine diPHENhydramine and diMENhydrinate Copyright © 2000-2018 TIPS Inc. Unauthorized reproduction of this manual is strictly prohibited and it is illegal to 59-6 reproduce without permission. This manual is being used during review sessions conducted by PharmacyPrep.

www.pharmacyprep.com

Health promotion

60 Health Promotion and Disease Prevention Questions Alerts! Common questions in pharmacy exam is to ask! A role of public health agency. Levels of health care prevention: It can be categorized as primary, secondary and tertiary preventions. Primary prevention

Secondary prevention

Tertiary prevention

Immunizations, smoking cessation. Lifestyle

Cancer screening. ASA 81 mg therapy or anticoagulants for patient with atrial fibrillations. Early detection and management of disease

Drug prophylaxis or treatment to prevent

Activity are designed to completely prevent the disease.

The drug of choice for secondary prevention of stroke associated with atrial fibrillation? warfarin The drug of choice for the prevention stroke in patient with transient ischemic attack (ASA)? ASA 81 mg

To manage clinical diseases in order to prevent them from progressing to avoid complications of disease.

Public Health units of public health agencies (PHA) offer health promotion and disease prevention programs to help Canadian of all ages learn more about health, including healthy lifestyles, communicable disease control and immunization. The role of public health agencies is to promote health; · Prevent and control chronic diseases and injuries. · Prevent and control infectious diseases. · Prepare for and respond to public health emergencies. · Serve as a central point for sharing Canada’s expertise with the rest of the world.

Copyright © 2000-2018 TIPS Inc. Unauthorized reproduction of this manual is strictly prohibited and it is illegal to 60-1 reproduce without permission. This manual is being used during review sessions conducted by PharmacyPrep.

www.pharmacyprep.com ·

Health promotion

Apply international research and development to Canada’s public health programs and Strengthen intergovernmental collaboration on public health and facilitate national approaches to public health policy and planning.

Health promotion (Wellness): a process enabling people to increase control over and to improve their health. In health promotion, pharmacists provide information and skills to individuals so that they can prevent specific diseases and participate in services for early detection and treatment of disease. The process involves a behavioural change approach such as in advising individuals on the importance of preventing and managing obesity. Health promotion activities from community pharmacies include the organizing of workshops, clinics where patients are advised on a specific topic, and written and other visual aids are available in the pharmacy for further information and for shop window dressing. Topics for health promotion or primary prevention in community pharmacy includes; · Smoking cessation · Diet, exercise, body weight (Eating well with Canada Food Guide) · Cardiovascular risk factors and prevention (Compendium of Therapeutic Choices) · Sun exposure or sunscreen · Travel Medicine (Specific countries travel.gc.ca) · Patient compliance for treatment · Immunization or vaccination programs (public health agency of Canada) · Screening Programs or Tests (Pap smear test, mammography, PSA, DRE), colorectal cancer screening. · Alcohol, drug abuse prevention Screening test Pap smear PSA DRE mammogram Fecal occult blood test (FOBT)

For detection of Cervical cancer associated with papilloma virus BPH and prostate cancer Colon cancer, prostate cancer Breast cancer Colon cancer, colorectal cancer, peptic ulcers, ulcerative colitis, Crohn’s disease

Sunscreen UVA AGING, WRINKLES, SKIN CANCER & EYE DAMAGE PREVALENT YEAR AROUND AT ALL TIMES ALL DAY AND PENETRATED UNTREATED GLASS ZnO, Titanium dioxide, Avobenzone blocks UVA.

UVB BURN OR SUNBURN, Q, PRIMARY CAUSE OF SKIN CANCER, AGING, EYE DAMAGE STRONG SPRING TO FALL BETWEEN 10.00 AM TO 4PM STILL ALL-DAY YEAR-ROUND RISK.

UVC SAFE STOPPED BY OZONE LAYER SHARTEST WAVES

Copyright © 2000-2018 TIPS Inc. Unauthorized reproduction of this manual is strictly prohibited and it is illegal to 60-2 reproduce without permission. This manual is being used during review sessions conducted by PharmacyPrep.

www.pharmacyprep.com

Health promotion

Sun protection factor (SPF): The average light-skinned person can stay in the sun with no sun protection for 10 min before start to burn. If person applied sunscreen with SPF 15. Person can stay in sun for multiple of 15 x 10 min = 150 min can stay in sun with screen without burn. SPF 15 x 10 min = 150 min SPF 30 x 10 min = 300 min SPF 50 x 10 min = 500 min Higher SPF also filters more UVB. i.e. SPF 15 blocks 93% of UVB SPF 30 blocks 97% of UVB SPF 50 blocks 98% of UVB SPF 100 blocks 99% of UVB Folic acid in pregnancy Primary prevention of neurotubule defect, it recommended to use 0.4 mg of folic acid daily. Starting from 3 months before conception and continue to the first trimester. Avoid vitamin A retinol in pregnancy. However, vitamin A with beta-carotene are safe. Immunization Flu vaccine Hepatitis A & B Pneumococcal Dukoral Zoster vaccine Factors influencing health promotion (wellness) activities in community pharmacy Positive factors Negative Factors Accessibility of pharmacy Lack or resource material, Lack of space Communication skills of pharmacist. Lack of confidentiality Strong pharmacist and patient relation Improper time management of pharmacy personal Environment within pharmacy conducive to health promotion

Copyright © 2000-2018 TIPS Inc. Unauthorized reproduction of this manual is strictly prohibited and it is illegal to 60-3 reproduce without permission. This manual is being used during review sessions conducted by PharmacyPrep.

www.pharmacyprep.com

Health promotion

Copyright © 2000-2018 TIPS Inc. Unauthorized reproduction of this manual is strictly prohibited and it is illegal to 60-4 reproduce without permission. This manual is being used during review sessions conducted by PharmacyPrep.

61 Collaboration and Teamwork Questions Alerts! Common questions in pharmacy exam is to ask! Teamwork and collaboration components includes communication, scope of practice, conflict management and management of patient care. Academic detailing goal is to enhance prescription practices. A collaborative care involves a number of health professionals working to treat a patient. The team generally comprises a physician, pharmacist, nurse practitioners, physiotherapist etc. To address this issue there were several collaboration models have been studied. Examples of collaborative traditional patient care models include biomedical model, bio-psycho-social model. Biomedical model: In this model, disease is defined as biophysical malfunction and goal of treatment is to correct the malfunction in order to cure the disease. The biomedical model includes pathophysiology of disease, objective tests and therapeutic interventions at centre of patient care. However biomedical model offers one dimensional approach to patient care that excludes the patient experience of illness and how this affects the other facets of life such as work disability, finances, and social networks because they are believed to lie outside of medicines responsibility and authority. Bio-psycho-social model (BPS): The bio-psycho-social model (body and mind) includes bio, psychological and social dimension of patient into the care plan. The biological component of this model examines the cause of the illness and how it affects the functioning of body. The psychological component of the model explores any potential psychological causes for the illness (example lack of self-control, emotional stressors, negative-thinking). The social component considers how different social factors (e.g., socioeconomic status, religion, culture) impact illness. In order to address all aspects of this threedimensional model, an integrated team approach involving allied healthcare professionals, such as physicians, nurses, psychologists, pharmacists, social workers and rehabilitation specialists, are critical for ensuring that more comprehensive patient care is provided. Recovery Model: In this model the patient is involved in a lifelong recovery process that involves a number of incremental steps across various facets of his or her life. The primary illness is seen as only one dimension in the patient’s recovery process. Other key aspects of this model include negotiating treatment approaches between patients and practitioners such that the patient feels empowered. Copyright © 2000-2018 TIPS Inc. Unauthorized reproduction of this manual is strictly prohibited and it is illegal to reproduce without permission. This manual is being used during review sessions conducted by PharmacyPrep.

61-1

Patient-centred cared model (PCC): In this model the patient individuality is central. The patient has right to have his or her needs, desires, beliefs, values, and goals respected and placed at centre of the care plan. Respect for patient individuality is part of the team commitment to understand the patient perspective of his or her own health status and continued care. Model for Collaborative working relationship (CWR) There are five stages in CWR Stage 0: Professional awareness Stage 1: Professional Recognition Stage 2: Exploration and Trial Stage 3: Professional Relationship Expansion Stage 4: Commitment to the collaborative working relationship Stage 0: exchange is minimal. Example pharmacist calling for refill request or alerting physician for possible side effects, drug interactions. Stage 1: The efforts are mostly from pharmacist. Example as pharmacist develop new services, pharmacist may be visiting physician to ask for referral of patients. Stage 2: Pharmacist still continues to be initiator. Recommend high quality and priority recommendations to physician. Value added services like Med check, screening. Stage 3: Continue developing relationship to offer patient care. Stage 4: Relatively high input, lengthy duration and great consistency. Pharmaceutical Care Delivery System · The major pharmaceutical care activities takes place in the following systems · Community pharmacy · Hospital pharmacy · Long term care facilities · Specialty hospital units Community Pharmacies: ·

Community pharmacies are considered one of the important components of the pharmaceutical care delivery system. However, health related services are primarily limited to dispensing medications and patient counselling.

Pharmacist collaboration: Pharmacists needs to work in collaborative relationship with physician and other healthcare providers. Academic detailing (AD) defined as process of outreach in which a knowledgeable heath professional (pharmacist) visit physicians to discuss issues of drug use and (often) overuse. AD is also known as counter detailing. The goal of academic detailing is to enhance appropriate prescribing practices. Soumerai and Avorn described eight components that contribute the success of academic detailing. · Establish credibility of agency developing the intervention · Conduct interviews with physicians to establish baseline knowledge · Focus the intervention on educational outreach, university-based educational detailing, and public interest detailing specific physicians. · Define clear objectives for intervention Copyright © 2000-2018 TIPS Inc. Unauthorized reproduction of this manual is strictly prohibited and it is illegal to reproduce without permission. This manual is being used during review sessions conducted by PharmacyPrep.

61-2

· · · ·

Stimulate physician interaction during the detailing visit Use concise graphic educational material during the presentation Highlight and reinforce the essential messages during presentation Provide positive reinforcement with a follow up visit to the physician

Pharmaceutical opinion: Pharmacist identify a potential drug related problem for patient and then provide the prescriber with a clinical recommendation to resolve the therapeutic problem. Pharmaceutical opinion or documentation is prepared in SOAP formats. S= subject; patient info O= Objective: lab results A= assessment: risk factors, symptoms P = therapeutic plan

Copyright © 2000-2018 TIPS Inc. Unauthorized reproduction of this manual is strictly prohibited and it is illegal to reproduce without permission. This manual is being used during review sessions conducted by PharmacyPrep.

61-3

Copyright © 2000-2018 TIPS Inc. Unauthorized reproduction of this manual is strictly prohibited and it is illegal to reproduce without permission. This manual is being used during review sessions conducted by PharmacyPrep.

61-4

www.Pharmacyprep.com

Sterile Preparations

62 Sterile Preparations Questions Alerts! Common questions in pharmacy exam is to ask! · The most common source contaminants in laminar airflow hood is personals. · Types of needles, syringes, vials and ampoules. The “gauge” measures the size of needle. The higher the gauge tinier the needle. · Cytotoxic preparations precautions. · Cytotoxic products sterile preparations use vertical laminar airflow hood and clean rooms.

Sterilization methods

Dry heat

Steam

Filtration

Gaseous

Radiation

Dry heat sterilization · Equipment. Oven · Method. Dry heat sterilization is carried out at 240 °C to 250 °C for 2 to 4 hrs. · Application. Glassware, fixed oils, glycerine, petrolatum, liquid petroleum (mineral oil), paraffin, and various heat stable powders, glassware and surgical instruments. · Dry heat method of choice when dry apparatus or dry containers are required, as in the handling of packaging of dry chemicals or non-aqueous solutions. · Advantages & disadvantages. Sterilization by means of heat requires higher temperatures and longer exposures than sterilization by steam. Heat transfer is slow, small volume of oil and thin layers of powder should be used. Steam (wet) sterilization · Equipment. Autoclave · Temp. 121oC 15 min, pressure 15 to 20 lbs · Method. In the presence of moisture, microorganisms are destroyed at a lower temperature than in dry heat. This is the method of choice when product can withstand such treatment. · Application. Solutions sealed in containers. Ampoule, vials, bulk solutions, glassware, surgical dressing, and instruments. Copyright © 2000-2018 TIPS Inc. Unauthorized reproduction of this manual is strictly prohibited and it is illegal to 62-1 reproduce without permission. This manual is being used during review sessions conducted by PharmacyPrep.

www.Pharmacyprep.com · ·

Sterile Preparations

Advantages. Rapid, inexpensive, effective, large volumes can be sterilized. Disadvantages. Cannot use for oily preparation (oil base ointment), cannot use for moisture sensitive preparations.

Filtration sterilization Physical removal of microorganisms by adsorption on the filter medium. Used for heat sensitive materials. · Equipment. Porcelain filters, siliceous earth filter, sintered glass filters, asbestos filters membrane filters. · Bacterial filtration: Microbial filter used in water filtrations 0.22 microns. Application. Thermo labile solutions of low viscosity. Advantages & Disadvantages. Depend on filter media, thermo labile solutions can be sterilized such as hormones, proteins. Gaseous sterilization · Equipment. Special oven, for admission of gas and humidity & hermetic. · Method. ethylene oxide · Ethylene oxide gas require 4 to 16 hours · Humidity of less than 20% RH · Ethylene oxide-carbon dioxide, pressure 30 psi, temperature 20 to 55 °C. · Application: Thermo labile powder, plastic/polymers, ophthalmic preparations, subcutaneous, vaginal inserts, plastic syringes, and tubing sets. Advantages & Disadvantages: Explosive hazard, toxic, not appropriate for solutions Radiation sterilization · Equipment: Ultraviolet lamp (laminar airflow hood), ionization (beta rays, gamma rays-from nucleus, X-rays) · Application: Thermo labile drugs (powdered) · Disadvantages: Highly specialized equipment required, effect of irradiation on products and their containers. Sterile Preparations To make sterile preparations aseptic techniques are procedure conducted under controlled condition to minimize the chance of contamination.

Question Alerts! 1) Doxorubicin preps are done in? Vertical or BSC 2) The most common contaminant? Personals, hand wash. 3) The gold standard reference for sterile prep is? USP Chapter 797 4) The work area in LAF from the edge should be in? 6 in (15 cm)

Sterility is the freedom from bacteria and other microorganisms. Formulations must be sterile, which is not a relative term an item is either sterile or not sterile. The word “sterile” refers to as free of living microorganism. If the sterile formulation is a solution, it must be free of all visible particulate material. Pyrogen free à Pyrogen is a chemical substance that is produced by microbial cell wall. Heating to high temperatures and double distillation can eliminate pyrogen. Aseptic preparation area: A limited access room of area in which laminar airflow hood is situated, usually separated from other pharmacy high traffic areas. Special measures should be taken to reduce airborne particles.

Copyright © 2000-2018 TIPS Inc. Unauthorized reproduction of this manual is strictly prohibited and it is illegal to 62-2 reproduce without permission. This manual is being used during review sessions conducted by PharmacyPrep.

www.Pharmacyprep.com

Sterile Preparations

Laminar Flows Hoods (Biological Safety Cabinets) that used for sterile preparations categorized as two types based on direction of airflow. Horizontal laminar airflow hood and vertical laminar airflow hood. Vertical laminar airflow (biological safety cabinet) hood is recommended for cytotoxic, anticancer antibiotics like doxorubicin and microbial preparations.

Horizontal Flow Hood

Vertical Flow Hood

HEPA filter: A HEPA filter is described as a high efficiency particulate air filter. It is employed with laminar flow for preparation of aseptic parenteral products. It has an efficiency of removing 99.97% particles of 0.33 microns or diameter larger. Class 100 clean room: A class 100 clean room is defined as an environment that contains no more than 100 particles per cu. Ft. of 0.5 microns or larger diameter size. Hypodermic Needle · HUB. Extension of needle that fits onto the syringe. · Bevel. portion of the needle that is ground · Heal. the back portion of the bevel · Cannula. shaft portion of the needle (made of steel) · Lumen. The needle holes.

Copyright © 2000-2018 TIPS Inc. Unauthorized reproduction of this manual is strictly prohibited and it is illegal to 62-3 reproduce without permission. This manual is being used during review sessions conducted by PharmacyPrep.

www.Pharmacyprep.com

Sterile Preparations

The gauge is thickness or the "inner diameter" of the bore of needle and ranges from 27 to 13. The smaller the number the greater the diameter. Needles discarded in sharp container. Insulin needle gauge 31-32 is the thinnest needle available. The 30G is thinner than 28G.

· · ·

Types of Glass Type І glass Type II glass Type III glass NP glass

is made of borosilicate the material, more resistant to water attacks. Is specially treated soda-lime glass Is the typical soda-lime glass Non-parenteral i.e. not suitable for injection

Type II glass is prepared by dealkalinizing the surface of type III glass by sulfur dioxide which will improve its resistance to breakdown and migration of alkali parenteral product. This internal coating will be weakened by repeated sterilization or exposure to alkaline detergents.

Tips 1.

2.

Dry heat

3.

4.

filtration and radiation ethylene oxide

5.

dealkalinizing by sulfur dioxide

6.

7.

Distillation

8.

9.

10. 13

free from bacteria Rabbit test and LAL test needle

11 14

microorganism, pyrogen & particulate Autoclave, 121oC ,At least 15 min. pyrogen free, sterile, particulate free, and isotonic

Vertical laminar air flow hood For microbial filtration 0.22mm in sterile H 2 O absolute form

12 15

heat sensitive products 0.22mm

16. · · · · · · · · · · · · · · · · · ·

How do you prepare type II glass, from type III glass? ( ) Filtration sterilizations ( ) Steam (wet) sterilization autoclave temperatures ( ) Gas sterilization ( ) Radiation methods ( ) Pyrogen test ( ) Hormones sterilized by ( ) Tubing’s sterilization method ( ) Proteins sterilization methods ( ) Petrolatum, waxes are sterilization methods ( ) Parenteral solutions should be: ( ) Sterility is? ( ) Cytotoxic drug preparations should be done in? ( ) Doxorubicin is an anticancer antibiotic prepared in? ( ) The size of HEPA filter that is used in laminar airflow hood? ( ) Parenteral preparations should be free from? ( ) Pyrogen are eliminated by? ( ) What part of the syringe that should not be touched? ( )

Copyright © 2000-2018 TIPS Inc. Unauthorized reproduction of this manual is strictly prohibited and it is illegal to 62-4 reproduce without permission. This manual is being used during review sessions conducted by PharmacyPrep.

www.PharmacyPrep.com

Drug Storage and Handling

63 Drug Storage Conditions Proper storage is very important key component of Quality Assurance to maintain medicines efficacy, stability. The drug is affected by the factors such as temperature, light sensitivity, chemical reactions. · Drug storage areas must maintain proper temperature and humidity conditions to ensure stability of all medications as recommended by manufacturer. · Temperature in refrigerators and freezers should be monitored and documented daily. · Before preparing any medication, each drug, ingredient and container should be visually inspected for damage defects and expiry date. · All the drugs and supplies use in the sterile room should be unpack before brining into the sterile room. · All the medications and supplies should be stored on shelves, cabinets, and cart, not on the floor. That way, the floors can be clean properly. · Chemo/toxins should be stored on an eye level or on lower shelves in order to avoid breakage. · All the drugs, supplies and equipment should be storage as the manufacturer indicated on the label. · All high-alert medications are safeguarded and that known safety requirements for storage, availability and labeling are followed. Stability: Refers to the extent that a pharmaceutical preparation retains specified properties and characteristics within specified limits that it possessed at the time of compounding or preparation. Cold Temperature: Temperature that does not exceed 8º C Refrigerated Temperature: Temperature that is between 2º C to 8º C Cool Temperature: Temperature that is between 8º C to 15º C Room Temperature: Temperature that is between 15º C to 30º C Temperature

Four ways to convert Celsius to Fahrenheit °F = [(°C) (9)/5] + 32 °F = °C x (9/5) + 32 °F = (°C x1.8) + 32 °F = (9°C+160) / 5

Four ways to convert Fahrenheit to Celsius °C = [(°F - 32) / 9] (5) °C = [(°F - 32) (5)] / 9 °C = (°F - 32) (0.56) °C = (5°F - 160) / 9

· · · · · ·

·

Deep freeze -10 to -72 oC Frozen 0 oC Refrigerator or cold 2 to 8 oC Cool 8 to 15 oC Room temp 15 to 25 oC Warm 30 oC F = 9C/5 + 32

Cold chain Management: Temp 2 to 8 oC (optimal temp 5 oC) Copyright © 2000-2018 TIPS Inc. Unauthorized reproduction of this manual is strictly prohibited and it is illegal to reproduce without permission. This manual is being used during review sessions conducted by PharmacyPrep.

63-1

www.PharmacyPrep.com

Drug Storage and Handling

Drug Insulin Triflurdine (Viroptic) Idoxuridine Xalaton (Latanoprost)

Recommended Storage Refrigerate at 2° to 8° C Refrigerate at 2° to 8° C Room temperature Prior to dispensing or unopened refrigerate at 2° to 8° C

Xalacom (Latanoprost + timolol)

Prior to dispensing or unopened refrigerate at 2° to 8° C

Etonogestrel/ethinyl estradiol vaginal ring (Nuvaring) Infliximab

Refrigerate at 2° to 8° C

Vaccine

Recommended Storage

Dukoral

Refrigerate at 2° to 8° C

Influenza virus Influenza live vaccine (FluMist)

Refrigerate at 2° to 8° C Refrigerate at 2° to 8° C

MMR

Refrigerate at 2° to 8° C

Hepatitis a Hepatitis B Hepatitis A and B Zostavax II All Biologicals Prolia

Refrigerate at 2° to 8° C Refrigerate at 2° to 8° C Refrigerate at 2° to 8° C Refrigerate at 2° to 8° C Refrigerate at 2° to 8° C Refrigerate at 2° to 8° C

Refrigerate at 2° to 8° C

Stability at room temperature 28 days After first use, store at a room temperature not to exceed 25°C for up to 6 weeks After first use, store at a room temperature not to exceed 25°C for up to 10 weeks After dispensing, store at 25°C for up to four month

Stability at room temperature Can be stored at RT for up to 2 wks one occasion only. After mixing buffer solution, should be consumed within 2 hrs.

Diluent may be stored in fridge or room temperature. Do not freeze.

Powdered antibiotics require reconstitution with distilled water, added at the time of dispensing. While many of these medications are stored in the refrigerator, some are not. Reconstituted products

Recommended Storage

Azithromycin suspension Amoxicillin suspension Amoxicillin/clavulin 400 mg Amoxicillin/clavulin 250 mg Clarithromycin suspension Erythromycin suspension Clindamycin suspension Phenytoin suspension

Room temperature for 10 days Refrigerate at 2° to 8° C for 14 days Refrigerate at 2° to 8° C for 7 days Refrigerate at 2° to 8° C for 10 days Room temperature for 14 days Refrigerate at 2° to 8° C Room temperature for 14 days Room temperature

Metronidazole suspension

Refrigerate at 2° to 8° C

Stability at room temperature

Protect from freezing and light Room temperature stable

Copyright © 2000-2018 TIPS Inc. Unauthorized reproduction of this manual is strictly prohibited and it is illegal to reproduce without permission. This manual is being used during review sessions conducted by PharmacyPrep.

63-2

www.PharmacyPrep.com

Drug Storage and Handling for 1 month

Chloramphenicol suspension Ciprofloxacin suspension Furosemide solutions Cefuroxime axetil suspension Cotrimoxazole suspension Norfloxacillin Cloxacillin

Room temperature for 2 weeks Room temperature for 2 weeks Room temperature for 2 weeks Room temperature until expiry Room temperature for 2 weeks Room temperature for 2 weeks

For cold chain follow National Guidelines for Vaccine Storage and Transportation.

Short dated products are those which will expire before the patient will finish using them.

Tips 1. 4. 7

Streptomycin Insulin Combination + Amoxicillin + clavulanate 10 Cotrimoxazole 13 2 to 8 oC also known as cold) · · · · · · · · · · · · · · · · ·

2. 5. 8

Xalatan Ampicillin Azithromycin

3. 6. 9.

Amoxicillin Clarithromycin Clindamycin

11 14

15 to 25 oC 0 to 4oC

12. 15.

8 to 15 oC Erythromycin

What temperature is fridge? ( ) What is room temperature in Canada? ( ) What is cool temperature? ( ) What is freezer temperature? ( ) Suspensions, store in refrigerator. Discard after 14 days. ( ) Reconstituted solution does not shake. Use immediately. Discard after 8 hours. ( ) Store in refrigerator, do not freeze and can be stored at room temperature for a month( ) Store unopened bottles in refrigerator. Opened bottles may be stored at room temperature up to 25 oC for up to 6 weeks ( ) What drug 200 and 400 mg is used within 7 days. 125 to 250 mg use within 10 days ( ) Refrigerate, if unused for 14 days discard it. ( ) Shake well before use and do not store in refrigerator. Discard unused portion after 14 days. ( ) Suspensions stored at room temperature ( ) Extemporaneous azithromycin suspension à Extemporaneous clarithromycin suspension à Extemporaneous clindamycin suspension à Cotrimoxazole suspension à Insulin stored at à

Copyright © 2000-2018 TIPS Inc. Unauthorized reproduction of this manual is strictly prohibited and it is illegal to reproduce without permission. This manual is being used during review sessions conducted by PharmacyPrep.

63-3

www.PharmacyPrep.com

Drug Storage and Handling

Copyright © 2000-2018 TIPS Inc. Unauthorized reproduction of this manual is strictly prohibited and it is illegal to reproduce without permission. This manual is being used during review sessions conducted by PharmacyPrep.

63-4

PharmacyPrep.Com

Pharmaceutical Care

64 Patient Care Patient care describes specific activities (job description) and services which an individual pharmacist offer services to patients that include dispensing, collaborating, implementing, developing and monitoring therapeutic plan that will produce specific therapeutic outcome for the patient. Patient care process includes Medication Assessment (gathering medication information) --> Care Plan --> Follow up evaluation, documentation The pharmaceutical or patient care is the services rendered by the pharmacist to improve patient quality of life within reasonable economic expenditure. Pharmaceutical care requires the pharmacist to incorporate some essential skills in practice that include. · Prescription processing and dispensing. · Making sure that the patient understands how to use the product, to ensuring that use of a medication will have the outcome desired. · Skills in assessment of patient health status. · Identification of potential and actual drug related problems or drug therapy problem (DRP or DTPs). · Identification of drug related problem and therapeutic choices · Developing and implementing therapeutic care plan. · Monitoring and evaluating patient progress with therapy · Documentation of finding and follow ups Drug-related problems (DRPs) or Drug therapy problem (DTP): The pharmacist’s focus is on the patient’s use of a product, to the patient’s situation as a whole including his or her need for the medication, the appropriateness of the product as prescribed, factors that could affect the patient’s use of the medication as prescribed, and whether the desired outcome will be achieved (from both the patient’s and physician’s points of view). An important part of this focus for the pharmacist involves the identification of current or potential drug related problems and their subsequent resolutions. DRPs No valid indication (unnecessary use) Drug NOT receiving or Need additional drug NOT taking/receiving appropriate drug OR INEFECTIVE DRUG Dosage is too low Dosage is too high

RELATED TO INDICATIONS

Examples

EFFECTIVENESS SAFETY

Copyright © 2000-2018 TIPS Inc. Unauthorized reproduction of this manual is strictly prohibited and it is illegal to 64-1 reproduce without permission. This manual is being used during review sessions conducted by PharmacyPrep.

PharmacyPrep.Com

Pharmaceutical Care

Adverse drug reactions Drug interactions Non-compliance

NON-COMPLIANCE

Drug-related problems (DRPs). That are described in the concept of pharmaceutical care include: 1. The patient is taking/receiving a drug for which there is no valid indication or unnecessary drug therapy. 2. The patient requires drug therapy for an indication and is not receiving it (includes symptoms of disease and of side effects). 3) The patient is not taking/receiving the appropriate drug (includes allergies, contraindications, or a drug that is not cost-effective, not working or not the drug of choice). 4) The patient is taking/receiving too little of a drug (includes insufficient dose, frequency and drug disease interactions, and drug food interactions). 5) The patient is taking/receiving too much of a drug (includes excessive dose, inappropriate frequency and drug-disease interactions). 6) The patient is not taking/receiving the prescribed drug appropriately (economic constraints, dispensing or administration error, non-compliance). 7) The patient is experiencing an adverse drug reaction (non-dose-related). 8) The patient is experiencing a drug-drug, drug-food or drug-laboratory test interaction. Developing therapeutic plan: A pharmacist must develop therapeutic plans, recommending therapeutic options, doses, scheduling/administration, required drug devices and compliance aids. Documentation of pharmaceutical care. Formulate a SOAP progress notes to describe and document the interventions intended or provided by the pharmacist. S = Subjective O = Objective A = Assessment Question Alerts! P = Plan 1) Patient age is categorized in? SOAP stands for Subjective data (S) separated

from objective data (O), A = assessment P = Plan SUBJECT OBJECTIVE (Findings) ASSESSMENT PLAN

S = Subjective (summary of case). MP is a 55-year-old man, receiving treatment of hypertension. MPs doctor prescribed hydrochlorothiazide 50 mg daily. Current BP 150/90, SrCr (80), FBG 6.5 mmol. MP using HCTZ 50 mg and have still high BP and require additional antihypertensive drug. MP Doctor adds Ramipril 10 mg.

Tips 1. 4. · ·

ARBs FARM

2 5

Refer to doctor SOAP

3 6

Pharmaceutical care improving patient quality of life

What is defined as the services offered by the pharmacist to improve patient quality of life within reasonable economic expenditure ( )? Finding, Assessment, Resolution, Monitoring ( )

Copyright © 2000-2018 TIPS Inc. Unauthorized reproduction of this manual is strictly prohibited and it is illegal to 64-2 reproduce without permission. This manual is being used during review sessions conducted by PharmacyPrep.

PharmacyPrep.Com · · · · · · · ·

Pharmaceutical Care

Subjective data, Objective data, Assessment, Plan ( ) Pharmaceutical care should be focus to? ( ) A 50-year-old patient currently using enalapril for hypertension. Experiencing dry cough. What alternative therapy should you recommend? ( ) A customer searching for OTC antidiarrheal medications. If you realized diarrhea is associated with clindamycin, what is appropriate action? ( ) Pharmaceutical care should be focus to à Write examples of drug related problem (DRP) à A 50-year-old patient currently using enalapril for hypertension. Experiencing dry cough. What is alternative preferably therapy is recommended?à A customer searching for OTC antidiarrheal medications. If you realized diarrhea is associated with clindamycin, what is appropriate action? à

Examples of DRPs: No indication Indication Wrong drug Low dose High dose Not administered properly ADR Drug Interactions Write the most common DRPs of the following drugs and patient presentations: · Alendronate --> · Statins --> · Metformin --> · Insulin --> · Salbutamol --> · Steroid inhalers --> · Anticholinergic drugs (ipratropium) --> · Contraceptive pills --> · Methotrexate --> · Minocycline --> · Metronidazole --> · NSAIDS --> · Warfarin --> · Accutane -->

Copyright © 2000-2018 TIPS Inc. Unauthorized reproduction of this manual is strictly prohibited and it is illegal to 64-3 reproduce without permission. This manual is being used during review sessions conducted by PharmacyPrep.

PharmacyPrep.Com

Pharmaceutical Care

Copyright © 2000-2018 TIPS Inc. Unauthorized reproduction of this manual is strictly prohibited and it is illegal to 64-4 reproduce without permission. This manual is being used during review sessions conducted by PharmacyPrep.

www.Pharmacyprep.com

Adverse Drug Reactions

65 Adverse Drug Reactions Questions Alerts!

Common questions in pharmacy exam is to ask! Clinical significant adverse drug reactions like most common and serious side effects ADRs affecting the cardiovascular system Cardiovascular Causative drug disorder Hypertension Venlafaxine dose dependant hypertension, sympathomimetics (pseudoephedrine) and mineral corticoids. Licorice. Postural hypotension Alpha-blockers, ACEi, diuretics, (syncope) antipsychotic (a1 blockade), vasodilator (hydralazine, nitrates) and opioids. Myocardial ischemia Levothyroxine, adenosine, amphetamines, beta agonist, betablockers (withdrawal), caffeine, ergotamine, nifedipine (short-acting), theophylline, verapamil. Oral contraceptive pills. Peripheral Beta-blockers vasoconstriction (DVT, Raynaud's phenomenon)

Tips/Comments >225 mg act on 5HT and NE

· · ·

·

· Hemorrhagic stroke

Anticoagulants, thrombolytic and antiplatelet drugs

·

Caution on standing Take first dose of alpha-blockers and ACEi on at bedtime In patients with hypothyroidism and cardiovascular disease, the initial dose of levothyroxine should be low and increased every 4 wks. Choose a more cardio selective agent such as atenolol, which has less affinity for beta 2adrenoceptors Choose a beta-blockers with vasodilators actions, e.g. carvedilol and labetalol These drugs should be avoided

Copyright © 2000-2018 TIPS Inc. Unauthorized reproduction of this manual is strictly prohibited and it is illegal to 65-1 reproduce without permission. This manual is being used during review sessions conducted by PharmacyPrep.

www.Pharmacyprep.com

Adverse Drug Reactions

Arrhythmias; prolonged QT interval (Ventricular tachycardia) Normal QTc Interval 72 hr then refer 3) Red irritated eye and NO itching and NO discharge than probable viral refer to doctor. 4) Gritty, sandy feeling is probable dry eye > 5days refer 5) Pain and vision changes refer to doctor 6) Blepharitis refer to doctor.

Conjunctivitis (red or pink eye). Inflammatory condition of the membrane that lines the inside of the eyelids and covers the exposed surface of the sclera. Conjunctivitis can be allergic, bacterial, and viral. Conjunctivitis diffuse redness in both eyes · Redness is more marked in the outer aspects of the eye and less around the cornea · Muco purulent discharges is more common with bacterial conjunctivitis · Clear discharge in viral or allergic conjunctivitis · Intense itching occurs with allergic conjunctivitis Red Eye

Bacterial or viral

Allergic

Symptoms Abrupt onset, purulent or mucopurulent discharge, lids endomatus +/- stuck AM. Self resolved in 7 to 10 days, chronic if >2 wks. Minimal itching. Non-Rx Clean gauze compresses avoid cleanser and avoid eye patches. Schedule Polymyxin B/gramicidin (eye drops), polymixin B/bacitracin (ointment). III (OTC)

Burning sensation, itchy eyes watery discharge, mild redness +/- lid swelling.

Rx

Artificial tears 4 to 6 times a day, ophthalmic antihistamine, oral antihistamine, mast cell stabilizers, and for chronic corticosteroids

Trimethoprim/polymyxin B (drops), erythromycin or bacitracin (ointment), sulfacetamide 10% solution

Allergen avoidance, cold compresses over the eyes, water irrigation BID, and avoid contact lenses.

Bacterial conjunctivitis: Common causes are S. aureus, S. pneumonia (most common in children), H. influenza (most common in children). Symptoms: Abrupt onset, purulent or mucopurulent discharge, lids endomatus +/- stuck AM. Self resolved in 7 to 10 days, chronic if > 2wks. Minimal itching. Copyright © 2000-2018 TIPS Inc. Unauthorized reproduction of this manual is strictly prohibited and it is illegal to 70-2 reproduce without permission. This manual is being used during review sessions conducted by PharmacyPrep.

www.Pharmacyprep.com

Ophthalmic, OTIC and Mouth Conditions

Non-prescription therapy: Clean gauze compresses, avoid cleanser and avoid eye patches. Polymyxin B/gramicidin (eye drops), polymixin B/bacitracin (ointment). Rx therapy: Trimethoprim/polymyxin B (drops), erythromycin or bacitracin (ointment), Sulfacetamide 10% solution Viral conjunctivitis Common cause: Adenovirus, herpes simplex virus Symptoms: Itchiness is minimal, redness is generalized. Discharge is profuse, serous Non-pharmacological: Give warm or cold compress to increase comfort. NonRx therapy: Ocular decongestants and/or lubricants may be useful. Rx therapy: Trifluridine (topical), Acyclovir, Famciclovir, and Valacyclovir (oral). Allergic conjunctivitis Common cause: ragweed, Grass pollen, Itchiness is severe Symptoms: Burning sensation, itchy eyes watery discharge, mild redness +/- lid swelling Non Pharmacologic therapy: Allergen avoidance, Cold compress over the eyes, water irrigation BID, and avoid contact lenses. Rx therapy: artificial tears 4 to 6 times a day, ophthalmic antihistamine, oral antihistamine, mast cell stabilizers, and for chronic corticosteroids Levocabastine; emedastine (H1 antagonist) for itchy and watery eye, Olopatadine (antihistamine and mast cell stabilzer), Nedocromil, Iodoxamide (mast cell stabilizing agent) alleviates. Ketorolac (Nonsteroidal anti-inflammatory eyedrop) for Drugs that cause dry eye itching & redness. · Anticholinergics: Antimuscarinic Other types of Conjunctivitis include: Chlamydial Conjunctivitis: drugs Trachomatis Fungal conjunctivitis: In rare cases: Rickettsial · First generation antihistamines conjunctivitis: Rare: Parasitic conjunctivitis: Rare Dry eye Symptoms: Dry eye, sandy, gritty sensation, photosensitivity and difficulty moving the eyelids · Etiology: Aqueous deficiency. Decrease lachrymal gland secretion. · Mucin deficiency: Damage or inflammation of goblet cells can be cause by condition erythema multiforme. · ·

· ·

· · · ·

B-blockers: propranolol, timolol Diuretics: hydrochlorothiazide, indapamide Isotretinoin Niacin (in hyperlipidemia) Phenothiazine antipsychotics (e.g. Chlorpromazine) TCA’s (amitriptyline)

Lipid deficiency: decrease lipid layer is common in patients with blepharitis. Epitheliopathies: defects in the corneal epithelium that can impair tear film stability.

Nonpharmacologic treatment · Cleanse eyes thoroughly · Blepharitis and hordeolum benefit from warm, moist compresses applied for up to 15 minutes, 3 to 4 times a day Copyright © 2000-2018 TIPS Inc. Unauthorized reproduction of this manual is strictly prohibited and it is illegal to 70-3 reproduce without permission. This manual is being used during review sessions conducted by PharmacyPrep.

www.Pharmacyprep.com ·

Ophthalmic, OTIC and Mouth Conditions

Cool, moist compresses have a soothing effect for conjunctivitis and dry eye.

Treatment Nonprescription Therapy: Artificial tear solutions: An ideal tear replacement product would posse: · Electrolytes in concentration similar to that normal tear. · An osmolality of 2000 to 280 mOsm · Viscosity of less than 20 centipoise · No cytotoxic · Preservative free. Pharmacotherapy: Substituted cellulose ethers (Carboxymethylcellulose 1%): Polyvinyl Polymers (Polyvinyl alcohol 1.4% and sodium hyaluronate) · Ointments: Petrolatum and carbomer · Artificial tear inserts – hydroxypropylcellulose · Pilocarpine, Acetylcysteine, Methylprednisolone · Instillation of artificial tears every 1-6 hours for a trial period of 48 hours · Emollients can cause blurring of vision and are better suited at night Any eye irritation that fails to respond to nonprescription therapy within 48 hours should be referred to an eye care professional for proper diagnosis Pharmaceutical agents Antihistamines – may cause photophobia or allergic reactions · Antazoline, pheniramine, pyrilamine Anti-invectives – polymyxin B combined with bacitracin or gramicidin Artificial tears – chemically inert and coat the eyes · Help them retain moisture · Protect from irritation · Slow turnover of tears · Examples dextran, methylcellulose, and hydroxypropyl methylcellulose, · Polyethylene glycol, polyvinyl alcohol and sodium carboxymethylcellulose Astringents – not to be used for hordeolum or allergic conjunctivitis · Zinc sulfates a mild astringent that clears eye secretions. Decongestants – can cause rebound hyperemia if overused · Contraindicated in patients with glaucoma · Examples naphazoline, oxymetazoline, phenylephrine, tetrahydrozoline, xylometazoline. · Emollients, soften eye tissue and protect it from drying such as Lanolin, mineral oil, petrolatum How to administer eye drops · Wash hands thoroughly. · Tilt the head back or lie down. · With eyes open, gently pull the lower lid below the eyelashes away from the eye to form a pouch. · Approach the eye from the side and hold the container near the lid (at least 2 cm away). Do not touch the lid or lashes. · Look toward the ceiling. Looking up moves the center of the eye away from the instillation site, minimizing the blink reflex. · Instill one drop into the pouch. Hold this position to let the drop fall as deep as possible into the pouch. Copyright © 2000-2018 TIPS Inc. Unauthorized reproduction of this manual is strictly prohibited and it is illegal to 70-4 reproduce without permission. This manual is being used during review sessions conducted by PharmacyPrep.

www.Pharmacyprep.com

Ophthalmic, OTIC and Mouth Conditions

Look down for several seconds and then slowly release the lower lid. Looking down brings the cornea into maximum contact with the drop. · Gently close (don’t squeeze) the eyes for 1 to 2 minutes while applying gentle pressure to the bridge of the nose for 30 to 60 seconds. Gentle pressure prevents the drops from being drained from the eye. · A tissue may be used to blot around the eye, but do not rub. Closing the eye helps prevent loss of solution caused by blinking. If the eye is closed too tightly, the medication may be expelled. · Don’t rub the eye. Try not to blink. · To apply several drops, wait 3-5 minutes after the instillation of each drop · Never contaminate the dropper tip or the top o the container by allowing it to touch the eye, eyelid, eyelashes, and fingers or counter surface. Question Alerts! 1) Difference of soft and hard lens? Eye care products 2) What is purpose of adding surfactants in contact lens Contact Lenses solutions? Removing contaminant, debris, facilitate Types of contact lenses disinfection. · Hard (rigid) gas permeable lenses or Rigid gas permeable (RGP) and hydrophobic. Silicone, fluorosilicone acrylate, polymethyl methacrylate (PMMA). ·

Soft lenses are hydrophilic. Hydroxyethyl-methacrylate (HEMA). Hard lens (RGP) Poly methyl methacrylate (PMMA) HYDROPHOBIC LIFE 5 YEAR UNTIL LOST DAILY WEAR TIME 12 HR HIGH FRAGILE Disposable are opened for each day. No regular solution requires.

Contact lens solutions (cleaning solutions) Two types: Surfactants remove loose debris and protein cleaners. In other words, remove proteins from soft lens. Surfactants: Disinfect and remove contaminants. Protein cleaners or enzyme cleaners contain papain, pancreatin, or Question Alerts! subtilism (enzyme). Remove protein 1) Administering eye drops? deposits by catalyzing the natural While instilling all ophthalmic drop, contact lens should be breakdown of debris into simple removed! compounds. 2) Tilt head back or lie down 3) After instilling eye drop why would you apply gentle Wetting and rewetting solutions: pressure to the bridge of nose? To keep the medication from Produce cushioning and lubricant going down the tear duct. (prevent systemic absorption) effect between lens and eyelid, Copyright © 2000-2018 TIPS Inc. Unauthorized reproduction of this manual is strictly prohibited and it is illegal to 70-5 reproduce without permission. This manual is being used during review sessions conducted by PharmacyPrep.

www.Pharmacyprep.com

Ophthalmic, OTIC and Mouth Conditions

between eye and cornea (removes dryness). Drugs interaction with contact lens · Oral contraceptive alters tear composition results decrease lubrication. · Antihistamine, Hypnotics, Sedative decrease blink rate (blink increases hydration). · Anticholinergics, antihistamines, TCA’s decrease tear volume · Isotretinoin may cause itching and decrease wear time in soft lens users. · ASA may cause ocular irritation, redness in soften wearers. · Disinfecting solutions kills bacteria · Preservative; maintain sterility of solution · Saline solution preservative minimize the risk of contamination · Wetting and rewetting solutions provide wetting, lubrication and cushioning functions. · Contact lens should be stored in disinfecting solutions. · Drying out is the major (75%) problem for soft lens users. Drugs cause discoloration of soft lens. Dopamine Nitrofurantoin Tetracycline Phenazopyridine Rifampin Pyrantel pamoate

Sulfasalazine Phenolphthalein

Mouth Ulcers (aphthous ulcer): Non-prescription Medications · Local anesthetics (topical): Benzocaine, lidocaine. · Oral analgesics: Acetaminophen, ASA, and NSAIDs · Protectant: hydroxycellulose; base agent (Zilactin, Oractane). Prescription Medications · Corticosteroids, Fluocinonide, Clobetasole and Triamcinolone. Dental Abscess: accumulation of puss in dental cavities. Drug of Choice: Pen V or Amoxicillin or Erythromycin (base for adults and estolate for children) Cold Sores Cold sore is oral herpes infection also called fever blisters is caused by herpes simplex virus 1 (HSV 1 ). Transmitted through direct contact. Usually appears on the lips also on hard palate or gums. Cold sores sign and symptoms begins with prodromal symptoms of mild burning or itching on the lips. · Small vesicles filled with clear fluid which eventually ruptures and crust over · Last for 3 to 10 days. · Cold sores improve without treatment. Cold sores also known as recurrent herpes labialis. Tingling sensation progressing to tiny painful grouped blister on lips, then crust. · Usually caused by activation of latent herpes simplex virus type I. Copyright © 2000-2018 TIPS Inc. Unauthorized reproduction of this manual is strictly prohibited and it is illegal to 70-6 reproduce without permission. This manual is being used during review sessions conducted by PharmacyPrep.

www.Pharmacyprep.com

Ophthalmic, OTIC and Mouth Conditions

Primary infection occurs between 6 and 36 months of age. 15% of adults have primary infection. Recurrent infection occurs in 20 to 45% of previously infected people. · Mostly get first infection with herpes when they are infant. Virus remains in the body and spread through physical contact. Abreva (docasanol) is used for treatment of recurrent cold sores. Acyclovir ointment.

·

· ·

Nonprescription medications Pharmacotherapy Topical anesthetics · Ester type: Benzocaine, tetracaine; contact sensitizers · Benzocaine: most common topical anesthetic used to relieve pain associated with canker and cold sores. External analgesics · Camphor, menthol, and benzyl alcohol · Counterirritants commonly found in cold sore balms Astringent: Burrow’s solution or cold compresses with tap water applied 3 to 4 times daily is helpful for cold sores · Sunscreen with SPF 15; recommended to prevent cold sores in those with recurrence after exposure to sun. Protectants · Petrolatum, ZnO, cocoa butter, allantoin, and calamine · Prevent drying of lesions from cracking or fissuring Heparin sodium and zinc sulfate (lipactin) – reduces pain duration · Shorten time required for lesions to heal Prescription medication. Antivirals like acyclovir CANKER SORES Recurrent aphthous stomatitis usually appear on the cheeks, tongue, and soft palate floor of the mouth. Canker sores. Visible manifestation of recurrent aphthous stomatitis · Streptococcus sanguis partly the cause · Autoimmune mechanism is also implicated. At least 20% is affected · Women twice as susceptible as men. Susceptibility appears to be inherited Canker sores sign and symptom. Painful, recurrent ulcers in the oral mucosa · 3-10 mm shallow lesions · Round with white center and red halo · Persist for 7-14 days. Treatment Topical anesthetics · Ester type. Benzocaine (contain up to 20% benzocaine), and tetracaine; contact sensitizers. · Applied to only small areas of the mouth to prevent a “cotton-mouth” feeling and loss of oral sensation. Protectants Copyright © 2000-2018 TIPS Inc. Unauthorized reproduction of this manual is strictly prohibited and it is illegal to 70-7 reproduce without permission. This manual is being used during review sessions conducted by PharmacyPrep.

www.Pharmacyprep.com

Ophthalmic, OTIC and Mouth Conditions

· Petrolatum, ZnO, cocoa butter, allantoin · Emollient mixtures or denture adhesives can alleviate pain Rx. Chlorhexidine gluconate and tetracycline mouthwashes help resolve cankers Oral thrush Also known as Candidiasis caused by fungus Candida albicans. Drugs commonly cause oral thrush are inhaled corticosteroids. To prevent oral thrush associated with inhalers. Rinse mouth with water after inhalation of corticosteroids spray and using aero chambers. Treatment: Nystatin suspension. Xerostomia (dry mouth): Xerostomia is a dry mouth conditions in which there are no salivary secretions and also caused by improper functioning of the salivary gland (Sjogren’s syndrome). Nonprescription medication: Ice chips, artificial saliva, and sugarless candies. Treatment. Artificial saliva Teething pain Nonpharmacological · Hard, smooth and clean products may be given to the child to bite and chew on such as frozen face cloth. Safe tethers cooled in refrigerator before use can be helpful. · The Canadian dental association recommends rubbing the back of a small, cold spoon on the gum. Non-prescription medication Oral analgesic. Acetaminophen and ibuprofen Topical anesthetic. Benzocaine 7.5% and 10% gel Treatment of eruption cysts · In general cysts rupture, spontaneously. · Rare cases surgically removed, if significant discomfort or interferes with feeding occurs. Dental Caries: Destruction of calcified tissue resulting from an infection. Dental caries most commonly caused by Streptococcus mutans. This bacterium produces acids that demineralized the enamel. Treatment 1. Tooth paste contains · Detergents or surfactants (sodium laurel sulfate, sodium N-lauryl sarcosinate) · Humectants (glycerin, propylene glycol) · Whitener (peroxides; sodium triphosphate) · Fluorides – reduce caries formation 2. Mouth wash contains · Cetylpyridinium chloride may cause staining of teeth · Chlorhexidine; may cause stains, taste change, discoloration of tongue 3. Triclosan · Antiplaque · Antimicrobial agent that helps prevents gingivitis, plaque cavities and tartar.

Copyright © 2000-2018 TIPS Inc. Unauthorized reproduction of this manual is strictly prohibited and it is illegal to 70-8 reproduce without permission. This manual is being used during review sessions conducted by PharmacyPrep.

www.Pharmacyprep.com

Ophthalmic, OTIC and Mouth Conditions

Trench mouth. This can cause acute necrotizing ulcerative gingivitis (ANUG) is caused by overgrowth of spirochete and fusiform microorganism. Gingivitis/periodontitis The infection of gingival tissue is gingivitis. Non-prescription · Mouth hygiene · Anesthetics; Benzocaine, and eugenol · Analgesic; Acetaminophen.

Gingivitis mouth rinse · Chlorhexidine mouth rinse

Endocarditis Caused by S. viridan and S. aureus. Prophylaxis · Amoxicillin 1 g before (1 hr) surgery, followed by 500 mg TID for 3 days. · Azithromycin 1 g/day followed by 500 mg OD x 2 to 3 days (for patients allergic to betalactam). · Clindamycin 600 mg followed by 300 mg QID x 3 days (for patients allergic to betalactam). Reference. Canadian Pharmaceutical Specialties OTIC disorders Excessive/impacted earwax · · · · ·

Overactive ceremonious glands Question Alerts! Narrowed ear canal 1) Ceremonius gland produce? earwax Large amount of hair in the 2) Earwax is removed by carbamide peroxide and mineral canal occurs often in oil. elderly. Ineffective or insufficient chewing or talking, especially in elderly. Improper removal methods.

Earwax softening agents · Carbamide peroxides the only approved as safe and effective agent for earwax removal. · To prevent vertigo, medication in the vial should be warmed in the hands and put 5-10 drops in the ear BID for 4 days. · Do not use if ear drainage, discharges, pain, and irritation or rash occurs. · Do not use if there is injury of perforation of eardrum. · If the patient feels pain or severe fullness upon instilling the drops, this might be an indication of ruptured tympanic membrane. Altitude and ear pressure: · This is caused by not functioning the Eustachian tube properly · Pain can be reduced or prevented through: Swallowing (chewing gum or eating candies) to activate the muscle that pull open the Eustachian tubes and helps to unblock the ear. Giving a bottle of milk or juice to the baby may reduced or prevent ear pain among babies. Copyright © 2000-2018 TIPS Inc. Unauthorized reproduction of this manual is strictly prohibited and it is illegal to 70-9 reproduce without permission. This manual is being used during review sessions conducted by PharmacyPrep.

www.Pharmacyprep.com · · ·

Ophthalmic, OTIC and Mouth Conditions

Yawning is effective in opening Eustachian tubes. Pinching the nostrils using the cheek and throat muscles, and forcing air back of the nose, may help in unblocking the ear. Decongestant may be a great help either an oral agent (Sudafel) taken an hour before descent, or topical agent (oxymetazoline) should be administered 10-15 minutes before descent.

Otitis Externa (swimmer ear) It is the inflammation of ear canal. This is commonly known as swimmer’s ear or hot weather ear. Most often it occurs during summer. 50% of this is cause by Pseudomonas aeruginosa, other common microbes include Staph, Bacillus, and Proteous organisms. Symptoms includes itching, moving pain in air, and fluid discharge from canal in severe cases, decrease or loss of hearing. Ear drainage: Clear or cloudy drainage ----> Otitis media, or CSF Bloody drainage---> ear trauma Drainage resulting from eczematous mild otitis externa can be self treated. Non-pharmacological · Hot compresses; pain, discontinue sticking · Cold compresses; swelling, itch · Avoid using shampoos · Do not manipulate with swabs · Removal earwax · Use blow drier after shower. Bath not shower OTC; Aluminum acetate 0.5% (Burosol), Benzothenium chloride 0.03% and Acetic acid 2% Prescriptions · Gentamycin otic solution (amino glycosides active against gram –ve, (Pseudomonas), and S. aureus (side effect: ototoxicity). · Ciprofloxacin ophthalmic solution (no ototoxicity) Otitis Media (OM) · · · ·

Otitis media is the infection of middle ear. Symptoms. Pain in the ear, and fever Acute otitis media (most common cause S. pneumonia, H. influenza, and M. catarrhalis). However, the types of OM chronic suppurative otitis media Otitis media with effusion. Drug of choice. Amoxicillin or +/- clavulanate, ceftriaxone, cefuroxime axetil, azithromycin. Children with frequent otitis media should vaccinated by pneumococcal vaccine.

Vertigo and dizziness Dizziness is refers to variety sensations such as light-headedness, fainting, spinning, and giddiness. Vertigo is defined as sensation of motion in response given bodily movement. Nausea and vomiting, pallor, and perspiration accompany vertigo. It is vestibular disease as result of lesions or disturbances in inner ear. e.g. Meniere's disease.

Copyright © 2000-2018 TIPS Inc. Unauthorized reproduction of this manual is strictly prohibited and it is illegal to 70-10 reproduce without permission. This manual is being used during review sessions conducted by PharmacyPrep.

www.Pharmacyprep.com

Ophthalmic, OTIC and Mouth Conditions

Meniere's disease prophylaxis; Diuretics (HCTZ, Triamterene), Betahistine (histamine agonist) is commonly used. Diet salt restrictions and avoid coffee, and smoking. Boils: Infected hair follicles in the ear canal that usually cause by S. aureus. This is self-limiting and is best treated by application of warm compress.

Tips 1. 4. 7. 10. 13. 16. 19. 22. 25.

· · · · · · · · · · · · · · · · · · · · ·

1 gtt OU otitis externa antibiotics & corticosteroids HSV2 VZV blepharitis polyvinyl alcohol

2 5 8

0.01% emollients, anesthetics, astringents and Acyclovir

23 26

11 14 17 20

1 gtt AU cold sores inflammation of the eyelid margin CMV pain in eye dry eye hydroxypropylmethylcellulose (HPMC) sterile & isotonic Acyclovir

3 6 9

Carbamide peroxide wax removal HSV 1

12 15 18 21

Epstein barr virus blurred vision diabetes Thimerosal

24 27

Tropicamide cerumenous gland

What ophthalmic conditions require referral to doctor? ( ) What is added in ophthalmic preparation to increase eye contact? ( ) the most allergic ophthalmic preservative? ( ) Benzalkonium chloride concentration as preservative in ophthalmic drops? ( ) Ophthalmic preparation should be? ( ) What eye drops that are used in eye exams? ( ) Cold sores are caused by? ( ) What is treatment of cold sores? ( ) What is not a treatment cold sores? ( ) Both eyes is directed as? ( ) Both ear is directed as? ( ) Earwax removal is? ( ) Swimmer’s ear is? ( ) Abreva is used for? ( ) What is active drug of valacyclovir? ( ) Ear wax glands are also known as? ( ) Mineral oil in ear is used as? ( ) Acyclovir is effective against? ( ) Blepharitis is? ( ) An autoimmune disease characterized by destruction of the lacrimal and salivary glands resulting in the inability to produce saliva and tears. ( ) Stye (hordeolum) require warm compress where as blepharitis require cold compress

Latanoprost side effect (Eye pigmentation and lengthening of eye lashes)

Copyright © 2000-2018 TIPS Inc. Unauthorized reproduction of this manual is strictly prohibited and it is illegal to 70-11 reproduce without permission. This manual is being used during review sessions conducted by PharmacyPrep.

www.Pharmacyprep.com

Ophthalmic, OTIC and Mouth Conditions

Copyright © 2000-2018 TIPS Inc. Unauthorized reproduction of this manual is strictly prohibited and it is illegal to 70-12 reproduce without permission. This manual is being used during review sessions conducted by PharmacyPrep.

www.Pharmacyprep.com

OTC Drugs, antihistamine, decongestants and antitussive

71 OTC Drugs, Antihistamine, Decongestants, Antitussives, and Expectorant Questions Alerts! Common questions in pharmacy exam is to ask! · Topical xylometazoline and oral decongestant pseudoephedrine, phenylephrine precautions and contraindications with MAOi (phenelzine, tranylcypromine, selagiline, rasagiline) hypertensive crisis. · Antitussives like dextromethorphan, codeine DIs with MAOi can cause serotonin syndrome. · What OTC drugs have abuse potential? Dextromethorphan, pseudoephedrine, diphenhydramine, and Tylenol # 1. Pharmaceutical Agents 1. Analgesics/antipyretics; acetaminophen, ASA or ibuprofen · Treat pain and fever. · Don’t administer ASA to infants, children, teenagers or young adults because of Reye’s syndrome. Antihistamines: Relieve rhinorrhea, sneezing and watery eyes associated with cold · Often cause drowsiness and drinking alcohol or taking other antihistamines or sedatives can increase effect. · Paradoxical excitability, nervousness and difficulty sleeping sometimes occur in children. · Contraindicated in patients with glaucoma, kidney or liver disease, prostatic hypertrophy and pregnant or breast-feeding. 1st gen Rapid onset and short duration Highly lipophilic Diphenhydramine Q6-8h No decongestant PM

2nd gen Slow onset and long duration Less lipophilic Cetrizine Q24h Fexofenadine Q12h 60 mg, SR Q24 120mg Desloratadine is approved for relief of nasal decongestant. Drug of choice to allergic rhinitis. AM or day time use

Copyright © 2000-2018 TIPS Inc. Unauthorized reproduction of this manual is strictly prohibited and it is illegal to 71-1 reproduce without permission. This manual is being used during review sessions conducted by PharmacyPrep.

www.Pharmacyprep.com

OTC Drugs, antihistamine, decongestants and antitussive

Can use for allergies, anaphylaxis, common cold, runny nose, and as sedative. Motion sickness and antiemetics

Allergies, common cold and runny nose

Antitussives. Dextromethorphan indicated only for dry, unproductive coughs when congestion is not present. · Contraindicated in patients with chronic, persistent cough, patients with lung disease and in women who are pregnant or breast-feeding Decongestants: pseudoephedrine, Questions Alerts! phenylephrine Phenylephrine or pseudoephedrine with MAOi cause · Oral agents are more hypertensive crisis. 2) Systemic decongestant in normotensive patient BP increases effective but caution in only in high doses. Whereas high BP patient increase BP in patients with heart disease, normal doses. hypertension, thyroid 3) topical decongestants SEs are mainly local and >3-5 d can disease, diabetes, glaucoma rebound congestion. and prostatic hypertrophy, patients taking antidepressants, and women who are pregnant and breast feeding. · Topical agents can cause rebound congestion if used for more than 3 to 5 days. · Oral decongestant should avoid in first trimester of pregnancy. ORAL DECONGESTANT Pseudoephedrine, phenylephrine, ephedrine Po Schedule II decongestant AVOID WITH MAOI (- BP)

TOPICAL DECONGESTANT Oxymetazoline 0.05% Nasal drops, and spray, ophthalmic solutions OTC For sinus and red eye As decongestant from colds, flu, hay fever, & sinusitis. AVOID WITH MAOI (- BP)

Expectorant. Guaifenesin is the only one available in Canada · Adverse effects are rare. · Overdose can cause nausea and vomiting · Contraindicated in patients with chronic, persistent cough, patients with lung disease and in women who are pregnant or breast feeding. Antihistamine: To treat Allergies, Allergic rhinitis, Insomnia, Motion sickness, Nausea and vomiting. Precaution. driving, operating machine, anticholinergic side effect, and constipation. Diphenhydramine or dimenhydrinate is the drug of choice for motion sickness. Meclizine have long half-life is the drug of choice in pilots and navigator. Decongestants. pseudoephedrine, xylometazoline,

Question Alerts! 1) Dextromethorphan concurrent with MAOi, SSRIs, TCAs can cause serotonin syndrome 2) Diphenhydramine can be taken 25-50 mg Q4-6h PRN 3) All of the antihistamine are taken once daily, except?

Copyright © 2000-2018 TIPS Inc. Unauthorized reproduction of this manual is strictly prohibited and it is illegal to 71-2 reproduce without permission. This manual is being used during review sessions conducted by PharmacyPrep.

www.Pharmacyprep.com

OTC Drugs, antihistamine, decongestants and antitussive

saline drops · Congestions and cough, shortness of breathe. · Caution. Uncontrolled blood pressure, uncontrolled diabetes, glaucoma. · DI. MAOi is not taken with oral decongestants. Antitussives: dextromethorphan, codeine used to treat cough or dry cough. Drug interactions: MAOi and SSRI can cause serotonin syndrome. Expectorants: Guaifenesin used to treat productive cough (cough with sputum). DEXTROMETHORPHAN Contraindicated in children 6 year age. Side effects: Nausea, Drowsiness, Dizziness Contraindications: CNS depression will increase if drug used with: Alcohol, Narcotics, Sedatives-hypnotics, Barbiturates, and depressants. Narcotics Antitussive: Codeine: 15-30 mg of dextromethorphan = 8 -15 mg of codeine Side effect: Drowsiness, Sedation, Constipation (Stimulant laxative bisacodyl or senna), nausea, and vomiting Expectorants § Gauifenesen (extract of tar) § Ammonium chloride Expectorants reduce sputum viscosity and allow more effective removal of secretions from the respiratory tract. Gauifenesen (extract of tar) Mechanism: · Increase ciliary action · Fluid from the respiratory tract less viscous Copyright © 2000-2018 TIPS Inc. Unauthorized reproduction of this manual is strictly prohibited and it is illegal to 71-7 reproduce without permission. This manual is being used during review sessions conducted by PharmacyPrep.

www.Pharmacyprep.com

OTC Drugs, antihistamine, decongestants and antitussive

Facilitate the removal of mucus High doses cause emesis

· ·

Side effects. Rare (drowsiness, nauseas, vomiting) Ammonium chloride: Mechanism: Irritant to stomach, Causes reflex increase in airway mucus secretions. High doses will cause acidosis in individuals with renal failure Decongestant Topical decongestants. Oxymetazoline and Xylometazoline · Decrease nasal airway resistance and nasal blood flow, but usually do not cause systemic sympathomimetic action. · Act rapidly within 10 min · Lead to rebound nasal decongestion (rhinitis medicamentosa) which usually occurs 5-10 days of treatment. · Short term treatment, 2-3 days · Topical: Onset of action 5-10 min. · Phenylephrine, Naphazoline, and Xylometazoline Side effects. Local burning sensation, Sneezing, Dryness of the nasal mucosa. Bradycardia, Tachycardia, Hypertension Nasal decongestants. Phenylephrine, and Propylhexedrine Systemic decongestants: · Pseudoephridine, Epinephrine, Phenylephrine and Phenylpropamine · Cause nasal vasoconstriction and decreased nasal edema within 30 minutes and continues for 6 hours (regular formulation) · Oral (onset of action 30 min) · Caution with patient, Hypertension, heart disease, hyperthyroidism, diabetes, narrow angle glaucoma, BPH. Side effects. CNS stimulation (mild), Insomnia, Headache, Irritation, Tachycardia or palpitation, Increase of BP in hypertensive patient. Affects blood sugar levels in diabetics. Contraindication. Uncontrolled hypertension, Severe coronary artery disease, MAOI, Glaucoma, BPH

Tips 1. 4. 7. 10.

glaucoma uncontrolled BP BPH low grade fever

2. 5. 8. 11

hypertension crisis diabetes watery eyes self-limiting viral infections of rhinovirus (30 to 50%) & corona virus (10 to 20%)

3. 6. 9. 12

runny nose sore throat sneezing malaise

Copyright © 2000-2018 TIPS Inc. Unauthorized reproduction of this manual is strictly prohibited and it is illegal to 71-8 reproduce without permission. This manual is being used during review sessions conducted by PharmacyPrep.

www.Pharmacyprep.com · · · · · · · · · · · · · ·

·

OTC Drugs, antihistamine, decongestants and antitussive

Common cold is caused by? (self-limiting viral infections of rhinovirus (30 to 50%) & corona virus ( ) Common cold symptoms(runny nose, sore throat, watery eyes, sneezing, low grade fever & malaise) ( ) Contraindications of oral decongestants ( ) MAOI + sympathomimetics like pseudoephedrine give --> ( ) Echinacea purpurea probably can be effective in the prevention and treatment of common colds in adults. Expectorants examples are? à Topical antihistamine examples à High risk groups for flu vaccine à Who should NOT take flu vaccine à Flu vaccine is taken every à When is the flu season in Canada à Flu immunization season à Contraindications of antihistamines à Cautions of oral decongestants à

Nonpharmacological treatment common cold? Bed rest, Drinking plenty of fluids and Humidifying the air

Copyright © 2000-2018 TIPS Inc. Unauthorized reproduction of this manual is strictly prohibited and it is illegal to 71-9 reproduce without permission. This manual is being used during review sessions conducted by PharmacyPrep.

www.Pharmacyprep.com

OTC Drugs, antihistamine, decongestants and antitussive

Copyright © 2000-2018 TIPS Inc. Unauthorized reproduction of this manual is strictly prohibited and it is illegal to 71-10 reproduce without permission. This manual is being used during review sessions conducted by PharmacyPrep.

www.Pharmacyprep.com

OTC Drugs for Nausea

72 OTC drugs for Nausea, Vomiting, Diarrhea Constipation, and Hemorrhoids NAUSEA & VOMITING Non-prescription anti emetic drugs · Dimenhydrinate is used for all types of nausea and vomiting (N&V) (except post chemotherapy N&V). Given 30 min before exposure · Meclizine is used for all types of N & V · Promethazine · Diphenhydramine is alternative for dimenhydrinate · Pyridoxine (vit. B 6 ) used only for pregnancy induced N&V (PANV) · Scopolamine: used only for motion sickness · Ginger root Sedation is common side effects of non-prescription antiemetic drugs (except pyridoxine). If alertness is required, scopolamine or promethazine + ephedrine or dexamphetamine (used by airline pilots). Pregnancy associated nausea and vomiting also known as “Morning Sickness”. Self-care measure Alter diet emphasize on small & frequent meals, avoid fatty or spicy foods. · Eat at all times of the day when nausea is less severe. · Eat before getting up from bed (in the morning). · Discontinue iron supplements (temporarily) because this causes nausea and vomiting. · Ginger root 250 mg qid may reduce nausea and vomiting. Nonprescription · Dimenhydrinate use only for 2 to 3 days refer to physician if ineffective. · Pyridoxine (vitamin B 6 ) can be used alone. No side effects & drug interactions. Prescription · Drug of choice Diclectin (pyridoxine 10 mg + doxylamine).

Copyright © 2000-2018 TIPS Inc. Unauthorized reproduction of this manual is strictly prohibited and it is illegal to 72-1 reproduce without permission. This manual is being used during review sessions conducted by PharmacyPrep.

www.Pharmacyprep.com

OTC Drugs for Nausea

Opioid induced nausea and vomiting (OINV) Opioid induced nausea and vomiting is caused by stimulation of chemoreceptor center and decreased GI motility. Alter administration schedule (make sure nausea does not interfere with meal). · If pain does not decrease, attempt an increase in opioid dosage (pain could be caused by nausea and vomiting). · Switch to another narcotic drug · Use other anti-emetic. Metoclopramide (drug of choice), and prochlorperazine, diphenhydramine, dimenhydrinate, ondansetron, haloperidol. Post chemotherapy nausea and vomiting (PCNV) Alter diet (emphasize on small & frequent meals. Avoid fatty or spicy foods). Non-prescription medicines are not useful. Prescription medicines are metoclopramide, dronabinol also known as THC (cannabinoids) and prochlorperazine. Low emitogenic drugs: Treatment start with dexamethasone as needed. Delayed nausea and vomiting: The drug of choice for moderate emitogenic is dexamethasone Drug of choice for acute emitogenic is 5HT 3 antagonist ondansetron+ dexamethasone. High emitogenic drugs: The drug of choice for high nausea and vomiting is dexamethasone + ondansetron + aprepitant Anticipatory nausea and vomiting: The drug of choice is benzodiazepine (lorazepam). Motion sickness Self-care measures · Avoid eating large meal within 3 hours of travel. · Avoid dairy products or food high in protein content, high-calories, or high in sodium before travel. · Avoid alcohol, smoking and bad smells. Treatment For short duration of exposure, dimenhydrinate is effective for most patients. Diphenhydramine is an alternative. Take oral medication at least 60 min in advance. Because motion induces gut stasis. The second dose can be used after 6 hrs. Scopolamine Trans dermal patch: Placed behind ears. 1 patch every 72 h, can be removed and reused within 72 h but should rotate site of application. · Side effects include constipation, dry mouth, blurred vision, skin rash, disorientation, delirium. · Should not be used in children. DIARRHEA Management · Rehydration and maintain electrolyte balance can reduce diarrhea symptoms · Children: Continue breastfeeding & oral rehydration solution (ORS) should be offered; otherwise, discontinue all food and drinks and give ORS. · Give ORS as soon as diarrhea begins until diarrhea is less frequent Copyright © 2000-2018 TIPS Inc. Unauthorized reproduction of this manual is strictly prohibited and it is illegal to 72-2 reproduce without permission. This manual is being used during review sessions conducted by PharmacyPrep.

www.Pharmacyprep.com

· · ·

OTC Drugs for Nausea

· Rapid re-feeding should immediately follow rehydration · If diarrhea with vomiting, give ORS 15 ml every 10 to 15 minutes (using a spoon). · Oral rehydration therapy (ORT) is the most effective treatment for children with acute diarrhea. Pregnant: Maintain fluid intake. Loperamide is safe in pregnancy. Elderly. Prompt rehydration is essential. Fruit juice pop, or tea with sugar are not suitable due to high carbohydrate content.

Symptoms of dehydration: Dry mouth (increased thirst), crying without tear, sunken eyes, less or low frequency or decreased urination and skin turger. Sunken soft spot in infants. Feeling weak and lightheadedness. Sweating or frequent urination is NOT a dehydration symptom. Treatment Nonprescription Loperamide (Imodium): · Not given to children under 2 years old also in children less than 12 years old without doctor’s advice. · Not recommended for acute dysentery/infections diarrhea (bloody stool with fever). · SEs include abdominal cramps, drowsiness, dry mouth, and skin rash. · Maximum dose is 16 mg/day (4 mg start then 2 mg after each loose bowel movement). Attapulgite (Kaopectate) may be used for drug-induced (mild to moderate) diarrhea. Not to be used for less than 2 days. Bismuth Subsalicylate · The bismuth subsalicylate (BSS) should be avoided in patients taking anticoagulants, salicylates, probenecid, or methotrexate. · Avoid in NSAIDs or ASA allergies · Not for children less than 2 years old due to Reye’s syndrome · Used in chronic diarrhea, for travelers diarrhea & H. pylori management · SEs. include black tongue and stools, and tinnitus Psyllium (Metamucil) · Should be taken within 2 hours of other medications because it reduces absorption of other medicines · Should be taken with at least 250mL water to prevent fecal impaction and/or esophageal obstruction Prescription · Cholestyramine – for treatment of bile acid induced diarrhea · Codeine – for patients who do not respond to non-prescription medicines · Clonidine – for diarrhea associated with opioid withdrawal and diabetic neuropathy · Diphenoxylate with atropine (Lomotil) less effective than loperamide · Octreotide – for chemotherapy-induced and AIDS-associated diarrhea · Herbal and other Remedies: · Herbal – chamomile, carob, marshmallow, slippery elm, bayberry · Probiotics – live microorganism (bacteria and yeast) ·

Refer to physician if diarrhea does not improve in 48 hours with high fever, blood in feces, severe pain in belly, children less than 6 months old, with vomiting for more than 4 to 6 hours with sign of rehydration; more than 6 BM in one day.

Copyright © 2000-2018 TIPS Inc. Unauthorized reproduction of this manual is strictly prohibited and it is illegal to 72-3 reproduce without permission. This manual is being used during review sessions conducted by PharmacyPrep.

www.Pharmacyprep.com

OTC Drugs for Nausea

Most cases, diarrhea in children is self-limiting and non life threatening. Focus on dehydration with ORT. Breast feeding should be continued during episodes of diarrhea. If not breast feeding appropriate food should be continued (avoid BRAT " banana, rice, apple sauce, toast' diet. is complex carbohydrate diet). After 24 hr normal condition can resume normal diet. take 7-10 days to become stools completely formed. If child vomiting and diarrhea does not stop in 4-7 hr should receive medical attention. NONINFECTIOUS diarrhea: Drug induced diarrhea could be caused by: · Antibiotics · Chemotherapeutic agents · Anti-inflammatory agents (NSAIDs, Colchicine) · Anti-arrhythmic (Quinidine) · Anti-hypertensive (Beta-blockers, ACE inhibitors) · Antacids (Mg-containing antacids, ranitidine, omeprazole) · Miscellaneous; Misoprostol and theophylline TRAVELLERS DIARRHEA caused mainly by E. coli, Shigella sp and Campylobacter jejuni Prevention. · Hot and cooked meals, cooked vegetables (no salad or no fresh salad). · Peeled fruits, boiled/bottled water, carbonated beverages without ice cubes, pasteurized milk (properly stored). · Bismuth sub salicylates as prophylactic agent. · Typhoid vaccine recommended for travelers. · Cholera vaccine for healthcare workers in endemic areas. Nonpharmacological: Children and elderly should use oral rehydration solution. Adults maintain hydration with canned juices, purified boiled or bottle water, clear salty soup, carbonated water. Prevention: Dukoral vaccine can be recommended by pharmacist. Prior to departure, travelers should see physician for appropriate antibiotics. Treatment (prescription) · Drug of choice is ciprofloxacin 500 mg BID X 3 days · Alternative azithromycin, and cefixime. · Cotrimoxazole of limited use due to widespread resistance CONSTIPATION Self-care measures · High-fiber diet (for children less than 2 years old should have dietary levels of fiber equal to or greater than their age + 5 g/day, 25 to 30 g intake for adults). · Minimum fluid intake of 1500 ml daily. Moderate physical activity. · Regular toilet routine (children should be encouraged to defecate 5 to 15 min after meal). · Heed the urge to defecate, weight loss for overweight patients. · Prune and other juices with sorbitol may also help. Drug inducing constipation. · Anticholinergic agents: antidepressants, antipsychotics, antiparkinson's (levodopa). Copyright © 2000-2018 TIPS Inc. Unauthorized reproduction of this manual is strictly prohibited and it is illegal to 72-4 reproduce without permission. This manual is being used during review sessions conducted by PharmacyPrep.

www.Pharmacyprep.com · ·

OTC Drugs for Nausea

Cation containing agent’s aluminum containing antacids, sucralfate, CaCO 3 , and Ca supplements, bismuth, and iron supplements. Other drugs. Verapamil, clonidine, diuretics, cholestyramine, NSAIDs, opiates, vinca alkaloids, sympathomimetics agents, and ganglion blockers.

Pharmacological treatment: laxatives Bulk-forming (psyllium, bran, and methylcellulose) · Adsorb water to soften the stool and increase the bulk, which stimulates peristalsis. · Should be taken with at least 250 mL water to prevent esophageal obstruction and/or fecal impaction. · Side effects include flatulence, bloating. Safe to use in pregnancy. · Contraindication in patients with fluid restriction and mechanical obstruction of the GIT. · Not to be taken within 2 hours with other medications because it reduces drug absorption. Osmotic laxatives. (Lactulose, and glycerin) · Mechanism: Retains water and allow the stool to pass easier through the bowel. · Osmotic laxative preferable for constipation in infants and children. · Lactulose not tolerated by most patients because of too much sweetness in taste. Can be used by diabetic patients. · Side effects include flatulence, abdominal cramps, and N&V. · Contraindicated in patients on galactose free diet. · Taken with fruit juice or milk only to improve palatability. · Lactulose is the drug of choice for hepatic encephalopathy, because it absorbs ammonia. · Glycerin: Softens the stool and lubricates the bowel by increasing water retention (osmotic properties) in the intestinal lumen. Also stimulates rectal contractions. SEs rectal irritation. Saline laxatives (Mg hydroxide, and Mg sulfate) · Mechanism: Same mechanism of action as osmotic laxatives. · Mg hydroxide (Milk of magnesia). · Side effects: Diarrhea, dehydration and electrolyte imbalance, hypermagnesemia · Contraindicated: Patients with cardiac or renal disease · Chilled before administration to increase palatability · Mg sulfate (Epsom salt) · Na phosphate (Fleet enemas) · Best administered on an empty stomach, 30 min AC or HS · SEs. includes hyperphosphatemia, hypocalcemia, hypokalemia, hypernatremia · CIs. for pregnant and lactating women. · Enemas · It is not recommended in children 2 y · SEs. abdominal pain and diarrhea · Taken with meal

Tips

· · · · · · · · · · · · · · ·

1

Psyllium

2

4 7 10 3 16

Dexamethasone Bismuth subsalicylate ice cubes sunken eyes crying without tears

5 8 11 14

Diclectin Vit B6 + Doxylamine Ciprofloxacin uncooked food fresh salads less frequent urine

3

Docusate sodium + senna or bisacodyl

6 9 12 15

Benzodiazepines contaminated water dry mouth loss of skin turger

DOC for pregnancy induced nausea and vomiting ( ) DOC for low emitogenic chemotherapy induced N& V( ) DOC for delayed chemotherapy induced N&V( ) DOC for anticipated nausea and vomiting ( ) symptoms of dehydration ( ) traveler’s diarrhea mainly caused by? ( ) black stools and tongue is side effect of? ( ) DOC for traveler’s diarrhea ( ) DOC in pregnancy for constipation ( ) DOC for opioids induced constipation ( ) What is the drug of choice for pregnancy induced nausea and vomiting à What are the self care measures recommended for N&V associated with PCNVà DOC for low emitogenic chemotherapy induced N&V à DOC for delayed chemotherapy induced N&V à DOC for anticipated nausea and vomiting à

Copyright © 2000-2018 TIPS Inc. Unauthorized reproduction of this manual is strictly prohibited and it is illegal to 72-8 reproduce without permission. This manual is being used during review sessions conducted by PharmacyPrep.

www.Pharmacyprep.com · · · · · · · · ·

OTC Drugs for Nausea

Symptoms of dehydrationà Traveler’s diarrhea mainly caused by à Black stools and tongue is side effect of à What type of food should be avoided by travelers to prevent infectious diarrhea à Drug of choice for travelers’ diarrhea à What are the most important self-care measure is recommended relieve constipation? Drug of choice in pregnancy for constipation? Drug of choice for opioids induced constipationà What are the self-care measures to relieve hemorrhoidsà

Copyright © 2000-2018 TIPS Inc. Unauthorized reproduction of this manual is strictly prohibited and it is illegal to 72-9 reproduce without permission. This manual is being used during review sessions conducted by PharmacyPrep.

www.Pharmacyprep.com

OTC Drugs for Nausea

Copyright © 2000-2018 TIPS Inc. Unauthorized reproduction of this manual is strictly prohibited and it is illegal to 72-10 reproduce without permission. This manual is being used during review sessions conducted by PharmacyPrep.

www.pharmacyprep.com

Analgesics and Topical Pain Relievers

73 Analgesics, and Topical Pain Relievers Questions Alerts! Common questions in pharmacy exam is to ask! · NSAIDs side effect and maximum dose · Triptans mechanism and onset of actions (sc is fastest) · Migraine prophylaxis. Amitriptyline, propranolol, verapamil, and nortriptyline.

Conditions Migraine Angina Neuralgia Ulcers Liver cirrhosis Referred pain (visceral pain) Appendicitis Urinary tract infections (pyelonephritis or pelvic) HEADACHE TENSION Associated with stress, tension etc. No nausea and vomiting.

Types of pain Throbbing pain Thrusting pain, can cause referred pain to left arm & finger Nerve pain, numbness, burning, tingling Epigastric pain Upper right quadrant Pain perceived a location other than the site. (e.g. angina) Lower right quadrant Flank pain (between ribs and hip)

MIGRAINE Unilateral headache (throbbing pain), often associated with nausea and vomiting. Decrease quality of life by interrupting activities. Associated with nausea and vomiting, triggers by light, smell, noise, and food etc.

CLUSTER A series of relatively short cyclical pattern or cluster. This can be painful and can wake up from sleep.

Headache: Generally, headache is characterized as tension, cluster and migraine. Tension: Associated with stress, tension etc. No nausea and vomiting. The drug of choice for tension headache? Acetaminophen or NSAIDs. Cluster headache: A series of relatively short cyclical pattern or cluster. This can be painful and can wake up from sleep. The drug of choice is verapamil.

Copyright © 2000-2018 TIPS Inc. Unauthorized reproduction of this manual is strictly prohibited and it is illegal to 73-1 reproduce without permission. This manual is being used during review sessions conducted by PharmacyPrep.

www.pharmacyprep.com

Analgesics and Topical Pain Relievers

Migraine · Unilateral headache (throbbing pain), often associated with nausea and vomiting. Decrease quality of life by interrupting activities. · Associated with nausea and vomiting, triggers by light, smell, noise, and food etc. Migraine Therapeutic Plan · Mild migraine attacks. ASA, ibuprofen, adjunctive dimenhydrinate, metoclopramide, and acetaminophen (weak evidence). ·

Moderate attacks. NSAID, triptans (the drug of choice), dihydroxy ergotamine (DHE) (weak evidence). Combination drugs acetaminophen + codeine+ caffeine, ASA + codeine + caffeine or ASA + butalbital + caffeine.

·

Severe and ultra severe attacks. Butraphanol, chlorpromazine, dexamethasone, ketorolac, meperidine, metoclopramide, prochlorperazine, and sumatriptan.

Mild Migraine attacks · ASA. Dose 650 to 1300 mg q4h buffered or soluble tablets (not enteric coated) · Ibuprofen: Dose 400 to 800 mg q6h, rapid dissolving tablets available · Acetaminophen: Weak evidence of benefit. Monitor AST/ALT. Moderate to severe migraine attacks · NSAIDs (ibuprofen, naproxen, mefenamic acid). · Triptans 5HT 1b/d agonists: Sumatriptan, rizatriptan, zolmitriptan, almotriptan and naratriptan. Sumatriptan side effects are chest tightness. Contraindicated: pregnancy. · Oral 25 to 50 mg, MR q2h (max 200 mg/24 h). · Intranasal 5 to 20 mg, 1 spray in 1 nostril per dose. · MR in 2 hours (max 40 mg/day). Do not shake and prime. · SC 6 mg, MR 1 hr/max 12 mg/24 hr. · Take at first sign of headache or at aura. · If NO relief first dose, DO NOT use second use. If it relieved, and second attack, use only after 2 hours. Avoid other triptans use with 24 hours. TRIPTANS. All triptans available as tablets. Sumatriptan subcutaneous is fastest onset of all triptans 10-15 min Oral disintegrating tablets (wafers), rizatriptan and zolmitriptan Inter nasal spray. Sumatriptan and zolmitriptan. Dosage forms HalfMAOi & life SSRI, SNRI Propranolol Sumatriptan Zolmitriptan Naratriptan Rizatriptan Fravotriptan Almotriptan

Tab, sq, spray RDT, wafers, spray tab RDT, wafers tab tab

2.5hr 3 hr 6 hrs 2-3 hrs 26 hrs 3-4 hrs

ü ü ü

Cimetidine

ü ü

Copyright © 2000-2018 TIPS Inc. Unauthorized reproduction of this manual is strictly prohibited and it is illegal to 73-2 reproduce without permission. This manual is being used during review sessions conducted by PharmacyPrep.

www.pharmacyprep.com

Analgesics and Topical Pain Relievers

Combination analgesics · Acetaminophen 300 mg + codeine 8 mg + caffeine 15 mg (Tylenol #1) · ASA+ codeine + caffeine (222) · ASA+ butalbital + caffeine (Fiorinal). Regulated as control drug part 2. Migraine prophylaxis: Propranolol, atenolol, metoprolol, nadolol, verapamil, amitriptyline, nortriptyline, topiramate, valproic acid, divalproex sodium and carbamazepine. Venlafaxine and pizotifen (serotonin antagonist) and lithium. Low Back Pain · Avoid unnecessary bed rest for uncomplicated back pain. As well as premature physical therapy. · For acute uncomplicated low back pain, NSAIDs are effective for pain relief particularly during the first few weeks. · For back pain and chronic soft tissue pain, tricyclic or other types of antidepressants have equivocal efficacy, but may be useful for their antidepressant effect. Pharmacological treatment · Acetaminophen · NSAIDs: ASA · Skeletal muscle relaxant (chlorzoxazone, methocarbamol) · Acetaminophen + methocarbamol (Robaxacet) · ASA + methocarbamol (Robaxasal)

Question Alerts! Low back pain? Keep normal activity, and minimize bed rest.

Sports Injuries Goals of therapy. · To reduce acute symptoms (pain, inflammation) and recurrences · To correct contributing factors (e.g. malalignment, muscle weakness) · To return the athlete’s weight-bearing capability, flexibility, range of motion, strength and proprioception to normal · To enable that athlete to participate comfortably and fully in all pre-injury activities General approach · R – Rest the injured part · I – Ice application to the injured part for 15 to 20 minutes ≥4 x/day for the 48 hours or longer. · C – Compress the injured part with elastic bandage if there is swelling. · E – Elevation · Other injuries requiring immediate medical attention eye, head, and nosebleed. · Along with the R-I-C-E therapy ASA or NSAIDs could be used for short period for pain and swelling. · Note: A patient with DVT and injury should not follow the full “RICE Therapy” because ice and compression could lead to stasis hence rests and elevation of limb would be options for such kind of patient. 1. Introduction to Wounds 2. Prevention and Management of Venous Leg Ulcers 3. Prevention and Management of Pressure Ulcers 4. Prevention and Management of Diabetic Foot Ulcers Copyright © 2000-2018 TIPS Inc. Unauthorized reproduction of this manual is strictly prohibited and it is illegal to 73-3 reproduce without permission. This manual is being used during review sessions conducted by PharmacyPrep.

www.pharmacyprep.com

Analgesics and Topical Pain Relievers

Pressure Ulcers: Also known as decubitus ulcers (rectal ulcer), and bedsores. Common sites of pressure ulcers are buttocks, thighs, back, ankle etc. Ulcer care. · Wound debridement, wound cleansing and dressing of wound. · Note that for wound cleansing, antiseptic agents/hydrogen peroxide and other wound cleaners may be toxic to the wound and should be avoided. · Cleansing or irrigation of wound should be done with normal saline. Stages of Symptoms treatment pressure ulcers Stage 1 Skin unbroken but inflamed Skin sealants Stage 2 Skin is broken to epidermis or dermis Hydrogel Stage 3 Stage 4

Ulcer extends to subcutaneous fat layer Ulcer extends to muscle or bone

Moist saline gauze Moist saline gauze

Multiple sclerosis Immunomodulators. Glatiramer, interferon beta1a, interferon beta 1b, fingolimod (spingosine-1-phosphate receptor agonist), dimethyl fumarate. Adhesion molecule inhibitor; Natalizumab Anti-CD52 monoclonal antibody. Alemtuzumab

Tips

· · · · · · · · · · · · ·

1.

Propranolol

2

4. 7 10

Avoid prolong bed rest Nausea & vomiting Amitriptyline

5. 8. 11

NSAIDS or Acetaminophen Throbbing pain; feels hitting w/ a hammer 5HT1 b/1d agonist Valproic acid

3 6 9 12

Triptans, alternatively ergot alkaloids Unilateral headache R-I-C-E Verapamil

What are the symptoms of migraine headacheà What is drug of choice for acute migraine attackà What are the drugs used to treat migraine prophylaxis -->? What is prophylaxis is recommended for migraine in-patient experiencing 3 to 4 migraine attacks every month and having constipation à What is mechanism of action of triptans à General approach for sports injuries ( ) What drugs are used for migraine prophylaxis? ( ) What are recommended self measures for back pain? ( ) What is the treatment for back pain? ( ) Migraine pain is? ( ) Pressure ulcers also known as à Mechanism of muscle relaxants à Drugs used in multiple sclerosis à

Copyright © 2000-2018 TIPS Inc. Unauthorized reproduction of this manual is strictly prohibited and it is illegal to 73-4 reproduce without permission. This manual is being used during review sessions conducted by PharmacyPrep.

www.pharmacyprep.com

Analgesics and Topical Pain Relievers

Select True or False statements · · · ·

A 45 yo man diagnosed with benign prostatic hyperplasia, and migraine prophylaxis. Propranolol appropriate therapy for migraine prophylaxis? after taking sumatriptan migraine headache does not relieve than double the dose of sumatriptan after taking sumatriptan migraine headache does not relieve than decrease the dose of sumatriptan after taking sumatriptan migraine headache does not relieve than do not use sumatriptan

Copyright © 2000-2018 TIPS Inc. Unauthorized reproduction of this manual is strictly prohibited and it is illegal to 73-5 reproduce without permission. This manual is being used during review sessions conducted by PharmacyPrep.

www.pharmacyprep.com

Analgesics and Topical Pain Relievers

Copyright © 2000-2018 TIPS Inc. Unauthorized reproduction of this manual is strictly prohibited and it is illegal to 73-6 reproduce without permission. This manual is being used during review sessions conducted by PharmacyPrep.

www.PharmacyPrep.Com

Asthma and COPD

74 Asthma and COPD Questions Alerts! Common questions in pharmacy exam is to ask! · Asthma triggers · Mild, Moderate and Severe asthma therapy · Theophylline drug interaction Predniosone side effects and dose

Asthma. Chronic inflammatory disorder of the airways, ↑ airways responsiveness, causes reversible obstruction. In asthma esinophils, mast cells and T lymphocytes plays significant role. Sensitivity, and hypersensitive of airways to specific and non-specific stimuli, such as air, odour, allergens, virus etc. Diagnosis: Spirometer (preferable method of diagnosis) (Forced expiratory volue per second (FEV 1 normal >80%) Peak flow meter (home monitoring) Bronchoprovocation challenge test, using methanacholine or histamine if diagnosis is in doubt. Sequence of asthma therapy SABA PRNà ICS à LABA à LTRAà PO CTS à iv CTS. MILD MODERATE SEVERE EMERGENCY FEV >80% FEV1 60-80% FEV1 10 Mu/l Hoshimoto HYPO HYPER

HYPERTHYROIDISM The drug of choice ANTITHYROID METHIMAZOLE AND PTU TREATMENT IF severe hyperthyroid symtpms Graves disease The drug of choice for thyroid toxicosis. Is lugol solution.

↑TSH (>10) = ↓ free T 4 5.5 13.

Hashimoto disease

14.

Thyroxin

16. Propylthiouracil 17. Lugol solution · Symptoms of hypothyroidism ( ) · Symptoms of hyperthyroidism ( ) · Hyperthyroidism ( ) · Hypothyroidism ( ) · What is the drug of choice for hypothyroidism (

3 6 9 12

Constipation Sensitive to heat Sensitive to cold Graves disease

15

Methimazole

18

)

Copyright © 2000-2018 TIPS Inc. Unauthorized reproduction of this manual is strictly prohibited and it is illegal to 80-3 reproduce without permission. This manual is being used during review sessions conducted by PharmacyPrep.

Pharmacyprep.com · · · · · · · · · · · · · ·

Thyroid disorders

What is the drug of choice for hyperthyroidism ( ) What is the drug of choice for hyperthyroidism in pregnancy ( ) What drug that are taken empty stomach ( ) Drug absorption is decreased if taken with calcium supp or dairy products, iron, antacids ( What drugs that stain ( ) Severe fever, sore throat and agranulocytosis are the side effects of ( ) Calcitonin is stimulated byà TSH is secreted from à In treatment of hypothyroidism with T 4 have effect on à Hypothyroidism is monitored by à T 4 metabolized to T 3 by deiodinase enzyme in --> Sweating is symptom of à Lugol solution is an oral drops of? à Thyroxine absorption is decreased by à

)

Select True or False Statements · · · · · · · · · · · · · · · · · ·

Hyperthyroidism in pregnancy: PTU is drug of choice, with the lowest possible doses used to maintain the maternal T4 level in the high normal range. (True) A 55 yo women using levothyroxin 75 mcg to treat hypothroidism. She is experiencing sweating, heat sensitive and diarrhea. Indicates overdose. (True) levothyroxin take in the morning empty stomach(True) levothyroxin take with full glass of milk (False) Lugol solution used to treat thyrotoxicosis, (Graves disease) (True) hypothyroidism, Total T 4 decrease(True) hypothyroidism, Free T 4 decrease(True) hypothyroidism, Total T 3 decrease(True) hypothyroidism, Serum TSH decrease(False) hypothyroidism, Free Thyroxine index decrease(True) FT4, TT4, TT3 and FTI decrease in hypothyrodism, only serum TSH increase. For hyperthyroidism, exactly opposite changes. (True) Myxedema; In this disease, the patient may have slow speech, a puffy face, slow pulse, low BMR and scanty hair. (True) Cretinism: The growth and height of the child is stunted. The patient has low BMR and a bloated face. The patient is also mentally retarded. (True) Goitre: It is also known as simple or non-toxic goiter. A dietary deficiency of iodine may be responsible for this. The neck of the patient is swollen. (True) Toxic nodular goiter: It is due to benign neoplasm or adenoma or may be because of long standing normal goiter. (True) Hoshimoto thyroiditis: It leads to hypothyroidism(True) Discontinue antithyroid if patient notice even a single rash (Pruritis Maculopapular rashesh associated with vasculitis) Why is it beneficial to add propranolol to a drug regimen of a patient diagnosed with hyperthyroidism? decrease heart rate, anxiety, tremors, and heat intolerance

Copyright © 2000-2018 TIPS Inc. Unauthorized reproduction of this manual is strictly prohibited and it is illegal to 80-4 reproduce without permission. This manual is being used during review sessions conducted by PharmacyPrep.

PharmacyPrep.Com

Contraception

81 Contraception Questions Alerts! Common questions in pharmacy exam is to ask! · Side effects of oral contraceptive pills · OCP DIs with phenytoin, carbamazepine, topiramate, rifampin, antibiotics. · OCP CI's. Coronary artery disease, DVT, PE and liver diseases. · What if patient miss one pill? What to do? · Emergency contraception like plan B side effects (N&V) Short term prevention (1 yr planning for pregnancy) Barrier methods IUD, Medroxy progestin inj Combined oral contraceptive pills, Nuvaring Breast feeding: progestin only Barrier methods: condoms. Male (latex, polyurethane, lambskin) · Protect against STIs including HIV (latex condoms only). · Lambskin: no protection against STIs. · Latex SEs: hypersensitivity in either partner, use water lubricants (oil-based ↓ integrity). Condoms: female · Not to be used with male condoms, shelf-life of up to 5 years. · inserted up to 8 h prior to intercourse & removed immediately after Diaphragm · Side effects: Toxic shock syndrome · Use with spermicidal, can be inserted 6 h before intercourse & used in breastfeeding women. · Use silicone diaphragm (allergy to latex). Sponge · Side effect: Toxic shock syndrome, spermicidal is released in a sustained fashion for 10 to 12h · Do not use during menstruation Cervical cap · Side effect: Toxic shock syndrome, can be left in place for up to 48h for multiple acts of intercourse. · Can be used in breastfeeding women, not to be used within 6wk of delivery Spermicides: nonoxynol-9 (Vaginal Contraceptive Film) Copyright © 2000-2018 TIPS Inc. Unauthorized reproduction of this manual is strictly prohibited and it is illegal to 81-1 reproduce without permission. This manual is being used during review sessions conducted by PharmacyPrep.

PharmacyPrep.Com

Contraception

Not effective against HIV or STI Intrauterine devices (IUD). copper-T IUD (Nova-T, Flexi-T) Contraceptives, oral combination · Avoid in lactating women during the first 6 wk postpartum, with caution in the first 6mo postpartum · SE: chloasma, hypertension, breakthrough bleeding or spotting, N/V, breast tenderness, and mood changes. · DI. rifampin & griseofulvin use backup barrier method during therapy · Obesity affects metabolism to compromise contraceptive efficacy. May decrease the effectiveness of contraceptive. · "ACHES" danger signal Abdominal pain/Chest pain & shortness of breath/Headaches/Eye problems/Severe leg pain (DVT). Diane-35 (EE 35 µg / cyproterone 2 mg) · Drug of choice in severe acne (Not for birth control) · Discontinue 3 to 4 mo after signs of acne have completely resolved Contraceptives, transdermal patch: EE/norelgestromin (Evra) · Apply a new patch once a wk on the same day of the wk for 3 wks · If off for >24 h start new patch & use backup method for 7 days · OCPs→ patch. The first patch on the first day of withdrawal bleeding · If later than the first day of withdrawal bleeding, use backup method for 7 days · Depot MPA→ patch, start on day of scheduled injection. · Not effective. Weight 90 kg or more · Do not apply on chest. Contraceptives vaginal ring: EE/etonorgestrel (NuvaRing) · Contraindication: 65 y Vertebral compression fractures Postmenopausal woman (not on estrogen therapy). Premature menopause (140 mm Hg, a diastolic blood pressure >90 mm Hg. Diagnosis criteria from joint national committee (JNC8) report recommendations for follow up in adults. Question Alerts! · /-180/110 à Evaluate or refer to source of care immediately or within 1 week depending clinical evaluation Maintain BP below · 140/90 uncomplicated hypertension (therapy starts) · 140/90 with target organ damage or CV disease · Isolate systolic hypertension >140/160/80 17. Ramipril What is the recommended sodium intake for a patient diagnosed with hypertension?( ) The drug of choice against uncomplicated hypertension age over 65 yo is ( ) The drug of choice against uncomplicated hypertension age less than 65 yo is ( ) The drug of choice for hypertension in pregnancy ( ) Use in non complicated hypertension and also indicated in opioids and benzodiazepine withdrawal symptoms ( ) Decrease BP in both supine & standing position, especially in elderly ( ) Diuretics that gives ototoxicity, hypokalemia, dehydration, allergy, nephritis and gout ( ) What drug turns urine into blue color ( ) What antihypertensive drug should be taken 1 hour before meals ( ) it is 3x more potent and used once daily and no sulfonil group ( ) it is use for hypertension and alopecia treatment ( ) The drug of choice for hypertensive crisis ( ) It causes salt and water retention which may lead to CHF ( ) The drug of choice for Reynaud phenomenon ( ) This drug may cause a sudden drop in blood pressure that can result in loss of consciousness ( ) Drugs may increase the effects of potassium supplements, potassium sparing diuretics, cyclosporine, leading to raise of potassium in the blood ( ) Hypertension with diabetes drug of choice is à Hypertension with renal disease drug of choice is à

Copyright © 2000-2018 TIPS Inc. Unauthorized reproduction of this manual is strictly prohibited and it is illegal to 85-4 reproduce without permission. This manual is being used during review sessions conducted by PharmacyPrep.

www.pharmacyprep.com · · · · · ·

Hypertension

Isolated systolic hypertension which drugs should not useà Cardio selective beta blockers are à The most beta 1 selective blockers that has been studied in lung dysfunction; à Name the cation most prevalent in the extracellular fluid of the body.à Why is bedtime the best time to dose terazosin?à It is an antihypertensive drug which is also used prophylaxis migraine ( )

Select True/False Statements A customer of your pharmacy checked two times blood pressure in your pharmacy blood pressure monitor and found to have average 190/95. What is appropriate to do? talk to him first and refer to doctor

Copyright © 2000-2018 TIPS Inc. Unauthorized reproduction of this manual is strictly prohibited and it is illegal to 85-5 reproduce without permission. This manual is being used during review sessions conducted by PharmacyPrep.

www.pharmacyprep.com

Hypertension

Copyright © 2000-2018 TIPS Inc. Unauthorized reproduction of this manual is strictly prohibited and it is illegal to 85-6 reproduce without permission. This manual is being used during review sessions conducted by PharmacyPrep.

PharmacyPrep.Com

86 Coronary Artery Diseases Questions Alerts! Common questions in pharmacy exam is to ask! · Atherosclerosis and plaques (Caused by high LDL) · Examples of coronary artery diseases angina, and myocardial infarction · Risk factors and investigations (biochemical marker CK-MB, Troponin-i, ECG) · Treatment of NSTEMI (Non ST segment elevated MI). ASA, BB · Treatment of STEMI (ST segment elevated MI). Alteplase, ASA

Copyright © 2000-2018 TIPS Inc. Unauthorized reproduction of this manual is strictly prohibited and it is illegal to 86-1 reproduce without permission. This manual is being used during review sessions conducted by PharmacyPrep.

PharmacyPrep.Com Types of angina

Stable/Exercise induced angina Due to coronary partial blood clot ↓ ST segment (subendothelial)

Prinzmetal/Varian/ Vasospastic angina ↓ Coronary blood flow. ↑ ST segment (transmural) Due to vasospasm effect of TXA2

Unstable/Crusendo angina Acute with platelet aggregations. Due to non-occlusive thrombus in an area of coronary atherosclerosis / or disrupted plaques

Definition: Angina is those symptoms of myocardial ischemia that occur when myocardial oxygen availability is insufficient to meet myocardial oxygen demand. Symptoms: discomfort or pain in the chest, arm, shoulder, back or jaw. Frequently worsened by physical exertion or emotional stress. Diagnosis: ECG and exercise stress test with echocardiography. Stable angina · Caused ↓ O 2 (Ischemia) supply due to ↓ blood flow. Precipitating factors exercise, cold weather, sexual activity and emotional stress.

·

Symptoms: Pain located over sternum and may radiate to left shoulder or arm, right arm or neck, or jaw. Duration 0.5 to 30 min. Patient description of symptoms pressure or heavy weight on chest, burning, tightness, deep, suffocating, squeezing, aching, and crushing. Symptoms occurring for weeks without worsening consider stable angina. Usually relieved by rest or nitroglycerin SL. Approximately 75% of ischemic episodes are silent and not detected especially in diabetes.

· · ·

Treatment: Acute angina chest pain use nitroglycerine SL spray or tab or Chewable ASA 80 mg. The drug of choice angina with hypertensionà cardio selective BBs, or NTG-LA or CCBs, Angina with hypertension and diabetes or chronic kidney diseaseà ACEi

·

Drug of choice 1. Secondary prophylactic treatment for all patient; ASA 325 mg enteric coated daily 2. Initial treatment of symptom with nitrates; nitroglycerin SL tablets 0.3-0.6 mg PRN or nitroglycerin SL spray 0.4 mg PRN 3. All patient who require regular treatment of symptoms should receive a beta-blocker; Metoprolol 25-100 mg bid, nadolol 20-240 mg once daily.

Second line therapies 1.

2.

If patient contraindication to beta blockers, such as reactive airway disease or experience side effects with beta-blockers, Verapamil 80 mg tid or Verapamil SR 240 mg once daily If patient intolerant to beta blocker and verapamil Nitroglycerine transdermal 0.4 mg/hr apply morning, remove after 12 hrs. or isosorbide mononitrate 20 mg in morning and afternoon 7 hr apart (Long acting nitrates to be used for a maximum 12 h/day. Nitrates should be used during high risk periods e.g. times when angina is common and overnight into early hours of morning.

Copyright © 2000-2018 TIPS Inc. Unauthorized reproduction of this manual is strictly prohibited and it is illegal to 86-2 reproduce without permission. This manual is being used during review sessions conducted by PharmacyPrep.

PharmacyPrep.Com Prinzmetal angina (vasospastic) Question Alerts! · Caused by spasm, do not increase MVO 2 Drug of choice to treat prinzmetal · Mainly due to atherosclerosis angina? · Symptoms. Pain usually occurs at rest awakens from sleep. · Characterized by recurrent, prolong attacks of severe ischemia. · Treatment. The drug of choice is CCBs (nifedipine, amlodipine) · Acute chest pain use nitroglycerine SL spray or tab Acute coronary syndrome (ACS): Term describes the symptoms that may lead to acute myocardial infarction (acute MI) or heart attack. Acute MI further characterized as STEMI and NSTEMI and as well as unstable angina. Diagnosis: Question Alerts! · Chest pain: Generally lasting for >30 min What are the biochemical · 12-lead ECG. ST segment elevation markers for MI diagnosis? · Cardiac isoenzymes: cardiac troponin T or I elevated. and CKMB elevated · Echocardiogram to identify the site and severity of wall motion abnormalities. · Patient presentation: Diaphoresis (sweating), nausea, vomiting, weakness, and shortness of breath, arm tingling, and syncope. May confuse as heartburn symptoms. NSTEMI. Partial blockade of coronary blood flow. Involves only subendocardial myocardium ST depression or NO ST elevation on ECG Positive: CK-MB and Troponin-I Drug of choice anticoagulants Heparin. or Antiplatelet (ASA or/and clopidogrel

STEMI Completely occlusive thrombus Effect entire thickness of myocardial wall. Cause myocardial necrosis. ST segmen t elevation on ECG More extensive damage Positive. CK-MB and Troponin I Drug of choice. Thrombolytic (Alteplase) are time dependant therapy or angioplasty. Stent: ASA + clopidogrel for 1 yr

Copyright © 2000-2018 TIPS Inc. Unauthorized reproduction of this manual is strictly prohibited and it is illegal to 86-3 reproduce without permission. This manual is being used during review sessions conducted by PharmacyPrep.

PharmacyPrep.Com UA: Unstable angina, STEMI: ST segment elevated myocardial infarction, NSTEMI: Non-ST segmented elevated Myocardial infarction, GPI; Glycoprotein IIb/IIIa receptor antagonists. NSTEMI · Caused by disruption of an atherosclerotic plaque or formation of platelet aggregation thrombus. · Symptoms: Crushing chest pain that can radiate to neck, back, shoulders, arms and jaw. Pain is similar to angina but more severe. May occur at rest and may be caused by less exertions. · Pain is NOT relieved by NTG · Diagnosis: Chest pain is NOT relieved by NTG and persist longer than 5 min. · Treatment: MONA therapy: Morphine à Oxygen à Nitrates à ASA; BBs without ISA or CCBs à Heparin or LMWH STEMI · The most common type MI (85%) is due to thrombus formation caused by precipitated by atherosclerosis plaque rupture. This propagated thrombus leads to occlusive thrombus. · The complete blockade due to occlusive thrombus results in persistent ischemia that clinically manifest as STEMI. If this is not treated, occlusion of coronary arteries can lead to sudden cardiac death. · Symptoms: similar to UA/NSTEMI, however it is common in women, elderly, and DM. Post MI Patient with stent gets ASA 81-325 mg+ clopidogrel 75 for one year.

Tips 1. 4. 7. 10 13

16 19

· · · · · · · · · · · · ·

Nitroglycerin Nitrates Heparin LDL >2.2 mmol/l Diltiazem

Atherosclerosis Neutralization

2. 5. 8. 11 14

17

Amlodipine Nitrites (Na Nitroprusside) Dihydropyridine Beta blockers verapamil

Nitric oxide (NO)

3. 6. 9. 12 15

18

Ca channel blockers ASA Clopidogrel Thrombolytic Ticlopidine

Room temperature

What is the most common cause of ischemic heart diseases (atherosclerosis) LDL What drug is effective for acute and chronic angina ( ) What are the treatment of choice in patients with coronary arterial spasm ( ) What is used for STEMI treatment (ST-segment elevation MI) ( ) What is used for NSTEMI treatment (Non-ST-segment elevation MI) ( ) What drug acts on peripheral vascular system that causes reflex tachycardia ( ) What appropriate drug for those who cannot take ASA ( ) Symptoms of stable angina à Chest pain, shortness of breath, pain radiating to left arm (True/False) What is the drug of choice for stable angina à ( ) Prinzmetal angina may causes due to à vasospasm What is the drug of choice for prinzmetal angina is à What antiplatelet drug has Neutropenia side effectà A patient is intolerant or allergic ASA, should get alternate drug of prophylaxis for vascular diseases à

Copyright © 2000-2018 TIPS Inc. Unauthorized reproduction of this manual is strictly prohibited and it is illegal to 86-4 reproduce without permission. This manual is being used during review sessions conducted by PharmacyPrep.

PharmacyPrep.Com Nitroglycerin act as smooth muscle vascular dilator due to à Venous pooling effect is caused by à Nitroglycerin storage conditions à · What is the drug of choice for STEMI à · What are the examples of LMWH à Enoxaparin, Dalteparin, Tinzaparin, Nadroparin · Mechanism action of LMWH àBlocks blood clotting factor 10a and IIa · What laboratory test is used for monitoring LMWH à None · Headache is side effect of à · Protamine sulphate is antagonist of heparin, which react by à · What kind of pain occurs in heart attack? Referred pain · Referred pain is? pain localized at one location however origin is from another organ · Patient with asthma and stable angina, the drug choice is? CCBs · · ·

Select True or False statements: · · · · · · · · · · · · · · · · · · · · ·

Symptom of coronary artery diseases? chest pain, sweating, shortness of breath (True) IV dosage of form of nitroglycerin is the fastest acting or have rapid onset of action? (True) Nitroglycerin onset of iv ( 1 to 2 min), SL ( 1 to 5 min), oral (40 min), ointment (20- to 60), patch ( 40 to 60) (True) Alteplase is least likely used after myocardial infarction after 6 hr of acute attack? (True) Nitroglycerin SL spray is used to relieve angina symptoms (True) nitroglycerin SL spray should spray on or under the tongue(True) nitroglycerin SL spray should store at room temperature(True) nitroglycerin SL spray do NOT require shaking before use(True) nitroglycerin SL spray relieve angina symptoms(True) nitroglycerin iv is faster acting than nitroglycerin SL (True) LMWH have predictable response thus no monitoring required(True) Heparin is the drug choice anticoagulant in pregnancy(True) Warfarin is monitored by PT and INR(True) warfarin should not be taken with vitamin K supplements(True) Protamine sulphate is antidote of heparin(True) With isoniazid take vitB 6 , with levodopa avoid vitamin B 6 , with warfarin avoid vit. K. (True) Isotretinoin and tretinoin avoid vitamin A because analogs of vitamin A and Phenytoin & methotrexate take folic acid(True) Coumarin derivatives anticoagulants is used as rat poisoning? (True) Angina, MI, transient ischemic stroke medical conditions are associated with ischemia. Omega 3 polyunsaturated fatty acids are abundant in fish oils? (True) Ecosapentanoic acid (EPA) and docosahexanoic acid (DHA) are derived from Omega 3(True)

Copyright © 2000-2018 TIPS Inc. Unauthorized reproduction of this manual is strictly prohibited and it is illegal to 86-5 reproduce without permission. This manual is being used during review sessions conducted by PharmacyPrep.

PharmacyPrep.Com

Copyright © 2000-2018 TIPS Inc. Unauthorized reproduction of this manual is strictly prohibited and it is illegal to 86-6 reproduce without permission. This manual is being used during review sessions conducted by PharmacyPrep.

www.pharmacyprep.com

Stroke

87 Stroke Questions Alerts! Common questions in pharmacy exam is to ask! ·

· · ·

Symptoms of stroke sudden dizziness, headache, confusion, blurred vision, and fainting. Stroke risk factors. Coronary artery disease, DVT, high cholesterol, and age. Transient Ischemic attack ASA. Acute stroke therapy. Alteplase or anticoagulant

Stroke symptoms: Headache, dizziness, blurred vision, confusion, and incoherent speech. F = face changes (ask patient to smile, one side drops) A = arms drifting (raising hand can drift) S = speech change (cannot repeat the same sentence) T = Time (every second 1 brain cell die) Lab tests: CT brain scan (to rule out hemorrhagic process). ESR (to test hypercoagulable state) ECG (exclude atrial fibrillation) MRI to confirm the diagnosis. Primary prevention of vascular diseases: Early recognition and management of modifiable risk factor such as poor diet, sedentary lifestyle, obesity, high BP, cholesterol, DM and smoking.

Secondary prevention of vascular disease (Transient ischemic attack) Management of risk factors after patient has suffered vascular event.

Drug of choice for the transient ischemic attack? ASA 81-325 mg The drug of choice for the secondary prevention in patient with atrial fibrillation? Warfarin or anticoagulants What is the drug of choice to prevent cardiogenic stroke? Warfarin or anticoagulants. Copyright © 2000-2018 TIPS Inc. Unauthorized reproduction of this manual is strictly prohibited and it is illegal to 87-1 reproduce without permission. This manual is being used during review sessions conducted by PharmacyPrep.

www.pharmacyprep.com

Transient ischemic attack (TIA); deficiency of oxygen (ischemia) in brain tissues. The drug of choice is ASA 81 mg

Stroke

Acute stroke: cerebral hemorrhagic, thrombotic and embolic stroke.

Pharmacotherapy Anti-platelets. ASA, Clopidogrel, Ticlopidine, Dipyridamole/ASA DOC: 2nd prevention of Noncardioembolic ischemic strokes ASA. initial therapy (50-325 mg/day for prevention) Clopidogrel – 75mg/daily, alternative agent, somewhat more effective < ASA alone Avoid grapefruit juice ASA + Clopidogrel; should NOT be used for long-term secondary prevention of ischemic events (-bleeding) Ticlopidine; 250mg bid, SE: diarrhea, skin rash, neutropenia (need monitoring). Not routine use Dipyridamole SR/ASA; 200/25g bid, ¯ risk of stroke (mostly for ischemic stroke) Anticoagulants; Warfarin, nicoumalone DOC : 2nd prevention of Cardioembolic ischemic strokes prevent cerebral and systemic emboli in patients with acute MI, valvular and nonvalvular AF and prosthetic cardiac valves. Nonvalvular AF and prior TIA/stroke ® require INR of 3.0 instead of 2.5. Drug of choice for stroke prevention? ASA Drug of choice to treat acute stroke? Thrombolytic (alteplase)

Copyright © 2000-2018 TIPS Inc. Unauthorized reproduction of this manual is strictly prohibited and it is illegal to 87-2 reproduce without permission. This manual is being used during review sessions conducted by PharmacyPrep.

www.pharmacyprep.com

Stroke

Tips ________________________________________________________________________ Tips format 002: Stroke 1. Headache 2. Dizziness 3. Blurred vision 4. Confusion 5. Incoherent speech 6. Warfarin 7 ASA 8 Clopidogrel 9. Ticlopidine 10 Alteplase 11 BP>140/90 12 LDL>2.6 mmol/L 13 · · · · · · · · · · · · ·

Chest pain

The artery that supply blood to brain? Internal carotid artery What is not a stroke symptom? ( ) Symptoms of stroke ( ) What is the initial symptoms of stroke ( ) What are the drugs of choice for long term prevention of atherothrombotic events ( ) What are risk factors for stroke ( ) What is the drug of choice for transient ischemic attack (TIA) ( ) What is the initial therapy for stroke prevention ( ) What drugs combination of drugs increases the risk of bleeding ( ) What drug may give neutropenia infrequently but is potentially serious and require monitoring of CBC every 1 to 2 wks ( ) What is not recommended routine protection of stroke ( ) What drug prevents cerebral and systemic emboli in patient with acute MI ( ) What drugs are used in within 3 hrs of acute ischemic stroke ( )

Select True or False Statements · · · · · ·

Dizziness and head ache are the initial symptoms of stroke. (True) Pathophysiologic of cerebral ischemia are associated with carotid atherosclerosis that can result into stroke. (True) ASA is the drug of choice for transient ischemic attack (TIA)? (True) Migraine headache is least documented risk factor of stroke? (True) Thrombolytics like alteplase should be used within 3 hours of stroke Seizures is not a symptom of stroke? (True)

Copyright © 2000-2018 TIPS Inc. Unauthorized reproduction of this manual is strictly prohibited and it is illegal to 87-3 reproduce without permission. This manual is being used during review sessions conducted by PharmacyPrep.

www.pharmacyprep.com

Stroke

Copyright © 2000-2018 TIPS Inc. Unauthorized reproduction of this manual is strictly prohibited and it is illegal to 87-4 reproduce without permission. This manual is being used during review sessions conducted by PharmacyPrep.

PharmacyPrep.Com

88 Congestive Heart Failure Questions Alerts! Common questions in pharmacy exam is to ask! · CHF symptoms dyspnea, fatigue, edema, weight gain · CHF Treatment. ACEi. furosemide, digoxin · Digoxin mechanism. + ve inotropic, -ve chronotropic and vagomimetic · Digitalis toxicity. Quinidine, thiazides, loop, erythromycin, tetracycline, verapamil · Digoxin CI's. Ventricular arrhythmias · Digitalis toxicity symptoms: severe nausea vomiting, anorexia, muscular weakness, bradycardia, and ventricular premature contractions. Severe toxic symptoms include: blurred vision, disorientation, diarrhea, ventricular tachycardia, AV blockade which progress to ventricular fibrillation.

Symptoms: Typical symptom of CHF include dyspnea, fatigue and fluid retention (edema). The primary manifestation of heart failure are dyspnea and fatigue that may limit exercise tolerance, and fluid retention the may lead to pulmonary or peripheral edema. Other symptoms may include paroxysmal nocturnal dyspnea, orthopnea (shortness of breath that prevents lying down), tachypnea (rapid breathing), cough, ascites and nocturia. Other symptoms include jugular venous distention, hepatojugular reflux, hepatomegaly (enlarged liver), bibasilar rales, pleural effusion (increase in fluid in pleural surfaces), tachycardia, pallor (pale skin) and S 3 gallop. Symptoms of advanced heart failure are the same but more severe. Causes of CHF. In 65% of patient’s coronary artery disease is the cause of heart failure, other causes include nonischemic cardiomyopathy example hypertension, thyroid disease or valvular disease. These patients usually have reduced left ventricular dysfunction; usually ejection fraction is 50 y, history of DVT, obesity, major surgeries (Hip, knee), paralysis (after stroke), CHF, CAD, deficiency of protein C and S and smoking. · Reynaud's phenomenon treatment AND intermittent claudication. · Drug should avoid in peripheral vascular disorders and BBs, ergot alkaloids

Fig 90.1

Copyright © 2000-2018 TIPS Inc. Unauthorized reproduction of this manual is strictly prohibited and it is illegal to 90-1 reproduce without permission. This manual is being used during review sessions conducted by PharmacyPrep.

www.PharmacyPrep.Com

Peripheral Vascular Diseases

Peripheral vascular disease occurs in arteries and veins of periphery, other than heart and brain.

Peripheral Vascular Diseases Disease of Veins Venous blood clots Pulmonary embolism Phlebitis Varicose veins

Disease of Arteries Arterial blockage Aortic aneurysms Buerger’s disease Reynaud's phenomenon Intermittent claudication

Venous Thromboembolism (Fig 90.1) Deep vein thrombosis (DVT) is obstruction of vein by blood clot. It is most common within the deep veins of calf muscles of the leg. The affected leg may become swollen and tender. The main risk is that the clot may become detached and give rise to pulmonary embolism. DVT symptom can cause pain in popliteal area. Calf swelling, pitting edema, entire leg swelling. Localized tenderness along the distribution of vein system. Diagnosis: Doppler Test and ultrasound. Non-pharms: Regular exercise of leg and anticoagulant therapy are used for prevention and treatment. The drug of choice for prevention and treatment if LWMH or heparin. Drug of choice for prophylaxis (prevention) of DVT in hip replacement or knee replacement surgeries? LMWH (enoxaparin) or heparin The drug of choice of DVT in pregnancy? LMWH or heparin. Treatment: LMWH (enoxaparin, dalteparin, tinzaparin, nadroparin), specific factor Xa inhibitors (fondaparinux) and unfractionated heparin (heparin) and warfarin. LMWH are approved for prophylaxis and treatment of DVT. Heparin is prophylaxis only. Warfarin is to target INR 2-3. Prophylaxis

Drugs (LMWH & heparin). Graduated compression stockings and intermittent pneumatic compression devices, and caval interruption by filters.

Treatment

For treatment of established DVT and/or PE start oral warfarin together with sc LMWH, or iv UHF. The LMWH or UHF is continued for minimum of 5 days or until the INR is therapeutic for at least 2 days. The duration of oral anticoagulant is dependent on the risk of recurrence of VTE. In most analysis of treatment of VTE, LMWH use has typically more effective and less costly overall compared to unfractionated heparin.

LMWH During Pregnancy

LMWH or heparins is the anticoagulant of choice during pregnancy. Injection sc twice a day to achieve the therapeutic levels. Warfarin or UFH may be used for about 6 weeks after delivery for secondary prevention.

Copyright © 2000-2018 TIPS Inc. Unauthorized reproduction of this manual is strictly prohibited and it is illegal to 90-2 reproduce without permission. This manual is being used during review sessions conducted by PharmacyPrep.

www.PharmacyPrep.Com

Peripheral Vascular Diseases

Arterial Thromboembolism Acute therapy

Heparinized (80 u/kg) followed by thromboembolectomy (thrombolytic treatment in all patients).

Chronic therapy

Warfarin (maintain INR 2-3 for cardiac source; 2.5-3.5 for prosthetic mechanical valve). If the risk of recurrence become high (e.g. cardiac source, significant vascular disease or abnormality of coagulation cascade) we can consider ASA in combination with warfarin Combination of glycoprotein IIb/IIIa receptor inhibitors with thrombolytic therapy may improve outcomes for arterial thrombosis

Intermittent Claudication

(Fig 90.1)

Intermittent claudication is a cramping pain, induced by exercise and relieved by rest, that is caused by inadequate supply of blood to the affected muscles. It is most often seen in the calf and leg arteries. The pulses are absent and feet may be cold. Antiplatelets agents

Antiplatelets agents reduce vascular death in high-risk patients about 25% and are equally effective in those with coronary artery disease and PAD. ASA is the drug of choice.

Clopidogrel

Clopidogrel may be more effective than ASA in patients with PVD but is usually reserved for those who cannot tolerate ASA or continue to have events while on ASA

ACE inhibitors

ACEi reduce the risk of ischemic events beyond that expected from lowering blood pressure in patients with (PAD).

Ramipril Ramipril demonstrated similar effects in patients with or without PAD. Contraindications Beta blockers (use cautiously in sever disease with hypertension). Rheologic modifiers à pentoxifylline

Reynaud's Phenomenon Reynaud's phenomenon is a condition in which arteries of the fingers Question Alerts! become spastic (vasospastic). This may result from atherosclerosis, Reynaud's Phenomenon is NOT connective tissue disease, ingestion of ergot alkaloids, or frequent use caused by plaques! of vibrating tools. Reynaud’s phenomenon is due to result of over sensitive blood vessels in the body extremities. It is characterized by a pale to blue to red sequence of color changes in extremities. Non-Pharmacological Choices. Minimize cold exposure, and use warm gloves Therapy. The drug of choice is CCBs (nifedipine XL, felodipine, amlodipine) and diltiazem Copyright © 2000-2018 TIPS Inc. Unauthorized reproduction of this manual is strictly prohibited and it is illegal to 90-3 reproduce without permission. This manual is being used during review sessions conducted by PharmacyPrep.

www.PharmacyPrep.Com

Peripheral Vascular Diseases

· Alpha blocker à prazosin · PGI 2 analog à iloprost · Peripheral vasodilator may relieve this condition. Contraindications: Avoid prescribing medications with vasoconstrictive potential: ergot derivatives, methysergide and beta-blockers.

CCB of dihydropyrid ine PGI 2 analog CI

Medication taken daily (as opposed to PRN) during the winter will increase tolerance to side effects (headache). DOC is CCBs CCB of dihydropyridine class (e.g. nifedipine XL 30 mg or felodipine 5 to 10 mg) should be used 60 minutes before cold exposure. i.v. iloprost (PGI 2 analog) may be useful for short term use; oral iloprost is less effective Ergot alkaloids, methysergide and beta blockers?

Tips

Find answers from the table 1. Smoking 4. Immobility 7. Effects extremities 10 sc LMWH

2. 5. 8. 11

Obesity Severe pain in legs Plaques Warfarin

3. 6. 9. 12

13

14

Pulmonary embolism

15

· · · · · · · · · · · · ·

Embolism

Increase age Pale finger tips iv heparin fondaparinux

Risk factors of venous thrombosis ( ) Symptoms of deep vein thrombosis ( ) Symptoms of Reynaud's phenomenon ( ) What is not a cause of Reynaud's phenomenon ( ) What is the initial treatment of established DVT or PE ( ) What is the first of a new class of antithrombotic agents, the specific factor Xa inhibitors. ( Examples of vascular diseases à Symptoms of deep vein thrombosis à What is the pharmacotherapy for deep vein thrombosis à Intermittent claudication symptoms à Reynaud's phenomenon symptoms à What is the drug of choice in Reynaud's phenomenon à What drugs should be avoided in Reynaud's phenomenon à

)

Select True or False Statements: · LMWH is the most important drug is used to prevent peripheral vascular diseases? (True/False) · Anticoagulants like warfarin is commonly used to treat acute peripheral embolic disorders. (True/False) · Beta blocker are least likely used to treat peripheral vascular diseases? (True/False) · beta blocker cause vascular constriction thus it is contraindicated in peripheral vascular diseases (True/False) · Reynaud phenomenon symptoms occurs in limb extremities (True/False) Copyright © 2000-2018 TIPS Inc. Unauthorized reproduction of this manual is strictly prohibited and it is illegal to 90-4 reproduce without permission. This manual is being used during review sessions conducted by PharmacyPrep.

www.PharmacyPrep.Com · · · · · ·

Peripheral Vascular Diseases

Calcium channel blockers are the drug of choice (True/False) Reynaud phenomenon can be trigger cold exposure (True/False) Reynaud phenomenon is may occur in old age (True/False) arterial plaques risk factor intermittent claudication (True/False) estrogen/progesterone have risk of blood clots in peripheral vascular system(True/False) LMWHs like enoxaparin, dalteparin, tinzaparin, and nadroparin are approved for both prophylaxis and treatment of venous thromboembolism (VTE) (True/

Copyright © 2000-2018 TIPS Inc. Unauthorized reproduction of this manual is strictly prohibited and it is illegal to 90-5 reproduce without permission. This manual is being used during review sessions conducted by PharmacyPrep.

www.PharmacyPrep.Com

Peripheral Vascular Diseases

Copyright © 2000-2018 TIPS Inc. Unauthorized reproduction of this manual is strictly prohibited and it is illegal to 90-6 reproduce without permission. This manual is being used during review sessions conducted by PharmacyPrep.

91 Chronic Pain Conditions Three types of pain Nociceptive pain: Caused by injury to body tissue. Aching, sharp, throbbing. E.g. osteoarthritis. Psychogenic pain: Related psychological factor. Neuropathic pain: Due to damage or dysfunction nerve, spine or brain. (touch sensitive). Examples diabetic neuropathy, post herpetic neuralgia, trigeminal neuralgia.

Trigeminal neuralgia · Trigeminal neuralgia is a cranial nerve disorder that involves malfunction of the trigeminal nerve (cranial nerve V), which carries sensation from the face to the brain. Malfunction of the trigeminal nerve produces bouts or severe, piercing pain lasting seconds to minutes. · Since bouts of pain are brief and recurrent, typical pain medications are NOT usually helpful. Carbamazepine (which works by binding to, and inhibiting the sodium channel) is the first line choice, but phenytoin may be prescribed if carbamazepine does not work, or if it produces severe side-effects. Baclofen, gabapentin, clonazepam, and valproic acid also help in some cases, although these are inferior to CBZ. CBZ diminishes the pain associated with trigeminal neuralgia within 24-48 hours. · CTC7 neuropathic pain. Clinical experience shown that other therapies are inferior than CBZ but may be helpful in adjunctive therapy. · if some relief from TN, but SEs are not tolerable good strategy is to try oxcarbazepine, or lower the CBZ dose or add beclofen.or phenytoin. · Dose for trigeminal neuralgia 200 mg bid. stat, increased by 200 mg daily to max 1000 mg daily (in divided doses). Post herpetic neuralgia (PHN) Herpes zoster (HZ), also known as shingles is a distinctive syndrome caused by reactivation of varicella zoster virus (VZV). The risk of HZ increases with age, approximately half of all cases occur in persons older than 60 years. One of the most common and debilitating sequele of HZ is post herpetic neuralgia (PHN), defined as pain persisting more than 3 months after the rash has healed. Copyright © 2000-2018 TIPS Inc. Unauthorized reproduction of this manual is strictly prohibited and it is illegal to 91-1 reproduce without permission. This manual is being used during review sessions conducted by PharmacyPrep.

Herpes zoster lesions contain high concentrations of VZV, which can be spread by contact and by the airborne route and which can cause primary varicella in exposed, susceptible persons. Less contagious than primary varicella. Herpes Zoster contagious 2 days prior to infectious lesions and until the lesions crust. Risk of transmission is reduced further if lesions are covered. PHN drug of choice TCAs/ gabapentin/pregabalin 2nd line. SNRI duloxetine or venlafaxine/topical lidocaine 5% patch. 3rd line. Opioids, tramadol, fentanyl, methadone, marijuana (cannabinoids). Acyclovir, famciclovir, and valacyclovir are nucleoside analogues that inhibit replication of human herpes viruses, including VZV. When taken orally, these agents reduce the duration of viral shedding, hasten rash healing, reduce the severity and duration of acute pain, and reduce the risk of progression to PHN. Antiviral are the most effective if started within 72 hours of rash.

Copyright © 2000-2018 TIPS Inc. Unauthorized reproduction of this manual is strictly prohibited and it is illegal to 91-2 reproduce without permission. This manual is being used during review sessions conducted by PharmacyPrep.

www.Pharmacyprep.com

Anxiety Disorders

92 Anxiety Disorders Questions Alerts! · Common questions in pharmacy exam is to ask! · Antidepressants mechanism and DI with MAOi · Benzodiazepine classifications (intermediate acting BZDs daily) for max 2 to 4 wks. Anxiety is a normal response to fear, threat, or psychological stress and is experienced occasionally by every one. · Anxiety disorder involves a state of distressing chronic but fluctuating nervousness that is inappropriate in certain circumstances. · Anxiety disorder can be categorized into several types, such as panic attack, social anxiety disorder, social phobia, obsessive-compulsive disorder, generalized anxiety disorder etc. Cognitive behavioral therapy Thought = what we think affects how we act and feel. E.g. I will not cope Emotions = What we feel affects what we think and do. E.g. sweating, anxious Behavior = What we do affects how we think and feel e.g. want to get away or not attend at all.

·

Panic attack and panic attack with or without agoraphobia Panic is acute, short lived, extreme anxiety with some physical symptoms. This could occur any anxiety disorder person with any specific situations. For example, a person with phobic of snake may panic when encounter with snake. Patient may actively avoid situations in which panic attacks are predicted to occur. Intolerance of physical symptoms of anxiety. 1s line SSRI, or SNRI 2nd line Benzodiazepine Often clinically a small dose of long acting BZD is prescribed with SSRI/SNRI to provide more immediate relief. Social anxiety disorder (SAD) and (or) social phobia · Excessive or unrealistic fear of social or performance situations. Intolerance of embarrassment or scrutiny by others. · Specific phobia: Excessive or unreasonable fear of a circumscribed object or situation usually associated with avoidance of the feared object (for example, an animal, blood, injections, heights, storms, driving, flying, or enclosed places). The drug of choice for Generalized social anxiety SSRI or SNRI Specific phobia like stage fear propranolol

Copyright © 2000-2018 TIPS Inc. Unauthorized reproduction of this manual is strictly prohibited and it is illegal to 92-1 reproduce without permission. This manual is being used during review sessions conducted by PharmacyPrep.

www.Pharmacyprep.com

Anxiety Disorders

Obsessive compulsive disorder (OCD) · Presence of obsessions. Recurrent, unwanted, and intrusive thoughts, images, or urges that cause marked anxiety (for example, thoughts about contamination, doubts about actions, distressing religious, aggressive, or sexual thoughts). · Compulsions. Repetitive behaviors or mental acts that are performed to reduce the anxiety generated by the obsessions (for example, checking, washing, counting, or repeating). The drug of choice is SSRIs. · 2nd line clomipramine · ·

Generalized anxiety disorder (GAD) Uncontrollable and excessive worry occurring more days than not, about a number of everyday, ordinary experiences or activities. Often accompanied by physical symptoms (for example, headaches or upset stomach). Intolerance of uncertainty.

Post traumatic stress disorder (PTSD): Occurs after a traumatic event to which patient responds with intense fear, helplessness, or horror; patients relive the event in memory, avoid reminders of the event, and experience emotional numbing and symptoms of increased arousal. Intolerance of experiencing trauma. The drug of choice for PTSD SSRI or SNRI. The drug of choice for Nightmare, prazosin. Reference: DSM-5. Diagnostic and statistical manual of mental disorder 5th ed. Anxiety clinical practice guidelines Cognitive behavioral therapy + BZD (daily) for 2 wks à SSRI or SNRI + CBT Anxiety disorders SSRIs Fluoxetine (Prozac) Fluvoxamine (Luvox) Paroxetine (Paxil) Paroxetine (Paxil CR) Sertraline (Zoloft) Dual action antidepressant Venlafaxine (Effexor XR) Azapirones Buspirone (BuSpar) 5HT1a agonist Benzodiazepines Propranolol

Cognitive behavior therapy (CBT)

PD

SAD

Y Y Y

Y Y

Y

Y

OCD

Y Y Y Y

GAD

PTSD

Y Y

Y

Y Y

Y (stage ear)

Reference: Can J Psychiatry, Vol 51, Suppl 2, July 2006 The Merck Manual of medical information Treatment choices are modified from data from Compendium of Pharmaceuticals and Specialties 2016 Copyright © 2000-2018 TIPS Inc. Unauthorized reproduction of this manual is strictly prohibited and it is illegal to 92-2 reproduce without permission. This manual is being used during review sessions conducted by PharmacyPrep.

www.Pharmacyprep.com

Anxiety Disorders

Tips Find answers from the table: 1 Benzodiazepines 2. 4 Sertraline 5. 7 Venlafaxine 8 · · · · · · · · ·

Flumazenil Paroxetine (Paxil) Paroxetine (Paxil CR)

3 6 9

Fluoxetine Fluvoxamine Buspirone

What are minor tranquilizers used to treat insomnia and anxiety ( ) What it is the antidote for benzodiazepine ( ) What drugs that are use for Obsessive-Compulsive Disorder (OCD) ( ) What drugs that are use for Post Traumatic Stress Disorder (PTSD) ( ) What drugs that are use for Generalized Anxiety Disorder (GAD) ( ) What drugs that are use for Social Anxiety Disorder (SAD) & social phobia ( Paroxetine is indicated for à Obsession is à Compulsion is à

)

Copyright © 2000-2018 TIPS Inc. Unauthorized reproduction of this manual is strictly prohibited and it is illegal to 92-3 reproduce without permission. This manual is being used during review sessions conducted by PharmacyPrep.

www.Pharmacyprep.com

Anxiety Disorders

Copyright © 2000-2018 TIPS Inc. Unauthorized reproduction of this manual is strictly prohibited and it is illegal to 92-4 reproduce without permission. This manual is being used during review sessions conducted by PharmacyPrep.

www.PharmacyPrep.Com

Depression

93 Depression Questions Alerts! Common questions in pharmacy exam is to ask! · Antidepressants first line SSRI, Bupropion, venlafaxine, Mirtazepine, mocolbamide. · Second line Tricyclic antidepressants · Third line MAOi · Serotonin syndrome. DIs with MAOi gives serotonin syndrome · Dual acting antidepressants like venlafaxine, desvenlafaxine, bupropion, mirtazepine · MAOi drugs interactions with sympathomimetics and antidepressants · Side effects SSRI, SNRI, TCAs like amitryptiline, MAOi. Depression is a feeling of intense sadness. It may follow a recent loss or other sad event but is out of proportion to that event and persists beyond an appropriate length of time. Signs and symptoms. Sadness, tearfulness, dejection, self-criticism, loss of ability to experience pleasure, loss of appetite, inability to sleep and loss of libido. Depression can be categorized into major depression, and bipolar depression. The first line treatment of major depression is SSRIs, dual acting venlafaxine, mirtazepine and moclobemide. (if one therapy fails try from the 1st line). The treatment duration for one episode of major depression is 1 year. The 2nd line therapy for major depression is TCA (Amitriptyline, nortriptyline) 3rd line is MAOi (phenelzine, tranylcypromine) The drug of choice for the treatment depression and anxiety? Fluoxetine or SSRI SSRIs: Fluoxetine, Sertraline, Paroxetine, Citalopram, Escitalopram, Fluvoxamine. · SEs. Sexual dysfunction, GI, CNS. Nausea is the common. · Discontinuation: taper slowly over four to 6 weeks. (particularly important for Paroxetine and venlafaxine). · Fluoxetine can be used in children & adolescents & pregnancy. SEs: insomnia, potent CYP2D6 inhibitor · Sertraline; least DI, Take with food, breastfeeding, SE : diarrhea · Citalopram; least DI, breastfeed patient Copyright © 2000-2018 TIPS Inc. Unauthorized reproduction of this manual is strictly prohibited and it is illegal to 93-1 reproduce without permission. This manual is being used during review sessions conducted by PharmacyPrep.

www.PharmacyPrep.Com · ·

Depression

Paroxetine; breast-feed Escitalopram: isomer of citalopram

SNRIs: Venlafaxine, desvenlafaxine and duloxetine. · SE: BP-(If dose >225 mg/day) Dual action: Bupropion, mirtazepine, and trazadone Bupropion: NE&DA reuptake inhibitor · DOC: depression & smoking cessation · CI: anorexia or bulimia nervosa, head trauma or prior seizure. · SE: insomnia, sexual dysfunction (low rate) Mirtazepine act on NE (directly) & serotonin (indirectly), alpha2 antagonist SEs. Sadation, weight gain (GI & sexual dysfunction is less). Trazadone: 5HT 2 antagonist with some serotonin reuptake inhibitory properties · Excessive sadation (300 to 400 mg daily) ®as a hypnotic (50-100 mg) · With other antidepressants · SEs. drowsiness, priapism TCAs: Desipramine, Nortriptyline, Amitriptyline, Imipramine, Clomipramine, Doxepin, Trimipramine · 2nd or 3rd line agent · SEs. Anticholinergic, orthostatic hypotension, · Clomipramine. obsessive-compulsive disorder · Amitriptyline: chronic pain, neuralgia, migraine prophylaxis, and depression. · Nortriptyline – elderly depressed patients Tetracyclic antidepressants – Maprotiline · Higher risk of seizures than the others MAOIs; Reversible: Moclobemide · Dietary precautions are NOT required at standard doses · SE: nausea, insomnia (persist) · Drug interactions: Avoid sympathomimetics (pseudoephedrine, ephedrine), meperidine. · Caution with opioids, antihypertensives, antipsychotics, SSRIs, Selegiline, excessive tyramine, alcohol. Reduce dose with cimetidine. Irreversible: Phenelzine, Tranylcypromine · Reserved for the treatment of resistant depression · Food and drug cautions must be followed during treatment and for 2 weeks after the last dose of the MAOi. · DI: Sympathomimetics may ↑ BP, Meperidine may cause agitation, hyperpyrexia, circulatory collapse · SSRIs, TCAs, levodopa may ↑ effects and side effects · Tyramine containing food may cause hypertensive crisis. Washout period Copyright © 2000-2018 TIPS Inc. Unauthorized reproduction of this manual is strictly prohibited and it is illegal to 93-2 reproduce without permission. This manual is being used during review sessions conducted by PharmacyPrep.

www.PharmacyPrep.Com

Depression

Generally, there is no need for a washout period, and a crossover technique can be applied (i.e., tapering one agent while titrating the other). · Exception · Irreversible MAOI (phenelzine/tranylcypromine)® other antidepressants 2 weeks · Moclobemide ® other antidepressants 5 days · Fluoxetine ® an irreversible MAOI 5 weeks · Caution: when starting other antidepressants after fluoxetine discontinuation. Serotonin syndrome is an acute condition due to increase serotonin levels and develops within minutes to hours (typically within 6 hours) after starting a medication, increasing the dose of a medication, or overdosing. Signs and symptoms of Serotonin syndrome Neurobehavioral confusion, agitation, coma, seizures Autonomic Hyperthermia (fever), diaphoresis (sweating), tachycardia, hypertension, diarrhea Neuromuscular myoclonus, rigidity, tremor (shivering), ataxia, nystagmus Most cases are mild and resolve spontaneously within 24 to 72 hours. Cardiac arrest, coma, and multiorgan system failure have been reported as consequences of serotonin syndrome. Serotonin Syndrome T =tremor H = hyperpyrexia e = excessive muscle tone S = shivering, seizure H = hypertension A = agitation N = neuromuscular pain C = coma D = Death

Serotonin syndrome occurs with the following agents. All SSRIs Miztazapine Venlafaxin Moclobemide Non selective MAOi Dextromethorphan Linezolide

The syndrome is possible when these agents are combined with. -each other -MAOIs -lithium, meperidine, pentazocine, and dextromethorphan Discontinued symptoms. Fatigue, nausea, dizziness, and lightheadedness, tremors, chills, insomnia, anxiety and diaphoresis are the most common effects of abrupt discontinuation of SSRIs.

Serotonin syndrome are tremors, delirium, excitement, anxiety, diaphoresis, rigidity, increase temp (fever), clonus, increase reflexes, increase HR, either increase or decrease blood pressure. Withdrawal. Flu, insomnia, nausea, imbalance, sensory disturbances, and hyperactivity. Tremors Symptoms Serotonin syndrome Withdrawal symptoms (FINISH) Autonomic symptoms Neuromuscular Dysfunction Cognitive

Diarrhea, Fever, Shivering, change in BP, and & N, V Tremors, seizure, myoclonus, hyper reflexes Agitation, Confusion, hypomania

Flu like fever, chills, dizziness, light headed, N &V, sleep disturbances Myalgia, lethargy, sensory disturbances Not present

Trazodone: potent postsynaptic serotonin (5HT 2 ) receptor antagonist, with weak serotonin reuptake inh. SNRI: Venlafaxine, duloxetine and desvenlafaxine. Desvenlafaxine is active metabolite of venlafaxine. Copyright © 2000-2018 TIPS Inc. Unauthorized reproduction of this manual is strictly prohibited and it is illegal to 93-3 reproduce without permission. This manual is being used during review sessions conducted by PharmacyPrep.

www.PharmacyPrep.Com

Antidepressants Mirtazepine Bupropion Trazadone Fluvoxamine Sertraline Fluoxetine Moclobamide

Depression

Highs Side effects Weight gain Sedation Drug interaction T 1/2 , Washout period DIs

Lows side effects Nausea & vomiting Low Sexual dys. & low weight gain Insomnia Drug interaction. Weight gain Sexual dysfunction

Tips Find answers from the table: 1. Phenelzine 3. Mirtazepine 5. Bupropion 7. Trazodone 9. Amitriptyline 11 TCAs · · · · · · · · · · · · · · · · · · · · · · · ·

2. 4. 6. 8. 10. 12.

Moclobemide Venlafaxine Lithium Fluoxetine SSRI MAOInh.

Drugs selectively blocks the prejunctional neuronal reuptake pumps in the CNS ( ) What is the most anticholinergic and sedative TCA ( ) What drug is used to treat depression with insomnia, what is therapy ( ) What antidepressant requires 5 weeks washout period ( ) Antidepressant also used for smoking cessation ( ) What drug is used to treat depression in sexual dysfunction, what is the best therapy ( ) An example of an irreversible non selective MAO ( ) it is the only reversible & selective inhibitor of MAO that is currently available ( ) Drugs acts directly on noradrenergic system & has low rate of GI and sexual side effects but is associated with sedation and weight gain ( ) Antidepressant also use for generalized anxiety disorder (GAD) ( ) Its is used in prophilactically in treating manic-depressive patient, treat manic episodes & bipolar depression ( ) What drug serum level should not exceed 1.5 mEq/L (6) SSRI onset of action is à Fluoxetine washout period à Drug of choice in depression with sexual dysfunction à Depression with insomnia à Depression with diabetes à Depression with diabetes, the drug of choice for major depressionà Higher dose of venlafaxine (225mg/day) have effect on à A patient on antidepressants and shows with dilated pupil, may be due toà TCA onset of action is à A substance found commonly in fermented foods which can be toxic when MAO inhibitors are usedà MAO is classified as à SSRI like fluoxetine inhibit cytochrome CYP2D6 (True/False)

Copyright © 2000-2018 TIPS Inc. Unauthorized reproduction of this manual is strictly prohibited and it is illegal to 93-4 reproduce without permission. This manual is being used during review sessions conducted by PharmacyPrep.

www.PharmacyPrep.Com · · · · · · · · · ·

Depression

Mirtazepine may cause higher weight gain (True/False) Avoid cheese with à Milk + MAOI à St. John wort has antidepressant effect (True/False) Serotonergic symptomsà The drug of choice against bipolar disorders and manic depression --> Lithium toxicity symptoms --> What antidepressant is used as pharmacotherapy for smoking cessation à Paroxetine therapeutic use --> What antidepressants are NOT used to treat bulemia and anorexia nervosa -- à

Select TRUE OR FALSE Statements · · · · · · ·

Buprapion, trazadone and mirtazepine have least sexual dysfunction (True/False) Take SSRI in the morning and TCAs in evening or bedtime (True/False) Normal blood levels of lithium in adult should not exceed is 1.5 mEq/L (T/F) Lithium concentration varies with Na+ ions (True/False) Li+ conc. increases with decrease Na+ and Li+ conc. decreases with Increase in Na+ (T/F) ACEI, NSAID, Thiazides, decreased renal perfusion à increase Li toxicity Fluoxetine (SSRI), dehydration and renal dysfunction increase Li+ toxicity(T/F)

Copyright © 2000-2018 TIPS Inc. Unauthorized reproduction of this manual is strictly prohibited and it is illegal to 93-5 reproduce without permission. This manual is being used during review sessions conducted by PharmacyPrep.

www.PharmacyPrep.Com

Depression

Copyright © 2000-2018 TIPS Inc. Unauthorized reproduction of this manual is strictly prohibited and it is illegal to 93-6 reproduce without permission. This manual is being used during review sessions conducted by PharmacyPrep.

Pharmacyprep.com

Antipsychotic Drugs

94 Anti Psychotic Drugs Symptoms: The schizophrenic individual is out of touch with reality, hallucinates, hears voices and exhibits bizarre behaviour. These symptoms are only one aspect of schizophrenia. Other symptoms are social withdrawal, and an inability to communicate or to concentrate.

Positive Symptoms Reality distortions: Delusions persecution Grandiosity, thought broadcasting, thought insertion, loose associations, gestures, mindreading, and being controlled. Hallucinations: Auditory (most common) Olfactory, somatic, and visual. Disorganizations. Disorganized speech Incoherent speech. Tangentiality and Loose association Disorganized behavior: Agitation, Hostility Assaultiveness, Uncooperativeness, Inappropriate sexual/social behavior, Inappropriate dress and catatonia. Treatment. 1st & 2nd genereration

Negative Symptoms Blunted affect: Lack of expressed emotion, Poor eye contact and inattentiveness, and Reduced spontaneity. Alogia: Lack of spontaneity of conversation and poor ability to concentrate Avolation/apathy: Lack of interest in activities, no motivation, social withdrawal and poor hygiene. Anhedonia: Loss of pleasure and Few recreational activates Attentional impairment. Lack of ability to concentrate on tasks or conversation Treatment. 2nd generation

Copyright © 2000-2018 TIPS Inc. Unauthorized reproduction of this manual is strictly prohibited and it is illegal to reproduce without permission. This manual is being used during review sessions conducted by PharmacyPrep.

94-1

Pharmacyprep.com · · · · ·

Antipsychotic Drugs

Severe case of psychosis (schizophrenia) or bipolar disorder à add mood stabilizers (Valproic acid, carbamazepine) Drug of choice for acute agitation in seniorsà Quetiapine or heloperidol 1st or 2nd generation 4 to 8 weeks no response, change to 2nd line Severe case of psychosis (schizophrenia) or bipolar disorder à add mood stabilizers For 1 episode of psychosis, continue for 1 to 2 yrs and for 2 episodes, continue treatment for 2 to 5 yrs

Antipsychotics side effects are associated extrapyramidal symptoms. Parkinsons like symptoms (tremors), akthisia (motor restlessness), tardive dyskinesia (inappropriate postures of neck, trunk and limbs), and dystonia. Parkinson’s symptoms include T = Tremor, R = Rigidity, A = Akithisia, P = Postural instability · 1st gen (haloperidol, loxapine, chlorpromazine, thioridazine) is effective in positive schizophrenic symptoms. However, 2nd gen (clozapine, risperidone, olanzapine, quetiapine) covers negative schizophrenic symptoms. Olanzapine, not effective for the treatment of resistance, and risperidone works for negative and positive symptoms. · Orthostatic hypotension is SEs of 2nd generation antipsychotics (can cause additive effects with other antihypertensive drugs).

Parkinson's Symptoms T = Tremor R = Rigidity A = Akinisia P = Postural unstability Extrapyramidal symptoms P = Parkinson's symptoms A = akthisia D = dystonia T = Tardive dyskinesia

1nd generation Antipsychotics Low potency · Chlorpromazine. SEs. QT c prolongation · Methotrimeprazine Intermediate potency · Loxapine, Perphenazine · Zuclopenthixol –injectable FGA , long T 1/2, Do not use in antipsychotic-naïve patients High potency · Fluphenazine, Flupenthixol · Haloperidol: with Lorazepam im for acute phase. SE: QT c prolongation · Pimozide; SE : QT c prolongation with dose > 8mg/day (avoid use with sertraline) · Thiothixene and Trifluoperazine 2ndgeneration Antipsychotics (SGAs) · Advantage in first-episode psychosis, in improving negative symptoms, mood and cognitive deficits and in preventing relapse and rehospitalization. · First line treatment · Depression in the acute phase effectivity SGAs>FGAs · SEs. -glucose abnormalities Clozapine: Only antipsychotic with proven efficacy in treatment-resistant schizophrenia Not first-line (SEs. agranulocytosis, need for regular blood monitoring). · Mechanism. Inhibit D (less), 5HT, H 1 , M and a1 Copyright © 2000-2018 TIPS Inc. Unauthorized reproduction of this manual is strictly prohibited and it is illegal to reproduce without permission. This manual is being used during review sessions conducted by PharmacyPrep.

94-2

Pharmacyprep.com · · ·

Antipsychotic Drugs

Side effects: Weight gain (greatest), hyperlipidemia, agranulocytosis > (1%), increase risk of diabetes, dyslipidemia, orthostatic hypotension, and high weight gain Therapeutic use: Drug of choice in resistance psychosis, -ve and +ve symptoms MON: CBC q week

Risperidone: · Risperidone-tabs oral disintegrating – do not split. · Side effects: high hyperprolactenemia (dose-related). EPS Olanzapine: SEs. Weight gain (greatest), hyperlipidemia, EPS (especially akathisia), least hyperprolactenemia · Use for acute phase: do not combine with parenteral BDZs. (cardiac & respiratory problems) · Olanzapine- orally disintegrating may stir into 125 mL of water, milk, coffee, orange juice or apple juice consume immediately. · Approved for acute treatment of mania Quetiapine: SEs.Increase risk of diabetes, hyperlipidemia (-TG), and ¯ thyroid hormone levels. Least EPS. Paliperidone is the active metabolite of risperidone. Least sedation Ziprasidone: Agonist activity at 5T 1A receptors and unlike other SGAs has antagonist activity of 5HT 1D . Must be taken with food. Aripiprazole: Partial agonist of D 2 and 5HT 1A . and potent antagonist activity at 5HT 2A receptor. Potential efficacy in -ve symptoms. least sedation. Extrapyramidal side effects. (EPS. dystonia, Parkinsonism, akathisia, tardive dyskinesia, tardive dystonia). Management. Prevention, use SGAs as first-line therapy. if EPS occurs, first reduce dose, consider switch to SGA if on FGA, anticholinergics (benztropine, procyclidine and trihexyphenidyl) should NOT be used prophylactically even with FGAs, and should usually only be used on a short-term basis to treat parkinsonism associated with FGAs, anticholinergics are generally not recommended with SGAs Akathisia - if dose reduction is not effective® b-blockers (e.g., propranolol 10 to 120 mg/day); anticholinergics are ineffective Dystonia (acute torticollis or oculogyric crisis) ® IM benztropine or diphenhydramine (acute), followed by reduction in dose or switch to SGA. Neuroleptic malignant symptoms: Fever, muscle rigidity, Autonomic disturbance, fluctuation in BP, Tachycardia, elevated WBC, and CK. NMS is treated by dantrolene, or bromocriptine

Copyright © 2000-2018 TIPS Inc. Unauthorized reproduction of this manual is strictly prohibited and it is illegal to reproduce without permission. This manual is being used during review sessions conducted by PharmacyPrep.

94-3

Pharmacyprep.com

Antipsychotic Drugs

Comparison of antipsychotic drug side effects Typical (1st generation) Atypical (2nd generation) High sedation (Chlorpromazine) Low sedation (respiradone) High sedation (clozapine) Low weight gain (haloperidol) High weight gain (clozapine) High tardive dyskinesis Low tardive dyskinesia – clozapine Low anticholinergic and sedation high anticholinergic side effects High Sexual Dysfuntion Low sexual dysfuntion (quitiepine) High EPS

Low EPS (Clozapine, quetiapine)

Tips

· · · · · · · · · · · · · · · · · · · · · · · · ·

1. Zuclopenthixol

2. Haloperidol

3. Chlorpromazine

4. Risperidone

5. Clozapine

6. Olanzapine

7. High EPS

8. Tardive dyskinesia

9. Weight gain

10. Agranulocytosis

11. WBC

12.

1st generation 4 to 8 weeks no response, change to 2nd generation. Severe case of psychosis (schizophrenia) or bipolar disorder à 2nd gen (clozapine, olanzapine) increase risk of lipids and diabetes, EXCEPT: Respiridone Least extra pyramidal symptoms à Highest EPS à Patient experiencing hallucination à Patient experiencing social withdrawal à 2nd generation coversà Schizophrenia is characterized by à Metoclopramide à Chlorpromazine à Tardive dyskinesia is caused by à TD symptoms à For resistance schizophrenia DOC à Mechanism of clozapine à Drug of choice for acute agitation in seniorsà it is not use in antipsychotic-naïve patients ( ) it is the only antipsychotic with proven efficacy in treatment-resistant Schizphrenia ( ) the most widely used treatment for psychotic agitation ( ) it is used in patients experiencing withdrawal ( ) this can cause lupus like syndrome ( ) this should not combine with parenteral benzodiazepines ( ) highest extrapyrimidal symptoms ( ) WBC should be monitored because of high agranulocytosis ( ) act on dopamine & serotonin receptors almost equally ( )

Copyright © 2000-2018 TIPS Inc. Unauthorized reproduction of this manual is strictly prohibited and it is illegal to reproduce without permission. This manual is being used during review sessions conducted by PharmacyPrep.

94-4

www.PharmacyPrep.Com

Dementia

95 Dementia · · · · · ·

Dementia is a decline in mental ability that usually progresses slowly, in which memory, thinking, judgment, and ability to pay attention and learn are impaired, and personality may deteriorate. Delirium: Acute mental disorder, with symptoms of acute agitation, hallucination, extreme excitement, and disorientation. Delirium tremens: Acute psychosis caused by alcohol withdrawal. Dementia is a decline in mental ability that usually progresses slowly, in which memory, thinking, judgment, and ability to pay attention and learn are impaired, and personality may deteriorate. Delirium: Acute mental disorder, with symptoms of acute agitation, hallucination, extreme excitement, and disorientation. Delirium tremens: Acute psychosis caused by alcohol withdrawal.

5 A OF ALZEHEIMERS APHASIA INABILITY TO EXPRESS ONSELF THROUGH SPEECH AMNESIA MEMORY LOSS ANOMIA INABILITY TO REMEMBER NAMES AGNOSIA INABILITY TO RECOGNIZE OBJECT APRAXIA MISUSE OF OBJECTS DUE TO FAILURE TO RECOGNISE Severity of dementia · Mild dementia. Forgetting daily activities. Example taking medicines, tel. phone number, finances, directions. · Moderate: Forgetting personal activities. Bathing, dressing, eating (remember upon reminders). · Severe: Forgetting personal activities but cannot recall upon reminders. · Terminal: Patient must be fed, immobile and mute. · Alzheimers dementia is due to? Deficiency of acetylcholine. Levy body dementia associated with psychosis. Diagnosis: MMSE. Minimental scale exam is used to assess cognitive impairment. Beta Amyloid plaques are found in? Alzheimers dementia. Drug of choice of Alzheimer dementia? Donepazil Drug of choice of Levy body dementia? Rivastigmine Symptoms: Memory loss, disorientation, impaired executive functions, behavioural distrubances, depression, psychotic disturbances, and inability to care for self. Copyright © 2000-2018 TIPS Inc. Unauthorized reproduction of this manual is strictly prohibited and it is illegal to reproduce without permission. This manual is being used during review sessions conducted by PharmacyPrep.

95-1

www.PharmacyPrep.Com

Dementia

Risk factors. Age, family history (genetic link), head injury, and cardivascular risk factors. To slow the progression of Alzheimers disease in patients with mild to moderate dementia. Donepezil 5 mg once daily, if no improvement after 4 to 6 weeks increase to 10 mg once daily. · Donepezil. Centrally active reversible, non-competitive. It selective and have greater affinity for AchEI in brain than periphery. Little or no hepatotoxicity. · Rivastigmine: is centrally selective arylcarbamate AchEI, it has short half life of 2hours, but able to inhibit AchEI upto 10 hours. Because of slow dissociation of carbamate enzyme it is referred as pseudoirreversible AchEI.

Donepezil

Drug of choice for Alzheimer's dementia. Mechanism

Side effects DUMBLESS Dosage Advantage Drug and Drug Interactions Monitor

Centrally active reversible, non competitive. It selective and have greater affinity for AchEi in brain than periphery. Little or no hepatotoxicity. · Reduces the hydrolysis of acetylcholine, increasing the amount available in the synaptic cleft. GI effects. Nausea, vomiting and diarrhea. · CNS effects. Fatigue, insomnia, headache · Other effects. Muscle cramps, anorexia, and difficulty in passing urine. ·

· · · · ·

Daily one dose in the morning or evening Initial 5 mg/day, target 10 mg/day adjust dose after 4 wk Toxicity may be ↑ by inhibitors of CYP2D6 or CYP3A4 (e.g. paroxetine, erythromycin, prednisone, grapefruit juice, nefazodone). Effectiveness may be ↓ by inducers of CYP2D6 or CYP3A4 (e.g. carbamazepine, phenytoin, rifampin). Periodic checks may be performed to see benefit of drug.

Rivastigmine Mechanism

· ·

Side effects Therapeutic use Dosage

· · · · · · ·

Inhibits non specific butyrylcholinesterase and reversible acetylcholinesterase or centrally selective arylcarbamate AchEi, It has short half life of 2 hours, but able to inhibit AchEi up to 10 hours. Because of slow dissociation of carbamate enzyme it is referred as pseudoirreversible AchEi. Weight loss due to reduced appetite. Nausea, abdominal pain. Depression, agitation, confusion, drowsiness, dizziness Weakness, trembling, sweating, malaise, convulsions (rare). Drug of choice to treat Lewy body dementia. Treatment of Lewy body dementia. Avoid antipsychotics Acute pain such as dysmenorrhea (painful periods). For rivastigmine, the initial dose is 1.5 mg PO BID, and can be doubled to 3 mg PO BID after 30 days, which is the minimum effective dose.

Copyright © 2000-2018 TIPS Inc. Unauthorized reproduction of this manual is strictly prohibited and it is illegal to reproduce without permission. This manual is being used during review sessions conducted by PharmacyPrep.

95-2

www.PharmacyPrep.Com

Dementia

Galanthamine Mechanism Therapeutic use Side effects Dosage

Selective, competitive, reversible acetylcholinesterase inhibitor and also enhances the action of acetylcholine on nicotinic receptors. Effective in Alzheimer’s and vascular dementia. Nausea, vomiting, abdominal pain, diarrhea, indigestion, decreased appetite and weight loss, headache, dizziness, tiredness, sleepiness or sleeplessness, confusion, runny nose, urinary tract infection, and falls. 16- 32 mg bid.

Memantine. NMDA receptor antagonist.

Tips

· · · · · · · · · · · · · · · · · ·

1 Donepezil 2 Rivastigmine 3 Galanthamine 4 Memantine 5 Tacrine 6 Mild dementia 7 moderate dementia 8 decrease Ach Amyloid plaques are seen in which disease? What is non-selective & limited use because of its hepatotoxicity ( ) What causes Alzheimer’s dementia ( ) What is the drug of choice of Alzheimer’s dementia ( ) The drug of choice or Effective in Alzheimer’s & vascular dementia ( ) The drug of choice or Effective in Lewy body dementia ( ) The drug that has anorexia side effect ( ) What is effective for patients with dementia associated with Parkinson’s disease ( ) N-methyl D-aspartate (NMDA) blocker (4) Memantin (Ebixa)à NMDA inhibitor. Forgeting instrumental activities of daily living (phone#s, driving direction, finances) ( ) Forgeting daily personal activities (bathing, feeding, dressing), recalls upon reminders. ( ) Alzheimers disease cause due to ( ) Delirium is à acute agitation, lack of judgement and thinking. Amnesia isà short loss of memory What are the risk factors for Alzheimers disease à Age, gender, and family history, genetic? Levy body dementia à dementia with hallucination, the DOC is rivastigmine Galantamine is classified as à competitive, reversible acetylcholinesterase inhibitor Rivastigmine mechanism? Inhibits non specific butyrylcholinesterase and reversible acetylcholinesterase.

Terminology · Aphasia : Impaired ability to communicate · Amnesia: short time loss of memory · Dysarthia: difficulty of speech

Copyright © 2000-2018 TIPS Inc. Unauthorized reproduction of this manual is strictly prohibited and it is illegal to reproduce without permission. This manual is being used during review sessions conducted by PharmacyPrep.

95-3

www.PharmacyPrep.Com

Dementia

Copyright © 2000-2018 TIPS Inc. Unauthorized reproduction of this manual is strictly prohibited and it is illegal to reproduce without permission. This manual is being used during review sessions conducted by PharmacyPrep.

95-4

www.PharmacyPrep.Com

Epilepsy

96 Epilepsy Simple Partial: CONSCIOUSNESS IS NOT IMPRAIED like somatosensory warning also called auras. Complex Partial: With alteration of consciousness. The patient may have periods of memory loss, automatism, and aberrations of behaviour. Generalized seizures. Have loss of consciousness and involvement of both hemispheres. Generalized seizure can be sub divided by EEG and clinical manifestation. Partial Seizure Comments Simple Seizures begin locally partial Consciousness not impaired With motor symptoms jerking, lip smacking, chewing motions With autonomic symptoms. Sweating, pupil dilation. With behavioral symptoms Complex Seizure begin locally partial With conscious impairment Begin as simple With or without automatism (automatism: picking at cloths is common may follow visual, auditory hallucinations)

Question alerts! What type of seizure conscious is NOT impaired?

Drug of choice 1st Carbamazepine 2nd Phenytoin 3rd Primidone 4th Gabapentin

Drug of choice 1st Carbamazepine 2nd Phenytoin 3rd Phenobarbital 4th Valproic acid

Generalized seizures Absence True absence seizures (petit mal)à10 to 30 seconds, causes alteration of consciousness, seizures (Petit starts with occasional blinking (Nystagmus). mal) The drug of choice is ethosuccimide. Avoid phenytoin and carbamazepine use in absence seizure. Myoclonic seizures Tonic-clonic seizure (grand mal)

Valproic acid, lamotrigine, and topiramate. Sudden very brief involuntary jerking of facial, limb, and trunk muscles or all body. Tonic seizures generally occur in children, muscle stiffening (tone) Clonic seizures. Sustained muscle contraction altering with relaxation. Sudden loss of consciousness, become rigid, falls to the ground, last about 1 minute.

Copyright © 2000-2018 TIPS Inc. Unauthorized reproduction of this manual is strictly prohibited and it is illegal to reproduce without permission. This manual is being used during review sessions conducted by PharmacyPrep.

96-1

www.PharmacyPrep.Com Status epilepticus

Epilepsy

Single prolonged seizure lasting more than five minutes. In some cases, no regain of consciousness between attacks. The drug of choice is iv diazepam.

Contraception Enzyme inducing drugs such as carbamazepine, oxcarbazepine, phenytoin, phenobarbital, primidone and topiramate may - metabolism of both estrogen and progestins thereby may reducing their concentration by up to 50%. Suggest need to increase the dose of estrogen from 35 mcg to 50 mcg. Midcycle bleeding may indicate that estrogen levels are too low to block ovulation. Drugs that has no interactions with oral contraceptive include gabapentin, valproic acid.

Anti-Epileptic Drugs Therapeutics Iminostilbene Derivatives: Carbamazepine (initial dose 25-33% of projected maintenance dose; increase dose q2-3 wks), adult 600-1600 mg/day divided bid to qid. Children 10-30 mg/kg/day bid or qid suspension. First line therapy for DOC for partial seizures, tonic clonic seizures, treat trigeminal neuralgia Carbamazepine: Side effects include transient neutropenia may worsen absence (petit mal) seizures Stevens Johnson syndrome. Strong drug interactions ↑ clearance of warfarin, OCs, TCAs and risperidone · Oxcarbazepine: SEs: hyponatremia, skin rash cross-reaction with carbamazepine and NO auto induction of liver enzymes. Hydantoin Derivatives: Phenytoin 300-400 mg/day, hs. Children · Available as: Phenytoin 100% suspension & chewable. Phenytoin sodium (capsule & iv) are phenytoin 90% sodium 10% · SE: Skin rash, gingival hyperplasia, and nystagmus · NOT effective for absence (Petit-Mal) seizures. · Phenytoin suspension and chewing tablets are 100% phenytoin · Gingival hyperplasia, recommend mouth hygiene and 0.15% chlorhexidine mouth rinse · Space 30 min before or after food · Swish and swirl then spit out. Benzodiazepines (BDZs): Clobazam, Clonazepam, Diazepam, Lorazepam, Nitrazepam · Rapid onset, tolerance · Tolerance to therapeutic effects. · Diazepam iv; used for status epilepticus Succinimide Derivatives: Ethosuximide. The drug of choice for absence seizures only, few DI and least teratogenicity. Initial dose 500 mg once daily, maintenance 1-2 g/d bid. GABA Derivatives: Gabapentin, Vigabatrin · SE: vision changes and trmors, and visual changes, · Gabapentin: No DI, can use in liver failure, “add on”drug, TID dosing · Vigabatrin: few DI, can use in liver failure, worsen absence seizures · DI: No interaction with oral contraceptives · Safe in pregnancy. Not metabolized. Can be used in liver failure. · Gabpentin: Administration with Al/Mg, Ca antacid may ↓bioavailability. · Gabapentin: drug of choice to treat trigeminal neuralgia, diabetic neuropathy. Copyright © 2000-2018 TIPS Inc. Unauthorized reproduction of this manual is strictly prohibited and it is illegal to reproduce without permission. This manual is being used during review sessions conducted by PharmacyPrep.

96-2

www.PharmacyPrep.Com · ·

Epilepsy

Vigabatrin: worsens absence seizure Pregabalin: Post herpetic neurologia (PHN) and deiabetic neuropathy (DeP)

Barbiturates: Phenobarbital, Primidone · Used for generalized seizure · Tolerance is common · Phenobarbital – take at bedtime (long t 1/2 ) · Primidone – metabolized to phenobarbital

Carboxylic Acid Derivatives: Valproic acid, Divalproex

· · · · · ·

SE: teratogenicity, GI (Valproic acid > Divalproex). Very low incidence of rash No ¯OCs efficacy DOC: mixed 1° generalized seizures (generalized tonic-clonic, myoclonus, absence) and alternate choice for absence seizures. Pregnancyà Neural tube defects No interactions with oral contraceptives Low incidence of rash, Steven Johnson’s syndrome

Other Anticonvulsants: Lamotrigine, Levetiracetam, Topiramate · Lamotrigine. SE: insomnia, rash, no enzyme induction · Levetiracetam – No DI · Topiramate. SEs. kidney stones, weight ¯, cognitive problems (limit use), ↓ efficacy of OC’S, word finding difficulties. ↓ efficacy of OC’S, expensive · Also used as weight loss drug

Tips 1 4 7 10 13

· · · · · · · · · · · · · ·

Carbamazepine Gabapentin clobazam steven jhonson syndrome Petit mal (absence) seizures

2 5 8 11 14

iv Diazepam Topiramate phenobarbital simple partial seizures Tonic clonic

3 6 9 12 15

Phenytoin Valproic acid gingerval hyperplasia Generalised seizure Status epilepticus

Antiepileptics drugs that have least drug interaction with oral contraceptives ( ) What is the drug of choice for status epilepticus ( ) What drugs used to treat partial seizures and tonic clonic seizures ( ) Drugs that can caus Steven johnson syndrome side effect ( ) Antiepileptic drugs not effective in absence (petit mal) seizures ( ) What is the drug of choice for trigeminal neuralgia ( ) What drugs enhances GABA activity, antagonizes amino acid ( ) What is the drug of choice for generalized seizure ( ) What antiepileptic drugs decrease the efficacy of oral contraceptives ( ) What type of seizure in-patient is NOT unconscious? ( ) Carbamazepine interferes with thyroid function test (T/F) Phenobarbital and phenytoin stimulates à Hepatic microsomal enzymes (HME) or CYP450. Carbamazepine is the drug of choice for à Trigeminal neuralgia What is the drug of choice against status epilepticus is à

Copyright © 2000-2018 TIPS Inc. Unauthorized reproduction of this manual is strictly prohibited and it is illegal to reproduce without permission. This manual is being used during review sessions conducted by PharmacyPrep.

96-3

www.PharmacyPrep.Com · · · · · · · · · · · · · · · · · · · · ·

Epilepsy

Avoid phenytoin in à Phenytoin overdose symptoms due to saturation zero order (saturated) kinetics? Nystagmus Gingival hyperplacia associated with phenytoin is treated by mouth hygiene and chlorohexidine mouth wash. Chlorhexidine is mouth rinse used for the treatment à gingivitis, stomatitis (mucositis) Carbamazepine, phenytoin, clonazepam à ↓ efficacy of OCP Topiramate cause à Weight loss, and kidney stones Gabapentin has the least à Drug interactions with OCP Phenytoin available as à Suspension (100%), iv (92%), chewable tablets (100%) and capsule (92%). Carbamazepine available as à Chewable tablets, liquid Clobazam has high à Tolerance Gabapentin indicated for à Diabetic, and post herpetic neuralgia Pregabalin indicated for à Diabetic and post herpetic neuralgia Lamotrigine à have low teratogenic (alternate DOC in pregnancy) Vigabatrin + carbamazepine à worsen absence seizures A patient is on phenytoin but now dose is increased, when is the appropriate time to measure steady stateà 5 to 30 days Phenytoin serum levels monitored for à 10 to 20 mcg/mL List the monitoring parameters for phenytoin.à Plasma phenytoin levels, LFT, CBC, BP, vital signs (with iv use). What is the drug of choice for absence seizure or petit-mal seizure is? Ethosuccimide. Phenytoin plasma concentration is increased by 50%, what is the first side effect seen. Overdose of phenytoin? Abnormal gait and nystagmus Enzyme inducers such as CBZ, Phenobarbital, phenytoin, oxcarbazepine, topiramate (>200 mg/day)? ¯ OCP effectiveness OCP may reduce level of lamotrigine.

Copyright © 2000-2018 TIPS Inc. Unauthorized reproduction of this manual is strictly prohibited and it is illegal to reproduce without permission. This manual is being used during review sessions conducted by PharmacyPrep.

96-4

www.PharmacyPrep.Com

Anti-Parkinson’s Drugs

97 Anti-Parkinson’s Drugs

Parkinson's diease is due to? Decrease in dopamine (DESTRUCTION OF NERVE CELLS IN BASAL GANGLIA). Symptoms: Dysarthria, gait disorders, postural instability, early stage resting tremors, rigidity, and bradykinesia. (Except. eye movements, nystagmus, cognitive impairment, hallucination, tardive dyskinesia). EARLY STAGE TREATMENT Patient who had PD for less than 5 yrs, treatment with MAOi-B, amantadine and anticholinergics may modestly improve mild symptoms. The drug of choice for MILD parkinson’s symtoms is selegiline. The drug of choice to treat Parkinson's disease in patient age over 70 yo? Levodopa+carbidopa Selegiline · SE: insomnia · Selegiline: may need ¯dose in women taking OCs · Drug Interactions: serotonin syndrome with antidepressants, triptans, meperidine, linezolid. NMDA receptor antagonists. Amantadine · Improves L-dopa-induced dyskinesia in the later stages of the disease · MOA: releasing dopamine from the presynaptic terminals or by blocking its reuptake · SE: leg edema, erythema, livedo reticularis · Caution: patients with cognitive deficits a (-confusion) Anticholinergic agents: Benztropine, Ethonopropazine, Procyclidine, Trihexyphenidyl · Effect on tremor, bradykinesia (little or no effect) · Use as monotherapy or as adjuncts to dopaminergic therapy · Benztropine – take with food COMT Inhibitors: Entacapone · COMT inhibitors (e.g., entacapone [Comtan], tolcapone [Tasmar]) decrease the degradation of levodopa and extend its half-life, thus relieving the end-of-dose wearing-off effect and reducing “off” time. · SEs. Relate to - dopaminergic activity in the brain (dyskinesia, confusion/hallucinations etc.) · ¯30% dose of Ldopa with COMT inhibitor.

Copyright © 2000-2018 TIPS Inc. Unauthorized reproduction of this manual is strictly prohibited and it is illegal to 97-2 reproduce without permission. This manual is being used during review sessions conducted by PharmacyPrep.

www.PharmacyPrep.Com · ·

Anti-Parkinson’s Drugs

Tolcapone. SEs: hepatotoxicity (only through the Special Access Program, Health Canada, for use in exceptional cases). Entacapone:SEs. diarrhea (often weeks to months after initiation), discoloration of the urine, NOT appear to cause hepatotoxicity.

Management of complications: The drug of choice for nausea and vomiting is domperidone 10 mg QID AC and HS. (Avoid Metoclopramide) Flucutation and dyskinesia are managed by? change sinemet CR, add COMT. Parkinsons+ psychosis? clozapine, quetiapine, and olanzapine.

Tips 1 deficiency of dopamine 3 Entracapone 5 Selegiline 7 levodopa/carbidopa 9 does not cross blood brain barrier 11 MAO- type B inhibitor · · · · · · · · · · · · · · · · · · ·

2 Amantadine 4 Levodopa 6 Tolcapone 8 Bromocriptine 10 tardive dyskinesia 12

it is a non selective dopamine agonist ( ) it is converted to dopamine within presynaptic dopaminergic neurons ( ) it is indicated in Parkinson’s disease and also for prevention and treatment )of Influenza A viral infections ( it a selective MAO type B inhibitor ( ) it help prevent peripheral metabolism of levodopa, which increase its availability to the brain ( ) PD is due to decrease in à Levodopa does penetrate into brain ( ) Dopamine does not penetrate into brain ( ) Carbidopa does not penetrate into brain ( ) Dyskinesiaà Metochlopramide an antiemetic drug should be avoided in PD patient All antipsychotics require caution in PD Levodopa/carbidopa is DOC in PD Selegline is a selective MAO- type B inhibitor Akinesia à Bradykinesia --> Over treatment of Parkinsonism drugs can result into à Sciatic nerve: are found in all foot branches Why do we add entracapone to levodopa treatment in parkinson’s -->

Copyright © 2000-2018 TIPS Inc. Unauthorized reproduction of this manual is strictly prohibited and it is illegal to 97-3 reproduce without permission. This manual is being used during review sessions conducted by PharmacyPrep.

www.PharmacyPrep.Com

Anti-Parkinson’s Drugs

Copyright © 2000-2018 TIPS Inc. Unauthorized reproduction of this manual is strictly prohibited and it is illegal to 97-4 reproduce without permission. This manual is being used during review sessions conducted by PharmacyPrep.

www.PharmacyPrep.Com

Antimicrobials

98 Antimicrobials Eye Infections

Eye Infections

Blepharitis

Hordeola Or Stye

Conjunctivitis

Keratitis

Hordeola (external hordeolu ‘stye’) or (internal hordeolum “acute meibomianitis) Causative agent S. aureus Site of infection At the edge of the eyelid or underneath it. Head and it ruptures spontaneously within days. Treatment Warm compresses + oral antistaphylococcal agent e.g. cloxacillin or flucloxacillin. Comments Incision and drainage are indicated and patient should be referred to an ophthalmologist if the patient does not respond to the treatment above. Conjunctivitis (pink eye or red eye) Causative Viral, bacterial, chlamydial & noninfectious, allergy, foreign body. agent Treatment Chlamydial disease in adults. Oral tetracycline (doxycycline 100 mg 12 hourly) or Erythromycin (safe in pregnancy) (500 mg 6 hourly) for 7 days, or Azithromycin (safe in pregnancy) 1 g PO as a single dose. Amoxicillin (safe in pregnancy). Gonococcal conjunctivitis in adults. Ceftriaxone 1 g IM as a single dose. Cefixime po tab & suspension. Comments Purulent or mucopurulent discharge suggests a bacterial or chlamydial cause. Symptoms Watery discharge may be associated with upper respiratory infection or adenovirus. Hallmarks of viral conjunctivitis are follicular reaction and preauricular lymphadenopathy. Viral conjunctivitis (“pink-eye”) Treatment Treatment is supportive

Copyright © 2000-2018 TIPS Inc. Unauthorized reproduction of this manual is strictly prohibited and it is illegal to reproduce without permission. This manual is being used during review sessions conducted by PharmacyPrep.

98-1

www.PharmacyPrep.Com

Comments

Antimicrobials

Topical corticosteroid therapy is controversial. Children are generally kept out of school for up to 2 weeks after the onset of the infection

Bacterial Conjunctivitis Causative agent Staphylococcus and/or Streptococcus for adults Haemophilus influenza more common in children Treatment Alternative agents include Gentamicin or Tobramycin eye drops (adults), and ointment (for children), or Fusidic acid eyedrops Conjunctivitis in newborn (ophthalmic neonatum) Causative agent Chlamydia trachomatis or Nisseria gonorrhea Treatment Chlamydia trachomatis. Erythromycin syrup (40 to 50 mg/kg/day) in 3 divided doses for 14 days. If needed treat parents for genital infection. Nisseria gonorrheae. Ceftriaxone 25 to 50 mg/kg IM as a single dose. If needed treat parents for genital infection Comments The best form of prophylaxis is 2.5% aqueous povidone-iodine solution. Canaliculitis Causative agent Treatment

Comments

Dacrocystitis Causative agent Site of infection Treatment

Keratitis Causative agent Site of infection Comments

Actinomyces, and rarely propiobacterium, nocardia or bacteroids. Mechanical expression of the exudative or granular material from canaliculi, combined with probing and irrigation of the nasolacrimal system with penicillin G (100,000 U/ml) eye drop solution. Patients should be referred to an ophthalmologist for definitive treatment. Streptococci including S pneumoniae or S. aureus but culture should guide definitive therapy Infection of nasolacrimal sac Acute infection: Oral amoxicillin-clavulanate or cefuroxime Chronic infections: Irrigate the outflow tract with an antibiotic solution such as penicillin G (100,000 U/ml) as a temporary measure. Definitive surgical decompression ultimately rests with the ophthalmologist Bacteria, Fungi, Herpes simplex virus, acanthamoeba Infection of the cornea It is a sight-threatening ocular emergency and requires prompt recognition and immediately referral to an ophthalmologist.

Herpes simplex keratitis (Viral) or keratoconjuntivitis Causative Herpes simplex (HSV-1) agent Treatment Epithelial disease: Topical antiviral agents e.g. acyclovir ointment applied to eye 5 x day, continued for at least 3 days after healing. Trifluridine (Viroptic) and idoxuridine (Herplex-D) ophthalmic drops. Copyright © 2000-2018 TIPS Inc. Unauthorized reproduction of this manual is strictly prohibited and it is illegal to reproduce without permission. This manual is being used during review sessions conducted by PharmacyPrep.

98-2

www.PharmacyPrep.Com

Antimicrobials

Stromal disease. Complex-combination of antiviral therapy and topical corticosteroids. Ear, Nose, and Throat Infections. (Upper respiratory Tract Infections) Common cold (acute rhinitis) Sinusitis Pharyngitis (Sore throat) Acute bronchitis Chronic bronchitis Pneumonia (CAP) Otitis media Acute otitis externa.

Causative agent Viruses e.g. Rhinoviruses Bacteria e.g. Streptococcus pneumonia Viruses e.g. Adenovirus, Bacteria S. pyogenes. Drug of choice for sore throat is amoxicillin or cefuroxime, azithromycin Viruses e.g. Myxoviruses H. influenza S. pneumonia, H. influenza, M. catarrhalis, M. pneumonia CAP DOC.in out patient. Amoxicillin, Azthromycin, Clarithromycin, Doxycyclin. S. pneumonia, H. influenza, M. catarrhalis DOC. Amoxicillin, amoxi/clav, azithromycin, clarithromycin, cefuroxime axetil. The most common etiology of acute otitis externa is bacterial infection. The microorganism P. aeruginosa (20-60%) and S. aureus 10-70%. Antibiotics aminoglycosides are active against both bacteria.

Prophylaxis of endocarditis prior dental surgeries: S. viridans is present in deep of teeth and also causes cavities. Amoxicillin 2 g 1hr before procedure or 2 hr after procedure, amoxicillin/Clavulanate. If patient is allergic penicillin give cephalexin, clindamycin, and macrolides. Dental abscess treatment: Amoxicillin 500 mg TID for 7 days If patient is allergic penicillin give cephalexin, clindamycin, and macrolides. Skin and Soft Tissue Infections

Bacterial skin infections Impetigo

Cellulites Normal skin flora

Folluculitis

Erysipelas

Necrotising fascitis

Gram (-) ve and Staphylococcus viridans, Streptococcus diphteroid Propionobacterium, actinobacter and yeast. Normal person carry 1012 bacteria on the skin, including Staphylococcus epidermidis and Propionibacterium acnes.

Cellulitis Causative agent Site of infection Treatment

An acute spreading infection of the dermis. Lesion is hot, usually red and swollen, edematous. Strep. pyogenes and/or Staphylococcus aureus Dermis Cloxacillin PO/IV and amoxicillin, cephalexin 7-10d, Clindamycin and cotrimoxazole and azithromycin and clarithromycin.

Copyright © 2000-2018 TIPS Inc. Unauthorized reproduction of this manual is strictly prohibited and it is illegal to reproduce without permission. This manual is being used during review sessions conducted by PharmacyPrep.

98-3

www.PharmacyPrep.Com Impetigo Causative agents Site of infection Treatment

Antimicrobials

Strep. pyogenes (hemolytic group A Strep) and/or S. aureus and presents as bullous, crusted or pustular eruption of the skin. Epidermis layer due to local invasion of causative agent. Cephalexin 7–10 d, if topical fusidic acid or mupirocin 7 –10 d (only topical, because unique mechanism (selectively bind to bacterial isoleucyl-tRNA) and no cross resistance with antibacteria).

Follculitis, boil (furuncles) and carbuncles Causative agent S. aureus Site of infection Invasion of S. aureus in the hair follicles causing minor abscess Treatment Penicillin's or amoxicillin Erysipelas

Causative agent/s Site of infection Treatment Necrotising fasciitis

Causative agent/s Site of infection Treatment

Erysipelas is a rapidly spreading infection of the skin (dermis) It has weel-defined spreading erythematous inflammation, usually accompanied by fever and other systemic manifestation Streptococcus pyogenes, Occasionally S. aureus Dermis of the face Penicillin or Amoxicillin, with or without Fluoxacillin as S. aureus may occasionally be the causative agent. Is an inflammatory response to infection of the tissue below the dermis, spreading with alarming rapidity along the facial planes causing disruption of the blood supply hence necrosis and gangrene Streptococcus pyogenes (B-haemolytic group A Strep) Micro aerophilic Streptococci with anaerobic bacteria Soft tissue below the dermis Benzyl penicillin, and clindamycin with or without Metronidazole, in addition to tissue debridment

Fournier’s gangrene Causative agent/s Treatment Comments

A form of necrotising fasciitis occurring around the groin area Diabetes and local trauma are the main predisposing factors Coliform (E. coli), Streptococci (Group A Strep) Penicillin and Cephalosporin's if allergic to beta lactams use Quinolones (ciprofloxacin) Pain, fever, and systemic toxicity are usually prominent factor of the disease Signs of gas formation and gangrene may also be seen

Infection of Skin by VIRUSES: Causative agent Papilloma virus Molluscum contagiosum (pox virus) Pox virus from sheep, goats

Disease and Characteristics Common wart Fleshy papule Paplovascular lesions, or skin lesions with systemic spread such as herpes simplex (vesicular lesions)

Copyright © 2000-2018 TIPS Inc. Unauthorized reproduction of this manual is strictly prohibited and it is illegal to reproduce without permission. This manual is being used during review sessions conducted by PharmacyPrep.

98-4

www.PharmacyPrep.Com Infection of Skin by fungi Causative agent Dermatophyte (keratinloving fungi) Tinea pedis

Antimicrobials

Disease and characteristics Ring worm or skin lesions can be part of systemic manifestation of the disease such as Cryptococcus neoformans and Blastomyces dermatitidis. Athlete’s foot treatment is clotrimazole 1% cream, miconozole, tolnaftate (topical). Where does athletes foot occurs? Between toes

Infection of the CNS

Bacterial Meningitis (Neonatal 6 weeks) Group B Strep, E. coli and other Children (>3 months) and adults: S. pneumonia, N. meningitis, H. influenza type B (can be prevented by vaccination) Elderly (>50 yrs), alcoholics, immunocompromised, head injuries: E. coli, S. pneumonia, L. monocytogenes Meningococcal Infection: Neisseria meningitis Haemophilus influenza type B Pneumococcus

Encephalitis

Polio Brain abscess Haematogenous

TB meningitis Causative agent Site of infection Treatment

Disease and characteristics Empirical treatment. Ampicillin + Gentamicin or Ampicillin + Ceftriaxone Empirical treatment. Cefataxime, Ceftriaxone or ampicillin or vancomycin

Empirical treatment: Ceftriaxone or ampicillin or vancomycin.

Spread by respiratory route, pharyngeal colonization in 5 to 10% of population. Affects 6 months to 5-year-old children (can be prevented by Hib vaccine). Spread through (respiratory route ®blood®meninges) Elderly patients: pneumonia, immunosuppressed, haematological malignancy. Very young (2 BETA LACTAM IV +CLARITHROMYCIN IV CEFUROXIME Na iv + gentamycin Ceftazidime + aminoglycoside

Bronchitis

Bronchitis Viral Bronchitis (95%)

Bacterial Bronchitis Treatment:

Etiology based on age group 10 years: Influenza, RSV, Adenovirus Chlamydia pneumoniae, Mycoplasma pneumoniae Routine antibiotic treatment is not recommended Antipyretic/analgesic: Acetaminophen Antitussives: Dextromethorphan Beta-agonist: Salbutamol

Copyright © 2000-2018 TIPS Inc. Unauthorized reproduction of this manual is strictly prohibited and it is illegal to reproduce without permission. This manual is being used during review sessions conducted by PharmacyPrep.

98-6

www.PharmacyPrep.Com

Antimicrobials

Urinary tract infection Lower UTI Bladder (cystitis)

Upper UTI Urethra (urethritis)

Ureteritis

Pyelonephritis

Urinary Tract Infection Causative agent The most common causative agen is E. coli. Cystitis Cotrimoxazole 3d, Nitrofurontoin 5d, Trimethoprim 3 day or Cephalexin 7d. If ineffective or allergic use ciprofloxacin 3 d. Urethritis Acute urethral syndrome (urea plasma and chlamydia infection). Doxycycline. During pregnancy used erythromycin. Pyelonephritis Bacterial infection of kidney substances. Ciprofloxacin 7-14 d, cephalosporin's, cotrimoxazole or amino glycosides + ampicillin (for severe). Uncomplicated UTI Infection in healthy patient with normal GU Tract DOC cotrimoxazole 3 days, nitrofurontoin 5 days.

Complicated UTI Factors that aqcuire bacteria and decrease efficacy of therapy such as abnormal genitourinary tract (BPH, stone, bladder cancer, and multidrug resistance) DOC ampicillin+aminoglycoside Ampicillin + vancomycin Blood/pus in urine (TURBID URINE)

Sexually Transmitted Infections Causative Nisseria gonorrhea agents Chlamydia (in children Chlamydia neonatrum) Syphilis Lymphogranuloma Trichomoniasis. vaginitis (colored discharge) Bacterial vaginosis. vaginitis (fishy smell) AIDS Condylomata acuminate Hepatitis B and C Chancroid (syphilis) Genital herpes Genital warts (papilloma virus) Comments Candida infections (Vulvovaginitis) is not STI thereby sexual partner does not require treatment. Symptoms of candida infections are white curdy discharge. If reoccur within 2 months, partner need treatment.

Copyright © 2000-2018 TIPS Inc. Unauthorized reproduction of this manual is strictly prohibited and it is illegal to reproduce without permission. This manual is being used during review sessions conducted by PharmacyPrep.

98-7

www.PharmacyPrep.Com

Antimicrobials

Question Alerts! 1) Chancroid (ulcer) are present in? 2) Trichomonas infection symptoms are? 3) In which STI’s partner require treatment?. Infections of the joint and bones

Joint and bones infections

Arthritis infections

Lyme disease

Osteomyelitis

Infectious arthritis Male/female The gonococcus bacteria may cause different symptoms in women than in men. Women differences may develop red sores on the hand feet, in addition to severe pain in the wrist and ankles. In men, the gonococcus will frequently attack only a single joint, most often the knee. Treatment Arthritis due to gonococcus can be treated with oral ampicillin. Surgery is generally not necessary or particularly helpful Lyme disease A tick-borne infection can cause arthritis and, in severe cases, heart and/or CNS complications. Causative agent Spirochete (Borelia burgdorferi) is transmitted to humans via deer tick (a tiny insect found not only in deer but in squirrels, rabbits, other rodents, birds, and household pets). Comment Prevalent during July to August The drug of choice doxycycline, amoxicillin, or cefuroxime auxetil Osteomyelitis It is a bacterial infection of the bone and bone marrow Causative Agent Staphylococcus aureus. Sequential therapy require (begin with IV and than changed to oral) for 3 weeks. Gastrointestinal Infection Normal person carries 1014 bacteria in his GI tract, 95 to 100% of which are anaerobic. The gut has a resident bacterial population. Stomach Infection with Helicobacter pylori is common and is associated with peptic ulcer disease and gastric cancer. The large These are predominantly anaerobes (99.9%) and an apparent negligible 0.1%) of intestine aerobes. Anaerobes Bacteriosides, Bifidobacterium, Clostridium, anaerobic cocci Aerobes Enterobacteriaceae, E. coli, Klebsiella, Proteus, Enterococci.

Bacteria Shigella Campylobacter jejuni Salmonella Clostridium difficile

Food poisoning Dysentery (traveler’s bloody diarrhea) Traveler’s diarrhea Eggs, poultry, meat Pseudomembranous colitis

Copyright © 2000-2018 TIPS Inc. Unauthorized reproduction of this manual is strictly prohibited and it is illegal to reproduce without permission. This manual is being used during review sessions conducted by PharmacyPrep.

98-8

www.PharmacyPrep.Com Escherichia coli S. aureus Clostridium perfringens Norwalk virus Entomoeba Bacillus cereus (B. cereus) Vibrio parahaemolyticus Listeria

Antimicrobials Meat food poisoning and traveler’s diarrhea Meat, mayonnaise, custard Acute gastroenteritis, reheated dishes Diarrhea in hospitalized patient, cruises. Amoebiasis Reheated rice (Be Serious!!!) Contaminated sea food Meat

Tips Steven-Johnson’s Syndrome (SJS). Rash, skin peeling, and sores on the mucus membrane. In Steven Johnson’s syndrome, a person has blistering of mucus membrane, typically in mouth, eyes and vagina. Patchy areas of rash. SJS can occur in all age groups. Due to SASPAN (sulfonylurea, anticonvulsant (phenytoin, carbamazepine, valproic acid), sulphonamide, penicillin, allopurinol and NSAIDs). Topical sulfadrugs are contraindicated because it may cause disease like SJS, this disease is life threatening. Treatment of SJS is cortisone

SJS may cause by: SASPAN S = sulfonylureas A = anticonvulstants S = sulfonamides P = penicillin A = Allopurinol N = NSAIDs

EYE INFECTIONS 1 Amoxicillin 2 Penicillins 3 Amoxicillin 4 Type 1 allergies 5 Type 2 allergies 6 Azithromycin 7 Minocycline 8 Clindamycin 9 Metronidazole 10 Doxycyclin 11 Ciprofloxacin 12 Cotrimoxazole · Methicillin is only IV and IM · Penicillin G benzathine has long half life · Naficillin is mainly hepatic elimination · Beta lactamase sensitive drugs: Pen G, Amoxi, Pen V and Ampicillin · Endocarditis prophylaxis is (Dental extraction prophylaxis) à · A child less than 2 y allergic penicillin, what is the drug choice for otitis media treatmentà · A patient has heart diseases and underwent prostatic valve surgery. Dentist plan to tooth extraction, what antibiotic is suitable for endocarditis prophylaxisà · Chewable antibioticsà · Beta lactams that should be taken empty stomach à · Aminopenicillins are: · Penicillin allergic patient, alternate drug of choice is? · Penicillins are ineffective in treatment of bacterial infections associated with à · MRSA infections are treated by à · P.colitis associated diarrhea is treated by à · Bacteria is inhabitant in GI, what location of GI tract is commonly found à · Type of bacteria mainly present in colon is à · Penicillin hypersensitive reactions

Copyright © 2000-2018 TIPS Inc. Unauthorized reproduction of this manual is strictly prohibited and it is illegal to reproduce without permission. This manual is being used during review sessions conducted by PharmacyPrep.

98-9

www.PharmacyPrep.Com · ·

Antimicrobials

If allergic penicillins the best alternate choice of antibiotics is macrolide. Penicillins gives what type hypersensitive reaction à Type 1 to V

UPPER RESPIRATORY INFECTIONS 1 Erythromycin 4 Type 1 allergy 7 Room temperature 10 Doxycyclin · · · · · ·

2 Clarithromycin 5 Type 2 allergy 8 Refrigerator 11 Ciprofloxacin

3 Azithromycin 6 Gastric upset 9 H. influenza 12 Cotrimoxazole

Azithromycin suspension stored at à Clarithromycin suspension stored at à Which macrolide suspension have to refrigerate after reconstitution à What antibiotics should caution and require monitoring in patient receiving warfarin What antibiotic potentiate the effect of digoxin and can cause digitalis toxicity à Azithromycin is the drug of choice in traveler diarrhea for patient traveling to à

LOWER RESPIRATORY TRACT INFECTIONS 1 Tetracyclin 4 Photosensitive 7 Room temperature 10 Calcium supplements

2 Doxycyclin 5 Must take empty stomach 8 Refrigerator 11 Dairy

3 Minocyclin 6 CC/PC 9 H. influenza 12 Cotrimoxazole

Tetracyclin are contraindicated are contraindicated in pregnancy and children. Tetracyclin can stain teeth causing discoloration. Oral or topical tetracycline are drug of choice for acne treatment Tetracyclin MUST BE taken on empty stomach. Tetracyclin binds à GI distress (abdominal discomfort, diarrhea) are most common SE. This can be prevented by taking with food or decreasing dose. · Expired tetracycline can lead à · Doxycyclin is the DOC à · Doxycyclin should be taken à · Minocyclin maybe taken with or without food. · Phototoxic reactions (sever skin lesions) can develop with exposure to sunlight. Photoxicity is the most common side effect of doxycyclin or demeclocyclin. · Epimerization is a à Tetracyclins = Take empty stomach Doxycyclins = Take with or after food Minocyclins = Regardless of food

· · · · · ·

Copyright © 2000-2018 TIPS Inc. Unauthorized reproduction of this manual is strictly prohibited and it is illegal to reproduce without permission. This manual is being used during review sessions conducted by PharmacyPrep.

98-10

www.PharmacyPrep.Com

Antimicrobials

SKIN INFECTIONS 1 Clindamycin 4 Photosensitive 7 Room temperature 10 Calcium supplements · · · · · · ·

2 Diarrhea 5 Must take empty stomach 8 Refrigerator 11 Dairy

3 6 9 12

P. colitis CC/PC H. influenza Cotrimoxazole

Most common complication of clindamycin is à Clindamycin is active against à Pseudomembranous colitis symptoms include: fever, abdominal pain, bloody stools. Clindamycin can cause à Clindamycin drug associated diarrhea is treated byà Clindamycin suspension can be stored at à Clindamycin should be taken à

INFECTIONS OF JOINTS AND BONES CNS INFECTIONS 1 Ciprofloxacin 4 Photosensitive 7 Room temperature 10 Calcium supplements · · · · · · · ·

2 moxfloxacin 5 Must take empty stomach 8 Refrigerator 11 Dairy

3 Nor floxacin 6 CC/PC 9 UTI 12 Cotrimoxazole

Fluroquinolones are indicated for UTI, Infectious diarrhea (Travellers diarrhea), lower respiratory tract infections, bone and joint infections (osteomylitis). Gatifloxacin, Moxifloxacin SE are à FluroquinolonesContraindicated in children, under 18 y, pregnant women due to itsà Antacids, bivalent and trivalent ions significantly decrease absorption of à Fluroquinolone increase INR in patient receiving warfarin, therefore monitor à Fluroquinolones can cause hypo or hyperglycemia, therefore monitor à Fluroquinolones at higher alkaline pH can cause à Cipro is the drug of choice in à

URINARY TRACT INFECTIONS 1 Metronidazole 4 Amoeba 7 Room temperature 10 Calcium supplements ·

2 Alcohol 5 Must take empty stomach 8 Refrigerator 11 Dairy

3 Trichomonas 6 CC/PC 9 anaerobic bacteria 12 Disulfiram like reaction

Alcohol with metronidazole can cause à

Copyright © 2000-2018 TIPS Inc. Unauthorized reproduction of this manual is strictly prohibited and it is illegal to reproduce without permission. This manual is being used during review sessions conducted by PharmacyPrep.

98-11

www.PharmacyPrep.Com · · · ·

Antimicrobials

Metronidazole is classified as: Antiprotozoal drug Metronidazole is effective against à Metronidazole discolor urine Metronidazole caution in pregnancy

SEXUALLY TRANSMITTED INFECTIONS 1 Cotrimoxazole 4 Photosensitive 7 Room temperature 10 Calcium supplements · · · · · · · · · ·

2 Sulphamethoxazole 5 Must take empty stomach 8 Refrigerator 11 Dairy

3 Minocyclin 6 PCP 9 UTI 12 pregnancy

Sulfamethoxazole + trimethoprim have à A 22 year old patient currently using cotrimoxazole for UTI, reported sever rashes on arms, neck and back, what are the possible reactionsà What are the folic acid synthesis inhibitorsà Patient with G6PD deficiency, takes sulfadrugs can cause à Hypersensitive reactions of sulfadrugs most commonly involve à Life threatening hepatitis caused by sulfadrug toxicity or sensitization rare SE, the signs and symptoms includeà Sulfamethoxazole have high frequency of skin hypersensitive reaction in patient with If used in last trimester of pregnancy, can cause kernecterus in new born. Cotrimoxazole suspension stored at room temperature in amber color glass bottle. P. carinii pneumonia (PCP) drug of choice is Cotrimoxazole.

FOOD TOXICITIES 1 Vancomycin 4 Clarithromycin 7 Minocycline 10 Doxycyclin · · · · · · · · · · · · · ·

2 Penicillins 5 Streptomycin 8 Clindamycin 11 Ciprofloxacin

3 Tetracycline 6 Azithromycin 9 Metronidazole 12 Cotrimoxazole

use in gram +ve anaerobic bacteria Bacteroid fragilis (abdominal infection) ( ) it should be stored at room temperature ( ) it is not effective for Mycoplasma bacteria ( ) it increase Warfarin INR, increase Digoxin & theophylline levels ( ) it is effective for H. pylori (used along with PPIs in triple therapy ( ) it is use in treatment of acne ( ) drug for treating methicillin-resistant Staphylococcus aureus infections ( ) it has the highest ototoxicity ( ) it is more active against gram-ve H. influenza than Erythromycin ( ) it is use for acne and rheumatoid arthritis ( ) must avoid alcohol while on this drug because it cause disulfiram like reactions ( ) it is use as prophylaxis in traveller’s diarrhea ( ) the drug of choice for UTI 7 traveller’s diarrhea ( ) it is use in chronic treatment of UTI ( )

Copyright © 2000-2018 TIPS Inc. Unauthorized reproduction of this manual is strictly prohibited and it is illegal to reproduce without permission. This manual is being used during review sessions conducted by PharmacyPrep.

98-12

www.PharmacyPrep.Com

Pharmacology

99 Anticancer Drugs and Chemotherapy Damage DNA

Alkylation: Mechlorethamine Cyclophosphamide Ifosfamide Chlorambucil Melphalan Busulfan Lomustine Carmustine Streptozolin Cisplatin Carboplatin Dacarbazine Procarbazine Altretamine/ Hexamethylmelamine Mitomycin

Action of DNA

Others *Actinomycin D *Etoposide *Teniposide *Amsacrine

Inhibit synthesis or functions

Antimetabolites 5-fluorouracil Cytarabine Mercaptopurine Thioguanine Methotrexate

Free radicals: *Bleomycin

Topoisomerase inhibitors Doxorubicin Daunorubicin Topotecan Irinotecan

Prostate cancer. The drug of choice for prostate cancer is Docetaxel, Cabazitaxel, Mitoxantrone, Estramustine and doxorubin. Breast cancer. The drug of choice for breast cancer is tamoxifen after breast surgery. Cervical cancer the drug of choice is radiation + cisplatin

Copyright © 2000-2018 TIPS Inc. Unauthorized reproduction of this manual is strictly prohibited and it is illegal to 99-1 reproduce without permission. This manual is being used during review sessions conducted by PharmacyPrep.

www.PharmacyPrep.Com

Pharmacology

Multiple myeloma the drug of choice is melphalan + prednisone Acute myeloid leukaemia the drug of choice is cytarabine + idarubicin/daunorubicin Skin cancer the drug of choice is topical 5-FU Treatment of extravasation is cold compress and hyaluronidase, sodium thiosulfate, DMSO (dimethyl sulfoxide) Definitions: Neoplasm = New and diseased form of tissue growth Benign neoplasms = Non-cancer form of tissue growth, which can be removed by surgery. No metastases. Malignant neoplasms = Cancer form of tissue growth. Invasive growth of cancer. Malignant neoplasms can be categorized as; Bone marrow = Leukemia (cancer of cells in blood) Connective tissue = Sarcoma Epithelium = Carcinoma Lymphoid tissue = Lymphoma (also named as Hodkins disease) Myeloid stem cells = Myeloid leukemia Endothelium = Kaposis sarcoma Skin (melanocytes) = Malignant melanoma Cell Cycle Phases

Cell cycle phase specific drugs

Cell cycle specific drug (phase-non specific) Cell cycle non specific agents

All cells must traverse the cell cycle phases before and during cell division. Anticancer drugs may act on specific phase. Tumor cells are more responsive to specific drugs G 0 phase – Resting phase G 1 phase – Synthesis of enzymes needed for DNA synthesis S phase –DNA replication (DNA synthesis) G 2 phase –Synthesis of components needed for mitosis. M phase – Mitotic tubule formation àVincristine and vinblastine More active against cells that are specific phase of cycle: G 1 phaseà L-aspraginase and prednisone S phaseà Methotrexate, 6-thioguanine, cytarabine G 2 specificà Bleomycin and etoposide M phaseà Vincristine and vinblastine, peclitaxol Alkylating agents, Antitumor antibiotic, Cisplatin Effective whether cancer cells are in cycle or resting phaseà radiation, nitrosoureas, mechlorethanime

Chemotherapy The treatment of cancer with drugs is called chemotherapy. Antineoplastic drugs, also referred to as chemotherapeutic agents, are drugs that are used to treat cancer. Side effects of chemotherapy Acute: · Extravasation (effects the adjacent tissue)

Copyright © 2000-2018 TIPS Inc. Unauthorized reproduction of this manual is strictly prohibited and it is illegal to 99-2 reproduce without permission. This manual is being used during review sessions conducted by PharmacyPrep.

www.PharmacyPrep.Com · · · · ·

Pharmacology

Vessicant drugs (damage to tissue/necrosis) Example: Bleomycin, cisplatin, dactinomycin, domorubicin, vincristine, vinblastin etc. Thrombophlebitis (inflammation associated with thrombus). Patient with cancer can develop thrombosis after chemotherapy. Due to activation of fibrinogen. Hypersensitive reactions. Example Etopiside, peclitaxol, rituximab, trastuzumab. Rapid tumor lysis syndrome Nausea and vomiting

Chronic, organ specific: Skinàalopecia, dry skin, nail changes, pigmentation (melanoma), and xerostomia. Alopecia is the loss of hair: Drug that cause alopecia is doxorubicin, daunorubicin, cyclophosphamide, vincristine, and paclitaxel. Vessicant agents include: dactinomycin, doxorubicin, mechlorethamine, mitomycin, vincristine, and vinblastine. Hair regrowth occurs after 1-2 months after stopping chemotherapy. Xerostomia: Dry mouth is one of the most common complications associated with radiation therapy. Reversible after 6 to 12 months of therapy. Can be managed by: sugar free hard candy, chewing sugar free gum stimulates salivation. Ice chips, sugarless candies, and commercially available saliva substitute or cholinergic agonist (Pilocarpine 5mg tab). Bone marrow depression (Myelosuppression) · Bone marrow depressionàNeutropenia, and thrombocytopenia · Neutropenia à treated by colony stimulating factors (G-CSF and GM – CSF) Filgrastim or pegfilgrastim. · Thrombocytopenia à for prevention use Oprelvekin (Inerleukin-11) · Complications: Bone marrow suppression is the most dose limiting side effect of cancer · Myelosuppression in general the onset is 7 – 10 days and peak is 10 – 14 days. Recovery count occurs usually occurs in 2 – 3 weeks. · Megaloblastic anemia by methotrexateàFolinic acid (leucovorin, 5-formyltetrahydrofolic acid). · Neutropenia associated anticancer drugsà can be treated by filgrastim (human granulocyte colony stimulating factor). · Least bone marrow depression anticancer drugs is Bleomycin · Cancer patient with anemia à Erythropoeitins are useful Cardiotoxicity: · Risk of CHF, commonly seen with doxorubicin, daunorubicin, epirubicin, mitoxantrone. · 5-FU, capecitabine. Cause coronary spasms, mimicking a myocardial infarction (avoid in know coronary artery disease patients). · Cardiotoxicity can be prevented or lessen by using cardioprotective agent Dexrazoxane Pulmonary toxicity: Pneumonitis, pulmonary fibrosis commonly seen with bleomycin, carmustine, cyclophosphamide, mitomycin, methotrexate, vinca alkaloids. Symptoms of pulmonary toxicity include SOB, non-productive cough, and rarely low-grade fever. Neurotoxicity: · Common with vincristine, vinblastine, Cytarabines, Methotrexate (very little), 5FU, interferon alpha. Copyright © 2000-2018 TIPS Inc. Unauthorized reproduction of this manual is strictly prohibited and it is illegal to 99-3 reproduce without permission. This manual is being used during review sessions conducted by PharmacyPrep.

www.PharmacyPrep.Com · · · ·

Pharmacology

Peripheral neuropathies associated with: Vincristine, peclitaxel: Peresthesia (numbness and tingling) can occur with vincristine, which often appears within few weeks of therapy. High dose of cytarabin may produce cerebllar toxicity that manifest initially as loss of eye-hand coordination and progress to coma. Fludrabine cause severe neurotoxicity Caramustine and other alkylating agents cause little or no neurotoxicity.

GI toxicity Mucositis or stomatitis: Mucositis: Generalized burning, and pain on the ventral surface of tongue. Floor of tongue, mouth looks erythromatus. Stomatitis: generalized inflammation of oral mucosa. · Mucositis or stomatitis: Common with Doxorubicin, Methotrexate, 5-fluorouracil, Actinomycin, Bleomycin capecitabine. · Recommend mouth hygiene, xylocaine, viscous sucralfate, nystatin, sodium bicarbonate, for severe cases peliformin (growth factor) can be used. · Avoid alcohol, antihitamine, steroids, spicy food · Mucositis treatment and prevention: · Topical anesthetics: Viscous lidocaine, or dyclonine HCL 0.5 or 1% · Corticosteroid provides anti-inflammatory action. · Capscisin: Produces burning and pain and ultimately desensitizes pain. · Sucralfate suspension may provide benefit by coating. · For Localized effect: use benzocain in orabase. · Mucositis prevention: · Chlorhexidine gluconate 0.12% (Peridex, Periogard) may reduce severity and frequency of mucositis infections. Nausea and vomitingà · Very high emetics anticancer drugs · Cisplatin · Streptozocin · Cyclophosphamide · High emetics anticancer drugs · Doxorubicin · Methotrexate (250 mg to 1000 mg) · Cytarabine · Lowest emetic anticancer drugs · Bleomycin · Methotrexate (under 50 mg) · Vincristine · Vinblastine · Tamoxifen Non-pharmacological therapy · Take small and frequent meals · Avoid high fat and heavy aroma · Take dry, starchy foods like crackers Copyright © 2000-2018 TIPS Inc. Unauthorized reproduction of this manual is strictly prohibited and it is illegal to 99-4 reproduce without permission. This manual is being used during review sessions conducted by PharmacyPrep.

www.PharmacyPrep.Com

Pharmacology

Management of nausea and vomiting associated with cancer chemotherapy: · The lowest emitogenic drugs nausea and vomiting can be treated by àDexamethasone PRN · High and very high emitogenic drugs associated acute: nausea and vomiting can be treated byà Dexamethasone+ Ondansetrons · DOC for delayed nausea and vomiting à Dexamethasone · Anticipatory nausea and vomitingà Benzodiazepine. Hepatotoxicity Hepatotoxicity monitor LFT, jaundice, or hepatitis: asparaginase, cytarabine, mercaptopurine, and methotrexate. Nephropathy: · Elevate BUN and electrolyte abnormalities: methotrexate may precipitate in kidney. Cisplatin and streptozocin. Amifostine may be used to protect the kidney from the nephrotoxicity associated with cisplatin. Sexual dysfunction: Cyclophosphamide, melphalan, and procarbazine associated with significant infertility in men and women. Hemorrhagic cystitis · It is a bladder toxicity that is seen most commonly after administration of cyclophosphamide and ifosfamide. · These drugs produce a metabolite called acrolein, which cause chemical irritation in bladder mucosa, resulting in bleeding. · Hemorrhagic cystitis caused by Acrolein can be prevented by excessive hydration and subsequent frequent urination. The other method is by administering uroprotecting agent called MESNA, which bind acroleine and prevent from contacting the bladder mucosa. Pulmonary toxicities: The most common with bleomycin, mitomycin, and carmustine. Rationale for combination therapy. Overcoming or preventing resistance. Cytotoxicity to resting and dividing cells. Biochemical enhancement of effect. Beneficial drug interactions rescue host cells. Some agents can be administered intrathecally: Methotrexate, Cytarabine’ Thiotepa Warning: Vincristine should be labelled as Intravenous only. Intrathecally vincristine causes death.

Tips Find the answers from the table: 1. alopecia 2. 4. Doxorubicin 5. 7. Vincristine 8. 10. Bleomycin 11. 13. xerostomia 14. 16. Mercaptopurine, 17. 19. Skin cancer 20. · ·

neutropenia Daunorubicin Paclitaxel Methotrexate 5-fluorouracil Thioguanine Cisplatin

3. 6. 9. 12. 15. 18. 21

chemotherapy Cyclophosphamide Tamoxifen Vinblastine Cytarabine, Methotrexate Streptozosin

Examples antimetabolites include --> Examples of alkylating anticancer drugs -->

Copyright © 2000-2018 TIPS Inc. Unauthorized reproduction of this manual is strictly prohibited and it is illegal to 99-5 reproduce without permission. This manual is being used during review sessions conducted by PharmacyPrep.

www.PharmacyPrep.Com · · · · · · · · · · · · · · · · · · ·

Pharmacology

Non pharmacological measures to prevent nausea and vomiting associated with caneer chemotherapy. Melonoma is à Metoclopramide and dexamethasone are more effective nausea related to à Methotrexate is used for à Which anticancer drugs cause pulmonary fibrosisà Hypertropy is à Hyperplasia is à Least emetic anticancer drug is à Cancer patient on cancer chemotherapy, reports shortness of breath, non productive cough, she may be using drug à DOC for delayed Nausea and vomiting à dexamethasone Mesna is à Doxorubicin preparation should be performed in à Hypertropy is à Hyperplasia is à Melatonin à Peclitaxel and docetaxel act on à Cancer estimated deaths in men: Lung cancer 31% and Prostate cancer 11% Cancer estimated deaths in women: Lung cancer 25% and Breast cancer 15% Examples antimetabolites include -->*5-fluorouracil, *Cytarabine, *Mercaptopurine, *Thioguanine and Methotrexate

Copyright © 2000-2018 TIPS Inc. Unauthorized reproduction of this manual is strictly prohibited and it is illegal to 99-6 reproduce without permission. This manual is being used during review sessions conducted by PharmacyPrep.

100 Pharmacognosy & Natural Products Natural Products Natural product Cranberry Vaccinium macrocarpon Dong Quai Angelica sinensis

Classification Antioxidant, bacteriostatic effect Tonic, immunostimulating, antispasmodic

Echinacea Echinacea angustifolia/purpurea

Antibiotic, antifungal immunostimulating

Evening Primrose Oenothera biennis

Anti-spasmodic

Feverfew Tanacetum parthenium

Anti-inflammatory, emmenagogue

Garlic Allium sativum

Antibiotic, antifungal, anti-viral

Licorice Glycyrrhiza glabra

Demulcent, diuretic, expectorant, laxative

Use Cleanses and stops infections in the urinary tract. Used to treat all symptoms of menopause as an alternative treatment to estrogen therapy. Regulates the hormonal system. Overall tonic for female reproductive system. Reduces high blood pressure and PMS. Caution: Contra-indication in pregnancy Stimulates and boosts immune function. Has cortisone-like activity that helps wound healing. Fights bacterial and viral infections. Contra-indication in auto-immune diseases (i.e. Multiple Sclerosis, AIDS) Used in treatment of multiple sclerosis and PMS. Helps prevent heart disease and stroke and maintains healthy skin. Excess consumption can result in oily skin. Helps prevent migraine headaches and also useful against swelling and arthritis. Stimulates digestion and improves liver function. Caution: Not to be used by lactating or pregnant women. Reduces high blood pressure and blood cholesterol. Immune support for respiratory system. Anti-cancer and digestive tonic. Caution: Not to be used by lactating women because it can pass to the breast milk and cause colic in infants. Gastric ulcers, adrenal insufficiency hypoglycemia. Good for coughs and other bronchial complaints. Caution: Contraindicated for those with high blood pressure or if pregnant.

Copyright © 2000-2018 TIPS Inc. Unauthorized reproduction of this manual is strictly prohibited and it is illegal to 100-1 reproduce without permission. This manual is being used during review sessions conducted by PharmacyPrep.

Ginger Zingiber officinale Ginkgo Biloba Ginkgo biloba

Ginseng Panax schin-seng

· · · · · · · · · · · ·

Diaphoretic, cholagogue, carminative, stimulant Anti-asthmatic, bronchodilator, platelet activating factor (PAF) inhibitor Tonic, stimulant, demulcent, stomachic

Relieves indigestion and abdominal cramping. Benefit in relieving motion sickness, dizziness, nausea and colds. Ginger lowers blood clotting. Increases blood flow to the brain. Improves memory loss. Alzheimer’s disease cerebral vascular insufficiency and inhibits blood clotting. with warfarin and Aspirin. Take with food. Stimulates both physical and mental activity. Anti-fatigue (insomnia, nervousness, poor appetite). Enhances immune system, inhibits exhaustion of adrenal gland and anti-stress.

Echinaceaà Common cold Saw palmettoà Prostate (BPH-Benign Prostate Hyperplacia) Garlic à lipid levels Feverfew à Migraine Gingkoà Increase memory St. Johns wart à antidepressant Bitter melonà anti-diabetic Prime rose oil à PMS (premenstrual syndrome) Atropineà anticholinergic Vincristine and vinblastinàanti-cancer Taxol (pecli-taxol)àYew plants (Himalayas)àseveral types of cancers Licorice can cause hypertension

Copyright © 2000-2018 TIPS Inc. Unauthorized reproduction of this manual is strictly prohibited and it is illegal to 100-2 reproduce without permission. This manual is being used during review sessions conducted by PharmacyPrep.

www.PharmacyPrep.com

Canadian Pharmacy Review

Abbreviations Abbreviatio n a ac pc c cc AD AS AU OD OS OU dr dx g

Meaning

Latin Phrase

Abbreviation

Meaning

Latin Phrase

Before Before meals After meal with With food Right ear Left ear Both ear Right eye Left eye Both eyes Dram Diagnosis Gram

Ante Ante cibum Post cibum

bid q qam q.d qod qid

Twice a day every Every morning Everyday Every other day Four times a day

bis in die quaque

p.o. pr p.r.n. pt

By mouth Per rectum As needed Pint

Per os

Q12h

BID

gr gtt h hr hs hx ID IM

Grain Drop Hour Hour At bed time History Intradermal Intramuscular

Every twelve hours quart prescription without One half (1/2) immediately suppository symptoms tablespoon

IU or U

International unit Intravenous IV piggyback Kilogram Liter Pound Microgram Milliequivalen t Milligram Milliliter Ounce post

IV IVPB kg L lb mcg mEq mg ml oz p

D= dexter S=sinister

Hora somni

units

mg

qt rx, Rx s ss stat supp sx T, Tbsp or tbs t, tsp tid TO tr tx ung VO aa/aaa Ad lib ud Q8H

Quaque die Quarter in die

Pro re nata

sine

teaspoon 3 X a day Telephone order tincture treatment ointment Verbal order Of each/ apply affected area As desired As directed

ter in die

Ad libitum

post

Copyright © 2000-2018 TIPS Inc. Unauthorized reproduction of this manual is strictly prohibited and it is illegal to reproduce without permission. This manual is being used during review sessions conducted by PharmacyPrep.

1

www.PharmacyPrep.com

Prefix a-;an-;ana abanteantiautobi bradycarcin contra DysEcten endo epi Ex. Gynec/o hemi hyper hypo infra inter intra iso macro mal meta

Suffix ac; al; ar; ary algia cele centesis crine crit cyte cytosis desis ectomy emesis emia genesis, genic, gen globin; globulin

Meaning No; not; without Away from Before; forward against self Two double both Slow cancerous Against; opposite Difficult; painful Outside; out Within; in Within Above; upon Out or extemporaneous woman half Above; excess Below; deficient Below; inferior between within Same; equal large Bad; poor; abnormal Change; after; beyond

Meaning Pertaining to pain Hernia, herniation Surgical puncture To secrete To separate Cell Condition of cells Binding together Surgical removal; excision Vomiting Blood Producing; forming protein

Canadian Pharmacy Review

micro Multi neo non oligo pan para per peri poly post pre primi retro semi

Prefix

Meaning Small many New Not Few; less All Near; beside Through Around Many After Before; in front of First Behind; back; upward Half

sub super supra sym syn tachy tri uni xero aa

Below; under Above; over; excess Above; on top of With Together; with Fast Three One Dry Apply affected area

Suffix otomy ous paresis pathy penia pepsia phagia phobia phonia plasty

Meaning Incision into Pertaining to Paralysis Disease Decrease number Digestion Eating, swallowing Abnormal fear Voice; sound Surgical repair

plegia pnea poiesis r/rhage; r/rhagia

Paralysis Breathing Formation Bursting forth

Copyright © 2000-2018 TIPS Inc. Unauthorized reproduction of this manual is strictly prohibited and it is illegal to reproduce without permission. This manual is being used during review sessions conducted by PharmacyPrep.

2

www.PharmacyPrep.com gram graph graphy ia; iac; ic ism itis lysis; lytic malacia megaly oid (o)logist (o)logy oma osis ostomy

Root Abdomin/o Aden/o Adip/o Aminio Andr/o Angi/o Aque/o Arteri/o Arteriol/o Arthr/o Ather/o Audi/o Aur/o bili Blephar/o Bronch/o Bronchiol/o Bucc/o Burs/o Calc/i Capnia Carcin/o Cardi/o Carp/o Cephal/o Cerebr/o Chol/e Cholangi/o Cholecyst/o Chondr/o

Record Instrument for recording Process of recording Pertaining to Condition Inflammation Break down Softening enlargement Resembling; like Specialist Study of tumor Abnormal condition Creation of an opening

Meaning abdomen Gland Fat amnion Male; man Vessel Watery Artery arteriole joint Fat; fatty plaque Hearing; sound ear Bile; gall eyelid bronchus bronchiliole Inside cheek joint calcium Carbon dioxide Cancer heart Wrist bone head cerebrum Bile; gall Bile duct gallbladder cartilage

Canadian Pharmacy Review rrhea rhesis sclerosis scope scopy somnia spasm stasis stenosis therapy thorax tocia tripsy trophy tropin

Root Cyst/o Cyt/o Dacry/o Dent Derm/o Dermat/o Dipl/o Dips/o Duoden/o Dur/a Electr/o Embry/o Encephal/o Enter/o Eosin/o Epis/i Erythr/o Esophag/o Fasci/o Femor/o Fet/o; fet/i Fibul/o Fund/o Gastr/o Gastr/o Gingiv/o Glauc/o Gli/o Glomerul/o Gloss/o

Flow; discharge Rupture Hardening Instrument for viewing Process of viewing Sleep Twitch Control; stop narrowing Treatment Chest; pleural cavity Labor; birth Crushing Growth; development Nourish; development; stimulate

Meaning Bladder, sac, urinary bladder Cell Tears Tooth Skin Skin Two, double Thirst Duodenum Dura mater Electricity Embryo Brain Intestines Red Vulva Red Esophagus Fascia Femur Fetus Fibula Fibula Fundus Stomach Gums Silver/gray Nerve cell Glomerulus Tongue

Copyright © 2000-2018 TIPS Inc. Unauthorized reproduction of this manual is strictly prohibited and it is illegal to reproduce without permission. This manual is being used during review sessions conducted by PharmacyPrep.

3

www.PharmacyPrep.com Coagul/o Cochle/o Col/o Conjunctiv/o Cor/o Corne/o Coron/o Cost/o Crani/o Cry/o Cut/o, cuti Cutane/o Cyan/o

clotting cochlea Colon Conjunctiva heart cornea heart rib cranium cold skin skin blue

Root ile/o ili/o Immune/o is/o Jejun/o Kal/i Kinesi/o Lact/o Lapar/o Laryng/o Ligament/o lingua Lip/o Lith/o Lumb/o Mamm/o Mast/o Melan/o Men/o Metacarp/o Metatars/o Morph/o Muc/o Myc/o Myel/o Miring/o Narc/o Nas/o Nat/o, natal Nephr/o Neur/o Noct/o

Meaning ileum ilium Protection Equal jejunum potassium tears Milk Abdominal wall larynx ligament Tongue fat stone lymph breast breast black Menses, menstruation Hand bones Foot bone Form shape mucus fungus Bone marrow, spinal cord eardrum sleep nose Birth, delivery kidney nerve night

Canadian Pharmacy Review Gluc/o Gonad/o Gravid/a/o Gyn/o; gyn/e Gynec/o Hem/o Hemangio/o Hemat/o Hepat/o Hidr/o Humer/o Hydr/o Hyster/o Gait

Glucose; sugar Sex glands Pregnancy woman Woman Blood Blood vessel Blood Liver Sweat Humerus Water; fluid Uterus imbalance

Root Orth/o Oste/o Ot/o Ovari/o oxi Pachy/o Pancreat/o Par/o Patello/o Pector/o ped Pelv/i Perine/o Peritone/o Peritone/o Phag/o Phalang/o Pharyng/o Phleb/o Phot/o Phren/o Pil/o Pneum/o Pod/o, podi Proct/o Psych/o, psych/i Pub/o Pulmon/o Py/o Pyel/o Quadr/i Radi/o

Meaning Straight Bone Ear Ovary Oxygen Thick Pancreas Bear; labor; childbirth Knee cap Chest Children Pelvic Pelvis Perineum Peritoneum Eat Finger and toe bones Pharynx Vein Light Diaphragm Hair Lung/air Foot Rectum Mind or soul Pubis; pubic bone Lungs Pus Renal pelvis Four Radius

Copyright © 2000-2018 TIPS Inc. Unauthorized reproduction of this manual is strictly prohibited and it is illegal to reproduce without permission. This manual is being used during review sessions conducted by PharmacyPrep.

4

www.PharmacyPrep.com

Canadian Pharmacy Review

Nyctal/o Ocul/o Onych/o Oophor/o

night eye nail ovary

Rect/o Ren/o Retin/o Rhabdomy/o

Ophthalm/o Opt/o Or/o Orch/o Orchid/o

eye eye mouth Testis; testicle Testis; testicle

Rheum Rhin/o Salping/o Sarc/o Semin/o

Root septi Sial/o Sinus/o Somat/o Spermat/o Sphere/o Sphygm/o Spir/o Spleen/o Spondyl/o Steth/o Stoma, stomat/o Synovi/o Tars/o

Meaning bacteria saliva sinus body sperm round pulse Breathe; breath spleen Vertebra; vertebral column Chest mouth joint Ankle bones

Rectum Kidney Retina Skeletal muscle; striated muscle Watery discharge Nose Fallopian tubes Flesh semen

Root Ten/o; tend/o Tendon/o; tendin/o Test/o Testicul/o Thorac/o Thromb/o Thyr/o Trache/o Tympan/o Urethra/o

Meaning tendon tendon Testis/testicle Testis/testicle Chest Clot Thyroid gland Trachea Eardrum Urethra

Ur/o Vas/o Ven/o Xanth/o

Urinary tract Vessel Vein yellow

Copyright © 2000-2018 TIPS Inc. Unauthorized reproduction of this manual is strictly prohibited and it is illegal to reproduce without permission. This manual is being used during review sessions conducted by PharmacyPrep.

5

www.PharmacyPrep.com

Canadian Pharmacy Review

Copyright © 2000-2018 TIPS Inc. Unauthorized reproduction of this manual is strictly prohibited and it is illegal to reproduce without permission. This manual is being used during review sessions conducted by PharmacyPrep.

6